0% found this document useful (0 votes)
90 views423 pages

Mathematics MODULE

The document is a comprehensive mathematics module for grades 9-12, covering various topics such as sets, geometry, polynomial functions, and calculus. It includes detailed explanations, examples, and methods for solving mathematical problems, along with review exercises and practice questions. Acknowledgements are provided for contributors to the module's development.

Uploaded by

abukaes70
Copyright
© © All Rights Reserved
We take content rights seriously. If you suspect this is your content, claim it here.
Available Formats
Download as PDF, TXT or read online on Scribd
0% found this document useful (0 votes)
90 views423 pages

Mathematics MODULE

The document is a comprehensive mathematics module for grades 9-12, covering various topics such as sets, geometry, polynomial functions, and calculus. It includes detailed explanations, examples, and methods for solving mathematical problems, along with review exercises and practice questions. Acknowledgements are provided for contributors to the module's development.

Uploaded by

abukaes70
Copyright
© © All Rights Reserved
We take content rights seriously. If you suspect this is your content, claim it here.
Available Formats
Download as PDF, TXT or read online on Scribd
You are on page 1/ 423

Contents

ACKNOWLEDGEMENTS .................................................................................................................... IV
UNIT ONE: FURTHER ON SETS ........................................................................................................ 1
1.1. INTRODUCTION TO CONCEPTS OF SETS. ............................................................................. 1
1.2. THE NOTION OF SETS ................................................................................................................ 2
1.3. OPERATIONS ON SETS .............................................................................................................. 4
SOLVED PROBLEM FURTHER ON SET......................................................................................... 8
UNIT TWO: GEOMETRY AND MEASUREMENT ....................................................................... 11
2.1. REGULAR POLYGONS ............................................................................................................. 11
2.2. FURTHER ON CONGRUENCY AND SIMILARITY ............................................................... 13
2.3. FURTHER ON TRIGONOMETRY ............................................................................................ 17
2.4. CIRCLES ...................................................................................................................................... 20
2.5. MEASUREMENTS ...................................................................................................................... 23
SOLVED PROBLEMS ON GEOMETRY AND MEASUREMENTS .............................................. 25
UNIT THREE POLYNOMIAL FUNCTIONS ................................................................................ 27
3.1. INTRODUCTION TO POLYNOMIAL FUNCTIONS ............................................................... 27
3.2. THEOREMS ON POLYNOMIALS ............................................................................................ 31
3.3. ZEROS OF A POLYNOMIAL FUNCTION ............................................................................... 32
4.4. GRAPHS OF POLYNOMIAL FUNCTIONS.............................................................................. 35
UNIT FOUR: EXPONENTIAL AND LOGARITHMIC FUNCTIONS.......................................... 38
4.1. EXPONENTS AND LOGARITHMS .......................................................................................... 38
4.2. THE EXPONENTIAL FUNCTIONS AND THEIR GRAPHS. .................................................. 40
4.3. GRAPHS OF EXPONENTIAL FUNCTIONS. ........................................................................... 40
4.4. LOGARITHMS. ........................................................................................................................... 43
4.5. THE LOGARITHMIC FUNCTIONS AND THEIR GRAPHS. .................................................. 46
4.6. EQUATIONS INVOLVING EXPONENTS AND LOGARITHMS. .......................................... 50
4.7. APPLICATIONS OF EXPONENTIAL AND LOGARITHMIC FUNCTIONS. ........................ 51
UNIT FIVE: SOLVING OF INEQUALITIES ................................................................................... 54
5.1. INEQUALITIES INVOLVING ABSOLUTE VALUE ............................................................... 54
5.2. SYSTEMS OF LINER INEQUALITIES IN TWO VARIABLES .............................................. 57
5.3. QUADRATIC INEQUALITIES .................................................................................................. 59
UNIT SIX. PLANE GEOMETRY ....................................................................................................... 62
6.1. THEOREMS ON TRIANGLES ................................................................................................... 62

I
6.2. SPECIAL QUADRILATERALS ................................................................................................ 65
6.3. MORE ON CIRCLES................................................................................................................... 67
6.4. REGULAR POLYGONS(5) ........................................................................................................ 69
UNIT SEVEN MEASUREMENT ........................................................................................................ 70
7.1. SURFACE AREAS AND VOLUMES OF PRISMS AND CYLINDERS. ................................ 70
7.2. PYRAMIDS, CONES AND SPHERES ....................................................................................... 73
7.3. FRUSTUMS OF PYRAMIDS AND CONES .............................................................................. 77
UNIT EIGHT FURTHER ON RELATION AND FUNCTIONS ..................................................... 81
8.1. REVISION ON RELATIONS ...................................................................................................... 82
8.2. SOME ADDITIONAL TYPES OF FUNCTIONS ....................................................................... 87
8.3. CLASSIFICATION OF FUNCTIONS ...................................................................................... 102
8.4. COMPOSITION OF FUNCTIONS ............................................................................................ 106
8.5. INVERSE FUNCTIONS AND THEIR GRAPHS ..................................................................... 109
REVIEW EXERCISES ON UNIT 8 ................................................................................................ 114
PRACTICE QUESTIONS ON UNIT 8 ............................................................................................ 115
UNIT NINE: COORDINATE GEOMETRY ................................................................................... 120
9.1. STRAIGHT LINES .................................................................................................................... 121
9.2. CIRCLE ...................................................................................................................................... 134
REVIEW EXERCISES ON UNIT 9 ................................................................................................ 142
PRACTICE QUESTIONS ON UNIT 9 ............................................................................................ 143
UNIT TEN STATISTICS AND PROBABILITY............................................................................ 146
10.1. STATISTICS .......................................................................................................................... 147
10.2. PROBABILITY ...................................................................................................................... 182
REVIEW EXERCISES ON UNIT 10 .............................................................................................. 209
PRACTICE QUESTIONS ON UNIT 10 .......................................................................................... 210
UNIT ELEVEN: MATRICES AND DETERMINANTS ................................................................ 215
11.1. MATRICES ............................................................................................................................ 216
11.2. DETERMINANTS AND THEIR PROPERTIES .................................................................. 236
11.3. INVERSE OF A SQUARE MATRIX .................................................................................... 247
11.4. SYSTEMS OF EQUATIONS WITH TWO OR THREE VARIABLES ............................... 254
11.5. CRAMER‟S RULE ................................................................................................................ 266
REVIEW EXERCISES ON UNIT 11 .............................................................................................. 269
PRACTICE QUESTIONS ON 11 .................................................................................................... 271

II
UNIT TWELVE:VECTORS AND TRANSFORMATION OF THE PLANE .............................. 276
12.1. REVISION ON VECTORS AND SCALARS ....................................................................... 277
12.2. REPRESENTATION OF VECTORS .................................................................................... 278
12.3. COMPONENTS OF VECTORS ............................................................................................ 282
12.4. PRODUCTS OF VECTORS ................................................................................................. 288
12.5. APPLICATION OF VECTOR ............................................................................................... 293
12.6. TRANSFORMATION OF THE PLANE ............................................................................... 300
REVIEW EXERCISES ON UNIT 12 .............................................................................................. 325
PRACTICE QUESTIONS ON UNIT 12 .......................................................................................... 329
UNIT THIRTEEN:SEQUENCE AND SERIES ............................................................................... 333
13.1. SEQUENCE ............................................................................................................................ 334
13.2 ARITHMETIC AND GEOMETRIC SEQUENCES................................................................. 335
13.3 THE SIGMA NOTATION AND PARTIAL SUMS................................................................. 337
13.4. INFINITE SERIES ................................................................................................................. 339
SOLVED PROBLEMS ON SEQUENCE AND SERIES ................................................................ 340
UNIT FOURTEEN: INTRODUCTION TO LINEAR PROGRAMMING ................................... 344
14.1. REVISION ON LINEAR GRAPHS ...................................................................................... 345
14.2. GRAPHICAL SOLUTIONS OF SYSTEMS OF LINEAR INEQUALITIES ....................... 347
14.3. MAXIMUM AND MINIMUM VALUES ............................................................................. 351
14.4. REAL LIFE LINEAR PROGRAMMING PROBLEMS ....................................................... 357
REVIEW EXERCISES ON UNIT 14 .............................................................................................. 362
PRACTICE QUESTIONS ON UNIT 14 .......................................................................................... 363
UNIT FIFTEEN:MATHEMATICAL APPLICATIONS IN BUSINESS ...................................... 366
15.1. BASIC MATHEMATICAL CONCEPTS IN BUSINESS .................................................... 367
15.2. COMPOUND INTEREST AND DEPRECIATION .............................................................. 376
15.3. SAVING, INVESTING AND BORROWING MONEY ....................................................... 382
15.4. TAXATION ............................................................................................................................ 387
REVIEW EXERCISES ON UNIT 11 .............................................................................................. 394
UNIT SIXTEEN: INTRODUCTION TO CALCULUS .................................................................. 395
16.1. INTRODUCTION TO DERIVATIVES. ................................................................................. 396
16.2. APPLICATIONS OF DERIVATIVES .................................................................................... 398
16.3. INTRODUCTION TO INTEGRATION .................................................................................. 399
SOLVED PROBLEMS ON INTRODUCTION TO CALCULUS ................................................... 400

III
ACKNOWLEDGEMENTS
I would like to express my sincere gratitude to all those who contributed to the development of this
Grade 9-12 Mathematics module.

First and for most, I extend my heartfelt thanks to my mentor and content editor, Mitiku Daba (Ph.D),
for his invaluable guidance and support in chronological order arrangement of chapters, contents and
sub-contents throughout this process. Their passion for mathematics and dedication for mathematics
and dedication to teaching inspires me all the time.

I also equally wish to acknowledge Feyisa Kebede (Ph.D), curriculum editor for their collaboration
and feedback, which greatly enhanced the content and clarity of the material. Special thanks to Mr.
Ahmed Awol and Gezahagn Kasehey for their assistance in reviewing the module and providing
insightful suggestions.

Additionally, I am grateful to Adama Science and Technology University (ASTU) for their provision of
Laptop computer and environment necessary for writing and development of this module.

Finally, I would like to thank ASTU Special School directors Mr. Garedew Jima (M.A) and Mekonnen
Kebede (M.A) for their unwavering encouragement and support during my studies.

Thank you all for your contributions to this module.

IV
UNIT ONE: FURTHER ON SETS

1.1. INTRODUCTION TO CONCEPTS OF SETS.

Sets are important concepts in mathematics. They are useful in defining and solving of mathematical
problems.
Sets and Elements
Sets is a collection of any object, things or numbers of well-defined.
 The individual objects in set are called elements of set.
 Repeating elements in a set does not add new elements to the set.
For example, the set {a, a, a} is the same as {a}.
 A set is known by its elements.
 A set is well-defined if and only if it can be decided that a given object is an element of the set.
Notation: we use capital letters to name sets and small letters to represent elements. The symbol „∈‟
stands for the phrase „is an element of‟ (or „belongs to‟). So, x∈A is read as „x is an element of A‟ or „x
belongs to A‟. We write the statement „x does not belong to A‟ as x∉A.
3.1. Description of sets
 A set may be described by three methods:
I. Verbal method
 This method may be used when there is a large number of elements or when all elements cannot be
named. We may describe a set in words.
Example:
a. The set of all whole numbers less than ten or
{all whole numbers less than ten}.
a. The set of all natural numbers. This can also be written as
{all natural numbers}
II. The listing method
If the elements of a set can be listed, then we can describe the set by listing its elements. The elements
can be listed completely or partially as illustrated in the
Complete listing method
In this method we list all elements of the set. We use this method if the set has small elements of the set.

Telegram channel https://t.me/johnson201485 1


b. Partial listing method
We use the partial listing method, if listing all elements of a set is difficult or impossible but the elements
can be indicated unambiguously by listing a few of them.
Example 1 Describe (express) each of the following sets using the listing method:
a. The set whose elements are a, 2 and 7. d. The set of non-positive integers.
b. The set of natural numbers less than 51. e. The set of integer
c. The set of whole numbers.
Solution:
a. First let us name the set by A. Then we can describe the set as A = {a, 2, 7}
b. The natural numbers less than 51 are 1, 2, 3, . . ., 50. So, naming the set as B we can express B by the
listing method as B = {1, 2, 3, . . ., 50}
c. Naming the set of whole numbers by, we can describe it as = {0, 1, 2, 3, . . .}
d. If we name the set by L, then we describe the set as L = {. . ., –3, –2, –1, 0}
e. You know that the set of integers is denoted by ℤ and is described by ℤ = { ., –3, –2, –1, 0, 1, 2, 3, .. .}
Note: The three dots indicate that elements are continue
III. Set builder method : In this method the elements are described with some elements
of the set. Every set can be described by this method.
Example 2: Express each of the following sets using set -builder method:

1.2. THE NOTION OF SETS

Empty Set, Finite Set, Infinite Set, Subset, Proper Subset


Empty Set: A set that contains no elements is called an empty set, or null set. An empty set is denoted by
either ∅ or { }.

Telegram channel https://t.me/johnson201485 2


In the give example above both sets are empty. The reason is the first one has no real root and in the
second any number is equal to itself. So, both of the given set is empty.
Finite and Infinite Set
 A set S is called finite, if it contains n elements where n is some non-negative integer.
 A set S is called infinite, if it is not finite.
Notation: If a set S is finite, then we denote the number of elements of S by n (S).
Example1: Find n (S) if:

Example 2: The following sets are an examples of infinite sets

Subsets: Set A is a subset of set B denoted by A ⊆ B, if each element of A is an element of B.


Note: 1. If A is not a subset of B, then we denote this by A ⊈ B.
2. For any set A, I ∅ ⊆ A. II. A ⊆ A
Example 3: a.

Power set: Let A be any set. The power set of A, denoted by P(A), is the set of all subsets of A. That is,
P(A) = {S|S ⊆ A}
Proper Subset: Set A is said to be a proper subset of a set B, denoted by A ⊂B, if A is a subset of B and
B is not a subset of A.
That is, A ⊂ B means A ⊆ B but B ⊆ A
Note:1. For any set A, A is not a proper subset of itself.
2. If a set A is finite with n elements, then
I The number of subsets of A is and
II The number of proper subsets of A is

Telegram channel https://t.me/johnson201485 2


Venn Diagrams, Universal Sets, Equal and Equivalent Sets
Venn diagrams: To illustrate various relationships that can arise between sets, it is often helpful to use a
pictorial representation called a Venn diagram. These diagrams consist of rectangles and closed curves,
usually circles. The elements of the sets are written in their respective circles.
 For example, the relationship A ⊂B can be illustrated by the following Venn diagram.
Example 1

Universal sets
The universal set is the totality of elements under consideration as elements of any set and including all
possible elements of the set.

Equality of sets : Given two sets A and B, if every element of A is also an element of B and if every
element of B is also an element of A, then the sets A and B are said to be equal. We write this as A = B
∴ A = B, if and only if A ⊆ B and B ⊆ A.
Example: a. Let A = {1, 2, 3, 4} and B = {1, 4, 2, 3}.
A = B, since these sets contain exactly the same elements.
b. E = {x∈ℝ/(x – 2) (x – 3) = 0} and F = {x∈ℕ/1 < x < 4}.
By listing completely, the elements of each set, we have E = {2, 3} and F = {2, 3}. We see that E and F
have exactly the same elements. So, they are equal.
Note: If A and B are not equal, we write A ≠ B.
Example: Let C = {–1, 3, 1} and D = {–1, 0, 1, 2}.
C ≠ D, because 2 ∈ D, but 2 ∉ C.

Telegram channel https://t.me/johnson201485 3


Equivalence of sets: Two sets A and B are said to be equivalent, written as A↔B (or A∼B), if there is a
one-to-one correspondence between them. Observe that two finite sets A and B are equivalent, if and
only if n (A) = n (B)

1.3. OPERATIONS ON SETS

Union of sets: The union of two sets A and B, denoted by A⋃B and read "A union B" is the set of all
elements that are members of set A or set B or both of the sets. That is, A⋃B = {x|x∈A or x∈B}
Example: a. A = {a, b, c, d, e} and B = {c, d, e, f, g}, then
A⋃B = {a, b, c, d, e, f g}
b. {a, b} ⋃ {c, d, e} = {a, b, c, d, e}
{1, 2, 3, 4, 5} ⋃ ∅ = {1, 2, 3, 4, 5}
Properties of the union of sets
For any sets A, B and C
1. Commutative property A⋃B = B⋃A
2. Associative property (A⋃B) ⋃C = A⋃(B⋃C)
3. Identity property A⋃∅ = A
Intersection of sets : The intersection of two sets A and B, denoted by A ⋂B and read as " A
intersection B ", is the set of all elements common to both set A and set B. common to both set A and set
B. That is, A⋂B = {x|x∈ A and x∈ B}.
Two or more sets are disjoint if they have no common element. A and B are disjoint, if and only if A ⋂ B = ∅.
Using the Venn diagram, In the Venn diagram,
is represented by the blue the sets A and B are disjoint shaded region:

Here A ⋂ B = ∅.
Example: a.

Properties of the intersection of sets : For any sets A, B and C and the universal set U
Commutative Property: A⋂B = B⋂A.
Associative Property: (A⋂B)⋂C = A⋂(B⋂C).
Identity Property: A⋂U = A

Telegram channel https://t.me/johnson201485 4


Difference and symmetric difference of sets
The relative complement (or difference) of two sets
The relative complement of a set B with respect to a set A (or the difference between A and B), denoted
by A - B, read as "A difference B", is the set of all elements in A that are not in B.
That is, A – B = {x|x∈A and x ∉ B}.
Note: A – B is sometimes denoted by A\B. (read as “A less B”)
A – B and A\B are used interchangeably.
Using a Venn diagram, A\B can be represented by shading the region in A which is not part of B
A\B is shaded in light green.
Example

Solution a
The complement of a set
Let A be a subset of a universal set U. The complement (or absolute complement) of A, denoted by A', is
defined to be the set of all elements of U that are not in A. i.e., A' = {x|x∈U and x ∉A}.
Note that for any set A and universal set U, A′ = U\A
Example: In copies of the Venn diagram on the right shade

Telegram channel https://t.me/johnson201485 5


The symmetric difference between two sets: Let A and B be any two sets. The symmetric difference
between A and B, denoted by A∆B, is the set of all elements in A⋃B that are not in A⋂B. That is A∆B =
{x|x∈(A⋃B) and x∉ (A⋂B)} or A∆B = (A⋃B) \(A⋂B).

A∆B = (A⋃B)\ (A⋂B) = (A\B) ⋃ (B\A).


Note: If A ⋂ B = ∅ then A ∆B = A ⋃ B.

Telegram channel https://t.me/johnson201485 6


Example: Let A = {–1, 0, 1} and B = {1, 2}. Find A∆B.
Solution: A⋃B = {–1, 0, 1, 2}; A⋂B = {1 }
∴ A∆B = (A⋃B)\(A⋂B) = {–1, 0, 2}
Example Let A = {a, b, c} and B = {d, e}. Find A∆B.
Solution: A⋃B = {a, b, c, d, e}; A⋂B = ∅
∴ A∆B = (A⋃B) \ ∅ = A⋃B = {a, b, c, d, e}

Equality of ordered pairs: Two ordered pairs (a, b) = (c, d), if and only if a = c and b = d.
Definition: Given two non-empty sets A and B, the set of all ordered pairs (a, b) where a∈A and b∈B is
called the Cartesian product of A and B, denoted by A × B (read "A cross B"). i.e., A × B = {(a, b) |a ∈A
and b∈B}.
Example 1 If A = { 1, 2, 3} and B = { 4, 5}, then
A × B = {(1, 4), (1, 5), (2, 4), (2, 5), (3, 4), (3, 5)}
Example 2 Let A = { a, b}, then form A × A.
Solution: A × A = {(a, a), (a, b), (b, a), (b, b)}.
Example 3 Let A = {–1, 0} and B = {–1, 0, 1}.
Find A × B and illustrate it by means of a diagram.
Solution: A × B = {(–1, –1), (-1, 0), (–1, 1), (0, –1), (0, 0), (0, 1)}
The diagram is as shown in the figure.
Note: n (A × B) = n (A) × n (B).

Problems Involving Sets


Number of elements in (A⋃B) For any finite sets A and B,
the number of elements that are in A⋃B is
n (A⋃B) = n (A) + n (B) – n (A⋂B).
In this figure, suppose the number of elements in the closed regions of the Venn diagram are denoted by

Telegram channel https://t.me/johnson201485 7


x, y, z and w.
n (A) = x + y and n (B) = y + z.
So, n (A) + n (B) = x + y + y + z.
n (A ⋃B) = x + y + z = n (A) +n (B) -y
i.e., n (A⋃B) = n (A) + n (B) – n (A⋂B).
Note: If A⋂B =∅, then n (A⋃B) = n (A) + n (B).
Example 1 Explain why n (A – B) = n (A) – n (A⋂B).
Solution: From Figure above, n (A) = x + y, n (A⋂B) = y
n (A) – n (A⋂B) = (x + y) – y = x, x is the number of elements in A that are not in B.
So, n(A – B)= x.
∴ n (A – B) = x = n (A) – n (A⋂B).
Then For any finite sets A and B n (A\B) = n (A) – n (A⋂B)
Example 2 Among 1500 students in a school, 13 students failed in English, 12 students failed in
mathematics and 7 students failed in both English and Mathematics.
i How many students failed in either English or in Mathematics?
ii How many students passed both in English and in Mathematics?
Solution: Let E be the set of students who failed in English, M be the set of students who failed in
mathematics and U be the set of all students in the school. Then, n (E) = 13, n (M) = 12, n (E⋂M) = 7
and n (U) = 1500.
i n (E⋃M) = n (E) + n (M) – n (E⋂M) = 13 + 12 – 7 = 18.
ii The set of all students who passed in both subjects is U\(E⋃M).
n (U\(E⋃M)) = n (U) – n (E⋃M) = 1500 – 18 = 1482.

SOLVED PROBLEM FURTHER ON SET


1. Which of the following are examples of the null set?
A. Set of odd natural numbers divisible by 2
B. Set of even prime numbers
C. { x : x is a natural numbers, x < 5 and x > 7 }
D. { y : y is a point common to any two parallel lines}
Solution: Let's go through each of the options to identify which ones are examples of the null set (empty set,
denoted by ∅):
A. Set of odd natural numbers divisible by 2:
o Odd natural numbers cannot be divisible by 2 (since odd numbers by definition leave a remainder
when divided by 2).
o This set is empty, so it is a null set.
B. Set of even prime numbers:
o The only even prime number is 2, as 2 is the only number that is both even and prime.

Telegram channel https://t.me/johnson201485 8


o This set is not empty, as it contains the number 2.
C. { x : x is a natural number, x < 5 and x > 7 }:
o There are no natural numbers that are both less than 5 and greater than 7 at the same time.
o This set is empty, so it is a null set.
D. { y : y is a point common to any two parallel lines}:
o Parallel lines never intersect, so there is no point common to them.
o This set is empty, so it is a null set.
Conclusion:
The null sets (empty sets) are:
 A. Set of odd natural numbers divisible by 2
 C. { x : x is a natural number, x < 5 and x > 7 }
 D. { y : y is a point common to any two parallel lines}
2. Which of the following sets are finite or infinite?
A. The set of months of a year.
B. {1, 2, 3, . . .}
C. {1, 2, 3, . . .99, 100}
D. The set of positive integers greater than 100.
E. The set of prime numbers less than 99.
F. The set of lines which are parallel to the x-axis
G. The set of letters in the English alphabet
H. The set of numbers which are multiple of 5
I. The set of animals living on the earth
Solution. Let's analyze each set:
A. The set of months of a year
Finite: There are 12 months in a year, so the set is finite.
B. {1, 2, 3, . . .}
Infinite: This set represents all positive integers (counting numbers), which goes on forever. Therefore, it
is infinite.
C. {1, 2, 3, . . .99, 100}
Finite: This set has a specific number of elements (100 elements), so it is finite.
D. The set of positive integers greater than 100
Infinite: This set includes all positive integers greater than 100, extending indefinitely, so it is infinite.
E. The set of prime numbers less than 99
Finite: There is a finite number of prime numbers less than 99. The set is limited, so it is finite.
F. The set of lines which are parallel to the x-axis
Infinite: There are infinitely many lines parallel to the x-axis, each having a different y-intercept, so this
set is infinite.
G. The set of letters in the English alphabet
Finite: The English alphabet has 26 letters, so this set is finite.
H. The set of numbers which are multiples of 5
Infinite: There are infinitely many multiples of 5, such as 5, 10, 15, etc., so this set is infinite.
I. The set of animals living on the earth
Infinite: While we may not know every animal species, there are billions of animals on Earth, and the
number continues to grow, so this set is effectively infinite.
Summary:
 Finite sets: A, C, E, G
 Infinite sets: B, D, F, H, I
3. In a survey of 600 students in a school, 150 students were found to be taking tea and 225 taking coffee, 100
were taking both tea and coffee. Find how many students were taking neither tea nor coffee?
Solution: To solve this problem, we can use the principle of inclusion-exclusion.

Telegram channel https://t.me/johnson201485 9


Given:
 Total number of students = 600
 Students taking tea (T) = 150
 Students taking coffee (C) = 225
 Students taking both tea and coffee (T∩C) = 100
We are asked to find how many students are taking neither tea nor coffee.
Step 1: Use the inclusion-exclusion principle
The formula for the number of students taking either tea or coffee (or both) is:
∣T∪C∣=∣T∣+∣C∣−∣T∩C∣|
Substituting the given values: ∣T∪C∣=150+225−100=275
So, 275 students are taking either tea, coffee, or both.
Step 2: Find the number of students taking neither tea nor coffee
The number of students taking neither tea nor coffee is the complement of those taking tea or coffee.
Thus, we subtract the number of students taking tea or coffee from the total number of students:
Students taking neither=600−275=325
Final Answer:
325 students are taking neither tea nor coffee.
4. Of 100 students, 65 are members of mathematics club and 40 are members of a physics club. If 10 are
members of neither club, then how many students are members of a. both club b. only the mathematics club c.
only the physics club
Let's break this down step-by-step.
Given:
 Total students = 100
 Students in the mathematics club (M) = 65
 Students in the physics club (P) = 40
 Students in neither club = 10
Step 1: Calculate the number of students who are members of at least one club
Since 10 students are members of neither club, the number of students who are members of at least one club is:
Students in at least one club=100−10=90\text{Students in at least one club} = 100 - 10 =
90Students in at least one club=100−10=90
Step 2: Use the principle of inclusion-exclusion
The principle of inclusion-exclusion gives the number of students who are members of either the mathematics or
the physics club (or both): ∣M∪P∣=∣M∣+∣P∣−∣M∩P∣
We know that ∣M∪P∣=90 (students in at least one club), so: 90=65+40−∣M∩P∣
Solving for ∣M∩P∣ (the number of students in both clubs): 90=105−∣M∩P ∣
∣M∩P∣=105−90=15
So, 15 students are members of both clubs.
Step 3: Calculate the number of students in only one club
 Only the mathematics club: The number of students who are members of only the mathematics club is:
∣M∖P∣=∣M∣−∣M∩P∣=65−15=50
 Only the physics club: The number of students who are members of only the physics club is:
∣P∖M∣=∣P∣−∣M∩P∣=40−15=25
Final Answers:
 a) Students in both clubs: 15
 b) Students only in the mathematics club: 50
 c) Students only in the physics club: 25

Telegram channel https://t.me/johnson201485 10


UNIT TWO: GEOMETRY AND MEASUREMENT
2.1. REGULAR POLYGONS

Definition : A polygon is a simple closed curve, formed by the union of three or more-line segments, no
two of which in succession are collinear. The line segments are called the sides of the polygon and the
end points of the sides are called the vertices
Types of polygons: Based on the measure of its interior angle of the polygon can be classified as
 Convex polygon: a polygon whose all measure of its interior angle is less than
 Concave polygon: a polygon at least one of the measures of its interior angle is greater than
Note:
 Polygons can be classified according to the number of sides.
 The number of vertices, angles and sides of a polygon are the same.
No of side No of interior angles Name of polygon
3 3 Triangle
4 4 Quadrilateral
5 5 Pentagon
6 6 Hexagon
7 7 Heptagon
8 8 Octagon
9 9 nonagon
10 10 Decagon
ANGLES OF POLYGONS
There are two types of angles of polygon: these are
i. Interior angle: is an angle of a polygon formed at the vertex on the inside of the polygon.
ii. Exterior angle: is an angle at a vertex of a polygon that is supplementary to the interior angle at
the vertex. It is formed between one side of the polygon and the extended adjacent side.
Example: In the polygon ABCD ∠DCB is an interior angle; ∠BCE and
∠DCF are exterior angles of the polygon at the vertex C
The sum of the measures of the interior angles of a polygon
Theorem 2:1 Angle sum theorem
The sum of the measures of the three interior angles of any triangle is
Note: The polygons are divided into triangles by drawing a diagonal from one vertex
Example: consider each of the convex polygons with all possible diagonal from one vertex.
No of sides No of triangles formed Sum of interior
of polygon from one vertex angles of polygon
3 1 1x1800  1800
4 2 2x1800  3600
5 3 3x1800  5400

Telegram channel https://t.me/johnson201485 11


Generally: a polygon of n side has
i. (n-3) diagonal from one vertex. ii) (n-2) triangles
Theorem 2: if the number of side of polygon is n, then the sum of the measure of all of its interior angle
is equal to ( ) .
Exercise
1. If the measures of the interior angles of a hexagon are ( ) ( ) and
( ) , find the value of x
2. Show that the measure of an exterior angle of a triangle is equal to the sum of the measures of the two
opposite interior angles.
Measures of Angles of a Regular Polygon
In each triangle, the degree measure of the central angle O is given by:

( )

Since the vertex angles at O of each isosceles triangle have equal measures, Namely , it follows that
all the base angles of all the isosceles triangles are also equal.

From this, it follows that the measures of all the angles of the polygon are equal; the measure of an angle
of the polygon is twice the measure of any base angle of any one of the isosceles triangles. So, the
polygon has all of its sides equal and all of its angles equal. A polygon of this type is called a regular
polygon.
Definition 2. 3: A regular polygon is a convex polygon in which the lengths of all of its sides are equal
and the measures of all of its angles are equal.
 Note that the measure of an interior angle of an n-sided regular polygon is , where
( – ) is the sum of the measures of all of its interior angles.
( )
 Then the measure of each interior angle of a regular n-sided polygon is
 A polygon is said to be inscribed in a circle if all of its vertices lie on the circle.
Example: Find the measure of an interior and exterior angle of a regular polygon with:
a. 10 sides b. 20 sides c. 12 sides
Solution: Then the measure of each interior angle of a polygon is given as
(n  2)1800 8(1800 )
a. The measure of an interior angle of a 10-sided regular polygon is   1440
n 10
(n  2)1800 18(1800 )
b. The measure of an interior angle of a 20-sided regular polygon is   1620
n 20
(n  2)1800 10(1800 )
c. The measure of an interior angle of a 10-sided regular polygon is   1500
n 12
And the measure of each exterior angle of a n side of regular polygon is given as
a. The measure of each exterior of 10 side is

Telegram channel https://t.me/johnson201485 12


b. The measure of each exterior of 10 side is
c. The measure of each exterior of 10 side is
Generally: For any regular n-sided polygon:
( ) iv. Measure of the sum of interior angle =(
i. Measure of each interior angle =
)
ii. Measure of each central angle =
iii. Measure of each exterior angle =
v. The sum of the measure of exterior angle of any polygon, taking one angle at each vertex is .
2.1.2. Properties of Regular Polygons
A symmetric properties of regular polygon : A plane figure can be divided exactly into two identical
parts by line ( )the line is called line of symmetry
 A figure is called symmetric figure if it has at least one line of symmetry
 Regular polygons are symmetric.
EXAMPLE 1

Note
 A regular n-side polygons has n-line of symmetry
 A regular polygon of odd number of sides has every line of symmetry, passes through the vertex.
Circumscribed (inscribed) regular polygons
 A polygon whose sides are tangent to a circle is said to circumscribe the circle.
 An inscribed polygon is a polygon all of whose sides are a chords of a circle.
 The circle is called circumscribed about the polygon if all sides polygons are chords of a circle.
Inscribe polygon is the same term as circumscribed circle.
EXAMPLE 2

Inscribed circle inscribed polygon or circumscribed


circle
Definition 2.4: The distance a from the center of a regular polygon to a side of
the polygon is called the apothem of the polygon. That is, the apothem a
of a regular polygon is the length of the line segment drawn from the
center of the polygon perpendicular to the side of the polygon.
Example 1 In the regular pentagon ABCDE is inscribed in a circle with center O and radius r.
Write formulae for the side s, perimeter P, apothem a and area A of the regular pentagon.
Solution: ∆OAB is an isosceles triangle. Draw the perpendicular from O to AB .
It meets AB at X. ∠AOB is a central angle of the regular pentagon.

Telegram channel https://t.me/johnson201485 13


3600
So M ( AOB)   72 0 NOW  AOX  BOX
5
Therefore AOX  BOX , Then M (AOX )  M (BOX )  1 M (AOB)  1  360   36 0.
0

 2 2 5 

Let s  AB , the length of side of regular pentagon.


Since  AOX  BOX , we have AX  XB AX  1 AB  1 s .
2 2
⁄ ⁄
(∠ ) ( (∠ )) THEN ⁄
Now in the right-angled triangle AOX you see that : Therefore, s  2r sin 360 . . . . . . . . .. . . . . . . (1)
Perimeter P of the polygon is P = AB + BC + CD + DE + EA
But since AB = BC = CD = DE = EA = s, we have P = s + s + s + s + s = 5s. Since from (1) we have s =
2r sin 36o, the perimeter of the regular pentagon is P = 5 × 2r sin 360
∴ P = 10r sin 360. . . . . . . . . . . . . . . . . . . . . . . . . . . (2)
To find a formula for the apothem, a, consider ∆AOX
( ) , Since (∠ ) and AO  r .
a
Then cos 36 0  so a  r cos 360
r
To find the area of the regular pentagon, first we find the area of ∆AOB. Taking the height and the base
of ∆AOB as OX and AB, respectively.
1
Since all these triangles are congruent, the area of each triangle is as
2
So, the area of the regular pentagon ABC . / ( )

180 0
Since 36 0  , where 5 is the number of their sides, we can generalize the above formulae for any n-
5
1800
sided regular polygon by replacing 360 by , as follows
n
Theorem 5.3
Formulae for the length of side s, apothem a, perimeter P and area A of a regular polygon
with n sides and radius r are
1. s rsin 3. 𝑎 = r cos
n n
2. P = nrsin 4. A= 𝑎P
n

Example 2
a. Find the radius of an equilateral triangle with perimeter 24 units.
b. Find the radius of a regular hexagon with perimeter 48 units.
c. Find the area of a regular hexagon whose radius is 5 cm.
d. Show that the length of each side of a regular hexagon is equal to the length of the radius of the
hexagon.

Telegram channel https://t.me/johnson201485 12


180 0
p  2nr sin 180 0
n p  2nr sin
n
180 0
24  2 x3 xr sin 180 0
Solution: a. 3 b. 48  2 x6 xr sin
6
24  6r sin 60 0

48  12r sin 30 0

3 8 3
4 r r units 48  6r  r  8 units
2 3
C. To find the area of the regular hexagon, we use the formula
A= P , where is the apothem and P is the perimeter of the regular hexagon.

Therefore, A= P . / . nrsin / ( )( x x sin ) units.
n n
D. We know that the length of a side s of an n-sided regular polygon is given by
180 0
s  2r sin Where r is the radius of the polygon
n
180 0 1
. If n = 6 then, s  2r sin  2r sin 30 0  2rx  r  s  r
6 2
Therefore, the length of a side s of a regular hexagon is equal to the radius r of the hexagon.

2.2. FURTHER ON CONGRUENCY AND SIMILARITY


A. CONGRUENCY
 Two geometric figures are said to be congruent if they have the same shape and the same size.
 Two line segments AB and CD are congruent if AB=CD
 Two angles are congruent if and only if their measures are equal.
CONGRUENCY OF TRIANGLE.
 Congruent triangles are triangles that have the same size and the same shape.
 Triangles that fit exactly are called congruent triangle.
 If two triangles are congruent; their corresponding sides and angles must be equal
A D

B C E F

The six congruent part of the triangle are the three corresponding angles and the three corresponding size
of and denoted by
These are

Telegram channel https://t.me/johnson201485 13


AB  DE  A  D 
 
BC  EF  Corresponding sides B  E  Corresponding angles
AC  DF  C  F 

Four Basic Principles of Congruent Triangle
1. SSS (Three sides) 3. SAS (Two sides and included angle)
2. ASA (Two angles and included side) 4. RHS (Right angle hypotenuse and sides)
Generally: Two triangles are congruent if the following corresponding parts of the triangles are
congruent.

Example:1
1. Given that if ̅̅̅̅ ̅̅̅̅ , then prove ̅̅̅̅ ̅̅̅̅
B
Explanation: Two columns proof
Statements Reasons
D ̅̅̅̅ ≡ 𝐵𝐶
1. 𝐴𝐵 ̅̅̅̅
E
2.
3. ̅̅̅̅
𝐴𝐵 ≡ 𝐵𝐶̅̅̅̅
A C 4.
5.
6. Therefore 𝐴𝐶𝐷 ≡ 𝐶𝐸𝐹 by ASA
∴ ̅̅̅̅
𝐴𝐷 ≡ ̅̅̅̅
𝐶𝐸
B. SIMILAR FIGURES
Figures that have the same shape but that might have different sizes are called similar.
In similar figures:
i. One is an enlargement of the other.
ii. Angles in corresponding positions are congruent.
iii. Corresponding sides have the same ratio.

Telegram channel https://t.me/johnson201485 14


Similar polygons: Two polygons of the same number of sides are similar, if their corresponding angles
are congruent and their corresponding sides have the same ratio. U
6 T
F 3 E 8
Example: 4 4 8

A D P S
2 2
B C 4 4
3
Q
6
The above hexagons are similar, which is denoted by provided that
i. A  P, B  Q, C  R, D  S , E  T and F  U
AB BC CD DE EF FA 2 1
ii.       
PQ QR RS ST TU UP 4 2

NOTE:
i. The ratio of corresponding sides of two similar polygons is called the scale factor.

ii. The scale factors of to is 1 2

iii. The scale factors of to is 2


Theorems on Similarity of Triangles
How do you know whether two triangles are similar?
There are also tests to determine whether two triangles are similar
These are:
i. AA-Similarity (Angle-Angle similarity)
ii. SSS-Similarity (Side- Side- Side similarity)
iii. SAS-Similarity (side-angle-side similarity)
A. AA-Similarity (Angle-Angle similarity): If two angles of one triangle are congruent to two
corresponding angles of another triangle, then the two triangles are similar. i.e. are
similar by AA-similarity from A with D, B with E and C with F if any two corresponding angles are
equal.
B. SSS-Similarity (Side- Side- Side similarity): Two triangles are similar, if two pairs of corresponding
sides of the two triangles are proportional and if the included angles between these sides are congruent.
C. SAS-Similarity (side-angle-side similarity): Two triangles are similar, if two pairs of corresponding
sides of the two triangles are proportional and if the included angles between these sides are congruent.

Telegram channel https://t.me/johnson201485 15


Theorems on Similar Plane Figures
Ratio of Perimeters and Ratio of Areas of Similar Plane Figures
Here we are going to see the way how to relate the side length, perimeter and areas of similar triangles
(polygons).
Let P1= Perimeter of the first triangle or polygon
P2= Perimeter of the second triangle or polygon
A1= Area of the first triangle or polygon
A2= Area of the second triangle or polygon
Theorem: If the ratio of the lengths of any two corresponding sides of two similar triangles or
polygons are , then

i. The ratio of their perimeters is given by

ii. The ratio of their Area is given by

NOTE: Area ratio= (Side length ratio)2 = (Perimeter ratio)2


Example: 2
A. The areas of two similar triangles are 144 unit2 and 81 unit2.
i. What is the ratio of their perimeters?
ii. If a side of the first is 6 units long, what is the length of the corresponding side of the second?
B. The sides of a polygon have lengths 5, 7, 8, 11, and 19 units. The perimeter of a similar polygon is 75
units. Find the lengths of the sides of the larger polygon.
Solution:
A1 144 12 4 3
A. i. The ratio of their perimeters is =    OR
A2 81 9 3 4

ii Let x unit be the corresponding side of the second triangle. Then,


6 4 (3)(6) 9
 x   4.5 units
x 3 4 2
B. Let the lengths of the corresponding sides of the other polygon be a, b, c, d and e.

Therefore, the lengths of the sides of the larger polygon are 7.5, 10.5, 12, 16.5, and 28.5 units.
Construction of Similar Figures
Enlargement: An enlargement with center O and scale factor k (where k is a real number) is the
transformation that maps each point P to point P′ such that

Telegram channel https://t.me/johnson201485 16


i. P′ is on the ray O P and ii. O P′ = k OP
If an object is enlarged, the result is an image that is mathematically similar to the object but of different
size. The image can be either larger, if k > 1, or smaller if 0 < k <1.
Note: If the scale factor of enlargement is greater than 1, then the image is larger than the object. If the
scale factor lies between 0 and 1 then the resulting image is smaller than the object. In these latter cases,
although the image is smaller than the object, the transformation is still known as an enlargement.
Eample: A tree casts a shadow of 30 m. At the same time, a 10 m pole casts a shadow of 12 m. Find the
height of the tree.
Solution:: Assuming both the pole and the tree are vertical to the ground, we have equality of their ratios
h 30
as  . Therefore, the height of the tree is 25m.
10 12

2.3. FURTHER ON TRIGONOMETRY

Angle: An angle is the union of two rays with a common end point.
In general, we associate each angle with a real number called the measure of the
angle. The two measures that are most frequently used are degree and radian.
i. Measuring angles in degrees
Definition: A degree, denoted by (0), is defined as the measure of the central angle subtended by an arc
1
of a circle equal in length to of the circumference of the circle.
360
Note:
1
 A minute which is denoted by ( ' ), is of a degree.
60
1
 A second which is denoted by ( '' ), is of a minute, From these we have 1o=3600‟‟ relation.
60
ii. Measuring angles in radians A radian (rad) is defined as the measure of the
central angle subtended by an arc of a circle equal in length to the radius of the
circle.You know that the circumference of a circle is equal to 2πr.
Since an arc of length r along the circle gives 1 rad, a complete rotation of length 2πr generates an angle
of 2π radians. On the other hand, we know that a complete revolution represents an angle of 3600. This
gives us the following relationship:
1 revolution = 3600 = 2π radians

Telegram channel https://t.me/johnson201485 17


i.e., 1800 = π radians, from which we obtain,
Therefore, we have the following conversion rules for degrees and radians.
1800
 To convert radians to degrees, multiply by


 To convert degrees to radians multiply by
1800
EXERCISE
a. Express each of the following radian measures in degrees:
i. π/6 ii. π iii. π /3 iv. 2 v. 3/4π vi. 5
b. Express each of the following in radian measure:
i. 2700 ii. 1500 iii. 2250 iv. 150

Trigonometrical Ratios to Solve Right-Ratios to Solve Right

Using similarity ratio of sides of triangles it is possible to drive the trigonometrical ratio on the triangles.
From the similarity ratio

The fractions or ratios in each of these proportions are called trigonometric ratio

The above discussion can be summarized and expressed as follows.

The angles 00, 300, 450, 600 and 900 are called special angles, because they have these exact
trigonometric ratios, its values are given as follows.

Telegram channel https://t.me/johnson201485 18


Example 1 A ladder 6 m long leans against a wall and makes an angle of 600 with the ground. Find the
height of the wall. How far from the wall is the foot of the ladder?
height of wall 3
Solution sin 60 0   height of wall  x6  3 3m
length of ladder 2
the foot of the ladder from wall 1
cos 60 0   the foot of the ladder from wall  x6  3m
length of ladder 2
Trigonometrical Values of Angles from Table (Sinθ, cosθ and tanθ, for 00 ≤ θ < 1800)
NOTE: If you know the value of one of the trigonometric ratios of an angle, you can use a table of
trigonometric ratios to find the angle. The procedure is illustrated in the following example.
Example : Find the measure of the acute angle A, correct to the nearest degree, if sin (A)0 = 0.521.
Solution: Referring to the "sine" column of the table, we find that 0.521 does not appear there. The two
values in the table closest to 0.521 (one smaller and one larger) are 0.515 and 0.530. These values
correspond to 310 and 320, respectively
Note that 0.521 is closer to 0.515, whose value corresponds to 310.
Therefore, m(∠A)= 310. (to the nearest degree)
 Using a right angle triangle you may have discovered the following
A. If m(∠A) + m(∠B) = 900, i.e., A and B are complementary angles, then
i. sin (∠A) = cos(∠B) B
ii. cos(∠A) = sin (∠B)
sin(  A)
B. tan(  A) =
cos( A) A C
C. sin (∠A) + cos2(∠A) = 1
2

How can you use the trigonometric table to find the sine, cosine and tangent of obtuse angles?
To find trigonometric value of an obtuse angle we see as follows. y
Let  Be an obtuse angle and
 Be supplementary angle of 
Then  and  are equal in magnitude different direction
Because OP‟ is obtained by reflecting OP along y-axis. x
    1800    1800   , becuase  ,  sup lemantary angle B 0 C
Then
   180 0      1800   becuase   

19

Telegram channel https://t.me/johnson201485


Therefore, using the trigonometric definition of each trigonometric function as follows
1. To find the sine of  as expressed on the coordinate P(a, b) , to relate with acute angle is shown as:
b b
sin   And sin    Is an acute angle in the 1st quadrant, which is
a b 2 2
a b 2 2

supplementary to obtuse angle 


b
Then from these two relations sin    sin   sin   sin(1800   )
a b
2 2

2. To find cosine of  as expressed on the coordinate P(a, b) , to relate with acute angle is shown as:
a a
cos   And cos   Is an acute angle in the 1st quadrant, which is
a b 2 2
a b 2 2

supplementary to obtuse angle 


a
Then from these two relations cos     cos  cos    cos(1800   )
a b 2 2

3. Similarly, to find tangent of  whose coordinate is P(a, b) , relating with acute angle is shown as:
a a a
tan   And tan  , which implies tan     tan  tan    tan(1800   )
b b b
Generally: For an obtuse angle  we have

2.4. CIRCLES
Symmetrical Properties of Circles
Circle is defined as the set of points in a given plane, each of which is at the same distance from a fixed
point of the plane. The fixed point is called the center and the distance is called radius of the circle

r
O
Note
A line segment through the center of a circle with end points on the circle is called a diameter
A chord of a circle is a line segment whose end points lie on the circle.
A line that divided the figure and one part of the figure is coinciding with the other part divided by the
line, then the line is called symmetric line and the figure is called symmetric figure.

20

Telegram channel https://t.me/johnson201485


Basic theorem of line on circle
-The line segment joining the center of a circle to the mid-point of a chord is perpendicular to the chord.
-The line segment drawn from the center of a circle perpendicular to a chord bisects the chord.
-If two chords of a circle are equal, then they are equidistant from the center.
-If two chords of a circle are equidistant from the center, then their lengths are equal.
-If two tangent segments are drawn to a circle from an external point, then,The tangents are equal in
length, and The line segment joining the center to the external point bisects the angle between the
tangents. Y C
M
O
Angle Properties of Circles
B
Basic terminologies of circle A X
Arc: A part of a circle (part of its circumference) between any two points on the circle, say between A
and B, is called an arc and is denoted
by ̂ or ̂
-Semicircle: Is an arc which is half of the circle is called semicircle, which is AC is a diameter. Then
̂ is semicircle of circle O.
-Minor and major arc: An arc is said to be a minor arc, if it is less than a semicircle and a major arc,
if it is greater than a semicircle. For example: ̂ is minor arc whilst ̂ is a major arc.
-Central angle: A central angle of a circle is an angle whose vertex is at the center of the circle and
whose sides are radii of the circle. For example: The angle ∠AOY, ∠COY are an example of central
angle.
-An inscribed angle in a circle is an angle whose vertex is on the circle and whose sides are chords of the
circle. And an arc subtend the inscribed angle is called subtended arc. For example: ∠CAB is an
inscribed angle and ̂ is subtended arc.
Basic theorem of angles on circle
-The measure of a central angle subtended by an arc is twice the measure of an inscribed angle in the
circle subtended by the same arc.
-Angles inscribed in the same arc of a circle (i.e., subtended by the same arc) are equal.
Q
-The angle inscribed in a semi-circle is a right angle.
-Points P, Q, R and S all lie on circle. They are called concyclic points. P R
Joining the point P. Q, R and S produces a cyclic quadrilateral.
S

21

Telegram channel https://t.me/johnson201485


The opposite angles of a cyclic quadrilateral are supplementary. i.e.
m(∠P) + m(∠R) = 180o and m(∠S) + m(∠Q) = 1800.
Arc Lengths, Perimeters and Areas of Segments and Sectors
Remember that: For r is a radius, d is a diameter of a circle O.
-Circumference of a circle = 2πr or πd.
-Area of a circle = πr2
-Part of the circumference of a circle is called an arc.
-A segment of a circle is a region bounded by a chord and an arc.
-A sector of a circle is bounded by two radii and an arc.
Arc length
The length ℓ of an arc of a circle of radius r that subtends an angle of θ at the center is given by
 r
 0
x2r 
360 1800
The area and perimeter of a sector
 r 2
The area A of a sector of radius r and central angle θ is given by A  xr 2 
360 0 3600
The perimeter P of the sector is the sum of the radii and the arc that bound it.
r
P  2r 
1800
The area and perimeter of a segment
The area A and perimeter P of a segment of a circle of radius r, cut off by
a chord subtending an angle θ at the center of a circle are given by
r 21
A  r 2 sin 
0
(sector area - triangle area)
360 2
Perimeter of segments bounded between a chord and arc is given as
 r
P  2r sin  (chord length + arc length )
2 1800
1
Note: The area formula for a triangle: A  ab sin  where a and b are the lengths of any two sides of
2
the triangle and θ is the measure of the angle included between the given sides
Example:

22

Telegram channel https://t.me/johnson201485


A. A square ABCD is inscribed in a circle of radius 4 cm. Find the area of the minor segment cut off
B
by the chord AB X

Solution: A minor segment is segment lie between chord ̅̅̅̅ and arc ̂ . A O C

(s t r r - tri n r ) In this case r=4cm and   900


D
( )

A. In Figure below, O is the center of the circle. If the


radius of the circle is
K
4cm and m (∠AKB) = 300, find the area of the segment bounded
by the chord AB and arc AKB. .O

Solution: first we find the measure of central angle m (∠AOB) =2m (∠AKB) =2(300) =600 A B
Then area of the major segment is=area of circle-area of minor arc
 r 2 1 2   8   40 
A  r 2    r sin    16    4 3    4 3 cm 2
 3   3 
0
 360 2 

2.5. MEASUREMENTS

Areas of Triangles and Parallelograms


A. Areas of triangles: There are different ways used to find the area of the triangle.
We Uses the following fact.
Case i. The area A of a right angle triangle with perpendicular sides of length a and b is given by
1
A ab
2

Case ii. The area A of any triangle with base b and the corresponding height h is given by A  1 bh
2
Case iii. The area A of any triangle with sides a and b units long and angle C (∠C) included between
1
these sides is A  ab sin( C )
2
NOTE: Heron‟s formula
The area A of a triangle with sides a , b and c units long and semi- perimeter

s
1
a  b  c  is given by A  s(s  a)(s  b)(s  c)
2

23

Telegram channel https://t.me/johnson201485


B. Area of parallelograms
-The area A of a parallelogram with base b and perpendicular height h is A  bh
1
-Area of a rhombus whose diagonals d1 and d2 is given as A  d1d 2 (how)?
2
Example:
i. The lengths of three sides of a triangle are 6 x, 4 x and 3x
inches and the perimeter of the triangle is 26 inches. Find the lengths of the sides of the triangle and the area of
the triangle
ii. Find the area of a rhombus whose diagonals are 5 inches and
6 inches long.
Solution:
i. Perimeter of triangle is equal to

Then from this the length of the sides are 12, 8 and 6 inches respectively.
ii. To find it‟s area we use heron‟s formula

A  s(s  a)(s  b)(s  c) For s 


1
a  b  c   13 then
2
A  13(13  12)(13  8)(13  6)  455 Square inches.
Further on Surface Areas and Volumes of Cylinders and Prisms
A. Prism: A prism is a solid figure formed by two congruent polygonal regions in parallel planes, along
with three or more parallelograms, joining the two polygons. The polygons in parallel planes are called
bases.
 A prism is named by its base. Thus, a prism is called triangular, rectangular, pentagonal, etc., if its
base is a triangle, a rectangle, a pentagon, etc., respectively.
In a prism,
- The lateral edges are equal and parallel.
- The lateral faces are parallelograms.
 A right prism is a prism in which the base is perpendicular to a lateral edge. Otherwise it is an
oblique prism.
In a right prism
- All the lateral edges are perpendicular to both bases.

24

Telegram channel https://t.me/johnson201485


- The lateral faces are rectangles.
- The altitude is equal to the length of each lateral edge.
- A regular prism is a right prism whose base is a regular polygon.
Surface area and volume of prisms
The lateral surface area of a prism is the sum of the areas of its lateral faces.
The total surface area of a prism is the sum of the lateral areas and the area of the bases.
The volume of any prism is equal to the product of its base area and its altitude.
If we denote the lateral surface area of a prism by AL, the total surface area by AT, the area of the base by
AB and its volume by V, then
 AL = Ph where P is the perimeter of the base and h the altitude or height of the prism.
 AT = 2AB + AL
 V = ABh.
B. Cylinder
A circular cylinder is a simple closed surface bounded on two ends by circular bases.
Surface area and volume of circular cylinders
1. The lateral surface area (i.e., area of the curved surface) of a right h
Circular cylinder denoted by AL is the product of its height h and r

The circumference C of its base. i.e. AL = hC OR AL = 2πrh


2. The total surface area (or simply surface area) of a right circular cylinder
Denoted by AT is two times the area of the circular base plus the area of the curved surface (lateral
surface area). So, if the height of the cylinder is h and the radius of the base circle is r, we have
AT = 2πrh + 2πr2 = 2πr (h + r)
3. The volume V of the right circular cylinder is equal to the product of its base area and height. So, if the
height of the cylinder is h and its base radius is r then V = πr2h

SOLVED PROBLEMS ON GEOMETRY AND MEASUREMENTS


1. The base of a right prism is an isosceles triangle with equal sides 5 inches each, and third side 4
inches. The altitude of the prism is 6 inches. Find the total surface area of the prism and the volume
of the prism.
2. Calculate the volume and total surface area of a right circular cylinder of height 1 m and radius 70 cm.
3. An agriculture field is rectangular, with dimensions 100 m by 42 m. A 20 m deep well of diameter 14
m is dug in a corner of the field and the earth taken out is spread evenly over the remaining part of the
field. Find the increase in the level of the field.

25

Telegram channel https://t.me/johnson201485


4. A glass cylinder with a radius of 7 cm has water up to a height of 9 cm. A metal cube of 5 1 2 cm edge is

immersed in it completely. Calculate the height by which the water rises in the cylinder.
Solution:
1. . Perimeter of the isosceles triangle = 5 + 5 + 4 = 14 inches and its semi-perimeter is=14/2=7

Area of the isosceles triangle = 7(7  5)(7  5)(7  4)  2 21 sq inches

 
AT = 2AB + AL=14x6+2( 2 21 )= 84  4 21 sq inches

V = ABh= 2 21x6  12 21 cubicinches .


2. To calculate the volume and total surface area of a right circular cylinder, we use the following formulas:
Volume of a Cylinder:

 r is the radius of the base of the cylinder


 h is the height of the cylinder
Total Surface Area of a Cylinder:

Where:

Given:
 Height h=1 m=100 cm (convert height to centimeters for consistency in units).
 Radius r=70 cm
Step 1: Volume Calculation

Substitute the values into the formula for volume:


( ) ( )
Step 2: Surface Area Calculation

Substitute the values into the formula for surface area:


( ) ( )( ) ( )

26

Telegram channel https://t.me/johnson201485


UNIT THREE POLYNOMIAL FUNCTIONS
Unit Outcomes: After completing this unit, you should be able to:
 Define polynomial functions.
 Perform the four fundamental operations on polynomials.
 Apply theorems on polynomials to solve related problems.
 Determine the number of rational and irrational zeros of a polynomial.
 Sketch and analyses the graphs of polynomial functions.
Main Contents of the Unit
3.1. Introduction to polynomial functions
3.2. Theorems on polynomials
3.3. Zeros of polynomial functions
3.4. Graphs of polynomial functions

3.1. INTRODUCTION TO POLYNOMIAL FUNCTIONS


At the end of this sub-unit, students will be able to:
 Define the polynomial function of one variable.
 Identify the degree, leading coefficient and constant terms of a given polynomial function.
 Give different forms of polynomial functions.
 Perform the four fundamental operations on polynomials.
Basic Terminologies of Polynomials
 An Algebraic Expression: - is a combination of ordinary numbers letters which represent numbers.
C). x  y
2 2
B). 5xy  2 x  z
2
Example1. A). 3xy
2x  y
 Term: - consists of products and quotients of ordinary numbers and letters which represents numbers.
Example2.
2
A). 3x yz …. It is one term, i.e., there is no addition or subtraction sign between them
B). 4 x y  2ac … Is an algebraic expression consisting of two terms, i.e. there is a
2

Subtraction sign between them


2
 Monomial: is an algebraic expression consisting of only one term. Example 3x yz And the degree of
2
monomial is the sum of the exponents of its variable. The degree of 3x yz is 4
 Binomial: is an algebraic expression consisting of two terms.
 Trinomial: is an algebraic expression consisting of three terms.
 Numerical Coefficient: - is a numerical factor of the term.
Definition of a Polynomial Function: -
Question: Define function, constant function, linear function, quadratic function and their domain and
range
Polynomial: is a monomial or the sum of terms that are monomials. In this chapter, you will study
polynomials in one variable.

27

Telegram channel https://t.me/johnson201485


Generally: Let n be a non-negative integer and let an , an 1,...,a1, a0 be real numbers with an  0 . The
n 1
function P( x)  a n x  a n 1 x  ...  a1 x  a 0 is called a polynomial function in variable x of degree n .
n

NOTE: i. an , an 1,...,a1, a0 are called the coefficients of polynomial function.


n
ii. an Is leading coefficient of the polynomial and an x is the leading term.
iii. a0 Is the constant term.
iv. The number n (the exponent of the highest power of x ), is the degree of the polynomial.
Note that the domain of a polynomial function is .
 A constant number is a polynomial function of degree zero (0). Example 2, 3-
How? What the leading term and leading coefficient?
 Zero (0) is a polynomial function that has no degree. Why? What the leading term and leading
coefficient?
n 1
 A polynomial expression in x is an expression of the form a n x  a n 1 x  ...  a1 x  a 0 , where n be
n

a non-negative integer with an  0

Example3: a). b). And etc.


 Using the terminology for polynomials. Linear functions are polynomial functions of degree 1.
Nonzero constant functions are polynomial functions of degree 0. Similarly, quadratic functions are
polynomial functions of degree 2.
Example4: which of the following are polynomial functions? For a polynomial function find degree,
leading Coefficient and constant term?
a) ( ) e) ( )
( )( )
h) ( )
( )
b) ( )
f) ( ) √( )
c) ( )
g) ( )
d) ( ) √
Answer :
Functions Degree Leading coefficient Constant term
( ) 4 2 2
0 - 1
( )
( ) - 0 0
( ) √ 0 √ √
( )( ) - - -
( )
( )
( ) √( ) 2 1 1
- - -
( )

28

Telegram channel https://t.me/johnson201485


- - -
( )
NOTE: Polynomial can be classified based on different ways. Some of them are based on their coefficients
and their degrees
Based on degree Based on their coefficients
Degree Name Example for both Polynomial If all of its coefficient is….
Constant 3
Linear  Over the set of If all of its coefficient is the
integer set of an integer.
Quadratic  Over the set of If it‟s entire coefficient is the
rational set of a rational.
Cubic  Over the set of If it‟s entire coefficient is the
real set of a real.
Quartic
Quintic
Operations on Polynomial Functions.
a. Addition And Subtraction of Polynomials:Polynomials are added or subtracted by combining of like
terms. Like terms contains the same variable factors and exponent.
Example5. A) and are like term B). and are not like term
 The degree of the sum or difference of two polynomial is less than or equal to the degree of polynomial with
higher degree.
 The sum of two polynomial functions f and g is written as , and is defined as:
( )( ) ( ) ( ), for all ∈ .
 The difference of two polynomial functions f and g is written as – , and is defined as:
( ) ( – )( ) ( ) ( ), for all ∈ .
Example6: for each of the following polynomial function ( ) and ( ), find ( )( ) and degree of
( ), ( ) , and degree of ( )( )
A). ( ) and ( )
B). ( ) and ( ) –
Answer
A. ( ) ( ) ( ) ( )
( ) ( ) ( ) ( )

Function

Degree 3 3 2 3

B. ( )( ) ( ) ( – )

=

29
Telegram channel https://t.me/johnson201485
( )( ) ( ) ( – )

=
Function

Degree 4 4 4 3
Note that if the degree of f is not equal to the degree of g, then the degree of ( ) ( ) and ( – ) ( ) is
the degree of ( ) or the degree of ( ), whichever has the highest degree. If they have the same degree,
however, the degree of ( ) ( ) and ( – ) ( ) might be lower than this common degree when they
have the same leading coefficient.

b. Multiplication Of Polynomials: The degree of the product of two non-zero polynomials is the sum of
the degree of the two polynomials in the product. That is let the degree of ( ) is and the degree of
( ) is , then the degree of ( )( ) is .
Note:The product of two polynomials and is found by multiplying each term of ( ) every term of ( )
and finally add their results.
The product of two polynomial functions and is written as , and is defined as:
( )( ) ( ) ( ), for all ∈ .
Example7: For each of the following find ( ) ( ), degree, leading coefficient , leading term ,
constant term and of , and
A. ( ) ; ( )
B. ( ) ( ) – –
Answer:
A. ( ) ( ) ( )( ) ( ) ( ) ( )
=( ) ( ) ( )
=
Function Degree leading leading constant
coefficient term term
2 1 5 -3
1 5 3 3
2+1=3 1*5=5 5*3=15 -12
B. ( ) ( ) ( )( – – ) ( – – ) ( – – )
=( ) ( )
Function Degree leading leading constant
coefficient term term
2 1 0 2
3 2 0 0
2+3=5 1*2=2 0 4

30
Telegram channel https://t.me/johnson201485
Division of Polynomials.: The quotient (division) of two polynomial functions f and g is written as ,
and is defined as: ( )( ) ( ) ( ) Provided that ( ) , for all ∈ .
Example: For each of the following, find the quotient and the remainder:
A. (x – ) (x – ) B. ( x –x x ) (x )
Answer a.
Quotient Remainder
A x x 0
B x x 0
From the above example the remainder is zero. Which means that the divisor is divide the dividend into
quotient.

3.2. THEOREMS ON POLYNOMIALS

At the end of this sub-unit, students will be able to:


 State and prove the Polynomial Division Theorem.
 Apply the Polynomial Division Theorem.
 State and prove the Remainder Theorem.
 Apply the Remainder Theorem.
 Apply the Factor Theorem.
A. Polynomial Division Theorem
If ( ) and ( ) are polynomials such that ( ) , and the degree of ( ) is less than or equal to the
degree of ( ), then there exist unique polynomials ( ) and ( ) such that
( ) ( ) ( ) ( )
For ( ) Dividend ( ) Divisor ( ) Quotient and ( ) Remainder
Where ( ) , or the degree of ( ) is less than the degree of ( ). If the remainder ( ) is zero,
( ) divides exactly into ( ).
Example: For each of the following pairs of polynomials, find the quotient ( ) and remainder ( )
that satisfy the requirements of the Polynomial Division Theorem:
a. (x) x –x (x) x b. (x) x x – x (x) x x–
Answer:
quotient ( ) remainder ( )
a 9
b

31
Telegram channel https://t.me/johnson201485
Remainder Theorem
Let ( ) be a polynomial of degree greater than or equal to 1 and let be any real number. If ( ) is
divided by the linear polynomial ( – ), then the remainder is ( ).
Example: In each of the following, express the function in the form ( ) ( – ) ( ) ( )
for the given number c, and show that ( ) is the remainder.
a. (x) x –x x
b. (x) –x x x –
c. When ( ) – – is divided by x + 1, the remainder is 15. What is the value
of ?
Answer:
a. (x) (x – ) (x x ) With remainder 29 ( ) ( ) – ( )
– Therefore, f (2) = 29 = Remainder.
b. ( ) ( )( – – ) (– ) With remainder –1. (– )
– (– ) (– ) (– ) – – – Therefore, f (–1) = –1 = Remainder.
c. – How? Show it
Remark If ( ) is divided by ( )( ) then the remainder will be linear in , which is of the
form . Thus ( ) ( )( ) ( ) . That is the remainder will be linear equation
because ( ) divided by degree 2
B. Factor Theorem: Let ( ) be a polynomial of degree greater than or equal to 1 and let be any real
number, then
i. – is a factor of ( ), if ( ) , and
ii. ( ) , if – is a factor of ( ).
Example: In each of the following, use the factor theorem to determine whether or not g (x) is a factor of f (x).
a. ( ) ; and ( )
b. ( ) ; and ( )
c. Find numbers and so that – is a factor of ( ) – – and (– ) .
Answer:
a. . / ( ) , then (x) x is not a factor of (x) x x
b. ( ) ( ) ( ) ( ) ; then ( ) ; is not a factor of
( )
c. And – ; how? Show it
3.3. ZEROS OF A POLYNOMIAL FUNCTION
At the end of this sub-unit, students will be able to:
 Determine the zero(s) of a given polynomial function.
 State the Location Theorem.
 Apply the location theorem to approximate the zero(s) of a given polynomial function.
 Apply the rational root test to determine the zero(s) of a given polynomial function.

32
Telegram channel https://t.me/johnson201485
Definition: For a polynomial function and a real number , if ( ) , then is a zero of .
Note that if – is a factor of ( ), then c is a zero of ( ).
Example:
a. (y) y – y y c. (x) x
b. (x) x – x – d. Determine the zeros of ( ) – .
e. Find a polynomial function f of degree 3 such that ( ) and the zeros of f are 0, 5 and 8.
Answer:
a. To find the zeros of the function we use different ways. The most important way is factorization
method. Then (y) y – y y y(y – y ) y or
y – y
y – y (y )(y ) y Or y
Then the zeros of (y) y – y y is -1, 0 and 1
b. (x) x – x – ( ) x – ( )

( ) ( ) ( ) ( )( )
c. Exercise
d. ( ) – ( ) –

( ) ( )


( ) ( ) ( )( )
√ √
Then the zeros of ( ) – are and
Note: A polynomial function cannot have more zeros than its degree.
e. Let ( ) ( – )( – ). Then, ( ) ( – )( – )
, then ( ) ( – )( – )
Zeros and Their Multiplicities
If ( ) is a polynomial function of degree n, n ≥ 1, then a root of the equation ( ) is called a zero
of .
By the factor theorem, each zero c of a polynomial function ( )generates a first-degree factor
( – ) of ( ). When ( )is factorized completely, the same factor ( – ) may occur more than once,
in which case c is called a repeated or a multiple zero of ( ). If ( – ) occurs only once, then c is
called a simple zero of ( ).
Definition If ( – ) is a factor of ( ) but ( – ) is not, then c is said to be a zero of multiplicity
k of .
Example: In each of the following, the indicated number is a zero of the polynomial function f (x).
Determine the multiplicity of this zero.
a. (x) x x – x b. – (x) x x x x

33
Telegram channel https://t.me/johnson201485
Answer:
a. (x) x x – x ( ) ( ). Therefore, 1 is the zero of multiplicity 2.
b. (x) x x x x ( ) . Therefore –1 is the zero of multiplicity 3.
Location Theorem : A polynomial function with rational coefficients may have no rational zeros. Most
of the standard methods for finding the irrational zeros of a polynomial function involve a technique of
successive approximation. One of the methods is based on the idea of change of sign of a function.
Consequently, the following theorem is given.
Theorem: Location Theorem: Let and be real numbers such that . If f is a polynomial
function such that ( ) and ( ) have opposite signs, then there is at least one zero of between
and .
Note: - This theorem helps us to locate the real zeros of a polynomial function. It is sometimes possible
to estimate the zeros of a polynomial function from a table of values.
Example:
a. Use the location theorem to verify that ( ) has a zero between and :
( ) – –
b. Use the Location Theorem to locate each real zero of ( ) between successive integers:
( ) – – –
Answer:
a. ( ) ( ) ( ) ( ) ( ) , and
( ) ( ) ( ) ( ) ( )
Therefore, using location theorem ( ) has zeros between -3 and -2
b.

Function -3 -2 -1 0 1 2 3
( ) – – 68 9 -2 -1 0 13 74
Then ( ) has zeros between -2 and -1
Rational Root Test
The rational root test relates the possible rational zeros of a polynomial with integer coefficients to the
leading coefficient and to the constant term of the polynomial.
Theorem: If the rational number , in its lowest terms, is a zero of the polynomials (x)
n
nx n xn x with integer coefficients, then must be a factor of and must
be a factor of
Example: Find all possible rational zeros:
a. ( ) – – b. ( ) –
1. Find all the rational zeros of the polynomial, and express the polynomial in factorized form:
a. ( )
1a. -1 and 3/2 are rational zeros of p. how? Show it
1b. 0, -1, -5/3 and 3/2 are rational zeros of p. how? Show it
2a. –1 and 2 are the only two rational zeros of p. how? Show it

34
Telegram channel https://t.me/johnson201485
4.4. GRAPHS OF POLYNOMIAL FUNCTIONS

At the end of this sub-unit, students will be able to:


 Sketch the graph of a given polynomial function.
 Describe the properties of the graph of a given polynomial function.
In your previous grades, you have discussed how to draw graphs of functions of degree zero, one andtwo.
In the present section, you will learn about graphs of polynomial functions of degree greater than two.
Note:
 The graph of polynomial function is a pictorial representation of polynomial.
 The graphs of polynomial function are smooth continuous curve.
 Smooth means does not have sharp corner.
 Continuous curve means does not have break, jump, and hole. It is continuous without break hole and
jump.
Example: From the given graph identify whether it is polynomial or not.
a. b. c. d.

Solution:
a. The graph is not a polynomial function, because there is a gap.
b. The graph is not a polynomial function, because there is a hole.
c. The graph is not a polynomial function, because there have sharp corner.
d. The graph is a polynomial function, because it is smooth and continuous.
Note: To draw the graphs of polynomial function we follow the following steps
A. X-intercept
An x-intercept is a point where a curve crosses the x-axis. The curve crosses or touch the x-axis at any
point where or ( ) . It is called the zeros of the function.
B. Y-intercept: A y-intercept is a point where a curve crosses the y-axis. The curve crosses the y-axis at
any point where .
Example: sketch the graph of the following polynomial function
a. ( ) b. ( )
Answer: To draw the graphs of each of the polynomial function, first we find the zeros polynomial or x-
intercept and y-intercept. Y
a. ( ) 2
The zeros of ( ) 1
( )
-2 -1 1 2 x
( ) ( ) -1
( )( )
-2

35
Telegram channel https://t.me/johnson201485
( )( )( )
Then the zeros of ( ) are -2, -1 and 1. This means
X-intercepts are (-2, 0), (-1, 0) and (1, 0) and The y-intercept is the value of y at x=0, if x=0
( ) , Then y-intercept is (0,-2)
Therefore the graph of ( ) is given as follows
b. Similarly to draw the graph of ( ) , We use the same steps Y
So the zeros of ( ) or x-intercept is (-2, 0), (-1, 0), (1, 0) and (2, 0) 4
And y-intercept is ( ) at then (0, 4) is y-intercept.
2
Then the graphs of the function ( ) is given as
Follows in the right side. -2 -1 1 2
BEHAVIOUR OF THE GRAPH OF POLYNOMIAL FUNCTION: -2
Note: The graphs of polynomial function: -4
 With n-degree meets (crosses) the x-axis at most n-times
 Crosses the y-axis exactly once.
 Has at most n-1 turning point.
 The graphs of even-degree polynomial function may not cross the x-axis.
Example:
a. ( ) , ( ) , …does not cross x-axis at all.
 The graphs are completely lies above x-axis.
b. ( ) , ( ) … does not cross x-axis at all.
 The graphs are completely lies below x-axis.
Example: you can observe from the graphs of above example and .
1. Local maximum and local minimum
 The points at which the graph changes direction are called turning point. If the turning point is peak,
then it is called local maximum and if the turning point is valley, then it is called local minimum.
 A point of that is either maximum point or minimum point is called local extremum of the
function.
2. FAR LEFT AND FAR RIGHT BEHAVIOUR OF POLYNOMIAL FUNCTION.
The graph of polynomial function may have several up and down fluctuations. How every the graph of
every polynomial function will eventually increase or decrease without bound as the graph moves far to
the left or far to the right from the origin.
So the leading term is said to be dominate a polynomial function ( )
as | | becomes large, because the absolute value of will be much larger than the
absolute value of any other terms.

36
Telegram channel https://t.me/johnson201485
In General: The behavior of the graph of polynomial function as x-decrease without bound to the left or
as x-increase without bound to the right can be determined
 By degree (even or odd) and By leading coefficient (positive or negative). So in a polynomial
( ) the dominant term is . The graph of ( ) eventually
rises or falls in the following four cases
Case 1: A positive leading coefficient and even degree ( and even ) 1
The graph is rise to the left and rise to the right without bound
Example: ( ) , ( ) , ( ) , and etc.
Case 2: A negative leading coefficient and even degree ( and even )
The graph is falls to the left and falls to the right without bound
Example: ( ) , ( ) , ( ) , and etc. 3
Case 3: A positive leading coefficient and odd degree ( and odd )
The graph is falls to the left and rise to the right without bound
Example: Example: ( ) , ( ) , ( ) , and etc. 4
Case 4: A negative leading coefficient and even odd ( and odd )
The graph is rise to the left and falls to the right without bound
Example: ( ) , ( ) , ( ) , and etc.
MULTIPLECITY AND X-INTERCEPT.
Let in a polynomial function ( ) , If ( ) is a factor of ( ),
then y
A. If the exponent k is even, then Graph of
i. The graph of ( ), touches the x-axis at x=c. case A
c x
ii. The sign of ( ), does not change from
one side to the other side of x= c.
iii. The graph has a local extremum at x=c. y
Graph of
B. If the exponent k is odd, then
case B
i. The graph of ( ), crosses the x-axis at x=c.
c c x
ii. The sign of ( ), does change from
one side to the other side of x= c.
iii. The graph has no a local extremum at x=c.
Exercise: sketch and state the properties of multiplicity and x-intercept of the following function.
a. ( )
b. ( )

37
Telegram channel https://t.me/johnson201485
UNIT FOUR: EXPONENTIAL AND LOGARITHMIC FUNCTIONS
Unit Outcomes: After completing this unit, students will be able to:
 Understand that the laws of exponents are valid for real exponents.
 Know specific facts about logarithms.
 Know basic concepts about exponential and logarithmic functions.
 Solve mathematical problems involving exponents and logarithms.
Main Contents
4.1.Exponents and logarithms.
4.2. The exponential functions and their graphs.
4.3. The logarithmic functions and their graphs.
4.4. Equations involving exponents and logarithms.
4.5. Applications of exponential and logarithmic functions.

4.1. EXPONENTS AND LOGARITHMS

A. An Exponent:
At the end of this subunit, students will be able to;
 Explain what is meant by exponential expression.
 State and apply the properties of exponents (where the exponents are real numbers).
There are many mathematical situations in which we multiply a number by itself repeatedly. Writing
such expressions in exponential form provide a short hand method for representing this repeated
multiplication of the same factor.
Example: A. Exponent
x x x x x x x x x
 Base
 10 is indicate the number of factor in the product, 2 appears as a factor of 10 times
Exponent
n
x x x x Base
B.
Exponent
n
x x x x Base
 2 appears as a factor of n times
 a appears as a factor of n times
Note: If n is a positive integer, and then is a product of n factor of .
Such that . In is called base, the exponent and is the power of .
Laws of Exponents
If and are a positive real number, and and are a rational number, then the following properties
hold true.
a. d. ( ) f. h. ⁄ √
b. ⁄
e. . / g. For i. (√ )
c. ( )

38
Telegram channel https://t.me/johnson201485
j. If and are non-zero real numbers then it is always true that for . / . /
k. Any non-zero numbers raised to zero is one. That is , if
Property of equality for exponential equations
l. For b > 0, b ≠1, x and y real numbers,
 , if and only if x = y
 , (x ≠ 0), if and only if a = b
Rational and Irrational exponents
Note:
A. If and are a positive real number, and and are integers with ,

(√ ) √ ( )
In √ is not real number if is a negative and is even natural number.
√ is a real number if is a negative and is odd natural number.
B. If is an irrational number and , then is a real number between and for all
possible choices of a rational numbers and such that .
Note:
A. For we have and a real number x:
i. for x ii. for x
B. For we have and a real number x:
iii. for x iv. for x
Example:
i. Use the laws of exponents to simplify each the following exponential expressions:
( )
a. b. c. d.
( )
Solution:
( )
a. c.
( )
b. ( ) ( )
( )
( ) ( ) ( )
d.
ii. Simplify each of the following expressions using one or more of the laws of exponents:
a. ( ) c.
( ) ( )
b. 4 5

Solution:
a. ( ) . / . /

b. 4 5 . / . /

( ) ( )
c.

39
Telegram channel https://t.me/johnson201485
4.2. THE EXPONENTIAL FUNCTIONS AND THEIR GRAPHS.

Competencies
At the end of this subunit, students will be able to:
 Define an exponential function.
 Draw the graph of a given exponential function.
 Describe the graphical relationships of exponential functions having bases reciprocal to each other.
 Describe the properties of an exponential function by using its graph.
The function defined b
(x) ( ) is called an exponential function with base b and exponent x.
The domain of f is the set of all real numbers.
Note that in the definition of an exponential function, the base is excluded because it yields
( ) Constant function
This is a constant function, not an exponential function.
Example: a The exponential function with base 2 is the function (x) with domain (– )
The values of ( ) for selected values of x follow:
X -3 -2 -3/2 -1 0 1 3/2 2 3
(x)= 1/8 ¼ 1/(2√ ) 1/2 1 2 2√ 4 8

Example The exponential function with base ½ is the function (x) . / with domain (– )
The values of ( ) for selected values of x follow:
X -3 -2 -3/2 -1 0 1 3/2 2 3
(x)=. / 8 4 (2√ ) 2 1 1/2 1/2√ 1/4 1/8

Generally: The function defined by (x) ( ) is exponential functions which have the
following general property.
1. The domain is (– ) 4. It is continuous everywhere.
2. The range is ( ) 5. If b > 1 it is increasing on(– )
3. It passes through ( ) 6. If b < 1 it is decreasing on(– )

4.3. GRAPHS OF EXPONENTIAL FUNCTIONS.

Example: sketch the graphs of each of the following exponential function and state its properties:
a. ( ) d. ( ) . / and ( ) . / on the same
b. ( ) . / coordinate.
c. ( ) And ( ) on the same
coordinate.

40
Telegram channel https://t.me/johnson201485
Solution:
a.First, recall that the domain of this function is the set of real numbers. Next, putting x = 0 gives y = 20 =
1, which is the y-intercept and (There is no x-intercept, since there is no value of x for which y = 0) The
values of ( ) for selected values of x is given as follows:

X -3 -2 -3/2 -1 0 1 3/2 2 3
Y
(x)= 1/8 1/4 1/(2√ ) 1/2 1 2 2√ 4 8
4 𝑥
Note that approaches zero as x decreases without bound. 𝑓(𝑥)
There is a horizontal asymptote at y = 0. 2
Furthermore, increases without bound when x increases without bound. 1
Thus, the range of f is the interval ( ) Finally, the graph is given as follows:-2 -1 2 x
b. First, recall again that the domain of this function is the set of real numbers. 1
0
Next, putting x = 0 gives y = (1/2) = 1, which is the y-intercept. Y
(There is no x-intercept, since there is no value of x for which y = 0) 4
The values of ( ) for selected values of x is given as follows: 𝑥
𝑓(𝑥) ( )
2
X -3 -2 -3/2 -1 0 1 3/2 2 3
1
(x)=. / 8 4 (2√ ) 2 1 ½ 1/2√ 1/4 1/8 -2 -1 1 2 X
Note that (1/2)x increases without bound when x decreases without bound.
Furthermore, (1/2)x approaches zero as x increases without bound:
There is a horizontal asymptote at y = 0. Y
As before, the range of f is the interval (0, ). 4 𝑓(𝑥) 𝑥

Finally, the graph is given as follows: 𝑓(𝑥) ( ) 𝑥


2
Note that, the graphs of (x)= and (x)=. / , they are symmetric
1
Between the two functions along the line y-axis. -2 -1 1 2 X
c. The domain for both functions is the set of real number. 𝑥
𝑔(𝑥)
Then to draw its graphs first we prepare table value for both functions. y
X -3 -2 -3/2 -1 0 1 3/2 2 3 4 𝑥
𝑓(𝑥)
(x)= 1/8 1/4 1/(2√ ) ½ 1 2 2√ 4 8 2
(x)= 1/27 1/9 1/(3√ ) 1/3 1 3 √ 9 27
1
The values of the function are greater than 1 for x > 0 and
Between 0 and 1 for x < 0. -2 -1 1 2 x
The graph approaches the x − axis as an asymptote on the left and
Increases without bound on the right.
y
d. Similarly, the domain for both functions is the set of real number. 𝑥
𝑔(𝑥) 4
Then to draw its graphs first we prepare table value for both functions.
𝑥 2
X -3 -2 -3/2 -1 0 1 3/2 2 3
𝑓(𝑥)
1
(x)=. / 8 4 (2√ ) 2 1 1/2 1/(2√ ) 1/4 1/8

-2 -1 1 2 x

41
Telegram channel https://t.me/johnson201485
(x)=. / 27 9 √ 3 1 1/3 1/( √ ) 1/9 1/27

The values of the function are greater than 1 for x < 0 and between 0 and 1 for x > 0.
The graph approaches the x-axis as an asymptote on the right and
Increases without bound on the left.
Properties of Exponential Functions
a. The exponential function y = bx (b > 0, b ≠ 1) has the following properties:
1. Its domain is (– , ).
2. Its range is (0, ).
3. Its graph passes through the point (0, 1)
4. It is continuous on (– , ).
5. It is increasing on (– , ) if b > 1 and decreasing on (– , ) if b < 1.
Exponential function with Base “e”
 Exponential functions to the base e, where e is an irrational
Then its graph is shown
Number, whose value is 2.7182818…, plays an important role
In both theoretical and applied problems.
 For any real number x, the function, f(x) = ex,
Defines the exponential function with base e,
Usually called the natural exponential function.
It can be shown that i . /
Examples: sketch the graphs of the exponential function f(x) = ex and f(x) = e-x
Solution: Since ex > 0 it follows that the graph of y = ex is similar to the graph of y = 2x and Since e–x >
0 it follows that 0 < 1/e < 1 and so f(x) = e–x = 1/ex = (1/e)x is an exponential function with base less than
1. Therefore, it has a graph similar to that of y = (1/2)x.
Consider a few values for x:
X -3 -2 -1 0 1 2 3
( ) 0.05 0.14 0.37 1 2.72 7.39 20.09
( ) 20.09 7.39 2.72 1 0.37 0.14 0.05

y
y 𝑔(𝑥) 𝑒 𝑥
5
5
3
3
𝑓(𝑥) 𝑒𝑥 1
1

-3 -2 -1 1 2 3 x
-3 -2 -1 1 2 3 x

42
Telegram channel https://t.me/johnson201485
Sketch the graph of the exponential function f(x) = ex.
and g(x) = e–x, on the same coordinate.
 From above example its graphs is given as follows
( ) ( ) y
 The domain is R.  The domain is R. 𝑥 𝑓(𝑥) 𝑒𝑥
𝑔(𝑥) 𝑒 5
 The range is (0, ∞)  The range is (0, ∞) 3
 Is an increasing function.  Is a decreasing function.
1
 The graph is intersects the  The graph is intersects the y-
y-axis at (0, 1). axis at (0, 1). -3 -2 -1 1 2 3 x
 Greater than 1, if x > 0  Greater than 1, if x < 0
 0 < f(x) < 1, if x < 0  0 < g(x) < 1, if x > 0

Generally:
 All members of the family f(x) = bx (b > 0, b ≠ 1) have graphs
 pass through the point ( 0 , 1)
 are above the x-axis for all values of x
 are asymptotic to the x-axis
 Have domain the set of all real numbers.
 Have range the set of all positive real numbers.

4.4. LOGARITHMS.

At the end of this subunit, students will be able to;


 Express what is meant by logarithmic expression by using the concept of exponential expression.
 Solve simple logarithmic equation by using the properties of logarithm.
 Recognize the advantage of using logarithm to the base 10 in calculation.
 Identify the ″ characteristic ″ and ″ mantissa ″ of a given common logarithm.
 Use the table for finding logarithm of a given positive number and antilogarithm of a number and
compute using logarithm.
Logarithmic function is an important notation of function in the context of solving problems, analyzing
graphs and computing inverse.
In the expression and ∈ we need to express in terms of .
 The logarithm of to the base is denoted by read as “ is the
logarithm to base .
 If and only if for and .
Example:
a. b. Because ( ) c. Because .
Because .
.
Example1: means “to what power of 2 is 8”.
Example2: Converting equations from exponential form to logarithmic form.

43
Telegram channel https://t.me/johnson201485
Exponential form Logarithmic form

( )

Generally: the value of is the answer to the question “to what power must b be raised to produces
?
Example: simplify each of the following.
a. b. √ c.

Answer:
a. Let then implies .
b. Let √
√ then ( √ ) √ implies .
c. Let then ( ) implies
Laws of logarithms (theorems on logarithm):
To simplify logarithmic expressions and solve logarithmic equations the following theorems are very
important.
Let which is positive numbers and ∈ , then
a. product rule for d. base change of logarithm
logarithm
e. if and only
b. quotient rule for
f.
logarithm g.
c. for any real number k h.
Example: If , then find the value of the following logarithm.
a. c. . / e.
b. f.
d.
g.
Solution:
a.
b. ( )

c. . / ( )

d.
Logarithms in base 10 (common logarithms)
The logarithm of a positive number to base 10 is called a common logarithm. The common logarithm is
usually the most convenient one to use for computations involving scientific notations because we use the

44
Telegram channel https://t.me/johnson201485
base 10 number system and also it has a special notation, which means a common logarithm is usually
written without indicating its base. For example, is simply denoted by .
So if a logarithm is given with no base, we take it to be base 10.
Example1: Find the common logarithm of the following
a. √ b. c. d.
Solution:
a. d.
b. √
c.
Note: If we write a number , as where , then logarithm of x can be read as
from a common logarithm table. The logarithm of m is called the mantissa of the logarithm of x, and c is
the characteristics of logarithm.
Using the logarithm table
The logarithm of any two decimal place number between 1.00 and 9.99 can be read directly from the
common logarithm table (a part of the table is given below for your reference).
So
Read the number at the intersection of row 5.2 and column 6
Reading the number in row 5.2 under column 6, we get 0.7210

Example2: Identify Mantissa and characteristics of each of


the following logarithm
a. b.
Solution:
a.
( )
Therefore, the characteristic is –4 and the mantissa is .
b. Similarly

. Therefore, the characteristic is 2 and the mantissa is .


Note: Numbers greater than 10 have logarithms greater than 1.
Antilogarithms:
Suppose log x = 0.6665. What is the value of x? In such cases, we apply what is called the antilogarithm
of the logarithm of x, written as antilog (log x). Thus antilog (log x) = antilog (0.6665).
We have to search through the logarithm table, for the value 0.6665 .We find this number located where
the row with heading 4.6 meets the column with heading 4. Therefore log 4.64 = 0.6665, and we have
x = 4.64 .
Example: Find:
a. antilog 0.8998 c. antilog (−6 ) e. antilog 0.8175
b. antilog 5.721 d. antilog(−0.2)

45
Telegram channel https://t.me/johnson201485
solution:
a. antilog 0.8998 = 7.94 c. antilog –6 = 0.000001
b. antilog 5.721 = 526000 d. antilog–0.2 = 0.6310
e. The number 0.8175 does not appear in the table. The closest value is 0.8176 and 0.8176 = log 6.57.
∴antilog 0.8175 can be approximated by 6.57 .
f. antilog 3.9058 = antilog (0. 9058 + 3) = 8.05 × 103 = 8050.
In general: Antilog (log c) = c for c>0 and log(antilog(c)=c for any real number c.
Computation with logarithms:
In this section you will see how logarithms are used for computations.
For instance, to find the product of 32 and 128 using logarithm to the base 2, you can do it as follows:
Lex

Remember that antilog (log c) = c.


In order to compute c you can perform the following two steps:
Step 1 Find log c, using the laws of logarithms.
Step 2 Find the antilogarithm of log c.
Example: Compute using logarithms:
a. b.

( )
Solution:
a. Let

nti ( x nti ( )
√ √
b. Let ( )

x ( )

( ) ( )
x nti ( x) nti ( ) nti ( )

4.5. THE LOGARITHMIC FUNCTIONS AND THEIR GRAPHS.

Competencies
At the end of this subunit, students will be able to:
 Define logarithmic function.
 Draw the graph of a given logarithmic function.
 Describe the properties of a logarithmic function by using its graph.
 Describe the graphical relationship of logarithmic functions having bases reciprocal to each other.
 Describe how the domains and ranges of exponential and logarithmic functions are related.
 Describe the relationships of the graphs of exponential and logarithmic functions.

46
Telegram channel https://t.me/johnson201485
Logarithmic function
For x  0 and 0  a  1, y = loga x if and only if x = a y.
The function given by f (x) = loga x is called the logarithmic function with base a.
Every logarithmic equation has an equivalent exponential form: y = loga x is equivalent to x = a y
A logarithmic function is the inverse function of an exponential function.
Generally:
The function defined by ( ) is called the logarithmic function with base b.
 The domain of f is the set of all positive numbers.
 The range of f is the set of all real numbers.
Example1: sketch the graph of logarithmic function ( )
Solution: Since the logarithm function is the inverse of the exponential function of the same base, its
graph is the reflection of the exponential function in the line y = x.
OR by preparing table value for ( ) it is possible to draw. Y
( ) ( ) 4 𝑥
𝑓(𝑥)
-2 ¼ ¼ -2 2
𝑓(𝑥) 𝑥
-1 ½ ½ -1 1
-2 -1 -1 1 2 x
0 1 1 0
-2
1 2 2 1 -4
2 4 4 2
3 8 8 3

Example 2: Graph the common logarithm function ( )


( )
Y
1/100 -2
4
1/10 -1 𝑓(𝑥) 𝑥
2
1 0
1
10 1 -10 -1 -1 1 10 x
100 2 -2
-4
1000 3

47
Telegram channel https://t.me/johnson201485
Example 3: sketch the graphs of a logarithmic function of y = log2 x and y= log10 x
From these graphs as the base becomes large, Y
It is approaches to y-axis to the left side and 4 𝑓(𝑥) 𝑥
𝑓(𝑥) 𝑥
Approaches to x-axis to the right side. 2
1
-10 -1 -1 1 10 x
-2
-4

Relationship between the Graphs of Logarithmic and Exponential Functions

Example4: sketch the of , and . /


, . /

Solution: The graphs of this exponential and logarithmic can be drawn by preparing its table value

( )
. /

1/27 -3 -3 1/27 27 -3 -3 27
1/9 -2 -2 1/9 9 -2 -2 9
1/3 -1 -1 1/3 3 -1 -1 3
1 0 0 1 1 0 0 1
3 1 1 3 1/3 1 1 1/3
9 2 2 9 1/9 2 2 1/9
27 3 3 27 1/27 3 3 1/27

48
Telegram channel https://t.me/johnson201485
 Generally
 Domain of y = logbx is equal to range of y=bx and
 Range of y = logbx is equal to Domain of y=bx
Properties of Logarithmic Functions
 The logarithmic function
y = logbx (b > 0, b ≠ 1)
has the following properties:
1. Its domain is (0, ).
2. Its range is (– , ).
3. Its graph passes through the point (1, 0).
4. It is continuous on (0, ).
5. It is increasing on (0, ) if b > 1and decreasing on (0, ) if b < 1.
6. The values of the function are negative for 0 < x < 1 and they are positive for x > 1 for b>1 and The
values of the function are positive when 0 < x < 1 and they are negative when x > 1 for 0<b<1
Natural logarithm:
 The function defined by f(x) = loge x = ln x (x  0, e 2.718281) is called the natural logarithm
function.
Note: y = ln x is equivalent to e y = x
Examples
x2 x2 1 1
 Expand and simplify the expression: ln solution 2 ln x  ln( x 2  1)  x
ex 2
Example
 Sketch the graph of the function y = ln x.
Solution
We first sketch the graph of y = ex.
The required graph is the mirror image of the graph of
y = ex with respect to the line y = x:

Properties Relating Exponential and Logarithmic Functions


 Properties relating ex and ln x:
eln x =x (x > 0)
x
ln e =x (for any real number x)
Examples: simplify each of the following
a. ln x = –3 solution eln x = e– 3 x = e–3
b. ex = 7 solution ln ex = ln 7 x = ln 7
c. ln x – ln 3 = 0 solution ln x = ln 3 x=3
3 3
d. lne solution lne = 3

49
Telegram channel https://t.me/johnson201485
4.6. EQUATIONS INVOLVING EXPONENTS AND LOGARITHMS.

At the end of this sub-unit, students will be able to:


 Solve equations involving exponents.
 Solve equations involving logarithms.
A. Solving Exponential equations
An exponential equation is an equation with unknown in exponent. Properties of exponent and logarithms
play major role in solving of exponential and logarithm equation
Example
a. Solve 27x+3 = 9x-1 answer x= -11
b. Solve 4x-2 = 64x answer x= -1
2x
c. Solve 2 = 11 answer x= 1.730
d. Solve 4x -2x+1-8=0 answer Exercise for student
:
Solution
a. 27x+3 = 9x-1 b. 4x-2 = 64x c. 22x = 11
3(x+3) 2(x-1) 2(x-2) 6(x)
3 =3 2 =2
3(x+3) =2(x-1) 2(x-2) =6x
3x+9=2x-2 2x-4=6x
3x-2x=-2-9 2x-6x=4 ( )
X= -11 X= -1

Solving Logarithmic equations


Use the properties of logarithms to solve the equation for x:
Example: Solve each of the following logarithmic equation
a. log33( x  1)  log33( x  1)  1 c. ln x = 2
b. log x  log(2 x  1)  log6
log 6 d. log3(5x – 1) = log3(x + 7)
Solution
a. log (. x  1)  log ( x  1)  1 b. log.x  log(2 xx  1) 
1)  log6
log 6
33 33
log x  log(2 x  1)  log6
log 6  0
x 1
log33 1 x(2 x  1)
x 1 log 0
6
x  1 11 x(2 x  1)
3 3 1)
 10
100  11
0
x 1 6
x  1  3( x  1) x(2 x  1)  6
x  1  3x  3 2 x22  x  6  0
3
4  2x (2 x  3)( x  2)  0 x is out of
2
x2 the domain of loglog xx,,
x  22
so it is discarded.
c. ln x = 2 d. log3(5x – 1) = log3(x + 7)
ln x 2
e =e 5x – 1 = x + 7
x = e2 4x = 8
x=2

50
Telegram channel https://t.me/johnson201485
EXERCISE: Show each steps for each of the following questions.

4 log 8 x  log 8 81 log 5 (y 12)  log 5 (y 12)  2


x  3 y  13 or 13
y  13
 1 1
log 5 42  log 5 6  log 5 k log10 y  log10 16  log10 49
4 2
k 7  y  14


 

4.7. APPLICATIONS OF EXPONENTIAL AND LOGARITHMIC FUNCTIONS.

(Exponential & logarithm Functions as Mathematical Models)


At the end of this sub-unit, students will be able to:
 Solve problems involving exponential and logarithmic functions from real life.
Exponential and logarithm function have different application on different areas. Some of them are:
 Growth/decay of population (bacteria)
 Types of Interest
 Concentration of Hydrogen ion
 Transparency light of a Lake
 Radioactive decay
 Assembly time
Now for this level we will see some of the first three areas with its examples.
A. Types of Interest
If a principal P is invested at an interest rate r for a period of t years, the amount A of the investment is
given by:
A  P 1  r  Simple interest (for 1 year)
nt
 r
A t   P 1  Interest compounded n times per year
 n
A  t   Pe rt Interest compounded continuously

51
Telegram channel https://t.me/johnson201485
We can use logarithms to determine the time it takes for the principal to increase to a given amount.
Finding Term for an Investment to Double
 A sum of birr 5000 is invested at an interest rate of 5% per year. Find the time required for the money
to double if the interest is compounded according to the following method.
(a) Semiannual (b) Continuous
A). Semi-annualy : We use the formula for compound interest with P = birr 5000, A(t) = birr 10,000, r =
0.05, n = 2
and solve the resulting exponential equation for t.

2t
 0.05 
5000  1   10000
 2 
1.025 
2t
2
log1.025 2t  log 2
2t log1.025  log 2 (Law 3)
log 2
t   14.04
2 log1.025
The money will double in 14.04 years.
b). Continously : We use the formula for continuously compounded interest with P = birr 5000, A (t) =
birr 10,000, r = 0.05 and solve the resulting exponential equation for t.

5000e 0.05 t  10,000


e 0.05 t  2
ln e 0.05 t  ln 2
0.05t  ln 2
ln 2
t   13.86
0.05
 The money will double in 13.86 years.
Time Required to Grow an Investment
Example: A sum of birr 1000 is invested at an interest rate of 4% per year. Find the time required for the
amount to grow to birr 4000 if interest is compounded continuously.

52
Telegram channel https://t.me/johnson201485
Solution :We use the formula for continuously compounded interest with P = birr 1000, A (t) = birr
4000, r = 0.04 and solve the resulting exponential equation for t.

1000e 0.04 t  4000


e 0.04 t  4
0.04t  ln 4
ln 4
t  34.66
0.04
The amount will be birr 4000 in about 34 years and 8 months.
Annual Percentage Yield: If an investment earns compound interest, the annual percentage yield (APY)
is: The simple interest rate that yields the same amount at the end of one year.
Example Find the APY for an investment that earns interest at a rate of 6% per year, compounded daily.
After one year, a principal P will grow to:
365
 0.06 
A  P 1  P 1.06183 
 365 
 The formula for simple interest is: A = P(1 + r)
 Comparing, we see that:1 + r = 1.06183
 Therefore, r = 0.06183.
 Thus, the annual percentage yield is 6.183%.
B. Concentration of Hydrogen ion: The concentration of hydrogen ions in a given solution is donated
by [H+] and is measured in moles per litre. For example, [H+] = 0.0000501 for beer and [H+] =
0.0004 for wine. Chemists define the pH of the solution as the number pH= −log [H ] + . The solution
is said to be an acid if pH < 7 and a base if pH > 7. Pure water has a pH of 7, which means it is
neutral.
a. Is beer an acid or a base? What about wine?
b. What is the hydrogen ion concentration [H+] of eggs if the pH of eggs is 7.8?
Solution
a. For beer pH = 4.3 < 7, so beer is an acid. And for wine PH = 3.4 < 7.So wine is an acid.

b.

53
Telegram channel https://t.me/johnson201485
UNIT FIVE: SOLVING OF INEQUALITIES
Unit Outcomes:
After completing this unit, students will be able to:
 Know and apply methods and procedures in solving problems on inequalities involving absolute value.
 Know and apply methods in solving systems of linear inequalities.
 Apply different techniques of solving quadratic inequalities.
Main content
5.1.Inequalities involving absolute value
5.2.Systems of liner inequalities in two variables
5.3.Quadratic inequalities

5.1. INEQUALITIES INVOLVING ABSOLUTE VALUE

Competencies
At the end of this subunit, students will be able to:
 Describe sets using interval notation.
 Solve inequalities involving absolute value of linear expression.
Introduction
Set: Is a collection of well-defined objects is called set.
Set can be described by:
 Verbal method
 Listing method (partial listing or complete listing method)
 Set builder method
Notation: For real numbers a and b where a < b,
 (a, b) is an open interval;
 (a, b] and [a, b) are half closed or half open intervals; and
 [a, b] is a closed interval, which is it include the boundary elements.
Note: If a and b are fixed real numbers with a < b, then
 [a, b] = {x: a ≤ x ≤ b and x∈ℝ} and (a, b) = {x: a < x < b, x∈ℝ} ( ] { } ,
 (-∞, a]={x: x ≤ a and x∈ℝ } and (a, ∞)={x: x > a and x∈ℝ }
The symbol "∞" is used to mean positive infinity and "−∞" is used to mean negative infinity.

54
Telegram channel https://t.me/johnson201485
Example: simplify each of the following using intervals
a. { x : x∈ℝ and x ≠ –2 } b. { x: 2x + 3 ≥ –5x} c. {x: 2x –1 < x < 3}
Solution:
a. ( )∪( )

b. x – x x x , )

c. {x: 2x –1 < x < 3} which means, 2x –1 < x, x < 3 and 2x –1 < 3 from this three equation we have x<1
Then the interval form of {x: 2x –1 < x < 3} is ( )
Absolute value:The number that shows only the distance from the point corresponding to zero (and not
the direction) is called the absolute value.
Definition: If x is a real number the absolute value of x, denoted by | | , is defined by

| | { Or | | {

Example:
a. | |
b. | | ( )
Note:
 The absolute value of a number is always non negative.
 The absolute value of a number is, in other word the distance of a number from zero without
considering the direction
Properties of absolute value :For any two real numbers x and y:
a. | | d. | | | |
b. | | | | e. √ | | | |
c. | | | || |
f. | |
| |
for
| |

g. | | | | | |, which is | | | | | | if x and y have the same sign and | | | |


| | if x and y have opposite in sign
Theorem 3.1 Solutions of the equation |x|
For any real number a, the equation |x|= a has:
 Two solutions x = a and x = –a, if a > 0;
 One solution, x = 0, if a = 0; and
 No solution, if a < 0.

55
Telegram channel https://t.me/johnson201485
Example: Find the solution set of the following equation.
a. | x | b. | – x| c. |x– | –
Solution:
a. | x | x r x
x r x
x rx

{ }

b. | – x| x x x

{ }

c. |x– | ,
* + ∅ , because the absolute value of any real number is always non-negative number.
Theorem 3.2 Solution of |x| and |x|
For any real number a > 0,
 The solution of the inequality |x| is – a < x < a.
 The solution of the inequality |x| is – a ≤ x ≤ a.
 In | x| < a, if a < 0 the inequality |x| < a has no solution.
 The solution of the inequality |x| , x=0 if a=0
Example:
a. | | c. | x | e. |x – | –
b. | | d. | x | f. |x – | –
Solution:
a. | |
x
x
x
x

Then the solution set is given as s. s . /


b. | |
x
x
x
x

Then the solution set is given as s. s 0 1

56
Telegram channel https://t.me/johnson201485
c. | x | x x x Then the solution set is given as s.

s 2 3

d. | x | , They have no solution, because the absolute value of any real number is greater than
or equal to zero.
e. |x – | – They have no solution, because the absolute value of any real number is greater than or equal
to zero.

f. |x – | – They have no solution, because the absolute value of any real number is greater than or
equal to zero.
Theorem 3.3 Solution of |x| and |x|
For any real number a, if a > 0, then
 The solution of the inequality |x| > a is x < – a or x > a.
 The solution of the inequality |x| is x ≤ – a or x ≥ a.
 The solution of the inequality |x| > a is * + if a=0 and if a<0
 The solution of the inequality |x| is if a<0
Example:
a. | x – | b. | | c. |x – | – d. |x – | –
Solution:
a. | x – | x or x x or x x or x

Then the solution set is given as s. s . /∪( )

b. | – x | x or x x or x x or x

c. |x – | – , Then the solution set is the set of real number. Because the absolute value of any real
number is always greater than or equal to zero.
d. |x – | – , Then the solution set is the set of real number. Because the absolute value of any real
number is always greater than or equal to zero.
5.2.SYSTEMS OF LINER INEQUALITIES IN TWO VARIABLES
Competency
At the end of this subunit, students will be able to:
 solve system of linear inequalities in two variables by using graphical method

57
Telegram channel https://t.me/johnson201485
When two or more linear equations involve the same variables, they are called a system of linear
equations.
An ordered pair that satisfies all the linear equations of a system is called a solution of the system. A
system of two linear equations in two variables often involves a pair of straight lines in the plane. The
solution set of such a system of equations can be determined from the graph and is the set of all ordered
pairs of coordinates of points which lie on both lines.

For instance: {

Note: In a system of equations, if “=” is replaced by “<”, “>”, “≤” or “≥”, the system becomes a system
of linear inequalities.
When two or more linear inequities involve the same variables, they are called a system of linear
inequities. An ordered pairs (region) that satisfies all the linear inequalities called the solution of the
system.
When we solve system of linear inequalities graphically if “≤” or “≥” we use a solid line if “<” or “>” we
use a broken line.
Example1: solve each of the following system of linear inequalities

a). { b). { c). {

Solution:
a. Consider the system of inequalities shown below:

The solution of this system is the set of all ordered pairs that
Satisfy both inequalities. This solution can be determined by
Graphing each inequality in the same coordinate plane as shown below.
Recall that the graph of each inequality is called a half-plane.
The intersection of the two half-planes represents the solution
To the system of inequalities. This solution is a region that
Contains the graphs of an infinite number of ordered pairs.
The graphs of
y = x + 2 (y ≥ x + 2)
y = -2x – 1 (y ≤-2x – 1)

58
Telegram channel https://t.me/johnson201485
Are the boundaries of the region and are included in the graph of the system.
b. Solve each system of inequalities by graphing

The solution is the ordered pairs in the intersection


of the graphs of y > x – 3 and y ≤ -1.
This region is shaded in green at the right.
The graphs of y = -1 and y = x – 3 are the boundaries of the region.
The graph of y = x – 3 is a dashed line and is not included in the graph of the system.

5.3.QUADRATIC INEQUALITIES

Competencies
At the end of this sub-unit, students will be able to:
 Solve quadratic inequalities by using product properties.
 Solve quadratic inequalities using the sign chart method.
 Solve quadratic inequalities using graphs.
Definition: An inequality that can be reduced to any one of the following forms:
Or
Or
Where and are constants and , is called a quadratic inequality.
Quadratic inequalities can be solved by using different methods. Some of them are:
a. Product Properties or algebraic method.
b. Sign Chart Method
c. Graphical method.
A. Solving Quadratic Inequalities Using Product Properties
The product of two real numbers is positive, if and only if either both are positive or both are negative.
This fact can be used to solve the given inequality.
Product properties:
1. m.n > 0, if and only if i m > 0 and n > 0 or ii m < 0 and n < 0.
2. m.n < 0, if and only if i m > 0 and n < 0 or ii m < 0 and n > 0.
B. Solving Quadratic Inequalities Using the Sign Chart Method.

59
Telegram channel https://t.me/johnson201485
As x is moved along the number line, the quantity is sometimes positive, sometimes zero,
and sometimes negative. To solve the inequality, you must find the values of x for which
is negative. Intervals where is positive are separated from intervals where it is negative
by values of x for which it is zero. To locate these values, solve the equation .
Factorizing the equation and dividing into three interval and determine the sign in those three intervals
give the solution of the required.
C. Solving Quadratic Inequalities Graphically.
In order to use graphs to solve quadratic inequalities, it is necessary to understand the nature of quadratic
functions and their graphs.
 If , then the graph of the quadratic function ( ) is an upward parabola.
 If , then the graph of the quadratic function ( ) is a downward parabola.
The graph of a quadratic function has both its ends going upward and downward depending on whether
is positive or negative. From different graphs you can observe that the graph of a quadratic function
( )
 Crosses the x-axis twice, if – .
 Touches the x-axis at a point, if – .
 Does not touch the x-axis at all, if – .
To solve a quadratic inequality graphically, find the values of x for which the part of the graph of the
corresponding quadratic function is above the x-axis, below the x-axis or on the x-axis.
Example: solve each of the following quadratic inequality graphically
A. B. C.
Solution: To solve the problem, first let us draw the graphs using the following information
a. and
X-intercept: The zeros of quadratic function which is
( )( +1) Y
Then x-intercept are (-5, 0) and (-1, 0) 𝑓(𝑥) 𝑥 𝑥
Y-intercept ( ) y, then ( ) , (0, 5) is y-intercept
5
Vertex . ( )/ ( )
-1 x
Therefore: The solution set of inequalities of -5
 is ( )∪( ) ( )
 is ( -∪, )
 is ( )
 is , -

60
Telegram channel https://t.me/johnson201485
Generally: In a quadratic equation for , the solution of quadratic inequalities
can be determined as follows using the following cases
Case one: If then the equation has no real roots. Moreover, the solution set of
 is the set of real number if
 is empty set if
 is empty set if
 is the set of real number if
Case two: If then the equation has exactly one real root which is , then the solution
set of
 is the set of real number if  is empty set if
 is 2 3 if
 is the set of real number
 is the set of real number if
except if
 is empty set if  is the set of real number
 is 2 3 if except if
Case three: : If then the equation has exactly two distinct real roots, let its roots are d
and e where d<e
 is ( -∪ , )if  is ( ) if
 is ( )∪ ( ) if  is , - if
 is ( )if  is ( -∪ , )if
 is , - if  is ( )∪ ( )if

61
Telegram channel https://t.me/johnson201485
UNIT SIX. PLANE GEOMETRY
Unit Outcomes
After completing this unit, students will be able to:
 Know more theorems special to triangles.
 Know basic theorems specific to quadrilaterals.
 Know theorems on circles and angles inside, on and outside a circle.
 Solve geometrical problems on quadrilaterals, circles and regular polygons.
Main contents:
 Theorems on triangles
 Special quadrilaterals
 More on circles
 Regular polygons

6.1. THEOREMS ON TRIANGLES


At the end of this sub-unit, students will be able to:
 Apply the incidence theorems to solve related problems.
Basic terminologies:
a. Collinear points: Three or more points that lie on one line are called collinear points.
b. Concurrent lines: Three or more lines that pass through one point are called concurrent lines.
The common point is called point of concurrency.

c. Bisector of line: A line that divides a line segment into two congruent line segments is called
B a
bisector of the line segment. D
d. Angle bisector: divides an angle into two equal angles. A
Any point of the bisector of an angle is equidistant from the sides of an angle, C

i.e., ≡
e. Perpendicular bisector: When a bisector of a line segment forms right angle with the line segment,
then it is called the perpendicular bisector of the line segment.
f. Incidence theorems: Theorems about collinear points and concurrent lines are called incidence
theorems.

62
Telegram channel https://t.me/johnson201485
Note: special segments on the theorem of triangles
Triangles have four types of special segments: these are
I. Median of triangle
II. Altitude of triangle
III. Angle bisector of triangle
IV. Side bisector of triangle
Special segment (line) of a triangle, which is median, altitude, angle bisector and side bisector of a triangle
are concurrent at a point. The intersection points of:
 Median is called centroid
 Altitude is called orthocenter
 Perpendicular side bisector is called circum-center
 Angle bisector is called in-center
A. Medians of triangle
A median of a triangle is a line segment drawn from any vertex to the mid-point of the opposite side.
 Every triangle will have three medians.
 Three medians of a triangle pass through the same point.
The lines are concurrent.
 The point intersection the medians of triangle are called centroid of triangle.
Theorem 6.1 The medians of a triangle are concurrent at a point of the distance from each vertex to the

mid-point of the opposite side.


Note:
 The medians of equilateral triangles are equal to each other.
 The length of the medians to the hypotenuse of a right triangle equals one-half of the length of the
hypotenuse. A
M
If is the medians of a right-angle triangle ABC to hypotenuse ,
C B
Then that is

B. Altitude of a triangle
The altitude of a triangle is a line segment drawn from a vertex, perpendicular to the opposite side, or to
the opposite side produced.
 Every triangle has three altitudes

63
Telegram channel https://t.me/johnson201485
 In triangle, the three-altitude pass-through the same point, this common point is called ortho-center of
triangle
 The three altitudes of a triangle are concurrent. i.e. (intersect at a point)
 Three altitudes of:
I. Acute-angle triangle lies inside
II. Right-angled triangle lies the vertex of right angled
III. An obtuse-angle triangle lies outside of triangle
Bisector of triangle:
There are two bisector of triangle:
 Angle bisector of triangle
 Side bisector of a triangle
The perpendicular bisector of a side of a triangle:
Theorem 6.2
The perpendicular bisectors of the sides of any triangle are concurrent at a point which is equidistant from
A
the vertices of the triangle.
Note: E O D

B C
F
 Every triangle has three perpendicular bisector and they intersect each other at a common point.
 The perpendicular bisector of the sides and of a triangle intersect at O, This
common point is called circum-center of the triangle
 The point O is equidistant from the three vertices A, B, C of triangle because O lies on
perpendicular sector of and of a triangle . Therefore radius
 If we draw a circle with center O and radius , it will pass through B and C. The three perpendicular
side bisector of triangle for:
a. An-acute triangle lies inside the triangle. c. Right-triangle lies on hypotenuse of triangle
b. An-obtuse triangle lies outside the triangle.
Theorem 6.3: The altitudes of a triangle are concurrent.
C. Angle bisector of a triangle
An angle bisector of a triangle is a segment that bisects an angle of a triangle has one end-point at a
vertex of a triangle and the other end-point on the triangle.

64

Telegram channel https://t.me/johnson201485


Theorem 6.4 The angle bisectors of any triangle are concurrent at a point which is equidistant from the
sides of the triangle. A
E
F O

B C
D

 Every triangle has three angle bisector and they intersect each other at a common point, this
common point called in-center.
 In triangle , and angle bisector, they intersect at a single point O such that
.
D. Altitude theorem: The altitude theorem is stated here for a right angled triangle. It relates the length
of the altitude to the hypotenuse of a right-angled triangle, to the lengths of the segments of the
hypotenuse.
Theorem 6.5 Altitude theorem
In a right-angled triangle ABC with altitude CD to the hypotenuse, AB

Note: The Square of the length of the altitude is the product of the lengths of the segments of the
hypotenuse.

6.2. SPECIAL QUADRILATERALS

At the end of this sub-unit, students will be able to:


 Apply theorems on special quadrilateral in solving related problems.
Quadrilaterals:
 Is any four-sided closed figure.
 The point A, B, C, D are called vertices.
 The two line segment and are diagonals.
 Adjacent side: are two side of quadrilateral that have common end-point
Example: and is a pair of adjacent side with common end-point A.
 The line segment and are called a pair of opposite side.
Special quadrilaterals: In this section, we consider the following special quadrilaterals: trapezium,
parallelogram, rectangle, rhombus and square.

65

Telegram channel https://t.me/johnson201485


A. Trapezium: A trapezium is a quadrilateral where only two of the sides are parallel. B
A
In the quadrilateral ABCD is a trapezium.
The sides and are non-parallel sides of the trapezium ABCD. D C

Note that if the sides and of trapezium ABCD are congruent, then the trapezium is called an
isosceles trapezium.
B. Parallelogram: A parallelogram is a quadrilateral in which both pairs of opposite sides are parallel.
In Figure the quadrilateral ABCD is a parallelogram, then AB//DC and AD//BC
Properties of a parallelogram and tests for a quadrilateral to be a parallelogram are
Stated in the following theorem:
Theorem 6.7
a. The opposite sides of a parallelogram are congruent.
b. The opposite angles of a parallelogram are congruent.
c. The diagonals of a parallelogram bisect each other.
d. If the opposite sides of a quadrilateral are congruent, then the quadrilateral is a parallelogram.
e. If the diagonals of a quadrilateral bisect each other, then the quadrilateral is a parallelogram.
f. If the opposite angles of a quadrilateral are congruent, then the quadrilateral is a parallelogram.
C. Rectangle A B
A rectangle is a parallelogram in which one of its angles is a right angle.
Some properties of a rectangle
a. A rectangle has all properties of a parallelogram. D C

b. Each interior angle of a rectangle is a right angle.


c. The diagonals of a rectangle are congruent.
D. Rhombus
A rhombus is a parallelogram which has two congruent adjacent sides. A B
Some properties of a rhombus
a. A rhombus has all the properties of a parallelogram. D
C
b. A rhombus is an equilateral quadrilateral.
c. The diagonals of a rhombus are perpendicular to each other.
d. The diagonals of a rhombus bisect its angles.
E. Square
A square is a rectangle which has congruent adjacent sides.

66

Telegram channel https://t.me/johnson201485


Some properties of a square
 A square has the properties of a rectangle.
 A square has all the properties of a rhombus.
Theorem 6.8
If the diagonals of a quadrilateral are congruent and are perpendicular bisectors of each other, then the
quadrilateral is a square.

6.3. MORE ON CIRCLES


At the end of this sub-unit, students will be able to:
 Apply the theorems on angles and arcs determined by lines intersecting inside, on and outside a circle
to solve related problems.
Circle: Circle is a plane figure, all points of which are equidistant from a given point called the Centre of
the circle
Basic terms on circle
a. Chord: a line segments whose end points are on the circle. Example
b. Diameter: is a chord which pass-through a Centre of a circle. Example
c. Arc: some part of circumference of circle. Example ̂
d. Central angle: an angle two radius and whose vertices are Centre of a circle.
e. Inscribed angle: In a circle, an inscribed angle is an angle whose vertex lies on the circle and whose
sides are chords of the circle.
f. Subtended angle(intercepted arc): angle <AOC is subtended by arc ̂
g. Inscribed angle (subtended arc): angle <AOC is intercepted by arc ̂
P X
h. Tangent line( ): is line which touches the circle exactly at one point.
Q
i. Secant line( ): is a line segment which touches the given circle at two points. O
𝑙
Note: Measure of a central angle: Note that the measure of a central angle is the
Measure of the arc it intercepts. So, m (∠ POQ) = m ( ̂ ).
𝑙
Angles and Arcs Determined by Lines Intersecting Inside and On a Circle
Theorem 6.9 : The measure of an angle inscribed angle in a circle is half the measure of the arc
subtending it. i.e. m (∠ ABC) = ½ m ( ̂ ).
Case 1: Suppose that one side of ∠ABC is a diameter of the circle with Centre O.

67

Telegram channel https://t.me/johnson201485


Case 2: Suppose that A and C are on opposite sides of the diameter through B.
Case 3: Suppose that A and C are on the same side of the diameter through B.
Corollary:
a. An angle inscribed in a semi-circle is a right angle.
b. An angle inscribed in an arc less than a semi-circle is obtuse.
c. An angle inscribed in an arc greater than a semi-circle is acute.
Theorem 6.10
Two parallel lines intercept congruent arcs on the same circle.

Angle formed by tangent and a chord:


Theorem 6.11 : An angle formed by a tangent and a chord drawn from the point of tangency is measured
by half the arc it intercepts.
An angle formed by two intersecting chords
Theorem 6.12 : The measure of an angle formed by two chords intersecting inside a circle is half the
sum of the measures of the arc subtending the angle and its vertically opposite angle.
Product or rectangle property of a circle.
If two chords intersect inside a circle the product of the lengths of the segments of one chord equals the
product of the lengths of the segment of the other. A D
i.e. C X
Angles and Arcs Determined by Lines Intersecting Outside a Circle.
B
Theorem 6.13 : The measure of the angle formed by the lines of two chords intersecting outside a circle
is half the difference of the measure of the arcs they intercept.

Theorem 6.14: The measure of an angle formed by a tangent and a secant drawn to a circle from a point
outside the circle is equal to one-half the difference of the measures of the intercepted arcs.
Given: Secant and tangent intersecting at P.

68

Telegram channel https://t.me/johnson201485


To prove: (∠ ) ( (̂ ) ( ̂ ))

Theorem : If a secant and a tangent are drawn from a point outside a circle, then the square of the length
of the tangent is equal to the product of the lengths of line segments given by
(P ) (P ) (P )

6.4. REGULAR POLYGONS(5)

Competencies
At the end of this sub-unit, students will be able to:
 Calculate the perimeters of regular polygons.
 Calculate the areas of regular polygons.
Introduction: A polygon whose vertices are on a circle is said to be inscribed in the circle.
The circle is circumscribed about the polygon.
A polygon whose sides are tangent to a circle is said to be circumscribed about the circle.
The pentagon PQRST is circumscribed about the circle. The circle is inscribed in the pentagon.
Perimeter of a Regular Polygon
The perimeter of a regular polygon is given as where n is the number of side and s is the length
of the side of regular polygon.

So ( . / . / . / in which d is the diameter of the circle

inscribed a polygon.
Area of a Regular Polygon
Theorem 6.16 : The area A of a regular polygon with n sides and radius r is

4 5

Example:
1. Find the area of a regular nine-sided polygon with radius 5 units.
2. Find the area of a regular twelve-sided polygon with radius 3 units

69

Telegram channel https://t.me/johnson201485


UNIT SEVEN MEASUREMENT
Unit Outcomes:
After completing this unit, you should be able to:
 Solve problems involving surface area and volume of solid figures.
 Know basic facts about frustums of cones and pyramids
Main Contents
 Revision on Surface Areas and Volumes of Prisms and Cylinders.
 Pyramids, Cones and Spheres
 Frustums of Pyramids and Cones
 Surface Areas and Volumes of Composite Solids
INTRODUCTION
We know that plane Geometry (sometimes called Euclidian Geometry) is a branch of Geometry
which deals about the properties of flat surfaces and plane figures, such as polygons, circles and so
on. Geometrical figures that have three dimensions (length, width and height) are called solid
figures. For example prisms, cylinders, cones, pyramids, spheres, hemispheres etc. These solid
figures have a volume. A branch of Geometry which deals about the surface areas and volume of
these solid figures is called Solid Geometry. In this unit you will learn more about surface areas and
volumes of solid figures. You will also study about surface areas and volumes of composed solids
and frustums of pyramids and cones.

7.1. SURFACE AREAS AND VOLUMES OF PRISMS AND CYLINDERS.

After completing this sub-unit, students will be able to:


 apply the formula for calculating surface area and volume of prism and cylinder.
A. Prism: is a solid object with identical ends, flat faces, and the same cross-sectional along its length.
 A cross-section is a shape made by cutting straight across the object
 It is a polyhedron, which means all faces are flat
 The side face of a prism is a parallelogram (for sided shapes with opposite sides are parallel).
 A prism is a solid figure which is bounded by two congruent polygons called the bases (upper
and lower bases).
 A prism is named by its bases (Triangular prism, rectangular prism, pentagonal prism).
 If the lateral faces are perpendicular to the bases, then the prism is called a Right-prism. (The lateral

70

Telegram channel https://t.me/johnson201485


faces are rectangles).
 If the lateral faces are not perpendicular to the bases, then the prism is called an Oblique prism.
(The lateral faces are parallelograms).
 The perpendicular distance, between the planes containing the bases is called Altitude of the prism.
 The union of the lateral faces and bases is called total surface (or surface)
Note: Upper base

a. In a prism
Lateral edge
 Lateral edges are equal and parallel
 Lateral faces are parallelograms Lateral face

b. In right prism
Base Lower base
 Altitude is equal to lateral edges and perpendicular to bases.
edge
 Lateral faces are rectangles.
c. In oblique prisms
 Altitude is shorter than lateral edges
d. A right prism with bases of regular polygon is called Regular prism
e. A right square prism whose altitude equals to length of edge of bases is called a Cube.
f. If bases are n-sided polygon prism then the prism has 3n edges and 2n vertices.
Surface area and Volume of Prisms
For a prism if its height= h
Lateral surface area = AL
Total surface area = AT
Base area = AB
Base Perimeter = p and Volume = V, then
 The lateral surface area- the sum of the area of lateral face (AL) = (perimeter) x (height) = ph
 Total surface area- the sum of lateral surface and the two bases (AT)= ph+2AB
 Volume- the product of base area and height (v)= ABh
Cylinders:It is a circular flat base and a flat top.
The lower is the same as the top base
h
From the base to the top the shapes stays the same.
It is one curved side. It is not a polyhedron as it has a curved surface r

71

Telegram channel https://t.me/johnson201485


Note:
 Circular cylinder: If the base of the cylinder is circular region, then it is called circular cylinder
 Right cylinder: if the line joins the two planes (directrix of the cylinder) is perpendicular to the planes
of the bases, then it is called right cylinder.
 Right circular cylinder: A circular cylinder which is right.
 Oblique cylinder: A cylinder which is not right is called oblique cylinder.
Surface area and Volume of cylinder:
 The lateral surface area- the product of circumference of the base and the height (AL) =
 The total surface area- the sum of the lateral surface area and the two bases
(AT)= ( )
 Volume- the product of base area and height
Example: 6
7
a. Find the total surface area and volume of the following prism.
1
b. The base of a right prism is an equilateral triangle of length 3 cm and 14 6
its lateral surfaces are rectangular regions. If its altitude is 8 cm,
Then find: the total surface area of the prism and the volume of the prism.
c. The radius of the base of a right circular cylinder is 2 cm and its altitude is 3 cm. Find the area of its lateral
surface, the total surface area and the volume.
d. A circular hole of radius 5 cm is drilled through the Centre of a right circular cylinder whose base has radius 6
cm and whose altitude is 8 cm. Find the total surface area and volume of the resulting solid figure.
e. If an equilateral triangle of radius 4 unit is drilled through the center of a right circular cylinder of
radius of base 4 units and height of 6 units, then find and of the resulting.
Solution:
a. Taking the base of the prism to be, as shown shaded in the following figure, we have:

( ) ( x ) Sq. units

( ) Sq. units
Sq. units
Cub. Units
√ √
b. . / ( ) . / . √ / .
√ √ √
√ .

72

Telegram channel https://t.me/johnson201485


c. The radius of cylinder r=2cm and h=3cm
r Sq. cm
r .
.
d. Let the figure the following is represent the given problem.
( )
( )
Fig.Problem for d
Volume of the resulting solid is given as:
Volume of a larger cylinder-volume of smaller cylinder
( ) ( )
e. (  √ )Sq. units (how show it)
(  √ ) units (How show it)
AT & V------- Exercise Fig.Problem for e
Exercise: A cylinder of base area 225cm2 and
height 15cm has the same volume as a cube. What is AT of the cube?

7.2. PYRAMIDS, CONES AND SPHERES


Competencies: At the end of this sub-unit, students will be able to:
 Calculate the surface area of a given pyramid or a cone.
 Calculate the volume of a given pyramid or a cone. Slant height Height
 Calculate the surface area of a given sphere.
 Calculate the volume of a given sphere.
A. Pyramids:
 Pyramid is a solid figure formed when each vertex of a polygon is joined to the same point not in the
plane of the polygon.
 The altitude of a pyramid is the length of the perpendicular from the vertex to the plane containing the
base.
 The slant height of a regular pyramid is the altitude of any of its lateral faces.
 A regular pyramid is a pyramid whose base is a regular polygon and whose altitude pass through
the center of the base and the apex (vertex) of the pyramid equidistant from each vertex of pyramid.

73

Telegram channel https://t.me/johnson201485


 If the base of the pyramid is a triangular region, it is called a tetrahedron.
 Quadrangular pyramid: A pyramid with base a quadrilateral.
Surface area and volume of pyramid:
AB =The area of the base
AL= The area of the triangular lateral faces , is the slant height.

AT = Total surface area the sum of lateral face area and base area

V = Volume of the pyramid is given as

Example:
a. An edge of a right square pyramid is 6cm long, if the length of the slant height is 5cm, then find the area
of its lateral surface, the total surface area and the volume.
b. The volume of a pyramid is . The pyramid has a rectangular base with sides 6 cm by 4 cm. Find the
altitude and lateral surface area of the pyramid if the pyramid has equal lateral edges.
c. Exercise 1: Find the altitude h AL and AT area of a right pyramid with volume of having a rectangular
base of dimensions 3cm by 4cm.
2: The volume a regular square pyramid is If its altitude is 10cm long, find the length of one edge of the
base and its total surface area.
3: If a lateral edge of a regular tetrahedron is x cm, find the measure of Altitude, surface area and volume
4: Show that the volume of a regular square pyramid whose lateral faces are equilateral triangles of side length s, is

Solution: The Square of pyramid is given as follows.


a. ( )

To find the volume we need to find the altitude h.


( ) ( ) ( )

b. The figure represent problem b is given as follows

( )

( ) ( ) √

74

Telegram channel https://t.me/johnson201485


. / √ And

√ √

. / . / ( √ √ ) .

B. Cones:The solid figure formed by joining all points of a circle to a point not on the plane of the
circle is called a cone.
 If the base of cone is circular region, the cone is called circular cone.
 Right circular cone: is a cone with the foot of its altitude at the Centre of the base
v
 Slant height: A line segment from the vertex of a cone to any point on
Altitud Slant
the boundary of the base (circle) is called the slant height. e height(𝑙)
Surface area and volume of a right circular cone: r
 The lateral surface area of a right circular cone is equal to half the product of its slant height and the
circumference of the base. That is,
( ) , where √

 The total surface area (AT) is equal to the sum of the area of the base and the lateral surface area.
That is,

( )

 The volume of a circular cone is equal to one-third of the product of its base area and its altitude.
That is,
, where V denotes the volume, r the radius of the base and h the altitude.

Example:
a. Calculate the total surface area and volume a right circular cone with length of altitude 8cm and
radius 6cm. Solution: h = 8cm, r = 6cm
b. The volume of a right circular cone is . The radius is 4cm.Find the length of the
perpendicular height.
c. Exercise 1: Calculate the total surface area and volume of a circular cone whose altitude and diameter
of base are equally x cm.
2: If the slant height of a circular cone is 13cm and the radius of a base is 12cm, then calculate; AL, AT
and V.

75

Telegram channel https://t.me/johnson201485


Solution:
a. √ √ √
( ) ( ) and ( )

b. ( )

C. Spheres:
A sphere is a closed surface, all points of which are equidistant from a point called the Centre or is a
three-dimensional figure made up of all points a given distance from center.
Surface area and volume of a sphere: r
The surface area (A) and the volume (V) of a sphere of radius r are given by
 Area of the sphere and Volume of the sphere

Surface area ad volume of a hemisphere (Half sphere)


Hemisphere is half of the spheres.
 Area of the hemisphere ( ) r
r
 Volume of the hemisphere . /

Example:
a. The diameter of a sphere is 6cm; find the area and volume of the sphere.
b. The radius of one sphere is twice as long as the radius of another sphere. If the volume of the smaller
sphere is 12 cubic units, then calculate the volume of the larger sphere.
c. If 18cm long wire whose radius of circular thickness is 4cm is melted to form a sphere, find the
surface area and volume of the sphere.
d. Find the surface area and volume a hemispherical metal of radius 12cm.
Exercise 1: A cylindrical container of base radius 8 cm has enough water in it. An iron ball of radius 3
cm is inserted in the cylinder. Assuming that the ball is completely immersed, how high does the
water level rise?
2: If a spherical stone with radius 6ocm is submerged in a cylindrical water tank whose base radius is 2m
then how much is the level of water raised?
Solution:
a. diameter ; x

76

Telegram channel https://t.me/johnson201485


Volume ( ) ( )

b. Let R be the radius of the larger sphere and r be the radius of the smaller sphere

√ Unit and √ Unit

( )

(√ )

c. To change in shape may change the surface area but doesn„t change the volume.
Then let radius of the sphere, A = area of the sphere radius of cylinder, h = altitude of cylinder
volume of cylinder, volume of sphere

( )

( ) ( )

( ) And

d. The radius of hemisphere is 12cm. Then


( ) And ( )

Exercise: If the radius of a sphere is doubled, what effect does this have on its volume and its surface
area?

7.3. FRUSTUMS OF PYRAMIDS AND CONES


At the end of the sub-unit, students will be able to:
 Define frustums of a pyramid and of a cone.
 Calculate the surface areas of frustums of pyramids or of cones.
 Calculate the volumes of frustum of pyramids or of cones.

77

Telegram channel https://t.me/johnson201485


A. Frustums of pyramids: If a pyramid or a cone is cut by a plane parallel to the base, the intersection of
the plane and the pyramid (or the cone) is called a horizontal cross-section of the pyramid (or the
cone).
 A frustum of a pyramid is a part of the pyramid included between the base and a plane parallel to the
base.
 The lateral faces of a frustum of a pyramid are trapeziums.
 The lateral faces of a frustum of a regular pyramid are congruent isosceles trapeziums.
 The slant height of a frustum of a regular pyramid is the altitude of any one of the lateral faces.
 The lateral surface area of a frustum of a pyramid is the sum of the areas of the lateral faces.

 In any pyramid, the ratio of the area of a cross-section to the area of the base is where h is the

altitude of the pyramid and k is the distance from the vertex to the plane of the cross-section.

In both figures
And
So in both conditions we have the above statement;

The ratio of the area of a cross-section to the area of the base is

Where h is the altitude of the pyramid and k is the distance from the
Vertex to the plane of the cross-section.
Surface area and volume of frustum of pyramids:
 The lateral surface area (AL) of a frustum of a regular pyramid is equal to half the product of the slant height ( )
and the sum of the perimeter (P) of the lower base and the perimeter (P') of the upper base. That is,

( ) 𝐴𝑏
 The total surface area of frustum of regular pyramid is the sum of
Lateral face area and the two upper and lower bases. 𝐴𝑏

( )

78

Telegram channel https://t.me/johnson201485


 ( √ ) Where A is the lower base area, A' the upper base area and h' is the

height of a frustum of a cone or pyramid.


Example:
a. The area of the base of a pyramid is . The altitude of the pyramid is 12 cm. What is the area of
a horizontal cross-section 4 cm from the vertex?
b. The lower base of a frustum of a regular pyramid is a square of side 6 cm, and the upper base has side
length 3 cm. If the slant height is 8 cm, find: its lateral surface area, total surface area and volume.
c. A frustum of a regular square pyramid whose lateral faces are equilateral triangles of side 10 cm has
altitude 5 cm. Calculate the volume of the frustum.
Solution:
a. Let be the area of the cross-section, and the base area.
V
k

10
b. ( ) ( ( )) h‟=5 h
h
D
C

c. √ A O

10 B
( √ ) √
√ √ (√ )
(√ )
. / . / ( √ ) √

. √ / 4 √ √ √ 5

B. Frustum of cone: A frustum of a cone is a part of the cone included between the base and a
horizontal cross-section made by a plane parallel to the base.
Surface area and volume of frustum of cone.
𝑟
 For a frustum of a right circular cone with altitude h and slant height , if the
circumferences of the bases are c and c', then the lateral surface area of
the frustum is given by 𝑟
( ) ( r r) (r r)

 The total surface area is the sum of the lateral surface area and the area of the two bases.

79

Telegram channel https://t.me/johnson201485


(r r) r r (r r) (r r )

 (r r rr )

Where r is the radius of the bigger (the lower base of the frustum) cone and r' is the radius of the smaller
cone (upper base of the frustum).
Example:
a. Calculate the surface area and volume of frustum of cone with length of bases radii 7cm and 4cm that
has a slant height 5cm.
b. Exercise. A frustum of height 12 cm is formed from a right circular cone of height 16 cm and base
radius 8 cm. Calculate the total surface area of the frustum and volume of the frustum.
Solution: r r n ( ) ( )
Total surface area of frustum of cone is given as follows
(r r) (r r ) ( ) ( )
And

(r r rr ) ( ) ( )

80

Telegram channel https://t.me/johnson201485


UNIT EIGHT FURTHER ON RELATION AND FUNCTIONS
Unit Outcomes:

After completing this unit, you should be able to:

 know specific facts about relations.


 know additional concepts and facts about functions.
 understand methods and principles in composing functions.
Main Contents:
8.1. REVISION ON RELATIONS.
8.2. SOME ADDITIONAL TYPES OF FUNCTIONS.
8.3. CLASSIFICATION OF FUNCTIONS.
8.4. COMPOSITION OF FUNCTIONS.
8.5. INVERSE FUNCTIONS AND THEIR GRAPHS.

81

Telegram channel https://t.me/johnson201485


8.1.REVISION ON RELATIONS
8.1.1. Inverse of a Relation
Definition 8.1: Let A and B be non-empty sets. A relation R from A to B is any subset of A  B . In
other words, R is a relation from A to B if and only if R  A B .
Definition 8.2: Let R be a relation from a set A to a set B . Then:

i. 
Domain of R  x  A:  x, y  belongs to R for some y  B  A . 
ii. 
Range of R  x  B:  x, y  belongs to R for some y  A  B . 
NOTE: If R is a relation from A to B, then:

i. Domain of R  A ii. Range of R  B


NOTE:
1. Let A be a set. A relation from A to A, simply read as a „relation on A‟, is any subset of A  A .
2. For any two sets A and B, empty set  is a relation from A to B. This is because  A  B .
Example 1: Let A  2, 5 and B  1, 3, 5 . Then:

a. R1   2, 1 ,  2, 3 is a relation from A to B. Because, R1  A×B .

Domain of R1  2  A and range of R1  1, 3  B .

b. R 2    A  B is a relation from A to B, and also from B to A.

c. R 3   2, 5 , 5, 3 , 5,5 is a relation from A to B. Because, R3  A  B .

Domain of R 3  2, 5  A and range of R 3  3, 5  B .

d. R 4   x, y  : x  B, y  A and x - y  0

 1, 2 , 1, 5 , 3, 5 is a relation from B to A. Because, R 4  B  A

Domain of R 4  1, 3  B and range of R 4  2, 5  A .

Example 2: Let A  1, 0, 2, 5 and let R   x, y  : x, y  A and y 2  x 2 is odd . List the elements of R

and give the domain and range of R.


Solution:  1  0 2 ,  1  2 2 , 0 2   1 , 2 2   1 , 0 2  52 , 52  0 2 , 22  52 and 52  22 are
2 2 2 2
odd.

Hence, R   1, 0 ,  0, 1 ,  1, 2 ,  2, 1 ,  0, 5 , 5, 0  ,  2, 5 , 5, 2   A  A is a relation on A.

Domain of R = Range of R  A  A

82

Telegram channel https://t.me/johnson201485


Definition8.3: Let R be a relation from A to B. The inverse of R, denoted by R 1 , is a relation from B to A,
given by R   b, a  :  a, b   R
1

From the above definition, we have that  x, y   R 1 if and only if  y, x   R .

For example, let A= 1, 1, 2 and B= 0, 1, 5, 6 .

Then R  1, 5 ,  1, 6 ,  1, 0  , 1, 1  A  B is a relation from A to B.

Domain of R  1, 1 and Range of R  0, 1, 5, 6 .

By definition, the inverse R 1 of R is given by R 1   y, x  : x, y   R

 R 1   5, 1 ,  6, 1 ,  0, 1 , 1, 1  B  A is a relation from B to A.

Domain of R 1  0, 1, 5, 6  Range of R and Range of R 1  1, 1  Domain of R

Moreover,  R 1    5, 1 ,  6,  1 ,  0,  1 , 1, 1  1, 5 ,  1, 6 ,  1, 0  , 1, 1  R


1 1

NOTE: From what we did above, you should have concluded that:
For any relation R ,

i. Domain of R 1  Range of R and Range of R 1  Domain of R .


ii. R 
1 1
R.

 When a relation R is described in set builder form as : R   x, y  : P  x, y 


Variable part property part

Then to determine the inverse R 1 of R we should either interchange the variables in the „variable part‟,
or in the „property part‟, but not both, and then write the rule in the simplest form, if necessary.

Example 3: Consider a relation R=  x, y  : y  2 x  1 . Determine R 1 .

Solution: To determine R 1 , we may use the following technique:

Given R :  x, y  ; P  x, y 

83

Telegram channel https://t.me/johnson201485


 R :  x, y  ; y  2x  1

 R 1 :  y, x  ; y  2x  1  interchanging x and y in the variable part or

 R 1 :  x, y  ; x  2 y  1  interchanging x and y in the property part (this is recommended)

 R 1 :  x, y  ; x 1  2 y  to write x  2 y  1 in its equivalence or

x 1
 R 1 :  x, y  ; y  .
2

x 1
of R is given by: R :  y, x  ; y  2 x  1 or R 1 :  x, y  ; y 
1
Therefore, the inverse .
2

R 1 can be written in any of its equivalent form such as

R 1   x, y  : 2 y  x 1 or R 1   x, y  : x  2 y  1

1
Or R   x, y  : x 1  2 y or R 1
  x, y  : 2 y  x  1 or any other equivalent form

Example 4: Determine the inverses each of the following relations.

a. R   x, y  : y is the father of x . d. R   x, y  : y  x 2  4

b. R   x, y  : x  5 . e. R   x, y  : 2  y  1, x  
c. R   x, y  : x  y  1 . f. R   x, y  : y  x and 2 x  y  3

Solution:

a. R   x, y  : y is the father of x
1
Then R   x, y  : x is the father of y
Here, domain of R  x : x has a father y  range of R and
1

range of R   y : y is a father y  domain of R 1 .

b. R   x, y  : x  5

84

Telegram channel https://t.me/johnson201485


1
Then R   x, y  : y  5
Here, domain of R  5  range of R and range of
1
* is real number+ domainof .

c. R   x, y  : x  y  1
 R 1   x, y  : y  x  1   x, y  : x  y  1  R , because x  y  y  x .

Therefore, R is its own inverse; since R 1  R .

d. R   x, y  : y  x 2  4

 R 1   x, y  : x  y 2  4   x, y  : y 2  x  4 or R
1
  x, y  : y  
x  4 or y   x  4 .

e. *( ) ∈ +.

f. R 1   x, y  : x  y and 2 y  x  3 .
8.1.2. Graphs of Inverse Relations
First, let us compare graphs of R and R 1 .For instance consider the following examples.


Example 5: Consider the relation R   3, 1 ,  4, 2 ,  6, 3 ,  5, 1 . 
 
i. Its inverse R   1, 3 ,  2, 4 , 3, 6 , 1,  5 .
1

ii. On a piece of squared paper, sketch the line y  x .

Note that reflecting a coordinate plane along the line with equation y  x , maps any point P  a, b  into

Q  b, a  . That is P  a, b  and Q  b, a  are mirror images one to the other along the line y  x . (See figure
10.1, below)

iii. Sketch R and R 1 on the paper, and mark points of


R by * and points of R 1 by Δ. (See figure 10.1 below)

Figure 8.2. Graphs of R and R-1

85

Telegram channel https://t.me/johnson201485


iv. When you fold the graph in figure 8.1 above, along the line y  x , you can see that the points on the

graph of R and the points on the graph of R 1 coincide.


Example 6: Let R   x, y  : x  y 2  . Draw the graphs of R and R 1 using the same coordinate axes.

Solution: The inverse of R is given by R   x, y  : x  y 2  .

The graphs of R and R–1 using the same coordinate axes are given below, figure 8.3.
y=x
When you fold the graph in figure 8.3 below,
y=x2
along the line y = x, you can see that x=y
2

the graph of R : and the graph

of R-1 : coincide.
Figure 8.3. Graphs of R and R-1
Notice that the boundary parabolas and

intersect at (0, 0) and (1, 1), which are also points

on the reflecting line y = x.


Thus, from the above two examples you should conclude the following.
 The graphs of R and R–1 are mirror images of each other on the line y = x. This means, if you
reflect the graph of R in the line y = x, you get the graph of R–1 and vice versa.
Example 7: For each of the following relations, draw the graph of R and its inverse R –1 using the same
coordinate axes.
a) *( ) + b) *( ) +

Solution: a) *( ) + then R:
To draw the graph of R, first sketch the boundary line :
x-intercept:  Then x-intercept is (1, 0)
y-intercept:  . The y-intercept is (0, 1)
Thus, the boundary line passes through points (1, 0) and (0, 1).
Take test point P(0, 0) which is below the boundary line .
; P (0, 0)   , which is true.
Therefore, the graph of R is the region which lies below or on the boundary line.

86

Telegram channel https://t.me/johnson201485


To find the inverse of *( ) +
 R–1: which is equivalent to , since
Therefore, the inverse of R is given by *( ) + , which is the same as R itself.
This shows that R is its own inverse.
Thus, the graphs of R and its inverse R–1 coincides. (See figure 10.4 below)

Figure 8.4 Figure 8.5

b) *( ) +
Then, the inverse of R is

*( ) + Or *( ) +
You can draw the graph of as previously done. (See figure 10.5 above)

8.2. SOME ADDITIONAL TYPES OF FUNCTIONS


8.2.1. Revision on Functions
Definition 8.4. A function is a relation in which no two distinct ordered pairs have the same first
coordinate and different second coordinate. That is if f is a function with domain A and range a subset of

B, we write f as  or →
 If  is given by a rule that maps x from A to y in B, then we write ( ).
Notation: If x is an element in the domain of a function , then the element in the range that is associated
with x is denoted by ( ) and is called the image of under the function .
This means *( ) ( )+
Example 1: Suppose  is the function that gives 5x – 1for any ∈ . The possible ways of
writing this function are

– ( ) – – → –
Example 2: Consider the relation R = {(1, 2), (7, 8), (4, 3), (7, 6)} Since 7 is paired with both 8 and 6,
and 8  6, the relation R is NOT a function.

87

Telegram channel https://t.me/johnson201485


Example 3: Consider the function ( ) | | +. Here the rule that define the function F is
| | which can be written as ( ) | | .
Domain of and range of * ∈ + , )
Example 4: ( ) √ is a function.
Domain of ( ) ( √ )
* ∈ + , -
Testing for Functions
Algebraic Test: If a relation is given as an equation, and the substitution of any value for results in one
and only one value of , then the equation defines function.
Example 5:
a) is a function; since for any value of , there is only one value of .
b) is not function; since for we have
Vertical Line Test:A set of points in the Cartesian plane is the graph of a function, if and only if no
vertical line intersects the set in more than one points.

For example, consider the following graphs of relation given below.

a) b) c)

Figure 8.6a Figure 8.6b Figure 8.6c

The graph in figure 8.6a does not define function, because a vertical line intersect the graph twice.
Whereas, The graphs in figures 8.6a and 8.6b do define functions, because any vertical line intersect the
graphs atmost once.

Even and Odd Functions

Note: A set S is said to be symmetric if and only if for any ∈ we have ∈

88

Telegram channel https://t.me/johnson201485


Example 6:

a) The sets , , , * +, * +, , (-5, 5), {-2, -1, 0, 1, 2}, {0}, [-5, 5], . /∪

. /, , - . / are examples of symmetric sets.

b) The sets , * +, * +, [-5, 5), { -1, 0, 1, 2}, { 5}, [0, 5], , - ( - are not
symmetric sets.
Definition 8.5. Let f : A  B be a function and let the domain A be symmetric set. Then is said to be
i) odd function if and only if for any ∈ ( ) ( ).
ii) even function if and only if for any ∈ ( ) ( ).
The evenness or oddness of a function is called its parity.
Example 7:
i) A constant function ( ) is even function, since ( ) ( ) for all ∈ .
ii) f (x) = x3 is odd function, since dom. of and f (–x) = (–x)3 = − x3 = − f (x).
iii) The function f (x) = x2 is even, since f (–x) = (–x) 2 = x2 = f (x).
iv) The function f (x) = x + 1 is neither even nor odd, since
f (–x) = –x + 1  –(x + 1) = – f (x) and f (–x) = – x + 1  x + 1 = f (x).
v) ( ) s is an even function, since ( ) s( ) s ( )
vi) ( ) sin is an odd function, since ( ) sin( ) s ( )
vii) The exponential function ( ) is neither even nor odd, since

( ) . / ( ) and ( ) . / ( )

viii) ( ) for if neither even nor odd, since the given domain , ) is not symmetric set.

Note:
ii. The trigonometric functions ( ) sin , ( ) s , ( ) t n and ( ) t are odd
functions.
iii. The trigonometric functions ( ) s , and ( ) s are even functions.

Remark:

i. If a function is an odd function, then its graph is symmetric w.r.t the y-axis.
ii. If a function is an even function, then its graph is symmetric w.r.t the origin.
NOTE: Using the definition of odd and even functions, we can justify the following:

89

Telegram channel https://t.me/johnson201485


even
a. even  even  even and  even .
even
even odd
b. even  odd  odd and  odd  .
odd even
odd
c. odd  odd  even and  even .
odd
Exponential and Logarithmic Functions
i. A function ( ) given by ( ) is called an exponential
function.
ii. A function ( ) given by ( ) is called a logarithmic
function.

Example 8:
a) The function ( ) and ( ) are an exponential function.
b) The function ( ) ( ) and ( ) ( ) are logarithmic

function.
8.2.2. Power Functions

Definition 10.6: A power function is a function which can be written in the form ( ) , where r
is a real number and a  ℝ, is a fixed number.

Note: Don't confuse power functions with exponential functions.


 Exponential Function: (a fixed base is raised to a variable exponent)
 Power Function: (variable base is raised to a fixed exponent)
Example 9:

a) ( ) , , ( ) , are power functions


b) , , ( ) , are power functions


The Behaviour of a Power Function


1. A power function ( ) , ∈ when „r‟ is integer
i. A power function ( ) , ∈ , when „r‟ is odd positive integer
For instance, consider a power function ( ) .
Here  and is odd positive integer.
 Domain of = Range of =

90

Telegram channel https://t.me/johnson201485


 ( ) ( ) ( ). Hence, ( ) is an odd function.
 Therefore, the graph of is symmetric w.r.t the origin.(i.e for every point (x, y) on the graph, (-x, -y)
is also on the graph).
 To understand the nature of the graph, let us calculate some of its functional values, as shown in the
following table.
2 1 0 1 2

( ) –32 –4 0 4 32

 Then the graph of is given below: Figure 8.7a. Graph of 𝒇(𝒙) 𝟒𝒙𝟑

 From the graph above, you can see that the graph is concave downward on the interval ( ), and
concave upward on the interval ( ). Point (0, 0) is the point on the graph of where the
concavity of the graph changes. The point (0, 0), where the graph is smooth, and changes its shape
from concave upward to concave downward, is called an inflection point.
ii. A power function ( ) , ∈ , when „r‟ is even positive integer
For instance, consider a power function ( ) .
Here  and is even positive integer. Then Domain of = and Range of = [0, )
 ( ) ( ) ( ). Hence, ( ) is an even function.
 Therefore, the graph of is symmetric w.r.t the y-axis. (i.e for every point (x, y) on the graph, (-x, y)
is also on the graph).
 To understand the nature of the graph, let us calculate some of its functional values, as shown in the
following table.
 Then the graph of is given below:

2 1 0 1 2

( ) 16 4 0 4 16

Figure 8.7b. Graph of 𝒇(𝒙) 𝟒𝒙𝟐

 Note that the graph given above is open upward for all for all ∈ . Hence, the graph has no
inflection point.
iii. A power function ( ) , ∈ , when „r‟ is odd negative integer

91

Telegram channel https://t.me/johnson201485


For instance, consider a power function ( ) . Here  and is odd negative
integer. Then Domain of = * + and Range of = * +

 ( ) ( ) . / . / ( ), for all ∈ .

Hence, ( ) is an odd function.


 Therefore, the graph of is symmetric w.r.t the origin.(i.e for every point (x, y) on the graph, (-x, y) is
also on the graph).
 To understand the nature of the graph, let us calculate 2 1 0 1 2
some of its functional values, as shown in the following
( ) 2
table.
 Then From the graph of ( ) given below, you can see that the graph is concave downward
on the interval( ), and concave upward on the interval( ).
 The concavity of the graph changes at point (0, 0). But (0, 0) is not the point on the graph of
( ) . Therefore, the graph of f has no inflection point.(that is point (0, 0) is not inflection
point)

𝟑
Figure 8.8a. Graph of 𝒇(𝒙) 𝟐𝒙

iv. A power function ( ) , ∈ , when „r‟ is even negative integer


For instance, consider a power function ( ) .
 Here  and , which is even negative integer.
 Domain of = * + and Range of = * +

 ( ) ( ) . / . / ( ), for all ∈ .

Hence, ( ) is an even function.


 Therefore, the graph of is symmetric w.r.t the y-axis.
 To understand the nature of the graph, let us
calculate some of its functional values, as shown
in the following table.

𝟐
Figure 8.8b. Graph of 𝒇(𝒙) 𝟐𝒙
92

Telegram channel https://t.me/johnson201485


2 1 0 1 2

( ) 1 1

 Then, the graph of is sketched as given here.


 The graph of is concave upward throughout its domain, * +.
 Thus, the graph of f on figure 1.8b has no inflection point.
1. A power function ( ) , ∈ , when r is a real number of the form
We now consider the behaviour of a power function ( ) , ∈ , when r is a real number of the
form , in its lowest term , where m and n are integers, with n  0.

i. When and is odd natural number (i.e ( ) , ∈ is odd)

Using the definition of radical, ( ) ∈ and , can be written as


th
( ) √ , read as „the n root of x ‟.

For instance, consider a power function ( ) √ .


 Here  and (odd natural, n > 1).
 √ is defined for all ∈ , and √ ∈ for all real number .
Therefore, Domain of and Range of =

 ( ) ( ) √ √ √ √ ( ) ( ), for all ∈

Hence, ( ) √ is an odd function.


 Therefore, the graph of is symmetric w.r.t the origin.
 To understand the nature of the graph, let us calculate some of its functional values, as shown in
the following table.

8 1 0 1 8

( ) √ 0 1

 Using the above values in the table,


the graph of can be sketched as follow:

 From the graph of ( ) given above, Figure 8.9a. Graph of 𝒇(𝒙) 𝑥

93

Telegram channel https://t.me/johnson201485


you can see that the graph is concave upward on the interval ( ), and concave downward on the
interval ( ).
 The concavity of the graph changes at point (0, 0), and (0, 0) is the point on the graph of .
Hence, point (0, 0) is the inflection point of the graph of .

Note: In general, all functions of the form ( ) √ , where n is an odd natural number, have
similar behaviour. They all pass through (−1, −1) and (1, 1). They are also increasing.

𝑓(𝑥) 𝑥𝑛

𝟏
Figure 8.9b. Graph of 𝒇(𝒙) 𝒙 𝒏 , for an odd natural number
n.
ii. ( ) , where is odd natural number

( ) ∈ and , can be written as ( ) .


For instance, consider a power function ( ) ..


 Domain of * + and Range of = * +

 ( ) ( ) ( ) ( ) ( ), for any odd n, and any ∈ * +


 Hence, in general, ( ) is an odd function, for odd n, and any ∈ * +

Thus, ( ) is an odd function.

 Therefore, the graph of ( ) is symmetric w.r.t the origin.


 To understand the nature of the 8 1 0 1 8
graph, let us calculate some of its
( ) 0 1
functional values, as shown in the √
following table.

94

Telegram channel https://t.me/johnson201485


 Using the above values in the table, the graph of can be sketched as follow:

𝑓(𝑥) 𝑥 𝑓(𝑥) 𝑥 𝑛

𝟏
Figure 8.10b. Graph of 𝒇(𝒙) 𝒙 𝒏 ,
Figure 8.10a. Graph of 𝒇(𝒙) 𝑥
where n is odd natural number.

Note: In general, all functions of the form ( ) , where n is an odd natural number, have similar
behaviour, as shown in figure 1.10b given above. From the graph we can see that
 They all pass through points (−1, −1) and (1, 1).
 They are decreasing on the intervals ( ) and on ( ).
 Here don‟t say „They are decreasing on the interval ( )∪( ).

iii. ( ) , where is even natural number

For instance, the power functions ( ) √ , ( ) √ , , and so


on are in this category.
 Domain of * ∈ + , ) and Range of =, )

 For even natural number n, ( ) ( ) ( ) ( ) √ ( ) ( ), for any


But the domain of , ) is not symmetric set.

Hence, in general, ( ) , for even natural number n, is neither even nor odd function. Thus, power

functions ( ) √ , ( ) √ , , are neither even nor odd function. Therefore,


the graph of has no symmetry.

𝟏
Figure 8.11a. Graph of 𝒇(𝒙) 𝒙𝟐 . 𝟏
Figure 8.11b. Graph of 𝒇(𝒙) 𝒙 𝒏 ,where n is even
natural number.

95

Telegram channel https://t.me/johnson201485


( ) , where is even natural n

For instance, the power functions ( ) ; ( ) ; , and so on


√ √ √

are in this category.


 Domain of * ∈ + ( ) and Range of =( )
 They are decreasing functions.

 For even natural number n, ( ) ( ) ( ) ( ) ( ), for any


But, the domain of f = ( ) , is not symmetric set. That is if ∈ , then ∉ .

Hence, in general, ( ) is neither even nor odd function.


 Thus, the graph of has no symmetry (see figure 1.12 below).
 In general, the graph of can be sketched as follow:

𝟏
Figure 8.12. Graph of 𝒇(𝒙) 𝒙 𝒏 , for an even natural
number
The following figures n. you some of the various possible graphs of power functions with rational
give
exponents .

Case 1. ( ) , where is even and is odd natural numbers for

 The power functions ( ) √ , ( ) √ , ( ) √ , and so, on are


in this category.
iv. For instance, consider the power function ( ) √ . Here
 Domain of ( ) and Range of =, ) * ∈ y +
 ( ) ( ) √( ) √ ( ), for all ∈
Hence, ( ) is an even function.
 Therefore, the graph of ( ) is symmetric w.r.t the y-axis (see figure 8.13a below).
 It is neither increasing nor decreasing function. But, is decreasing on ( - and increasing on
, ).

96

Telegram channel https://t.me/johnson201485


𝑓(𝑥) 𝑥

𝒎
Figure 8.13b. Graph of 𝒇(𝒙) 𝒙 𝒏 , for
𝟐
Figure 8.13a. Graph of 𝒇(𝒙) 𝒙 𝟓. even m, odd n and m < n.

In general, the graph of a power function ( ) , where is even and is odd


natural numbers, and is as sketched in figure 10.13b above.
In the above graphs, (0, 0) is the pointed end where two curves meet. That is the graph form sharp corner
at the point (0, 0). In this case point (0, 0) is called a cusp.

Case 2. ( ) , where is odd and is even natural numbers.

 The power functions ( ) √ , √ , ( ) √ ( ) √ ,


and so on are in this category.
 In general, for all such power functions,
 Domain of , ) * + and Range of =, ) * ∈ y +
 Since, the domain of is not symmetric set, is neither even nor odd function.
 Therefore, its graph is has no symmetry. (see figure 10.14 below). Thus, it is increasing function
throughout its domain.

𝒎
Figure 8.14. Graph of 𝒇(𝒙) 𝒙 𝒏 , for odd m and even n.

Absolute Value (Modulus) Function: : The absolute value of a real number x is the distance of the point
P with coordinate x on a number line from the origin O.
Definition: For any real number , the absolute value or modulus of , denoted by | |, is defined by

a, if a  0
a  .
 a, if a  0

97

Telegram channel https://t.me/johnson201485


Some properties of the absolute value

1. x  0 , for any ∈ . x x
4.  , for any x and ∈ .
y y
2.  x  x , for any ∈ .
5. x  y  x  y , for any x and ∈ .
3. xy  x y , for any x and ∈ .
6. x  x and  x  x , for any ∈ .

7. x  a if and only if x   a , provided that a  0 . In the case a  0 , x  a has no solution.


Absolute Value (Modulus) Function
Definition: The absolute value or modulus function is given by f  x   x , for ∈ .

 In the usual manner, you can draw the graph of f  x   x as follows:

For x  0, y  f  x   x  x and for x  0, y  f  x   x   x .

x 3 2 1 0 1 2 3

f  x  x 3 2 1 0 1 2 3
y  x 0 yx Figure 8.15. Graph of 𝒇(𝒙) | 𝒙 |.

Properties of modulus function ( ) | |


1. Domain of and range of * ∈ + ).
2. f is continuous function, since the graph has no hole or break in it.
3. The graph of f makes a sharp corner or a cusp at P  0, 0 .
4. f   x    x  x  f  x  , for each ∈ . Therefore, f is an even function.
5. The graph of f is symmetrical with respect to the y  axis.
8.2.3. Signum Function
Definition: The signum function, denoted by sgn x and read as signum x, is a piece-wise function, defined

1, for x  0

by y  f  x   sgn x  0, for x  0
1, for x  0

Example 10: For signum function f  x   sgn x , find

 5
a. f  2 b. f   
 3

c. f 4 2  2 8 

98

Telegram channel https://t.me/johnson201485


Solution:

a. f  2  sgn 2  1, since 2  1.

 5  5 5
b. f     sgn     1, since   0 .
 3  3 3

c.    
f 4 2  2 8  sgn 4 2  2 8  0, since 4 2  2 8  0 .

NOTE: The signum function can be described in terms of modulus function as follow:

x
x  x  1, for x  0
 , if x  0 x 
sgn x   x , because  undefined, for x  0 .
0, if x  0 x 
 x
  1, if x  0
 x

Graph of Signum Function: The table of values and the graph of f  x   sgn x are as given below.

x 3 2 1 0 1 2 3
sgn x 1 1 1 0 1 1 1

Properties of Signum Function ( )

For signum function f  x   sgn x ;


Figure 8.16. Graph of 𝒇(𝒙) 𝒔𝒈𝒏 𝒙.
f  1, 0, 1
1. Domain of ( ) and Range of .
2. It is piece - wise function.
3. is not continuous function, since the graph has hole or break in it.
4. f   x   sgn   x    sgn x   f  x  for each ∈ . Therefore, f is an odd function.
5. The graph of is symmetrical with respect to the origin, since it is odd function.
Example 11: Sketch the graph and give the domain and range of each of the following functions.

a. f  x   x  sgn x b. f  x   x sgn x
Solution:
a. f  x   x  sgn x .
To sketch the graph of , let us redefine as follow:
 x  1, for x  0

f  x   x  sgn x  0  0  0, for x  0
 x  1, for x  0

99
x 4 3 2 1 -2/3 0 1 3/2 2 3 4
sgn x 1 1 1 1 1 0 1 1 1 1 1
x  sgn x 5 4 3 2 -5/3 0 2 5/2 3 4 5
in ℝ n ( )∪* +∪( )

b. f  x   x sgn x
x 4 3 2 1  2 0 1 3 2 3 4
3 2 𝑓(𝑥) 𝑥 𝑠𝑔𝑛 𝑥

sgn x 1 1 1 1 1 0 1 1 1 1 1
x sgn x  x 4 3 2 1 2 0 1 3 2 3 4
3 2

in ℝ n ( )

8.2.4. Greatest Integer (Floor) and Ceiling Functions

Definition: The greatest integer function, denoted by f  x    x  is defined as the greatest integer .

NOTE:
1. The greatest integer is also called the floor of x .
2. The greatest integer is the integer that is immediately to the left of x ( or x itself, if x is an integer).

Example 12: Find  x  when:

b. x  2.56 c. x  0.6 e. x  2.4


c. x  7.2143 d. x  0.6 f. x  3
Solution:
a. 2.56 is between 2 and 3 on number line.

What is the largest among the integers that is less than or equal to 2.56 ? You can see that it is 2.
Thus, 2.56  2
b.
c. Similarly, x  7.2143 is between 7 and 8 .

 7.2143  7

100
d. 0.6 is between 1 and 0 on number line.
NOTE: The greatest integer function which can be written in the form of
Thus domain of .and range of .

Example 13: Let f  x    x  . Complete the following table.

x 2  x  1 1  x  0 0  x 1 1 x  2 2 x3 3 x  4

f  x 3

Solution:
x 2  x  1 1  x  0 0  x 1 1 x  2 2 x3 3 x  4

f  x 2 1 0 1 2 3

The graph of the greatest integer function y   x  lies on or below the line y  x so it provides an integer

floor for x .

The Least Integer (Ceiling) Function:

The Least integer (ceiling) function, denoted by f  x    x  is defined as the smallest integer .

Definition: The ceiling function also has type of Thus domain of .and range of .

Example 14: Find  x  when:

a. x  2.8 b. x  2.8 c. x  12
Solution:
a. 2.8 is between 2 and 3 on number line.

b. 2.8  2 since x  2.8 is between 3 and 2 .  2.8  3

c. 12  12 why?

101
 1  5  9
Example 16: Solve the equation  x     x     x    3 .
 2  2  2
1
Solution: We can solve this equation more easily by using the substitution x  m.
2
Therefore, we have the equation: m  m  2  m  4  3 .
Now using the identity  x  n   x   n , we have:

m  m  2  m  4  3


 3 m  9
 m  3
1
 3  m  4 but m  x 
2
1
3 x 4
2
1 1
 3  x  4
2 2
 3.5  x  4.5
 x  3.5, 4.5

8.3. CLASSIFICATION OF FUNCTIONS


8.3.1. One-to-One Functions
Definition: A function f : A  B is said to be a one-to-one (an injection), if and only if, each element
of the range is paired with exactly one element of the domain, i.e., f  x1   f  x2   x1  x2 for any x1
and x2  A .
NOTE: The above definition is the same as saying x1  x2  f  x1   f  x2  .

Example 1: Show that defined by f  x   5x  2 is a one-to-one function.


Solution: Let ∈ such that f  x1   f  x2  . We need to show that x1  x2 .
f  x1   f  x2   5x1  2  5x2  2
 5x1  2  2  5x2  2  2
 5x1  5x2  x1  x2
The Horizontal Line Test: If f is a one-to-one function, no horizontal line intersects the graph of f more than
once. Conversely, if the graph of a function has this property, then the function is one-to-one.
Using the horizontal line test:
𝒚 𝟓𝒙 𝟐

Horizontal line
Crossing
point

102
Therefore f  x   5x  2 is a one-to-one function.

defined by f  x   x is not one-to-one function.


2
Example 2: Show that

Solution: Take x1  2 and x1  2 .

Obviously, x1  x2 i.e. 2  2 But f  x1   f  x2   2  4   2  .


2 2

This means there are numbers ∈ for which x1  x2  f  x1   f  x2  does not hold.

Using the horizontal line test: 𝒚 𝒙𝟐


Horizontal line
Crossing Crossing
point point

A horizontal line crosses the graph of y  x 2 at two points. Thus, f is not a one-to-one function.
Example 3: Which of the following are one-to-one functions?
a. F   x, y  : y is the father of x . c. G   x, y  : x is a dog and y is its nose .
b. H   x, y  : y  x  2 
Solution: Only G is one-to-one. Why?
8.3.2. Onto Functions
Definition: A function f : A  B is onto (a surjection), if and only if, Range of f  B .

NOTE: Range of f  B means for any y  B there is x A such that y  f  x  .

Example 4: Let f be defined the following Venn diagram shown in a below.


Range of f  B . Therefore, f is onto.

a. b.

In figure b above, Range of f  1, 2, 4  Range of f  B . Thus, f is not onto.


Example 5:
be f  x   x .
2
a. Let
Take y  4 . Since for any ∈ . Thus f is not onto.

103
) be given by f  x   x .
2
b. Let ,

Take y  0,  . Since y  0 , for all ∈ , ). Thus, x2  y has a solution. Indeed, if x  y ,

then, f  x   f  y y
2
 y . Thus, f is onto.

Example 6: Show that , defined by f  x   x  1 is one-to-one as well as onto.


Solution:
i. To show f is one-to-one:
Let ∈ such that f  x1   f  x2  . We need to show that x1  x2 .
f  x1   f  x2   x1  1  x2  1
 x1  1 1  x2  1 1
 x1  x2 Therefore, f is one-to one.
ii. To show that f is onto:
Let ∈ be arbitrary. We need to show that it has a pre-image x in satisfying f  x   m . But if
we take x  m 1 , then f  x   f  m 1  m  1  1  m . Since ∈ is arbitrary, this proves that
every real number has a pre-image in . That is the range of f is . Hence f is onto.
Definition: A function f : A  B is one-to-one correspondence (a bijection), if and only if, f is one-
to-one and onto.

a. The graph of f  x   x passes the horizontal line test. Thus, f is one-to-one. The range of f  x   x
5 5

is . So f is onto.
Therefore f is a one-to-one correspondence.
3x1  1 3x2  1
b. f  x1   f  x2     3x1  3x2
5 5
 x1  x2 for any ∈ is n t n .
5 y 1
For any ∈  f is onto .
, , gives x 
3
Therefore f is a one-to-one correspondence.
c. For ∈ ,
f  x1   f  x2   5x1  5x2
5x1
 1
5x2
 5x1  x2  1  50
 x1  x2  0
 x1  x2

104
Thus, f is one-to-one. But f is not onto because negative numbers cannot be images. For instance, take
y  25 . Since 5x  0 , for every ∈ , it is not possible to have ∈ , for which 5x  25 . Thus,
f is not onto.
Therefore, f is not a one-to-one correspondence
d. The graph of f  x   x passes the horizontal line test. Thus, f is one-to-one. The range of f  x   x
5 5

is . So f is onto. Therefore f is a one-to-one correspondence.


3x1  1 3x2  1
e. f  x1   f  x2     3x1  3x2
5 5
 x1  x2 for any ∈ is n t n .

5 y 1
For any ∈ , , gives x   f is onto . Therefore f is a one-to-one correspondence.
3
5x1
f. For ∈ , f  x1   f  x2   5x1  5x2  1  5x1  x2  1  50
5x2
 x1  x2  0  x1  x2

Thus, f is one-to-one. But f is not onto because negative numbers cannot be images. For instance, take

y  25 . Since 5x  0 , for every ∈ , it is not possible to have ∈ , for which 5x  25 . Thus,
f is not onto.
Therefore, f is not a one-to-one correspondence

g. For x1 , x2 1,  , f  x1   f  x2  ,
  x1  1  1   x2  1  1   x1  1   x2  1
2 2 2 2

 x1  1  x2  1
2
 
2

 x1 1  x2  1

 x1 1  x2  1,  x1  1, x2  1   x1  x2 Thus, f is one-to-one.

Also  x 1  0 , for any x  1,    x  1 1  1


2

 f  x   1, x 1,   Range of f  1,   0, 


Therefore, f is not a one-to-one correspondence

105
8.4. COMPOSITION OF FUNCTIONS
Given two functions, it is sometimes possible to construct a third function based on the two functions. In
this subunit we will illustrate some of the methods of constructing such a third function.
8.4.1. Combination of functions
NOTE: Recall the following.

1. Let f and g be two functions. Then,


a.  f  g  x   f  x   g  x  c.  fg  x   f  x  g  x 
f  f  x
b.  f  g  x   f  x   g  x  d.    x   ; where g  x   0
  x  1   x  1
2
g g  x
2. Domain of  f  g   Domain of  f  g
 Domain of  fg   Domain of f Domain of g
f 
3. Domain of     Domain of f Domain of g  \  x : g  x   0
g

Example 1: Let f  x   x2  1 and g  x   x  1 , find f  g , f  g , f  g and


f
and give the domain of
g
each of the new functions.
Solution:
  f  g  x   f  x   g  x 
 x2  x

Domain of  f  g   Domain of f Domain of g

  f  g  x   f  x   g  x 
  x 2  1   x  1

 x2  x  2

Domain of  f  g   Domain of f Domain of g

  f  g  x   f  x   g  x 
  x 2  1   x  1

 x3  x2  x 1 then Domain of
f  f  x  x2 1
    x    x  1; x  1
g g  x x 1

106
f 
Domain of     Domain of f Domain of g  \  x : g  x   0
g
* + * +
Example 2: If f  1, 3 ,  2, 4 , 5, 7  , 8, 9  and g  1, 4 ,  2, 0 , 3, 5 , 8, 1 , then find

f
f  g, f  g, f  g, and  3 f .
g
Solution:
 f  g  1, 3  4 ,  2, 4  0 , 8, 9  1  1, 7  ,  2, 4 , 8, 10 

 f  g  1, 3  4 ,  2, 4  0 , 8, 9 1  1, 1 ,  2, 4 , 8, 8

 f  g  1, 3 4 ,  2, 4  0 , 8, 9 1  1,121 ,  2, 0 , 8, 9

Here do Domain of  f  g   Domain of  f  g


 Domain of  fg   Domain of f Domain of g  1, 2, 8

f  3   9  
  1,  ,  8,  
g  4   1  

f 
Domain of    1, 8 . Why?
g

 3 f  1,  3 3 ,  2,  3 4 , 5,  3  7  , 8,  3  9   1,  9 ,  2, 12 , 5,  21 , 8,  27 

8.4.2. Composition of functions


Definition: Let f : A  B and g : B  C be functions. Then, then compositions of g by f , denoted
by , is given as ( )( ) ( ( )).
Example 3: Given the following Venn diagram find:
a.  gof  a  b.  gof  d  c.  gof  c 

107
Solution:
a.  gof  a   g  f  a   g 1  e

b.  gof  d   g  f  d   g  3  h

c.  gof  c   g  f  c   g  2  f

NOTE: The domain of composite function gof is the set of all real numbers x such that x is in the domain of f

and f  x  is in the domain of g .

Example 4: Given f  x   x  1 and g  x   3x  2 . Find:


2

a.  gof  4 b.  gog  1

Solution:
a.  gof  4  g  f  4  g  42  1  g 15  3 15   2  47
 gog  1  g  g  1  g 3 1  2  g  1  3 1  2  1

Example 5: Let f  x   x  , find f  f  2  .


1
x
 
 1  1   x2  1   x 
Solution: f  f  x     x       f  f  x     2 
 x  x 1   x   x 1
 x
 22  1   2  29
So, f  f  2      2  
 2   2  1  10
Example 6: Let f  x   5x  4 and  gof  x   7 x  1. Find g  x  .
Solution: Since gof and f linear, try a linear function g  x   ax  b .
g  f  x    g  5x  4  a 5x  4  b  5ax  4a  b
Now, g  f  x    7 x 1  5ax  4a  b  7 x  1
7 28 33
 5a  7  a  and 4a  b  1  b  1  4a  b  1  
5 5 5
7 33
Thus, g  x   x  .
5 5
Example 7: Let f  x   4x  1 and g  x   3x  k . Find the value of k for which  gof  x    fog  x 

108
.Solution:  gof  x    fog  x   g  f  x    f  g  x  
 g  4 x  1  f  3x  k 
 3  4 x  1  k  4 3x  k   1
 12x  3  k  12x  4k  1
2
 k  4k  1  3  k 
3
Example 8: Let f  x   x  9 and g  x   9  x 2 find  gof  x  and its domain.
2

Solution:  gof  x   g  f  x    f     9
2
9  x2  9  x2

 9  x2  9   x2
From this, it might appear that the domain of the composition is the set of all real numbers. This,
however is not true. Because the domain of f is the set of all real numbers and the domain of g is
3  x  3 , then domain of  fog  x  is 3  x  3 .

8.5. INVERSE FUNCTIONS AND THEIR GRAPHS


1. f 1 is a function, if and only if f is one-to-one. If the inverse of f is g , then g is denoted by f 1 . In
this case f is called invertible.
2. If f  x   y , then x  f  y  .
1

3. Domain of f  Range of f 1 and Range of f  Domain of f 1 .


 fog   x   g 1of 1
1
4.
Steps to find the inverse of a function
1. Interchange x and y in the formula of f .
2. Solve for y in terms of x .
3. Write y  f  x  .
1

Definition: The function I : A  A , given by I  x   x is called identity function.

Definition: A function g is said to be an inverse of a function f , if and only if,


g  f  x   I  x   f  g  x  .

Example 1: Find the inverse of each of the following functions:


a. f  x   4 x  3 c. f  x   3 x  8 e. f  x  
2x
, x 1
1 x
x  x 
b. f  x   , x 1 d. f  x   ln   2, x 1 4x 1 2
x 1  x 1  f. f  x   , x
3x  2 3

109
Solution:
 x3  x3
a. f   x, y  : y  4x  3 and f 1   x, y  : x  4 y  3   x, y  :  y   y  f 1  x   .
 4  4

b. f 1  x   x 
y y  y 
x d. f 1  x   x  ln   2, y 1
y 1 y 1  y 1 
 x  y  1  y  y 
 x  ln   2, y 1
 xy  x  y  y 1 
 xy  y  x  y
ln 

2 
 y 1 
 y  x 1  x e ex
, y 1
y
x  ex   2, y  1
y , x 1 y 1
x 1
3y  2
x  ex  , y 1
Therefore f 1  x   y  , x 1 y 1
x 1
c. f 1  x   x  3 y  8  x  3 y  8  ex  y 1  3 y  2

 x3  y  8  yex  3 y  ex  2

 y  f 1  x   x3  8 ex  2
 f 1  x   y 
ex  3
 2x  4 y 1 2
e. f   x, y  : y   and f. f 1  x   x  , y
 1 x  3y  2 3

f 1   x, y  : x 
2y   x  3 y  2  4 y  1
, y  1
 1 y 
 3xy  4 y  2x  1
 f 1   x, y  : x  xy  2 y, y  1
2x 1 4
 f 1  x   y  , x
 x  3x  4
f 1
  x, y  : y   , x  2 3
 x  2
Note that to find range of a given function use: Range of f  Domain of f 1 .
For example, in example above range of in 2 3
Example 2: Show whether or not each of the following pairs of functions are inverses of each other.
3x  5 x4
a. f  x   2 x  5 and g  x   c. f  x   2 x  4 and g  x  
2 2
1 3x  1 x 1
b. f  x  and g  x   d. f  x   2 x  1 and g  x  
x 3 x 2
Solution: Functions f and g are inverses of each other if and only if f  g  x    x  g  f  x   .
 3x  5 
a. f  g  x   2    5  3x
 2 
 f  g  x    3x  x
 f and g are not inverses of each other.

110
 f  g  x   x
f  g  x  
1 1 1
b.   x
3x  1 3x  1  3x 1
3
x x x
 f and g are inverses of each other.
 x4
c. f  g  x   2  4  x  f and g are inverses of each other.
 2 
 x 1 
d. f  g  x   2   1  x  2
 2 
 f  g  x   x  f and g are not inverses of each other.
Example 3: Sketch the graph each of the following function and its inverse on the same coordinate:
a. f  x   2x 1 b. f  x   x , x  0
2
c. f  x    x  2, x  0
2

Solution:
a. f 1  x   x  2 y  1  y  f 1  x  
x 1 b. f 1  x   x
2

c. f 1  x   y   x  2, x  2

111
Exercise 8.8

1. Determine the inverse of each of the following functions and determine the domain and
range of its inverse. Is the inverse a function?
1
a. f  x   log3 2 x c. h  x   1  x e. f  x   g. g  x   5  x  7
x 1
1
g  x   5x  13 d. k  x    x  2  f. g  x   5 x3  1 h. f  x  
2
b.
e 1
x

2. Are the following functions inverses of each other( in the respective domain )?
x2
a. f  x   3x  2; g  x   d. f  x   x3 ; g  x   3 x
3
b. f  x   x ; g  x   x2 e. f  x    x5  3; g  x   5  x  3

x 1  x 1 2 x  1
c. f  x   2 x5  1; g  x   5 f. f  x   ; g  x 
2 x2  x 1
3. Which of the following functions are invertible? If they are not, can you restrict the domain
to make them invertible?
1
a. f  x   x3 b. g  x   4  x
2
c h  x   x  5 d. f  x   log x
2

3
4. Which of the following functions are invertible and find its inverse if it is invertable.
a. c. e. c.

b. d. c.

5. For each of the following functions, sketch the graph of f and f 1 on the same coordinate
plane, along y  x .
a. f  x   x3 b. g  x    x  7 3 c. f  x    x  5 d. f  x   2 x  3 e. h  x   3 x  3 f. f  x   3 x  7  5
3

Telegram channel https://t.me/johnson201485


11
2
1. A relation from A to B is any subset of A  B .
2.  f  g  x   f  x   g  x  ;  fg  x   f  x  g  x  ;  f   x   f  x  , g  x   0 .
g g  x

3. R   b, a  :  a, b   R .
1

4.  R 1   R , for any relation R .


1

5. Domain of R 1  Range of R and Range of R 1  Domain of R .


6. A function is a relation in which no two of the ordered pairs in it have the same first element.
7. f  x   ax , r 
r
is called power function.
m

8. f  x   ax n , m even and n odd has a cusp at the origin.

9. x   x, for x  0 .
  x, for x  0

10. x  x 2 .
1, for x  0
11. sgn x  
0, for x  0 .
1, for x  0

12. The floor function or the greatest integer function f  x    x  maps into .
13. The Least integer (ceiling) function f  x    x  maps into .
14. f is one-to-one, if and only if f  x1   f  x2   x1  x2 , for any x1 , x2  Domain of f .
15. A numerical function f f is one-to-one, if and only if no horizontal line crosses the graph of f more
than once.
16. f : A  B is onto, if and only if Range of f  B .
17. f : A  B is a one-to-one correspondence, if and only if f is one-to-one and onto.
18.  fog  x   f  g  x   .
19. Domain of  fog   Domain of g .
20. f 1 is a function, if f is one-to-one.
21. g and f are inverse functions of each other, if and only if g  f  x    x and f  g  x    x .
22. To find f 1
 Write y  f  x  .
 Interchange x and y in the above equation to obtain x  f  y  .
 Solve for y and write y  f  x  .
1

Telegram channel https://t.me/johnson201485


11
3
REVIEW EXERCISES ON UNIT 8

1. Find the inverse of each of the following relation and determine whether the inverse is a function.
a. R   2,  2 ,  3,3 ,  4, 4  c. R   5, 13 ,  6, 10.8 , 3, 11.4  ,  10,14 

b. R   2, 1 ,  2,3 ,  2,7  d. R   4, 15 ,  8,  20 ,  4, 16.5 , 3, 15.25

2. Find the inverse of each function. Is the inverse a function?


1
f  x   x2  9
x
a. c. g  x   e. h  x   2
3 x2
4  3 4 x
f  x    x2  9 d. g  x   f. f  x   5 x  2  2
2
b.
2

3. Find the domain of f  x   x x .

4. a. Give the intersection points of y  x5 and y  x7 .

c. Are these points common to y  x n , where n is an odd natural number?

d. For each of the following functions, what is the effect of 4, when f  x  is compared with f  x3 ?

i. f  4 x3 ii. f  x3  4

e. For f  x3 , compare f  a  b  and f  a   f  b  for any ∈ . What do you notice?

f. Is the property f  a  b   f  a   f b  for any ∈ generally true for any ( ) ∈ ?

5. a. Show that y  sgn x is odd.


1
c. If h  x    sgn x  1 , then show that h   x   h  x   1.
2

6. Sketch the graph the following functions, find the domain and the range.
a. f  x   x  sgn x b. f  x   x sgn x
3
c. f  x   sgn  x  2  3

a. Evaluate the following: a. 3.15 b.  27.05 c. 15  d.  9 

 1
7. Check if  2 x    x    x   by taking different values of x .
 2

8. Find fog , fof , gof , gog for: a. f  x   1  2 x; g  x   x b. f  x   log x; g  x   3x  1

Telegram channel https://t.me/johnson201485


11
4
If f  x   3x  2 and g  x   x  9 , find;
2
9.

 f 
a.  f  g  2 b.  gof  3 c.    5  d.  f  g 10
g
10. Suppose f is a function on the set of integers defined by:

f 1  5 and for all n  1, f  n  1  n  f  n   . Find f  6  .

11. Let f  x   x  4 and g  x   2x  5 .Find:

c.  g 1of 
1
d.  fog 
1
a. g 1of 1 b. fog 1

PRACTICE QUESTIONS ON UNIT 8


CHOOSE THE BEST ANSWER FROM THE GIVEN ALTERNATIVES
x2
and g ( x)   2 , then f g (x)  is equal to:
1
1. If f ( x) 
x2 x
x
A. x 2 B. x 2 C. x D.
x
 x 
2. If f ( x)  ln   2  , for x  1, then which of the following is the inverse of f ?
 x 1 
ex  2 ex
A. g ( x)  C. g ( x)  2
ex  3 ex 1
ex  2
x
B. g ( x)  D. g ( x)  e x 1  2
ex 1
3. Which one of the following is a one-to-one function?
A. f  1, 6, 2, 7 , 3, 7 , 4, 8 C. k :    given by h( x)  x  3
B. k : 0,  given by h( x)  log x D. g  x, y  : x is a student and y is his/her rank 

4. If f ( x)  ln x  1 and g ( x)  x 3  7 , then what is the domain of  fg (x) ?


A.  2,  B.  1,  C.  2,  D. 
1
5. If f ( x)  , then which of the following is equal to f 1 ( x) ?
e 1
x

 1 
ln 1  x   ln x  B. e  x  1
1
A. C. ln   D.  x
 x 1 e 1

Telegram channel https://t.me/johnson201485


11
5
6. If f ( x)  3 1  e  x , which of the following is equal to f 1 ( x) ?

A.
 1 
ln  3 
 x 1 
B.

1
ln x 3  1 
C. ln 1  x 3  
D. 1  e  x 
3

7. Which of the following functions is a one-to-one correspondence?


A. f : '  , f ( x)  tan x , where ' is the domain of f .
B. g :   , g ( x)  2 x C. h : 0,   0, , h( x)  x 2 D. r : 0,
  0, , r ( x)  x  5
8. Which one of the following is a one-to-one correspondence function from A  0, 1 to B  1, 2 ?
1 3
A. f ( x)  x B. f ( x)  x 1 C. f ( x)  2x  1 D. f ( x)  x 2  1
3
2x
9. The inverse of the function defined by g ( x)  is equal to:
x3
2x 3x x3 x2
A. g 1 ( x)   B. g 1 ( x)   C. g 1 ( x)   D. g 1 ( x) 
x3 x2 2x 3x
10. If f : A  B and g : B  C are functions, then which one of the following is true about the
composition function?
A. Domain of gf   Domain of f . C. Domain of gf  
 Domain of f .
B. Range of gf  
 Range of g . D. Range of gf   Range of f .
11. If the point 3,  2 is on the graph of y  f (x) , which point is on the graph of y  f 1 ( x) ?
1   1
A.  ,  2 B. 3,  1 C.  2, 3 D.  3,  
3   2
12. Which of the following is the inverse of the relation R  x, y  : 2 y  x  1 ?
A. x, y  : x  2 y  1 B.  y, x  : 2 x  y  1 C.  y, x  : 2 y  x  1 ND. x, y  : y  2 x  1
1
13. and g ( x)  x , then which of the following is true?
If f ( x)  x 
x
1
A. Domain of fg is x | x  0 C. f    f  x 
 x
2x 2  1
B. Domain of gf is x | x  0  
D. f x   g x 2 
x

14. Which of the following is NOT true?


A.  
The range of the inverse of the relation R  x, y  | y  x 2 and y  4 is y | 2  y  2 .
B. The domain of the relation R  x, y  | x  y  2 and x  0 is x | x  0 .
C. If R is a relation on the set of real numbers, then the graph of R 1 can obtained by reflecting the
graph of R along the line y  x .
D. A function is a relation.

Telegram channel https://t.me/johnson201485


11
6
15. If f :   is a function, then which of the following is necessarily true?
A. If f is one-to-one, then it is onto. C. If f is odd, then ff is odd.
B. If f is onto, then it is one-to-one. D. If f is odd, then  f is even.
16. Let f ( x)  3 . Then which of the following is true for each real numbers x and y ?
x

f x  y   f x   f  y  C. f x 2  1   
1
A. f x2
3
B. f xy   f x  f  y  D. f x 1  f x 
 1
 , if x  3
17. Given functions f ( x)  x  x  3 and g ( x)   x  1
2
, then gf (2) is equal to:
 x 2  1, if x  3

1
A. B. 3 C. 26 D. 82
4
18. Which of the following represents the graph of the relation R  x, y  | x  y ?
A. Y C.
Y

X
X

Y Y
B. D.

X X

x
19. If f ( x)  and g ( x)  x  1 , then which of the following is true?
1 x
2x  1
A. gf ( x)  C. The domain of gf ( x)  x | x  1
1 x

The domain of  fg ( x)  x | x  1 D. gf (5) 


1
B.
2
20. The power function f ( x)  x 4 is symmetric with respect to:
A. the line y  0 B. the origin C. the line y  x D. the line x  0
21. Which of the following is NOT true about the function f ( x)  x 2 ?
A. It is an even function. C. Its range is  .
B. Its graph has a vertical asymptote. D. Its domain is  \ 0 .

Telegram channel https://t.me/johnson201485


11
7
22. Which of the following relation is the inverse of the relation
R  x, y  : y  2 x  6 and 2 y  x  8 ?
   
x, y  : y  x  3 and y  2 x  8 C x, y  : y  x  3 and y  2 x  8
1 1
A.
 2   2 
B. x, y  : x  2 y  6 and 2 x  y  8 D. x, y  : y  2 x  6 and 2 y  x  8
23.  
If R is a relation given by R  x, y  : y  9  x 2 and y  2 x  1 , then which of the following is
the domain of R 1 ?
A.  7, 5 B.  4, 2  C.  4, 2 D.  7, 5
3 3x
24. If f ( x)  and fg ( x)  , then which of the following true about g (x) ?
x 1 2 x
6 2 2
A. g ( x)  3x B. g ( x)  C. g ( x)  D. g ( x) 
2 x x 1 x
25. Which of the following functions is an odd function?
A. 
f ( x)  x 3 x  1 B f ( x)  x 3  7 C. f ( x)  x 2 x  x 2  D. f ( x)  x  1
1
26. Which of the following is NOT a property of y  x 3 ?
A. Its range is  . C. The graph is symmetric with respect to the origin.
B. f (ab)  f (a) f (b) D. The graph is decreasing.
27. Which of the following functions can be a possible representation of the graph shown below?
A. f ( x)  1  x Y
B. f ( x)   x  1
C. f ( x)  x  1
D. f ( x)  x  1 X

Let f :   and g :  \ 0   \ 0 be two functions defined as f ( x)  x  1 and g ( x) 


1
28. .
x
Which of the following statements is NOT true?
x 1
A.  
f f 1 ( x)  x for all x   . B. f g ( x)  
x
C. f g (2)  
1
2
D. g  f (1)   0

29. The graph shown below is the graph of a certain relation R. What is the range of R 1 ?
Y

A.  2, 
B.  1, 
C.  ,  2
X
D.  ,  1

Telegram channel https://t.me/johnson201485


11
8
30. Given a relation R  x, y  : y  x  3 and y  2 x  6 , which one of the following is true
about its inverse?
 
R 1  x, y  : y  x  3 and y   x  3
1
A. C. Range of R 1   , 3
 2 
 
R 1   y, x  : y  x  3 and y   x  3 D. Domain of R 1   3, 6
1
B.
 2 
31. Let f ( x)  x  4 , then which one of the following is true about f 1 ( x) ?
A. The range of f 1 ( x) is  4,  C. The domain of f 1 ( x) is the set of real numbers.
B. f 1 ( x)  x 2  4 . D. The x  intercepts of f 1 ( x) are  2 and 2 .
32. Given f ( x)  log 3 x and g ( x)  x 2  3 , then which of the following is true?
A. Domain of  fg (x)  0,  C. Domain of gf (x)   , 
Range of  fg (x)  1,  D. gg ( x)  log 3 x 
2
B.
33. Which one of the following graphs represents the graph of the inverse of the relation whose graph
is shown below?

A. C.

B. D.

Telegram channel https://t.me/johnson201485


11
9
UNIT NINE: COORDINATE GEOMETRY
Unit Outcomes:

After completing this unit, you should be able to:

 understand specific facts and principles about lines and circles.


 know basic concepts about conic sections.
 know methods and procedures for solving problems on conic sections.
Main Contents:

9.1.STRAIGHT LINE.
9.2.CIRCLE.

Telegram channel https://t.me/johnson201485


12
0
9.1. STRAIGHT LINES
9.1.1. Review on Equation of a Straight Line
 distance between points on a coordinate plane;
 division of a line segment (how to find the coordinates of a point on a line segment that divide the
segment in a specified ratio);
 mid-point formula for a line segment;
 slope and angle of inclination of a line;
 equation of a straight line;
 how to tell whether two lines are parallel or perpendicular by looking at their slopes.
A. Distance between points in a plane
Suppose P (x1, y1) and Q (x2, y2) are two distinct points on the xy-coordinate plane. We can find the
distance between the two points P and Q by considering three cases.

a) b) c)

Figure 9.1

CASE I: When P and Q are on a line parallel to the x-axis (that is, ̅̅̅̅ is a horizontal segment)
As in the Figure 3.1a, since the two points P and Q have the same y-coordinate (ordinate), the distance
between P and Q is given by PQ = |x2 – x1|
For instance; the distance between points A(3, 5) and B(4, 5) is
|x2 – x1| = |4  (3)| = |7| = 7 ; since A and B have the same ordinate.
CASE II: When P and Q are on a line parallel to the y-axis (that is, ̅̅̅̅is a vertical segment)
As in the Figure 3.1b, since the two points P and Q have the same x-coordinate (abscissa), the distance
between P and Q is given by PQ = |y2 – y1|
For instance; the distance between points P(1, 4) and Q(1, 6) is
|y2 – y1| = |6  4| = |10| = 10 ; since P and Q have the same abscissa.

Telegram channel https://t.me/johnson201485


12
1
CASE III: When ̅̅̅̅ is any line segment (the general case), As in the Figure 3.1c the distance between P
and Q is given by
√( ) ( ) √( ) ( )
Example 1: Find the distance between points (√ √ ) and B( √ , √ ).
Solution: Take ( ) (√ √ ) and ( ) ( √ √ ). Then
√( ) ( )

= √( √ (√ )) ( √ ( √ ))

√(√ ) ( √ √ ) √ √

√ √ units
Example 2: What are the possible values of b if the point A(b, 4) is 10 units away from the point B(0, –
2)?
Solution:Given : AB = 10 units where A(b, 4) and B(0, –2).
√( ) ( )

 √( ) ( ( )) √
   or
Therefore, the possible values of b are and 8.
B. Division of a Line Segment
Given a line segment PQ with end point coordinates P (x1, y1) and Q(x2, y1), let R be a point between P
and Q that divide the line segment ̅̅̅̅ internally in the ratio m : n. Then the coordinates (x0, y0) of R are

given by ( ) . /

This formula is called the SECTION FORMULA.

Telegram channel https://t.me/johnson201485


12
2
Example 3: Find the coordinates of the point R that divides the line segment with end- points A(6, 2) and
B(1, – 4) such that .
Solution:
Put (x1, y1) = (6, 2), (x2, y2) = (1, – 4), m = 2 and n = 3. Using the section formula, you have
( ) ( ) ( ) ( )
( ) . / . /

. / . /

Example 4: A line segment has end-points P(–4, –3) and Q(2, 5). Find the coordinates of the points that
trisect the segment ̅̅̅̅.
Solution: Let ( ) and ( ) be two points that trisect ̅̅̅̅. This implies that A and B divide ̅̅̅̅
into three congruent smaller segments. That is PA = AB = BQ
First point A divides the ̅̅̅̅ in the ratio 1:2, and hence Q(2, 5)
PA : AQ = 1 : 2

 ( ) . /
B
( ) ( ) ( ) ( )
. / . /
A
Secondly point B divides the ̅̅̅̅ in the ratio 2: 1, and hence
P(4, 3)
PB : BQ = 2 : 1
( ) ( ) ( ) ( )
 ( ) . / . /

Therefore, points that trisect ̅̅̅̅ are . / and . /.

The Mid-Point Formula


 A point that divides a line segment into two equal parts is the mid-point of the segment. It is
equidistant from the endpoints of the segment.
 The formula for the midpoint of a line segment can be obtained by making m = n = 1.
The mid-point of the line segment with endpoints (x1, y1) and (x2, y2) is given by
y

( ) . /

Telegram channel https://t.me/johnson201485


12
3
Example 5: Find the midpoint of the line segment with endpoints A(5, 4) and B(3, 4).
Solution: Put (x1, y1) = A(5, 4) and (x2, y2) = B(3, 4).
Then by midpoint formula, the coordinates of the midpoint M of ̅̅̅̅ are
( )
( ) . / . / ( )
Example 6: The mid-point of a line segment is M(−3, 2). If one end-point of the segment is
P(1, −3), then find the coordinates of the other end-point.
Solution: Put the midpoint (x, y) = M(−3, 2) and one endpoint (x1, y1) = P(1, 3).
To find the other endpoint Q(x2, y2).
Then by midpoint formula, ( ) . /  ( ) . /
 

Therefore, the other end point of the segment is ( ).
C. Gradient (slope) of a Line
 A straight line may be steep or may rise very slowly. The number that describes the steepness of a
line is called the GRADIENT (SLOPE) of the line.
 In coordinate geometry, the gradient of a non-vertical straight line is the ratio of "change in y-
coordinates" to the corresponding "change in x-coordinates". That is, the slope of a line through P and
Q is the ratio of the vertical distance from P to Q to the horizontal
distance from P to DEFINITION: If (x1, y1) and (x2, y2) are points on a
line with
x1  x2, then the SLOPE or GRADIENT of the line, denoted by m, is
given by

NOTE:
1. Vertical lines have no gradient.
2. The gradient of a horizontal line is zero.
y y y y

x x x x

Figure 7a: Figure 7b: Figure 7c: Figure 7d:


 A horizontal line.  A vertical line.  An inclined line to i) An inclined line to
 Slope m = 0  Slope m = 0 the right. the left.
 m > 0 (positive) ii) m < 0 (negative)

Telegram channel https://t.me/johnson201485


12
4
( )
Example 7: The slope of a line passing through points (−5, 1) and (3, 0) is .

NOTE : Three distinct points are said to be LIE ON THE SAME LINE (or COLLINEAR) if the line
that contains any two points also contains the third.
R
C Q

B
A P
Points A, B and C are collinear. Points P, Q and R are not collinear.
RULE :
1. Three distinct points are A, B and C are lie on the same line (or collinear) iff
̅̅̅̅ ̅̅̅̅( ̅̅̅̅)

2. A horizontal line and vertical lines are always perpendicular.


Example 8: Check whether the following triplets of points are collinear or not.
a. ( ) ( ) ( ) b. (√ ) ( √ ) (√ √ )
Solution
a. ̅̅̅̅ and ̅̅̅̅
( ) ( )

 ̅̅̅̅ ̅̅̅̅ . Therefore, points A, B and C don‟t lie on the same line.
√ √ √ √ √ √
b. ̅̅̅̅ √ , and
√ √ √ √

√ (√ ) √ √
̅̅̅̅ √ .
√ √ √ √

 ̅̅̅̅ ̅̅̅̅ . Therefore, points P, Q and S are collinear.

Example 9: Find the possible values of k so that the following triplets of points are collinear.

a. ( ) ( ) ( ) b. ( ) ( ) ( )
Solution
a. ̅̅̅̅ and ̅̅̅̅

Since, A, B and C are collinear, we have ̅̅̅̅ ̅̅̅̅   


b. and ̅̅̅̅

Since, P, Q and R are collinear, we have ̅̅̅̅ ̅̅̅̅   ( )( )


  ( )( )  k = 2 or k = 3
Therefore, the possible values of k are 2 and 3.

Telegram channel https://t.me/johnson201485


12
5
D. Slope of a Line in Terms of Angle of Inclination: The angle measured from the positive x-axis to a
line, in anticlockwise direction, is called the INCLINATION OF THE LINE or the ANGLE OF
INCLINATION OF THE LINE.

If  is the angle of inclination of a line always .

Here, there are four cases: Let be a line with angle of inclination θ.

y y y y
θ 0

θ 0
θ θ
x x x x

Figure d:
Figure a: Figure b: Figure c:
 An inclined line to
 A horizontal line.  A vertical line.  An inclined line to the left.
 θ 0  θ 0 the right.  900 < θ 0
 Slope m = 0  Slope m =  00 θ 0
 m < 0 (negative)
 m > 0 (positive)

REMARK: Let be a non-vertical line with an angle of inclination θ. Then slope of = tan θ
0 0
 If is vertical line, then θ = 90 and slope of = tan 90 =
Recall the following trigonometric values of special angles:
sin θ cos θ tan θ
00 = 0 rad. 0 1 0
300 = rad. √ √ √

450 = rad. √ √ 1

600 = rad. √ √

900 = rad. 1 0

1200 = rad. √ √

1350 = rad. √ √ 1

1500 = rad. √ √ √

Telegram channel https://t.me/johnson201485


12
6
Example 10: Find the slope of the lines having each of the following angles of inclinations:
a. θ = 600 b. θ = rad c. θ = 1350

Solution :
a. t n t n √
b. t n t n t n

c. t n t n t n √ √
Example 11: Find the angles of inclinations of the lines having each of the following slope:
a. b. √ c.
Solution :
a.  t n  0

b. √  t n √  rad

c.  t n  0

E. Different Forms of Equations of a Line


(i) HORIZONTAL LINE : If a line is horizontal and crosses the y-axis at the point (0, b) then
the equation of the line is y = b.
For instance; the equation of a horizontal line passing the point A(3, 2) is y = 2.
(ii) VERTICAL LINE : If a line is vertical and crosses the x-axis at the point (a, 0), then
the equation of the line is x = a.
For instance; the equation of a vertical line passing the point A(3, 2) is x = 3.
x = 3 y

y=2

x
A

(iii) TWO-POINT-FORM : The equation of a non-vertical line passing through two points
(x1, y1) and (x2, y2) is given by 𝑦 𝑦 𝑦 𝑦
𝑥 𝑥 𝑥 𝑥

Example 12. Give the equations of the lines passing through the points P(–1, 3) and Q(4, 0).

Telegram channel https://t.me/johnson201485


12
7
Solution. Put (x1, y1) = (–1, 3) and (x2, y2) = (4, 0)

 
( ) ( )

 ( )

 ------------------------- standard form of equation of line

Or ------------------------- general form of equation of line


(iv) POINT-SLOPE FORM: The equation of a non-vertical line having slope m and
passing through the point (x0, y0) is given by
y – y0 = m (x – x0)

(v) SLOPE-INTERCEPT FORM: The equation of a non-vertical line making y -


intercept y = b and having slope m is given by
y = mx + b

This form of equation of line is called STANDARD FORM of equation of line.


(vi) INTERCEPTS FORM: The equation of a non-vertical line making intercepts a
and b on x- and y-axes, respectively, where a  0 and b  0, is given by 1

𝑥 𝑦
𝑎 𝑏

GENERAL FORM EQUATION OF A LINE: Any equation of the form Ax + By + C = 0, where


or is called the GENERAL FORM EQUATION OF A LINE.
The general form of the line can be reduced to various forms as given below:
i) If and , then
Ax + By + C = 0  0.x + By + C = 0
 By + C = 0

 ,

which is an equation of a horizontal line whose slope is m = 0 and y-intercept .

It has no x-intercept, unless c = 0.


ii) If and , then
Ax + By + C = 0  Ax + 0.y + C = 0
 Ax + C =

Telegram channel https://t.me/johnson201485


12
8
 , which is an equation of a vertical line which has no slope. Its x-intercept is .

iii) If both and , then

Ax + By + C = 0  Ax + By + C = 0  By = Ax   . /,

which is an equation of a slant line having slope and y-intercept .

Its x-intercept is obtained when y = 0

 . /  ,  is the x-intercept.

Example 13. Find the equation of lines passing through (1, 2) and making angle 30° with positive y-axis.
Solution. Put (x1, y1) = (1, 2)
The line makes 30° with +y-axis implies the angle of inclination of the line is
 = 300 + 900 = 1200
 The slope of the line is t n t n √
Therefore, the equation of the line is y – y1 = m(x – x1)
 y–2= √ (x – 1)
 y= √ √
The y-intercept is √
√ √
When y = 0, √ √ 

Example 14. Find the slope and intercepts of the line having equation

Solution. First write the given equation in the standard form of equation of line.

. / . /





Or

Thus, the x-intercept is and the y-intercept is

Telegram channel https://t.me/johnson201485


12
9
Example 15. If the slope of a line passing through the point A(3, 2) is , then find points on the line which

are 5 units away from the point A.


Solution. Equation of the line passing through (3, 2) having slope is given by

y–2= ( )

 4y – 3x + 1 = 0
Let (h, k) be the points on the line is 5 units away from the point A. then by distance formula

√( – ) ( – ) =5

 ( – ) ( – ) ------------------------------------- (i)
Since (h, k) is on the line, we have
4k – 3h + 1 = 0

 ------------------------------------- (ii)

Putting equation (ii) in (i) and on simplifying, we get


25h2 – 150h – 175 = 0
or h2 – 6h – 7 = 0
or (h + 1) (h – 7) = 0
or h = –1, h = 7
Putting these values of h in (ii), we get k = –1 and k = 5. Therefore, the coordinates of the
required points are either (–1, –1) or (7, 5).
F. Parallel and Perpendicular Lines
Slopes can be used to see whether two non-vertical lines in a plane are parallel,
perpendicular, or neither.
Parallel Lines
THEREOM: If two non-vertical lines ℓ1 and ℓ2 with slopes m1 and m2, respectively, are parallel to each
other, then they have the same slope.
That is if ℓ1 ℓ2, then m1 = m2
The converse is also TRUE. Suppose ℓ1 and ℓ2 be two non-vertical lines with slopes m1 and m2,
respectively. if m1 = m2, then ℓ1 ℓ2.
NOTE: If two lines are vertical, then they are always parallel.

Telegram channel https://t.me/johnson201485


13
0
Suppose ℓ1 and ℓ2 be two non-vertical lines with slopes m1 and m2, and angles of inclinations θ and β,
respectively. as shown in the figure below.
If ℓ1 is parallel to ℓ2, then θ = β. Consequently, m1 = tan θ = tan β = m2

Example 16. Write an equation for the line that passes through the point (6, 2) and is parallel to the line
3x + 4y = 10.
Solution: Let us first figure out the slope of this other parallel line. Transforming the given equation given
to slope-intercept form and read the slope. 3x + 4y = 10 

The slope of the given line is .If our line is to be parallel to this line, its slope must be the same.

Thus, the slope of the required new line is

Now, the new line passes through the point (x1, y1) = (6, 2) and having slope

By point - slope form of equation of a line, we have y (x )

y ( ) (x )

 y x is the required equation.

Perpendicular Lines: If two non-vertical and non-horizontal lines ℓ1 and ℓ2 with slopes m1 and m2,
respectively, are perpendicular to each other, then the product of their slopes is 1.
That is if ℓ1 ℓ2, then m1 . m2 = 1
The converse is also TRUE.
Suppose ℓ1 and ℓ2 be two non-vertical and non-horizontal lines with slopes m1 and m2, respectively.
if m1 . m2 = 1, then ℓ1 ℓ2.
Example 17. Find the equation of the straight line passing through (1, 2) and perpendicular to the line x +
y + 7 = 0.
Solution : Let m be the slope of the line whose equation is to be found out which is perpendicular to the
given line x + y + 7 = 0. The slope of the given line y = (– 1) x – 7 is m = – 1.
Therefore, using the condition of perpendicularity of lines, we have (– ) – or is the
slope of the required new line.

Telegram channel https://t.me/johnson201485


13
1
Hence, the required equation of the line is y – 1 = (1) (x – 2) or y – 1 = x – 2 or x – y – 1 = 0.
Example 18. Consider the triangle determined by A(4, 4), B(1, 7), and C (6, 1). Write an equation for
the altitude drawn to side ̅̅̅̅ .
Solution: The height or altitude drawn to side ̅̅̅̅ is simply a line that is perpendicular to line ̅̅̅̅ and
contains the opposite vertex point B.
First compute the slope of line segment ̅̅̅̅ . (x1, y1) = A(4, 4), and (x2, y2) = C(6, 1)

( )

The altitude drawn to side ̅̅̅̅ is perpendicular to side ̅̅̅̅ and so its slope must be the negative reciprocal
of slope of ̅̅̅̅ . That is the slope of the required altitude is
Hence, the required altitude passes through B(1, 7) and having slope .
Therefore, the equation of the required altitude is y – 7 = 2(x – 1) or y – 7 = 2x – 2 or y = 2x +
9.1.2. Angle Between Two Lines on the Coordinate Plane
When two lines are intersecting, the angle between the two lines can be determined using their slopes.
DEFINITION: The angle between two intersecting lines ℓ1 and ℓ2 is defined to be the angle θ measured
counter-clockwise from ℓ1 to ℓ2.

THEOREM: If β is the angle between the two lines ℓ1 and ℓ2 measured from ℓ1 to ℓ2 counter-clockwise
and m1 and m2 their slopes, respectively, then the tangent of β is given by
n n
t n , where .
n n

Example 19. Given points P (2, 3), Q (–4, 1), C(2, 4) and D(6, 5), find the tangent of the angle between
the line that passes through P and Q and the line that passes through C and D when measured from the line
that passes through P and Q to the line that passes through C and D counter-clockwise.
Solution: Let m1 be the slope of the line through P and Q and m2 be the slope of the line through C and D.

Then, and

t n .  t n

Example 20. Find the acute angle between the line and √

Telegram channel https://t.me/johnson201485


13
2
Solution: Let m1 be the slope of the line , and m2 be the slope of the line

√ .  and

√ √
Let be the angle between the two lines. t n

 . /

9.1.3. Distance between a Point and a Line on the Coordinate Plane


DEFINITION Suppose a line ℓ and a point P(x, y) are given. If P does not lie on ℓ, then we define the
distance d from P to ℓ as the perpendicular distance between P and ℓ.P ℓ

If P is on ℓ, the distance is taken to be zero. d

The distance from a point to a line can be obtained using the following theorem.
THEOREM: The distance d from the point (h, k) to the line ℓ : Ax + By + C = 0, where A  0 or B0
| |
is given by √

Example 21. Find the distance of the point (3, 1) from the line y = x + 1.
Solution: First write the line in the general form y = x + 1 
Put (h, k) = P(3, 1) and ℓ : x  y + 1 = 0 . Then the distance from P to ℓ is
| | | ( ) ( ) | | | √
√ √ ( ) √ √

Distance between two parallel lines :The distance d between two parallel lines y = mx + c1 and y = mx +
| |
c2 is given by √

Example 22. Find the distance between the lines 3x + 4y = 9 and 6x + 8y = 15.
Solution : The equations of lines 3x + 4y = 9 and 6x + 8y = 15 may be rewritten as :

3x + 4y = 9  and 6x + 8y = 15 

Since, the slope of these lines are same and hence they are parallel to each other with

and

| |
Therefore, the distance between them is given by
√ ( )

Telegram channel https://t.me/johnson201485


13
3
EXERCISE 9.1 INSTRUCTION: Work-out each of the following questions correctly.
1. Let ℓ be the line containing point (1, 5) and its angle of inclination be 1350. Find the distance from ℓ to
the point.
2. What is the equation of a line that passes through ( ) in the xy-plane if is parallel to a line that
passes through points ( ) and ( ), where ?
3. Find the locus of points which are equidistant from two points A(2, 5) and B(0, 1).
4. Show that two lines with equations and are perpendicular if

5. Determine the value of k so that the line ( ) ( )


a) is parallel to the x-axis. b. is passes through the point (1, 1) c) has y-intercept y = 3
6. Given the line . Find the possible equation(s) of line(s) passing through the point (2,
0
1) and making an angle of 45 with the given line.
7. Find the acute angle formed between the following pairs of lines.
a)
b) √ √ ( √ ) (√ √ )
8. Find the obtuse angle formed between the pairs of lines √ √ √
9. Find the distance between the given line and the given point below.
a) ; (3, 1) b). √ ;( ) c)
10. Find the value of h such that the distance from the line to the point (h, 2) is 6.
9.2. CIRCLE
Circles A Circle is the locus of a point that moves in a plane with a fixed distance from a fixed point. The
fixed distance is called the RADIUS of the circle P r
C
and the fixed point is called the CENTRE of the circle.
Let a circle has center C (h, k) and radius r. For any point P(x, y) on the circle,
PC = r and by the distance formula you have,
√( ) ( )
By squaring both sides, we obtain

(𝑥 ) (𝑦 𝑘) 𝑟

The above equation is called the STANDARD FORM of the equation of a circle, with centre C (h, k)
and radius r.
Conversely, for r > 0, an equation ( ) ( )
represents a circle with center (h, k) and radius r.
NOTE: If a circle is centered at the origin (0, 0), then and , and the equation becomes

Telegram channel https://t.me/johnson201485


13
4
Example 26. An equation
a. ( ) ( )

 ( ) ( ) is an equation of a circle with center C. / and radius r = 6.

b. ( )  ( ( )) ( ) √
is an equation of a circle with center C( ) and radius r = √ .
Consider a circle with center C (h, k) and radius r having equation ( ) ( )
Expanding the equation gives ------------- (a)

Letting and , then equation (a) can be written as
𝑥 𝑦 𝐷𝑥 𝐸𝑦 𝐹 -------- (b)

This form of equation is called GENERAL FORM of equation of a circle.


For instance; consider an equation of a circle ( ) ( )


This equation is general form of a circle with center (1, 3) and radius r = √
NOTE: An equation of the form doesn‟t always represent a circle.
To check whether an equation represent a circle or not, first change it to the
form ( ) ( ) using COMPLETING SQUARES METHOD. Then
i) if , then it is a circle,
ii) if , then it is a point, and
iii) if , then it doesn‟t represent any plane figure.
Now consider an equation, and apply completing the square method
------------------- (c)
 ------------------ (rearranging the equation)

 ⏟ ⏟ ------ (Add both sides . / and . / )

 𝐷 𝐸 𝐷 𝐸 𝐹
(𝑥 ) (𝑦 )
------------------- (d)

Telegram channel https://t.me/johnson201485


13
5
Here, there are three cases :

Case i. If , then √ ∈ and is an equation of

a circle with center . / and radius √

Case ii. If , then √ and represent a

point, since the equation is satisfied by the point . / only. Sometimes the equation is said to be a

point circle.

Case iii. If , then √ ∉ and represent

nothing, since the equation has no solution.


Example 27. Identify whether each of the following represent a circle or not.
a. c.
b. d.
Solution
a.


 ( ) ( )
 ( ) ( ) . That is there is no real number r such that
Therefore, doesn‟t represent a circle. Moreover, it doesn‟t represent any
plane figure.
b.
First divide both sides by 4 to make the coefficients of and each 1.

 . / ( )

 . / ( )

 . / ( ) √

Therefore, represent a circle with center . / and

Telegram channel https://t.me/johnson201485


13
6
c.
 ( )
 ( )
Therefore, represent a point P( ).
d.

 ( ) ( )
 ( ) ( )
Therefore, is a circle with center ( ) and √
Remark: Consider a circle with center C and radius r. Let P be a point in the plane.
CASE I: If CP < r, then P is the interior point of the circle.
CASE II: If CP = r, then P is the point on the circle.
CASE III: If CP > r, then P is the exterior point of the circle.
Example 28. Check whether each of the following points lie inside, on or outside the circle with equation
a) A(1, 1) b. ( ) c. ( )
Solution: First write the equation in its standard form.

 ( ) ( )
 ( ) ( )
Thus, the circle has center C(2, 1) = (h, k) and radius r = 4
a. A(1, 1) = (x, y) :
√( ) ( ( )) √ √
Therefore, point A is the interior point of the circle.
b. ( ) ( ):
√( ) ( ( )) √ ( ) √
Therefore, point B lies on the circle.
c. ( ) ( )
√( ) ( ( )) √ √
Therefore, point A is the exterior point of the circle.
Circles and Lines

Telegram channel https://t.me/johnson201485


13
7
Given a circle and a line, there are three possibilities:
 The line may cross the circle at two points; or
 The line may touch the circle; or
 The line may not meet the circle at all.
These possibilities can be checked using distance as follow:
1. If the distance from the centre of a circle to a line is less than the radius of the circle, then the line
intersects the circle at two points. Such a line is called a SECANT line to the circle.
2. If the distance from the centre of a circle to a line is equal to the radius of the circle, then the line
intersects the circle at only one point. Such a line is called a TANGENT line to the circle and the point of
intersection is called the POINT OF TANGENCY.
3. If the distance from the centre of a circle to a line is greater than the radius of the circle, then the line
does not intersect the circle.

𝓵 𝓵
O O

Figure a: is secant Figure b: is tangent


Figure b: is neither
line to circle O. line to circle O.
tangent nor secant line to
P is point of tangency. circle O.
NOTE: If a line is tangent to a circle of center O and radius r, and P is the point of tangency, then
̅̅̅̅
𝑂𝑃 𝑟
i. ̅̅̅̅ , and ii.
Example 29. Check whether the following lines are secant, tangent or neither to the circle with equation
. If tangent, find the point of tangency.
a. b. c.
Example 30. Find the equation of the circle with center (3, 7) and tangent to the line given with equation
.
Solution : Center : C(3, 7) = (h, k)
Tangent

| | | ( ) ( ) |
 √ √ ( )

| |

√ √

Telegram channel https://t.me/johnson201485


13
8
This, the circle has center (3, 7) and radius √ . Therefore, the equation of the circle is

( ) ( ) √
( ) ( )
Or
Example 30. Find the equation of the tangent to the circle with center at ( )and point of tangency at
( ). 𝓵 P(-1, 3)
Solution : It is given that center : O(2, 0) = (h, k)
point of tangency : P( ) ( ).
The tangent line is passing through P( ) ( ) and O(2,0)
perpendicular to ⃡ . Slope of ⃡ is

Thus, the slope of is

Therefore, the equation of the tangent line is


( )
 ( ( ))

NOTE: If a line ℓ is tangent to a circle (x − h)2 + (y − k)2 = r2 at a point T(xo, yo), then

the equation of the tangent ℓ is given by

Example 31. Give the equation of the line tangent to the circle with equation
(x + 1)2 +(y − 1) 2 =13 at the point P(–3, 4).
Solution : Center of the circle : C(–1, 1) = (h, k) and
Point of tangency : P(–3, 4) = T(xo, yo).
Then the equation of the tangent line is

( )
 ( )


Therefore, the equation of the tangent is

Telegram channel https://t.me/johnson201485


13
9
EXERCISE 9.2
1. Find the equation of the circle with
a. center (2, 5) and passes through the point P (1, 2).
b. diameter having endpoints A (5, 7) and B (2, 3).
c. center (3, 7) and tangent to the y – axis.
2. Write each of the following equations in standard form, and determine if the graph of the equation is
a circle, a point, or neither. If a circle, obtain the center and radius.
a. d.
b. e.
c. f.
3. Find the possible value(s) of „k’ so that the equation represent circle?
4. Find the equation of the circle passing through points ( ) n ( ) which has its center
on the line .
5. A point moves so that the sum of the squares of its distances from the two lines and
is 7. Show that the locus is a circle and find the equation of the locus.
6. Find the smallest and largest distance from the point P(1, 4) to the circle

7. Find the area and circumference of the circle whose equation is .


8. Two circles in a plane are said to be concentric if they have the same center. Show that the circles
whose equations are and are
concentric.
9. Find the points of intersection(s) of the given circles and the given lines.
a. C : ( ) ( ) c. C :
: :
b.C : :
10. Find the equation of the tangent line to the circle at ( ).
11. Find the equation of the circle that is tangent to the x-axis and has center at (5, 4).
12. Find the equation(s) of the line(s) passing through the point ( √ ) and tangent to the circle
.
13. Find the equation of the circle passing through points ( ) ( ) ( ).
14. A circle has Centre at (5, 12) and is tangent to the line with equation 2x − y + 3 = 0. Write the
equation of the circle.

Telegram channel https://t.me/johnson201485


14
0
y2  y1
1. The slope of a line through  x1 , y1  and  x2 , y2  is given by .
x2  x1
2. Two point form: If  x1 , y1  and  x2 , y2  with x1  x2 are given, then the line through them has as
y y 
equation y  y1   2 1   x  x1  .
 x2  x1 
3. Point-slope form: If a point  x1 , y1  and slope m are given, then the equation of the line is
y  y1  m  x  x1  .
4. Slope-intercept form: If the slope m and y  intercept b are given, then the equation the line is
y  mx  b .
5. Two lines are parallel if and only if they have the same angle of inclination.
6. The slope of a non-vertical line is tan  , where  is the angle of inclination of the line, with
0    180o .
mn
7. The angle  between two non-vertical lines is given by the formula tan   where m and n
1  mn
are the slopes of the lines.
8. Two lines are perpendicular if and only if the angle between them is 90o .
9. If two perpendicular lines are non-vertical, then mn  1 , where m and n are their slopes.
10. The general form of equation of a line is Ax  By  C  0 , where A  0 or B  0 are fixed real
numbers.
C
11. The distance from the origin to the line Ax  By  C  0 is given by .
A 2  B2
Ah  Bk  C
12. The distance from P  h, k  to Ax  By  C  0 is .
A 2  B2
13. If the xy  coordinate system is translated to a new x ' y ' coordinate system with origin at C  h, k  ,
then the translation formulae are: x '  x  h and y '  y  k
14. The standard form of the equation of a circle is  x  h    y  k   r 2 , where  h, k  is the centre and
2 2

r is the radius.
1
15. The equation x2  y 2  Bx  Cy  D  0 represents a circle of positive radius r  B2  C2 - 4D and
2
 B C
center   ,   if and only if B2  C2 - 4D  0 .
 2 2
16. The line that touches a circle at only one point is called a tangent line and its equation is
y  y0   x0  h 
 where  x0 , y0  is the point of tangency and  h, k  is the centre of the circle.
x  x0 y0  k

Telegram channel https://t.me/johnson201485


14
1
REVIEW EXERCISES ON UNIT 9

1. Write each of the following in the general form of equation of a line.


1
a. y  3 b. x  9 c. y  x  4 d. y  2x  5 e. y  3  4  x f. 3x  7  4 y
2
2. Give the equation of the line that satisfies the given conditions:
a. passes through P  2, 3 and has slope 1 .

b. passes through P  3, 7  and Q  6, 10 .

c. parallel to the line with equation y  3x  4 and passes through A  3,  2 .

d. perpendicular to the line with equation 6 x  2 y  4 and y  intercept 4 .


3. Find the tangent of the acute angle between the following lines:
a. 2x  y  2  0 c.  x  5 y  2  0
3x  y  1  0 y  4x  7  0
b. x  6 y  5  0 d. x  6 y  5  0
2 y  x 1  0 2 y  x 1  0
4. Find the distance from the given point to the line whose equation is given.
a. P  4, 3 ; 2 x  3 y  2  0 c. A  2, 4 ; 3x  4 y  0 e. R 1, 9 ;  5x  12 y  13  0

b. Q 1, 1 ; 2 x  2 y  4  0 d. B  2, 0 ; 3x  4 y  0

5. If the length of the perpendicular drawn from the point 15x  8 y  5  0 to the straight line is 10 units,
then find all the possible values of y .
6. Find the distance between the pairs of parallel lines whose equations are given below:
a. y  2x  4and y  2x 1 c. 2x  3 y  2  0 and 2x  3 y  6  0
b. 4 y  3x 1and 8 y  6 x  7  0 d. x  3 y  6  0 and x  3 y  24  0
7. Write the equation of each circle with the given conditions:
a. centre at O  3,  7  and radius r .

b. centre at P  3,  7  and tangent to 2x  4 y  4  0 .

c. end points of its diameter are A  3,  7  and B  4, 3 .

Telegram channel https://t.me/johnson201485


14
2
8. Find the equation of the tangent line to the circle with equation  x  3   y  4   20 at P 1, 0  .
2 2

9. Find the equation of the circle with radius 5 and concentric with circle x2  y 2  8x  7 .

PRACTICE QUESTIONS ON UNIT 9


CHOOSE THE BEST ANSWER FROM THE GIVEN ALTERNATIVES
1. What is the equation of the line passing through the mid-point of the line segment with end points
 1, 3 and 3 , 1, and perpendicular to the line whose angle of inclination is double the angle of
inclination of the line 2x  y  7 ?
A. 2 y  4 x  10 C. 2 y  x  3
B. 4 y  5x  3 D. 4 y  3x  11
2. Which of the following points are collinear?
A. (1,3), (1,3), (3,3) C. (1,1), (0,0), (2,1)
B. (2,0), (0,2), (3,5) D. (2,1), (1,1), (2,0)
3. What is the coordinate of the point R on the line segment with end points P(-3, 0) and Q(0, -3)
PR 2
such that  ?
RQ 3

 7  7   9 6
A.   1,  B. (1,1) C.   ,0  D.   , 
 5  5   5 5

4.Line l passes through 0, 5 and  5, 0 . What is the angle between the y  axis and l in radian
measure?
   3
A. B. C. D. 
4 3 2 4
5.Consider a circle whose center is on the x  axis . If a line given by y  x is tangent to the circle at point
2, 2 , what is the equation of the circle?
x2  y2  8 C. x  4  y  8
2 2
A.

B. x  22  y 2  4 D. x  1  y 2  5
2

6.Suppose l1 and l 2 are perpendicular lines intersecting at 2, 1 . If the angle of inclination of l 2 is 45 0 ,

what is the equation of l1 ?


A. y  x  2 B. y   x  3 C. y  x  1 D. y  2 x  5

Telegram channel https://t.me/johnson201485


14
3
7.If x 2  6 x  y 2  k  0 is equation of a circle with radius 2 , then what is the value of k ?
A. 13 B. 5 C. 4 D.  4

 passes through 0, 1 , which one of the following is the


3
8.If a line with angle of inclination of
4
equation of the line?
A. y  x  1 B. y  x  1 C. y   x  1 D. y  x  1

9.A circle of radius 1 unit is internally tangent to the circle x 2  y 2  2 y  8 at 3, 1 . Then, the equation
of the smaller circle is:
x 2   y  1  1 C. x  1   y  2  1
2 2 2
A.

B. x  22   y  12  1 D. x  4   y  1  1
2 2

10. What is the equation of the circle that is tangent to the line 3 y  4 x  5 and has center at 2, 1 ?
2 2
 2  11 
A. x    y    2 C. x 2  y 2  4 x  2 y  16
 5  5

B. x  22   y  12  4 D. x 2  4x  y 2  2 y  1

11. Which one of the following is equation of a circle whose center is on y  axis and radius is 3 ?

x2  y2  6y  0 C. x 2   y  2  3
2
A.

B. x  22  y 2  9 D. x 2  2x  y 2  8

12. The equation of the line that passes through 2,  1 and perpendicular to 3x  4 y  6 is:
A.  4x  3 y  5 B. 4 x  3 y  5 C. 4 x  3 y  11 D.  4 x  3 y  11
13. If the graphs of the equations ax  by  c  0 and Ax  By  C  0 are perpendicular, then which of
the following is necessarily true?
A. aA  bB  0 B. aB  bA  0 C. aA  bB  0 D. aB  bA  0
14. Let C be the circle whose equation is x 2  y 2  2x  2 y  2  0 . Let L be the line whose equation is
y  x  4 . Which one of the following is true?

A. L is tangent to C . C. C is below L .
B. L intersects C at two points. D. C is above L .
15. If the end points of a diameter of a circle are 2, 3 and 5, 1 , then which one of the following
equations represent the circle?

Telegram channel https://t.me/johnson201485


14
4
2
 7
A.  x     y  2  13 C. x 2  y 2  4 y  7 x  13  0
2

 2

B. x 2  y 2  7 y  4 x  13  0 D. x 2  y 2  7 x  4 y  13  0

16. Which of the following equations represents the circle passing through 1, 1 and  1, 1 ?

A. x  12   y  12  8 C. x  1   y  1  2 2
2 2

B. x  12   y  12  8 D. x  1   y  1  2 2
2 2

17. What is the equation of the tangent line to the circle with equation x  3   y  4  20 at P(1, 0) ?
2 2

x  1 D. y  2x  1
1 1
A. y x B. y   C. y  2x  1
2 2

18. Consider a circle whose equation is given by x  3   y  4  20 . Then what is the equation of the
2 2

tangent line to the circle at a point P1, 0  ?


A. x  2 y  1  0 B. 2x  y  1  0 C. x  2 y  1  0 D. 2x  2 y  1  0

Telegram channel https://t.me/johnson201485


14
5
UNIT TEN STATISTICS AND PROBABILITY
Unit Outcomes:

After completing this unit, you should be able to:

 know specific facts about types of data.


 know basic concepts about grouped data.
 know principles of counting.
 apply facts and principles in computation of probability.

Main Contents:
10.1. STATISTICS
10.2. PROBABILITY

Key terms

Summary

Review Exercises

Telegram channel https://t.me/johnson201485


14
6
INRODUCTION

The word statistics comes from the Italian word "statista" meaning stateman. It was used to signify the application
of recorded data for purposes of the state. When statistics is used in its plural sense, it means a body of numerical
facts and figures. The numerical facts are called statistical data, or simply data. When it is used in its singular form,
statistics is a branch of mathematical science, and is concerned with the development and application of methods
and techniques for the collection, organization, analysis and interpretation of quantitative data. We will confine
ourselves to this second meaning of statistics through this unit.
10.1.STATISTICS
10.1.1. Types of Data

ACTIVITY 10.1
1. Classify the following data as qualitative or quantitative:
a. beauty of a picture d. number of children living in a house
b. size of your shoe e. colour of your skin
c. type of a car f. blood type(group)
2. Classify the following variables as discrete or continuous:
a. size of a shirt d. number of rooms in a house
b. number of members of a football club e. heights of students in a class
c. price of a kilo of sugar f. life-time of an electric bulb
The first step in applying statistical methods is the collection of data; this is the process of obtaining counts or
measurements. The data obtained can be classified into two types: qualitative or quantitative data.

Qualitative data (or categorical data), is obtained when a given population or sample is
Definition 10.1: classified in accordance with an attribute that cannot be measured or expressed in numbers,
while Quantitative data is that obtained by assigning a real number to each member of the
population, under study.

Honesty, height, weight, intelligence, income, efficiency, width, sex, pressure, distance, religion, social status.
Solution: Honesty, intelligence, efficiency, sex, religion and social status are qualitative, while height, weight,
income, width, pressure and distance are quantitative.
[If I.Qs (intelligent quotients) are used to measure intelligence, then it will be quantitative.]
Definition 10.2: A number, which is used to describe the attribute and which can take different values is
called a variable.
For example, in your class the height, weight or age of different individuals varies, and can be expressed in
numbers. Therefore, these quantities (height, weight, age,…) are variables.

NOTE:
 Variables are denoted by letters such as x, y, z,...
 A variable may be either discrete or continuous.

Telegram channel https://t.me/johnson201485


14
7
A Discrete Variable is one which takes only whole number values. It is usually obtained by
Definition 10.3: counting. There is a gap between consecutive values i.e. it varies only by finite jumps.
A Continuous Variable is one which takes all real values between two given real values.

Example 2: Which of the following are discrete variables? Which are continuous?
Number of students in a class, weight of students, length of a road, number of chairs in a room, temperature of a
room, the number of heads in a sequence of coin tosses, the result of rolling a die, the number of patients in a
hospital and number of houses along a street.
Solution: Number of students in a class, number of chairs in a room, the number of heads in a sequence of coin
tosses, the result of rolling a die, the number of patients in a hospital and number of houses along a street are
discrete. They can have whole number values only. On the other hand, weight of students, length of a road and
temperature of a room are continuous variables. They can take fractional or decimal values. For instance, weight of
students could be given by values like 50.1kg , 49.73kg ; length of a road could be given by values like 6.5km,

2.63km, while temperature of a room could be given by values like 20o c, 14o c .
NOTE:
1. Discrete data have no decimal places, the average of these values can be fractional. For example,
families can have only a discrete number of children: 1, 2, 3, etc. However, the average number of
children per family can be 2.2.
Frequently, you‟ll use bar charts to graph discrete data because the separate bars emphasize the distinct
nature of each value. However, it‟s appropriate to use other graphs as well.

2. We can use histograms and scatterplots to graph continuous data. These graphs are designed to
handle values that fall on a continuous spectrum and have decimal places.

Telegram channel https://t.me/johnson201485


14
8
Height and Weight are Continuous Variables
Discrete vs. Continuous Data Summary
Discrete Data Continuous Data
Specific values that you cannot divide Infinite number of fractional values between any two values
Counting Measuring
10.1.2.Introduction to Grouped Data
A Frequency distribution is a table which shows the list of all values of data obtained and
the number of times these values occur (frequency). The raw data obtained will be
Definition 13.4: organized and summarized into a grouped frequency distribution table for the purpose of
summarizing a large amount of data.

Example 3: Consider the following data. It represents the number of patients that a doctor visits per day for 150
working days.

Solution: The data given is raw data or ungrouped data. To summarize the raw data into a grouped frequency
distribution, follow these steps:

Steps to prepare a grouped frequency distribution table:


1. Determine number of classes required (usually between 5 and 20).
2. Approximate the interval of each class or class width using the following formula:

Telegram channel https://t.me/johnson201485


14
9
To prepare the frequency distribution, first you decide the number of classes. For this case, let the number of classes
maximum value - minimum value 29  0
be 5. Class interval   Class interval  w   5.8
number of classes required 5

NOTE:
From the formula, the class interval, w, is calculated as 5.8. For practical purposes, it will be convenient to
choose the class interval to be a whole number. For this case, you can take class interval as 6. (This is obtained
by rounding 5.8 to the nearest whole number). Therefore (See the grouped frequency distribution below).

In the above frequency distribution, you are considering frequencies of each class. But, in reality you may be
interested to know about other issues such as how many days the doctor visited fewer than 8 patients. To answer
such a question, the frequency distribution given above may not always be suitable. For such a purpose, you need
to construct what is called a cumulative frequency distribution.
A cumulative frequency distribution is constructed by either successively adding the frequencies of each class called
“less than cumulative frequency” or by subtracting the frequency of each class from the total successively called
“more than cumulative frequency”.
The cumulative frequency distribution of the above data of patients that a doctor visits per day is as follows.

Note that the above frequency distribution is for a discrete variable.

The first and the last elements of a given class interval are called class limits.
Definition 10.5:

Telegram channel https://t.me/johnson201485


15
0
Example 4: For the above table, give the lower- and upper-class limits for the second and the fourth classes.
Solution: For the second class 6 – 11, 6 is called the lower class limit and 11 is called the upper class limit, while
the lower limit and the upper limit of the fourth class are 18 and 23 respectively.

Exercise 0.1

Describe whether each of the following is qualitative or quantitative.


a. Beauty of a student. d. Neatness of our surrounding h. Type of degre
b. Volume of water in a barrel. e. The number of people on a jury. g. City
c. Score of a team in a soccer match. f. My friend is very happy.
1. Identify whether each of the following is discrete or continuous.
a. Yield of wheat in quintals e. The number of pages in a book
b. Rank of students by examination results f. The number of songs in an album
c. Volume of water in a barrel g. The volume of water in a bottle
d. Sex of a student h. The time it takes for a car battery to die

2. The following are scores of 40 students in a statistics exam.

Prepare a grouped frequency distribution, using 7 classes. Answer the following questions.

i. What is the class interval?


ii. What is the lower-class limit of the second class?
iii. What is the upper-class limit of the second class?
iv. What is the frequency of the first class?
v. Determine the cumulative frequency distribution?
vi. What is the frequency of the fourth class?
vii. What is the cumulative frequency at the sixth class?

1. The average of the upper and lower class limit is called the class mark or class midpoint.
The class mark serves as representative of each data value in a class (or the class itself).
Definition 10.6:

2. The correction factor is half the difference between the upper class limit of a class and
the lower class limit of the subsequent class.

Telegram channel https://t.me/johnson201485


15
1
Example 5: For the following distribution which shows the test scores of 60 students in a mathematics test corrected out of
100, give the correction factor.

Solution: In this distribution, the correction factor i 1  26  25  0.5 or 1  51  52   0.5 or 1  76  75   0.5
2 2 2
Why do you need the correction factor?
Previously, you saw that a cumulative frequency distribution of discrete values may help to answer some questions. But, there
could be more questions to answer. For example, in Example 5 above, suppose you are asked „To which class does a mark of
9.5 belong? Or, how many students have scored less than 9.5?‟ To solve such problems, you have to smoothen the distribution
and fill the gaps. In order to smoothen, you add the correction factor to the upper limits of each class and subtract from the
lower class limits of each class to get what are called class boundaries.
Then the class 25.5–50.5 includes variable values that are 25.5 and above, but below 50.5.
Steps to construct a frequency distribution:
1. Find the highest and lowest values. 3. Select the number of classes desired
2. Find the range (i.e., highest value – lowest value).
4. Find the class interval by dividing the range by the number of classes and rounding up.
5. Select a starting point (usually the lowest value); add the class interval to get the lower limits.
6. Find the upper class limits. 8. Find the frequencies.
7. Tally the data. 9. Find the cumulative frequency.
Example 6: Construct the frequency distribution table for the following data using 6 classes.
12 18 27 31 40 42 14 20 27 32 40 51
56 40 32 27 20 14 21 29 14 34 40 60
16 23 31 36 45 65
Solution:
 Find the highest and lowest values=Highest value  65 and lowest value  12
 Calculate the range of the data set=Range  65 12  53
 Select the number of classes desired= Number of classes desired is 6.
 Class interval 
53
 8.83 . Round
8.83 to 9 which is called class width.
6
Now construct a frequency distribution table.
Data (Interval) Frequency Cumulative frequency
12  20 8 8
21  29 6 14
30  38 6 20
39  47 6 26
48  56 2 28
57  65 2 30

Telegram channel https://t.me/johnson201485


15
2
Exercise 10.2
1. A teacher in a school has given a project to her students to make a survey of the size of two kinds of trees in a
forest nearby. The following is the frequency table that the students made about the circumference of 100
randomly selected trees of each of two kinds A and B.

i. What is the class interval? iv. What is the frequency of the first class?
ii. What is the lower class limit of the second class? v. Complete the following table about Tree type A.
iii. What is the upper class limit of the second class?
vi. Make a similar table for Tree type B.
vii. Draw histograms to illustrate both frequency distributions.

10. The following are yield in quintals of wheat harvested by thirty farmers per hectare.

Prepare a grouped frequency distribution, using 11 classes. Answer the following questions.
a. What is the lower class limit for the third class?
b. What is the lower class boundary for the seventh class?
c. Determine the correction factor for this frequency distribution.
d. What is the class mark of the second class?
e. Find the difference between the class marks of the eighth and ninth classes.

Telegram channel https://t.me/johnson201485


15
3
10.1.3.Measures of Location for Grouped Data
When you want to make comparisons between groups of numbers, it is good to have a single value that is
considered to be a good representative of each group. One such value is the average of the group. Averages are also
called measures of location or measures of central tendency. The most commonly used measures of central tendency
are Mean (or Arithmetic mean), Median, Mode, Quartiles, Deciles and Percentiles.
First, let us recall the summation notation. Let x1 , x2 , x3 ,..., xn be a number of values where n is the total number
th
of observations with xi the i observation.
n
The symbol  x is called sigma or the summation notation and i
i 1
i is called an index, with I  1 the starting index
n
and I  n the ending index. Thus, x
i 1
i  x1  x2  x3  ...  xn .

The Mean(or Arithmetic Mean)

The mean of a set of data is equal to the sum of the data items divided by the number of
items contained in the data set. If are values, then their mean is given by

Definition 10.6:

If is a set of data items, with frequencies with frequencies

, respectively, then their mean is given by

Example 7: Calculate the mean of the following data:

a. 3,  7, 5, 13,  2 b. 7, 7, 2, 6, 7, 10, 8, 10, 2, 7, 10, 7, 2, 7, 6, 10, 6, 7, 8, 7, 6, 7, 10, 6, 10

Solution:

x1  x2  x3  ...  xn 3   7   5  13   2 
a. x   2.4
n 5
b. Let us prepare a frequency distribution table.

x 2 6 7 8 10

f 3 5 9 2 6

Telegram channel https://t.me/johnson201485


15
4
2  3  6  5  7  9  8  2  10  6 6  30  63  16  60
x  7
359 2 6 25

Example 8: A group of 5 water tanks in a farm have a mean average height of 4.7 metres. If a sixth water tank
with a height of 2.91 metres is erected, what is the new mean average height of the water tanks?

Solution: Let x1 , x2 , x3 , x4 , x5 be the five water tanks and x6 be the sixth water tank.

x1  x2  x3  x4  x5
From the given we have  4.7  x1  x2  x3  x4  x5  5   4.7   23.5
5
x1  x2  x3  x4  x5  x6
The new mean average height of the water tanks is given by
6
x1  x2  x3  x4  x5  2.91 23.5  2.91 26.41
 but x1  x2  x3  x4  x5  23.5    4.402
6 6 6
Therefore, new mean average height of the water tanks is 4.402meters .

NOTE:
To calculate the mean for two graphs of data, first we need to calculate the number of values of each group and
also the total number of values.
Generally, to find the mean of two groups of data,

Example 9: One group of 8 students has a mean average score of 67 in a test. A second group of 17 students has
a mean average score of 81 in the same test. What is the mean average of all 25 students?

Mean1  Number of values of 1st group  Mean 2  Number of values of 2nd group
Solution: Mean 
Total Number of values obtained from the groups
67  8  8117 536  1377
 Mean    76.52
25 25

NOTE: Properties of the mean

1. Given values of a population function, if the mean is , then


.
2. If a constant is added to each value of a population function, then the new mean is equal to the
old mean plus .
3. If each value of a population function is multiplied by a constant , then the new mean is equal to
the old mean times .
4.
MeanThefor
mean of a sumdata
grouped or difference of a population function is the sum or difference of the means.

Telegram channel https://t.me/johnson201485


15
5
The procedure for finding the mean for grouped data is similar to that for ungrouped data, except that the mid points
of the classes are used for the x values.
Example 10: Calculate the mean average of this grouped frequency table for students‟ test scores.

Solution: If you have to use what you know so far to calculate the mean, we need to know the total number of
students that took the test and the total number of marks that they scored.
The total number of students is 100, but we have a problem when it comes to the total number of marks. Since you
have grouped data, you cannot obtain individual marks. For instance, 13 students scored between 26 and 30. But,
there is no way you can tell the total mark of the 13 students.
The way out of this problem is to approximate each student‟s mark by the middle mark of the class interval, as in
the following table:

Now, total number of students  100 ; Total marks (approximate)  2455 .


2455
Therefore, approximate mean   24.55 .
100

Steps to find the mean from a grouped distribution:

lower class limit  upper class limit


1. Find the class mark (mid-point) xc for each class, by .
2
2. Multiply xc by its corresponding frequency and add.
n

f1 xc1  f 2 xc2  ...  f n xcn fx i ci


3. Divide the sum obtained in step 2 by the sum of the frequencies. x   i 1

f1  f 2  ...  f n n

f
i 1
i

Telegram channel https://t.me/johnson201485


15
6
Example 11: The following is the age distribution of 20 students in a class. Find the mean age of these students.

Solution: The mean x 


 fx
c

490
 24.5 years .
f 20

Example 12: The following is the frequency distribution of a grouped data. Find the mean of the data.
Class Interval Frequency  f 

37 2
8 12 2
13  17 10
18  22 6
Solution:
Class Class mid- Frequency fxc
Interval point  xc  f
37 5 2 10
8 12 10 2 20
13  17 15 10 150
Therefore, the mean
18  22 20 6 120
 fx 300
x   15 .  f  20  fx  300
c

f 20 c

The procedure for finding the mean for grouped data assumes that all of the raw data values in each class are equal
to the class mark of the class. In reality, this is not true. However, using this procedure will give us an acceptable
approximation of the mean, since some values usually fall above the class mark and others fall below the class mark
for each class.

Telegram channel https://t.me/johnson201485


15
7
Exercise 10.3
1. Find the mean for the following frequency distribution tables.
a. b. c.
Weight(kg) Frequency Marks Frequency Age Frequency
10 12 4 13  15 6
13  15 7 16  18 6
16  18 10 19  21 3
19  21 13 22  23 2
23  25 16
2. Forty-six randomly selected light bulbs were tested to determine their life time (in hours) and the following
frequency distribution was obtained. Find the mean hour of life time.

3. The following are quintals of fertilizer distributed to fifty farmers.

a. Find the average number of quintals of fertilizer distributed to the farmers from the raw data.
b. Prepare discrete frequency distribution and calculate the mean.
4. Using the data given in Question 3 prepare two grouped frequency distributions, using 6 and 9 classes.
Answer the following questions.
i. Find the mean of each. iii. Write your generalizations.
ii. Are the four means you calculated equal?
5. A school has three classrooms for grade 11, namely, 11A, 11B and 11C . The number of students in these

classrooms is 28, 20 and 22, respectively. All the students took examination and the average score of the

students of 11A, 11B and11C is 60, 70 and 70, respectively. What is the average score in this examination for

all grade 11 students?

Telegram channel https://t.me/johnson201485


15
8
The median (md): You should remember that median of a set of data is the middle number when the data is
arranged in either increasing or decreasing order of magnitude. It is a half-way point in a data set, when the data is
arranged in order (called a data array). The median will be a value in the data or will fall between two values.
Example 13:
a. The following data shows the age to the nearest year of 7 students in a class. What will be the median of
this age distribution? 6, 8, 5, 6, 10, 7, 3 .
b. Find the median from the following data. 60, 63, 59,72,50, 49 .

Solution:
a. Arranging in an increasing order gives 3, 5, 6, 6, 7, 8, 10. Since the number of observations is 7 and this
 n 1   7 1 
th th

number is odd, therefore, md    item    item  4 item which shows the median is 6.
th

 2   2 
b. First you have to arrange in increasing order giving, 49, 50, 59, 60, 63, 72. Since n = 6, which is even, you
th th th th
n n  6 6 
  item    1 item   item    1 item
will use the second formula: md    2    2 
2 2
2 2
3 item  4 item 59  60 119
th th
md     59.5
2 2 2
Exercise 10.4
1. Consider the following data which shows the amount of milk in liters sold by a farmer in one month.

a. Find the median from the raw data. b. Prepare a frequency distribution table.
2. The following data shows score of fifty students in Mathematics exam

a. Find the median from the raw data.


b. Prepare a discrete frequency distribution table and calculate the median.
3. Find x if;
a. 15, 12, 11, x  6, 7, 5, 4 are in order and their median is 10.
b. 8, 10, x 1, x  3, 17,19 are in order and their median is 14.

Telegram channel https://t.me/johnson201485


15
9
Median for grouped data: So far, you have seen how to find the median from raw data and, from the above
Exercise, you should have been able to find the median from a single valued frequency distribution table. In the
next part, you will see the steps to find the median of a grouped frequency distribution.
Steps to find the median of a grouped frequency distribution:
1. Prepare a cumulative frequency distribution and Find the class where the median is located. It is the lowest
n
class for which the cumulative frequency equals or exceeds
2
n 
2. Determine the median by the formula   cf b 
md  BL   2 i
 fc 
 
where, BL  Lower boundary of the class containing the median (median class)
n  total number of observations   f  .
cfb  the cumulative frequency in the class preceding ("coming before") the class containing the median.
fc  the number of observations (frequency) in the class containing the median.
i  the size of the class interval.(i.e. width of the median class).

NOTE: First use the correcting factor to prepare a cumulative frequency table.
Example 14: Find the median of the following distribution.
Class Interval Frequency ( )
1 4 8
58 12
54
Solution: The correcting factor is  0.5 (uniform for all 9  12 3
2
classes). 13  16 25
From this, you can prepare the class boundary column and the 17  20 13
cumulative frequency column as follows. 21  24 7

Class Interval class boundaries Frequency  f (Cumulative frequency)

1 4 0.5  4.5 8 8
58 4.5  8.5 12 20
9  12 8.5 12.5 3 23
13  16 12.5 16.5 25 48
17  20 16.5  20.5 13 61
21  24 20.5  24.5 7 68
 f  68

Telegram channel https://t.me/johnson201485


16
0
The median class is that class containing the  68  item  34th item . It is the third class.
th

 
 2 
Therefore the median class is 12.5 16.5
n
Thus, BL  12.5,  34, f c  25, i  16.5  12.5  4, cfb  23 .
2
Therefore,  n
  cf 
b

 34  23 
 md  12.5  1.76  14.26
md  BL   2  i  12.5   4
 fc   25 
 
The median is 14.26 .
Example 15: The following is the height of 30 students in a class. Find the median height.

146  145
Solution: The correcting factor is  0.5 (uniform for all classes).
2
From this, you can prepare the class boundary column and the cumulative frequency column as follows.

The median class is that class containing the  30  item  15th item . It is the second class.
th

 
 2 
Therefore the median class is 145.5 151.5.
n
Thus, BL  145.5,  15, f c  9, i  151.5  145.5  6, cfb  7 .
2
 n 
Therefore,   cf b   15  7 
 md  145.5  5.333  150.83
md  BL   2  i  145.5   6
 fc   9 
 
The median height is 150.83 .

Telegram channel https://t.me/johnson201485


16
1
Exercise 10.5

The following data shows Age of forty students in a class.

a. Find the median from the raw data.


b. Construct a grouped frequency distribution, with 5 classes.
c. Find the median from the frequency distribution table.
1. Calculate the median of each of the following sets of data about students in a class.

2. The mark that students scored in an examination is grouped in class intervals as shown in the following
table. What is the median of the mark?

Class Interval (Mark) Number of Students

55  64 8
65  74 12
75  84 20
85  94 6
95  104 4
3. The amounts of drops of water in drip irrigation were registered from 80 sample drip holes in one day and the
data are as follows.

Telegram channel https://t.me/johnson201485


16
2
a. Find the median from the raw data.
b. Construct a grouped frequency distribution, with 10 classes and find the median.
3. Calculate the median of the following sets of data about score of students in an exam.

a. Find the mean and median score of the students. b. Compare the mean and the median.

The mode (mo ) In statistics, the word mode represents the most frequently occurring value in a data set.

The Mode of a set of data is the value in the data which appears most frequently in the
Definition 10.7: set of values.

NOTE: The mode can usually be determined by observation. It is possible for distribution to have more than
one mode or no mode at all.

Example 16: Find the mode of each of the following.


a. 3, 6, 14, 17,19, 22 c. 4, 8, 7, 4,8, 2,3
b. 2, 5, 6,5, 4, 2,3, 2

Solution:

a. Every member appeared only once. Hence there is no mode for this distribution.
b. In this observation, the most frequent value is 2. Therefore, the mode is mo  2 since it appears three

times. This data has only one mode and is called unimodal.

Telegram channel https://t.me/johnson201485


16
3
c. Here both 4 and 8 appear twice but the rest appear only once. Hence the modes are 4 and 8. This
distribution has two modes. Such distributions are said to be bimodal.

xample 17: Given that 1, 2, x,5, y,8 . Find the values of x and y if the mode of the resulting numbers is 5 and the

mean is 4 .

1 2  x  5  y  8
Solution: The mean of the numbers is 4  4
6

 x  y  16  24

 x  y  8 …….(1)

Since the mode of the numbers is 5 either x  5 or y  5 .

Let y  5 . Now substitute y  5 in equation (1) above, then x  3 .

Therefore, x  3 and y  5 .

Exercise 10.6
1. The following represent days in a month at which salary was paid for forty-two consecutive months.

a. What is the mode of this data?


b. At which date is salary paid mostly?
2. The following data represents shoe sizes of different models of shoes displayed in a boutique.

a. Determine the mode shoe size in the shop? b. What does this mode describe?
3. Calculate the mean and the mode.

Mode of grouped data: NOTE:


Before we find any mode(s) that might exist, check the following points:
1. The class interval of all classes should be equal (Uniform class interval).
2. We need a column of class boundaries which can be obtained from the class limits.

Telegram channel https://t.me/johnson201485


16
4
Steps to calculate the modal value from grouped data:

1. Identify the modal class. It is the class with the highest frequency.

 d1 
2. Determine the mode using the following formula: Mode  mo  BL   i
 d1  d 2 
where, BL  lower class boundary of the modal class.

d1  the difference between the frequency of the modal class and the frequency of the preceding class (pre-modal
class).
d2  the difference between the frequency of the modal class and the frequency of the subsequent class (next
class).
i  the size of the class interval.

Example 18: The following table gives the age distribution in a certain class. Compute the modal age (in years).
Solution: The modal class is the 3rd class because its frequency is the largest.
BL  19.5, d1  10  6  4, d2  10  2  8, i  24 19  5

 d1 
mo  BL   i
 d1  d 2 
 4 
 mo  19.5   5
 48
20
 mo  19.5   19.5  1.67  21.17 years
12
 mo  21.17 years
2
Example 19: The following is a distribution of the size of farms (in 1000m ) in a woreda. Find the mode of the
distribution.

Size of farm 5 14 15  24 25  34 35  44 45  54 55  64 65  74


No of farms 8 12 17 29 31 5 3
th
Solution: The modal class is the 5 class because its frequency is the largest.
BL  44.5, d1  31  29  2, d2  131  5  26, i  54.5  44.5  10

 d1 
mo  BL   i
 d1  d 2 
 2  5
 mo  44.5   10  mo  19.5   45.21
 2  26  7

Telegram channel https://t.me/johnson201485


16
5
Exercise 10.6

1. The daily profits (in Birr) of 100 shops are distributed in the following table. Find the modal value.

2. The amounts of drops of water in drip irrigation were registered from 80 sample drip holes in one day and
the data are as follows.

b. Find the mode from the raw data.


c. Construct a grouped frequency distribution, with 10 classes and find the mode.
3. The number of farmers who got a prize for their productivity and the type of prize they got is given as
follows. Determine the mode prize.

Quartiles, deciles and percentiles


The median divides a distribution into two equal halves. There are other measures that divide the data into four, ten
and a hundred equal parts. These values are called quartiles, deciles and percentiles, respectively. These measures,
which are recognized as measures of location, will be discussed for both ungrouped and grouped data.
Quartiles, deciles and percentiles for ungrouped data
Quartiles: are values that divide a set of data into four equal parts. There are three quartiles, namely, Q1 , Q2 and
Q3
Steps to calculate quartiles for ungrouped data:
1. Arrange the data in increasing order of magnitude.
2. If the number of observations is:

Telegram channel https://t.me/johnson201485


16
6
th
  kn   kn  
 k  n  1 
th
  4    4  1 
     item
a. odd, Qk    item. b. even, Qk   .
 4   2 
 
 
Example 20: Find Q1 and Q3 for the following data. 25, 38, 42, 46, 31, 29, 21, 9, 5.

Solution: Arranging in increasing order of magnitude, we get,


 k  n  1   1 9  1 
th th

Qk    item  Q1    item  Q1   2.5  item


th

 4   4 
This means Q1 is half way between the 2 nd and 3rd items .

Therefore, Q1  2nd item 


2

1 rd
3 item  2nd item   9   21  9   9  6  15
1
2
 3  n  1 
th
 3 10 
th

Q3    item    item   7.5  item.


th

 4   4 
It is half way between the 7  x7  and 8th  x8  items .
th

Therefore, Q3  x7  0.5  x8  x7   38  0.5  42  38  38  2  40


Deciles are values that divide a set of data into ten equal parts. There are nine deciles, namely,
D1 , D2 , D3 , D4 , D5 , D6 , D7 , D8 , D9 .
Steps to calculate deciles for ungrouped data:
1. Arrange the data in increasing order of magnitude.
2. If the number of observations is:
th
  in   in  
 i  n  1 
th
  10    10  1 
     item
a. odd, Di    item. b. even, Di   .
 10   2 
 
 
Example 21:Find D2 and D7 for the following data: 25, 38, 42, 46, 50, 31, 29, 21, 9, 5.

Solution: Arranging in increasing order of magnitude, we get, 5, 9, 21, 25, 29, 31, 38, 42, 46, 50.
th
  in   in  
  10    10  1 
Di        item
 2 
 
 

Telegram channel https://t.me/johnson201485


16
7
  2 10    2 10   
th

   1 
 23
th
  10   10   item  D2  
 D2   item=2.5 item  15 and
th
 2   2 
 
 

  7 10    7 10   
th

   1 
  item   7  8  item  7.5th item  40
th

D7    10   10
   
2  2 
 
 
3. Percentiles
Percentiles are values that divide a data set into a hundred equal parts. There are ninety nine percentiles, namely,
P1 , P2 , P3 ,..., P99 . Percentiles are not the same as percentages. If a student gets 85 correct answers out of a possible
100, he obtains a percentage score of 85. Here there is no indication of his position with respect to other students.
th
On the other hand if a score of 85 corresponds with the 96 percentile, then this score is better than 96% of the
students under consideration. Were your average and percentile in your grade eight exams the same?
Steps to calculate percentiles for ungrouped data:
1. Arrange the data in increasing order of magnitude.
2. If the number of observations is:

 t  n  1 
th

a. odd, Pt    item.
 100 
th
  tn   tn 
  100    100  1 
     item
b. even, Pt   .
 2 
 
 
Example 22: Find P42 and P75 for the following data. 25, 38, 42, 46, 50, 31, 29, 21, 9, 5.

Solution: Arranging in increasing order of magnitude, we get, 5, 9, 21, 25, 29, 31, 38, 42, 46.
Number of observation is odd.

 t  n  1 
th

So, Pi    item.
 100 

 42  9  1 
th
 42 10 
th

P42    item    item  4.2 item


th

 100   100 

Here P42 is between the 4 and 5 item, i.e. x4  0.2  x5  x4 


th th

Telegram channel https://t.me/johnson201485


16
8
Therefore, P42  25  0.2  29  25  25  0.2  4  25  0.8  25.8

 75  9  1 
th
 75 10 
th

P75    item    item  7.5 item  40


th

 100   100 

Note that P75  40 . That is, 75% of the data values are less than P75 and the rest are above it.

Quartiles, deciles and percentiles for grouped data


You have just discussed quartiles, deciles and percentiles for ungrouped data. When we have a very large set of
data, grouping the data in a frequency distribution will make it easier.
th
1. Quartiles: The k quartile for a grouped frequency distribution is:

 kn 
  cfb 
Qk  k th quartile   BL   4  i , k  1, 2, 3 and
 f k 
 
BL  lower class boundary of the k th quartile class

cfb  the cumulative frequency before the k th quartile class

fk  the number of observations (frequency) in the k th quartile class


i  the size of the class interval
Steps to calculate quartiles for grouped data:
1. Prepare a cumulative frequency distribution.
th
th  kn 
2. Find the class where the k quartile belongs: the   item .
 4
3. Use the formula above.
Example 23: The following is the distribution of the marks of students in Mathematics test out of 50 . Find the
first quartile, Q1 of the following distribution.

Range of marks 20  25 26  31 32  37 38  43 44  49

Frequency 15 30 25 20 10

Telegram channel https://t.me/johnson201485


16
9
Solution: First prepare a cumulative frequency distribution table

Range of marks Frequency Cumulative


frequency
20  25 15 15
26  31 30 45
32  37 25 70
38  43 20 90
44  49 10 100
th
 kn 
Number of observations n  100 , Qk is the   item
 4
th
 100 
 Q1    item  25 item which falls in the 2 class. cfb  15, f1  30 and i  6
th nd

 4 
 kn 
 4  cfb 
Qk  BL   i
 fk 
 

 1100  
  15   25  15  6  25.5  10  6  25.5  2  27.5
 Q1  25.5   4  6  25.5 
 30  30 30
 
 

Therefore, Q1  27.5

Example 24: Find Q1 , Q2 and Q3 of the following distribution.

th
 kn 
Solution: n  40 and Qk is the   item
 4

Telegram channel https://t.me/johnson201485


17
0
th
 40 
Q1    item  10th item which falls in the 2 nd class. cfb  5, f1  7 and i  5
 4 

 kn 
 4  cfb 
Qk  BL   i
 fk 
 

 1 40  
 5 10  5 5  24.5  5  5  24.5  25 =28.07
 Q1  24.5   4  5  24.5 
 7  7 7 7
 
 

 2  40 
th

Q2    item  20 item. Q2 (the same as the median) is found in the 3 class.


th rd

 4 
 2  40  
  12   20  12  5  29.5   8  5  29.5  5  34.5
Q2  29.5   4  5  29.5   
 8  8 8
 
 

 3  40 
th

Q3    item  30 item. It is found in the 4 class.


th th

 4 
 3  40  
  20   30  20  5  34.5   10  5 
Q3  34.5   4  5  34.5   
 18  18  18 
 
 
 Q3  34.5  2.78  37.28

2. Deciles: The j th deciles for grouped frequency distributions is calculated in a similar way as follows.
Steps to calculate deciles for grouped data:
th
 jn 
1. Find the class where the j th quartile belongs, which the class that contains the   item .
 10 
 jn 
  cfb 
Use the formula D j  j decile   BL   10  i, j  1, 2,...,9
th
2.
 f c 
 
Where BL  lower class boundary of the j th decile class. and n  f
cfb  the cumulative frequency before the j th decile class.
fc  the number of observations (frequency) in the j th decile class.

Telegram channel https://t.me/johnson201485


17
1
i  the size of the class interval
Example 25: Find D3 , D7 and D9 of the following data

Solution:
 3  36 
th

 item  10.8 item. It is found in the 2 class.


th nd
a. D3 is 
 10 
 3  36 
 10  6 
So, D3  49.5   10  49.5  4.8  54.3 .
 10 
 
7  36 
th

b. D7 is   item   25.2  item. It is found in the 3 class.


th rd

 10 
 7  36 
 16 
So, D  59.5   10 .
7  10  59.5  5.41  64.91
 17 
 

 9  36 
th

 item   32.4  item which is found in the 3 class.


th rd
c. D9 is 
 10 
 9  36 
 10  16 
So, D9  59.5   10  59.5  9.65  69.15 .
 17 
 
3. Percentiles
The j th percentile for grouped frequency distributions is calculated in a similar way as follows:
Steps to calculate percentile for grouped data:
th
th  jn 
1. Find the class where the j percentile belongs, which the class that contains the   item .
 100 
 jn 
 10  cfb 
Use the formula Pj  j decile   BL    i, j  1, 2,...,99
th
2.
 fc 
 
3. Where BL  lower class boundary of the j th percentile class.

n f and cfb  the cumulative frequency before the j th percentile class.

Telegram channel https://t.me/johnson201485


17
2
fc  the number of observations (frequency) in the j th percentile class.
i  the size of the class interval
Example 26: Find P20 , P50 and P68 for the following frequency distribution.

Solution:

 20  36 
th

 item   7.2  item. It is found in the 2 class.


th nd
a. P20 is 
 100 
 20  36 
 100  6 
So, P20  49.5   10  49.5  1.2  50.7 .
 10 
 

 50  36 
th

 item  18 item. It is found in the 3 class.


th rd
b. P50 is 
 100 
 50  36 
 100  16 
So, P50  59.5   10  59.5  1.17  60.67 .
 17 
 

 68  36 
th

 item  24.48 item which is found in the 3 class.


th rd
c. P68 is 
 100 
 68  36 
 16 
So, P  59.5   100 .
50  10  59.5  4.99  64.49
 17 
 
Example 27: As shown in the table below, a measurement is grouped into five class intervals with the frequency
distribution.

Class interval 45  55 55  65 65  75 75  85 85  95
Frequency 26 38 33 16 7

th
Find the first quartile Q1 and the 75 percentile P75 of the measurement.

Telegram channel https://t.me/johnson201485


17
3
Solution: First prepare a cumulative frequency distribution table
Class interval Frequency Cumulative
frequency
45  54 26 26
55  64 38 64
65  74 33 97
75  84 16 113
85  94 7 120
th
 120 
a. Q1    item  30 item which falls in the 2 class. cfb  26, f1  38 and i  10
th nd

 4 
 1120  
  26   30  26 10  54.5  1.05  55.55
 Q1  54.5   4 10  54.5 
 38  38
 
 
Therefore, Q1  55.55

 75 120 
th

 item  90 item which is found in the 3 class.


th rd
b. P75 is 
 100 
 75 120 
 100  64 
So, P75  64.5   10  64.5  7.88  72.38 .
 33 
 

Exercise 10.7
1. Find Q1 , Q2 and Q3 for each of the following data sets:

a. 78, 68, 19, 35, 46, 58, 35, 35, 31, 10, 48, 28

b.

c.

Telegram channel https://t.me/johnson201485


17
4
2. Find Q1 , Q2 and Q3 for each of the following data. It is a distribution of marks obtained in a mathematics

exam (out of 40).

a. Find Q2  Q1 , Q3  Q2 and Q3  Q1 . Write your conclusion.

b. From the above data, if students in the top 25% are to be awarded a certificate, what is the minimum mark for
a certificate?
c. If students whose scores are in the bottom 25% of the marks are considered as failures, then what is the
maximum failing mark?
d. Find the percentile of the farmers who received more than 20 quintals.
e. If a farmer receives more than 75 percentile, find the minimum amount of quintals of fertilizer s/he receives.
3. Find Q2 , Q3 , D4 , D8 , P12 , P24 , P87 for each of the following data sets:

a.

b.
Class Interval

Frequency

10.1.4. Measures of Dispersion


The measures of central tendency are not adequate to describe data. Two data sets can have the same mean but
they can be entirely different. Thus to describe data, one needs to know the extent of variability. This is given by
the measures of dispersion.
The degree to which numerical data is spread about an average value is called
the variation or dispersion of the data.
Definition 10.8:
Range, Variance and standard deviation are the three commonly used measures of dispersion.
1. Range: Range is the difference between the maximum and the minimum values in a data set.
Range  xmax  xmin
Example 28: Find the range of
a. 12, 20, 25, 81,67, 45, 54 b.
Solution:
a. xmax  81, xmin  12 . Range  xmax  xmin  81 12  69

Telegram channel https://t.me/johnson201485


17
5
b. Range  8  4  4
Range for grouped data

Range for grouped data is defined as the difference between upper class boundary of the
highest class Bu(H) and the lower class boundary of the lowest class Bl(L). That is,
Definition 10.9: R=Bu(H)- Bl(L).

Example 29: Consider the following data, what is the range of this distribution?

Solution: From the grouped frequency distribution, the range is calculated using;

Bu  H   22.5 and Bl  L   4.5

 R  Bu  H   Bl  L   22.5  4.5  18
Advantages and limitations of range
Advantage of Range
 It is simple to compute
 It is fixed value
Limitation of Range
 It only depends on extreme values.
 It doesn‟t consider variations of values in between.
 It is highly affected by extreme values.
 It does not tell us anything about the variability of other data.
2. Variance and standard deviation
The standard deviation is the most commonly used measure of dispersion. The value of the standard deviation tells
how closely the values of a data set are clustered around the mean. In general, a lower value of the standard
deviation for a data set indicates that the values of the data set are spread over a relatively small range around the
mean. On the other hand, a large value of the standard deviation for a data set indicates that the values of that data
set are spread over a relatively large range around the mean.
Variance is a measure of dispersion that takes into account the spread of all data points in a data set. It‟s the
measure of dispersion the most often used, along with the standard deviation, which is simply the square root of
the variance. The variance is mean squared difference between each data point and the centre of the distribution
measured by the mean.
Definition 10.10: Variance is the average of the squared deviation of each item from the mean.

Telegram channel https://t.me/johnson201485


17
6
Properties of Variance
 It is always non-negative since each term in the variance sum is squared and therefore the result is either
positive or zero.
 Variance always has squared units. For example, the variance of a set of weights estimated in kilograms
will be given in kg squared. Since the population variance is squared, we cannot compare it directly with the mean
or the data themselves.
Variance for ungrouped data
If x1 , x2 , x3 ,..., xn are n observed values, then variance for the sample data is given by
n

 x  x 
2

 x1  x    x2  x   ...   xn  x 
2 2 2 i
Variance ( s 2 )   i 1

n n
where, x  mean , s  variance and n  number of values
2

The quantities x  x is in the above formula are the deviations of x from the mean.
Steps to calculate variance for ungrouped data:
a. Calculate the mean of the distribution.
b. Find the deviation of each value from the mean and square it.
c. Add the squared deviations.
d. Divide the sum obtained in step c by n .
Definition 10.11: The positive square root of variance is called standard deviation.
Properties of Standard Deviatio: It describes the square root of the mean of the squares of all values in a
data set and is also called the root-mean-square deviation.
 The smallest value of the standard deviation is 0 since it cannot be negative.
 When the data values of a group are similar, then the standard deviation will be very low or close to zero.
But when the data values vary with each other, then the standard variation is high or far from zero.
n

 x  x 
2
i
Standard deviation  sd   variance  sd  i 1

n
Example 30: Find the variance and standard deviation of the data 20, 16, 12, 8, 18, 5, 9, 24.
20  16  12  8  18  5  9  24
Solution: The mean x   14
8

x  x 
2
 302

Telegram channel https://t.me/johnson201485


17
7
n

 x  x 
2

 x  x    x2  x   ...   xn  x 
2 2 2 i
302
Variance ( s 2 )  1  i 1
 s2   37.75
n n 8
Standard deviation  sd   variance
n

 x  x
2
i
 sd  i 1
 37.75  6.14
n
Example 31: Find the variance and standard deviation of the data 6, 7, 10, 11, 11, 13, 16, 18, 25.
6  7  10  11  11  13  16  18  25
Solution: The mean x   13
9
n

x  x    x  x   ...   x  x  
xx x  x x  x
2 2
x

2 2 2 i

6 7 49 Variance ( s )  2 1
 2 n i 1

n n
7 6 36 280
10 3 9  s2 
 31.11
9
11 2 4
Standard deviation  sd   variance
11 2 4
13 0 0 n

 x  x 
2
i
16 3 9
 sd  i 1
 31.11  5.58
18 5 25 n
25 12 144 If x1 , x2 , x3 ,..., xn are values with corresponding frequencies
x  x   280
2
f1 , f 2 , f3 ,..., f n , the variance is given by

  xi  x
2
fi
f1  x1  x   f 2  x2  x   f3  x3  x  ...  f n  xn x
2 2 2 2

Variance ( s 2 )   i 1
n


n
fi
i 1

Steps to calculate variance from frequency distribution:


a. Calculate the mean of the distribution.
b. Find the deviation of each value from the mean and square it.
c. Multiply the squared deviations by their corresponding frequencies and add.

d. Divide the sum by f i .

Example 32: Find the variance and standard deviation of the following data.

Telegram channel https://t.me/johnson201485


17
8
165
Solution: The mean x   6.88
24
n

 f x  x
2
i i
146.63
Variance ( s 2 )  i 1
n  s2   6.11 and  sd   variance  6.11  2.47
f i 1
i
24

NOTE: In a grouped frequency distribution, every class is represented by its


Variance for grouped data
class mark or class nmidpoint.
 f x  x
2
i i
The variance for grouped data is given by, s 2  i 1 where xi is the midpoint of each class (class
n

f i 1
i

mark).
Steps to calculate variance from a grouped frequency distribution:
a. Find the class mark for each class.
b. Find the mean of the grouped data.
c. Find the deviation of each class mark from the mean and square it.
d. Find the sum of the squared deviations, then Divide the sum obtained in step d by f i .

Example 33: The following set of raw data shows the lengths, in millimeters, measured to the nearest mm, of 40
leaves taken from plants of a certain species. This is the table of frequency distribution.
Length (mm) Frequency  fi 
25  29 2
30  34 4
35  39
n
7
Solution: The mean x 
fx i ci
1720
40  44 10 i 1
n
  43
f
40
45  49 8 i 1
i

50  54 6
55  59 3

Telegram channel https://t.me/johnson201485


17
9
Length (mm) Frequency class mark fxi xi  x  xi  x 
2
f i  xi  x 
2

 fi   xi 
 xi  43  xi  432 f i  xi  43
2

25  29 2 27 54 16 256 512


30  34 4 32 128 11 121 484
35  39 7 37 259 6 36 252
40  44 10 42 420 1 1 10
45  49 8 47 376 4 16 128
50  54 6 52 312 9 81 486
55  59 3 57 171 14 196 588

 f  40  f x  1720
i i  f x  x
i i
2
 2460

 f x  x
2
2460
 Variance ( s 2
) i i
  61.5
f i 40

 f x  x
2
2460
Standard deviation  sd     61.5  7.84
i i

f i 40

Example 34: Find the variance and standard deviation of the following distribution.

Solution: The mean x  fx


i i

105
7
f i 15

 f x  x
2
250
Variance ( s 2
) i i
  16.67
f i 15

 f x  x
2
250
Standard deviation  sd     16.67  4.08
i i

f i 15

Telegram channel https://t.me/johnson201485


18
0
Merits and Demerits of standard deviation
Merits: It is rigidly defined and free from any ambiguity.
 It is based on all observations.
 It exhibits the scatter of dispersion of the various items of a series form its arithmetic mean and thereby
justifies its name as a measure of dispersion.
Demerits: It attaches great weight to extreme values, as squared deviations are used.
 The process of squaring deviations and then taking the square root of their mean is complicated.
 It cannot be used for comparing the dispersion of two, or more series given in different units.

NOTE: Properties of the variance and standard deviation


1. If a constant is added to each value of a population function, then the new variance is the same as the old
variance. The new standard deviation is the same as the old standard deviation.
2. If each value of a population function is multiplied by a constant , then the new variance is times the
Exercise 13.8
1. old variance and thethe
Find new standard
range, deviation
variance is times
and standard the oldfor
deviation standard
each ofdeviation.
the following data.
a. 18, 2, 4, 6, 10, 7, 9, 11

b.

c.

2. If the standard deviation of x1 , x2 , x3 ,..., xn is 3 , then what is the standard deviation of

2 x1  3, 2x2  3, 2x3  3,..., 2 xn  3 ?


3. The standard deviation of the temperature for one week in a certain city is zero. What can you say about
the temperature of that week?
4. Two basketball players scored points for their team. The scores were recorded for 9 games as follows:

a. Calculate the standard deviation of the points of each player.


b. Which player, A or B, is more consistent in scoring points for his team? How do you know?
5. Consider the following raw data representing yield of Barley (in quintals) of three farmers from their
respective hectare of land for consecutive 8 years.

Telegram channel https://t.me/johnson201485


18
1
a. Determine the range, variance and standard deviation of each of the three farmers.
b. Who of the farmers has higher variation in yield? What does this tell?
c. Who of the farmers has lesser variation in yield?
d. Who of the farmers has consistent yield?
6. If the standard deviation of the scores 1, 2, 3, 4, 5, 6 and 7 is 2 , then find the standard deviation of

13, 24, 35, 46,57, 68 and 79 .


7. If the variance of x is 4 , then find the variance of 3  5x .

10.2. PROBABILITY
1. An Experiment is an activity (measurement or observation) that generates results (outcomes).
2. An Outcome (Sample point) is any result obtained in an experiment.
3. A Sample Space (S) is a set that contains all possible outcomes of an experiment.
4. An Event is any subset of a sample space.
Example 1: When a "fair" coin is tossed, the possible results are either head (H) or tail (T). Consider an
experiment of tossing a fair coin twice.
a. What are the possible outcomes? c. Give the event of H appearing on the second throw.
b. Give the sample space. d. Give the event of at least one T appearing.
Solution:

a. HH, HT,TH,TT c. A   HH,TH

b. HH, HT,TH,TT d. B   HT,TH,TT

NOTE: In tossing a coin, if the coin is fair, the two possible outcomes have an equal chance of occurring. In
this case, we say that the outcomes are equally likely.

Probability of an event (E)


If an event E can happen in m ways out of n equally likely possibilities, the probability of the occurrence of an
event E is given by

number of favourable outcomes nE m


PE   
total number of possible outcomes n  S  n

Telegram channel https://t.me/johnson201485


18
2
Example 2: A box contains 4 red and 5 black balls. If one ball is drawn at random, what is the probability of
getting a
a. red ball? b. black ball?
Solution: Let event R  a red ball appears and event B  a black ball appears. Then,
n  R 4 n  B 5
a. P  R   b. P  B   
nS  9 nS  9

10.2.1.Permutation and Combination


In the previous example of tossing a fair coin twice, the number of all possible outcomes was only four. To find the

probability of the event A   HH,TH , you have to count the number of outcomes in event A (which is 2) and

divide by n  S  . Thus, we have

n  A 2 1
P  A   
nS  4 2
Now, if the experiment is tossing a coin five times, what is the total number of possible outcomes? If an event E is

defined by "3 heads and 2 tails", then how do you find n  E  ? From this, you can observe that counting plays a

very important role in finding probabilities of events. In this section, you shall see some mathematical techniques
which will help you to simplify counting problems. When the number of possible outcomes is very large, it will be
difficult to find the number of possible outcomes by listing. So you have to investigate different counting
techniques which will help you to find the number of elements in an event and a possibility set.
Fundamental principles of counting
There are two fundamental principles that are helpful for counting. These are the multiplication principle and the
addition principle.
Multiplication principle
Before we state the principle, let us consider the following example.

Example 3: Suppose Nuria wants to go from Harrar via Dire Dawa to Addis Ababa. There are two minibuses from
Harrar to Dire Dawa and 3 buses from Dire Dawa to Addis Ababa. How many ways are there for Nuria to travel
from Harrar to Addis Ababa?

Solution: Let M stand for Minibus and B stand for Bus.

Telegram channel https://t.me/johnson201485


18
3
In the above illustration, Nuria has two possible choices to go from Harar to Dire Dawa and three alternatives from
Dire Dawa to Addis Ababa.
The total number of ways is 2  3  6 .
Example 4: Suppose there are 5 seats arranged in a row. In how many different ways can five people be seated on
them?
rd
Solution: The first man has 5 choices, the 2 nd man has 4 choices, the 3 man has 3 choices, the 4th has two
th
choices, and the 5 has only one choice. Therefore, the total number of possible seating arrangements is
5  4  3 2 1  120 .
Generalized Principle of Counting
Suppose that a task consists of n choices performed consecutively. Suppose that choice 1 can be performed in m1
ways; for each of these, choice 2 in m2 ways; for each of these, choice 3 in m3 ways and so on. Then the task can
be performed in m1  m2  m3  ...  mn ways.

Example 7: A corporation has a board of directors cosisting of 10 members. The board must select from among its
members a chairperson, vice chairperson, and secretary. In how many ways can this be done?
Solution:
1. Select chairperson- there are 10 ways (people) to do this.
2. Select a vice chairperson- there are 9 ways ( 9 people left after chairperson is chosen) to do this.
3. Select a secretary- there are 8 ways to do this.
Therefore, the total nmber of ways to complete the task is 10  9  8  720 ways.
Addition principle

NOTE: Two events are said to be mutually exclusive, if both cannot occur simultaneously.
In tossing a coin, Head and Tail are mutually exclusive events because they cannot appear at the same time.
Example 5: A question paper has two parts where one part contains 4 questions and the other 3 questions. If a
student has to choose only one question, from either part, in how many ways can the student do it?
Solution: The student can choose one question in 4  3  7 ways.

Telegram channel https://t.me/johnson201485


18
4
Combined counting principles

The fundamental counting principles can be extended to any number of sequences of events.
Example 6: A question paper has three parts: language, arithmetic and aptitude tests. The language part has 3
questions, the arithmetic part has 6 questions and the aptitude part has 5 questions. If a student is expected to
answer one question from each of two of the three parts, with arithmetic being compulsory, in how many ways can
the student take the examination?
Solution: The student can either take language and arithmetic or arithmetic and aptitude. This gives
3 6  5  6  48 possibilities.
Example 7: The HIV/AIDS Club of a certain school has 6 members: A, B, C, D, E, and F. They are going to line
up, from left to right, for a group photo. After lining up in alphabetical order (ABCDEF), Mr. F complains that he
is always last whenever they do things alphabetically, so they agree to line up in reverse order (FEDCBA) and take
another picture. Then Ms. D complains that she's always stuck next to Mr. C, and that she never gets to be first in
line. Finally, in order to avoid bruised egos, they all agree to take pictures for every possible left-to-right line-up of
the six people. How many different photos must be taken?

Solution: To arrange the six people in a line we need to make six dependent decisions:
1. Choose first person (6 options);
2. Choose second person (5 options)
3. Choose third person (4 options)
4. Choose fourth person (3 options)
5. Choose fifth person (2 options)
6. Choose last person (1 option)
According to the Fundamental Counting Principle the number of outcomes is 6  5  4  3 2 1  720 .

Exercise 10.9
1. In an experiment of selecting a number from 1 – 10, which of the following cannot be an event?
a. The number is “even and prime”. c. The number is multiple of 3.
b. The number is “even and multiple of 5”. d. The number is zero.
2. In an experiment of tossing three coins at a time,
a. Determine the sample space. b. Find the probability of getting two heads.
3. A box contains 2 Red and 3 Black balls. If two balls are drawn at random,
a. Determine the possible outcomes
b. Find the probability of getting 2 Red balls.
c. Find the probability of getting 1 Red and 1 Black balls.

Telegram channel https://t.me/johnson201485


18
5
4. Suppose you have six different books. In how many different ways can you arrange these books on a
shelf?
5. There are three gates to enter a school and two doors to go into a classroom. In how many different ways
can a student get into a class from outside?
6. In a classroom there are 50 students from whom 27 are female students. If one student is selected at
random, what is the probability of getting male student?

For any positive integer factorial denoted as is defined as


Definition 10. 12: .
We define .

10!
Example 8: Calculate a. 4! b. 6! c.
6!
Solution:
a. 4!  4  3  2 1  24 10! 10  9  8  7  6!
c.   10  9  8  7  5040
b. 6!  6  5  4  3  2 1  720 6! 6!

Permutation

A Permutation is the number of arrangements of objects with attention given to


Definition 10.13: the order of arrangements.

The number of permutations of a set of n objects taken all together is denoted by P  n, n  or n Pn and is equal to

n! .
Thus, P  n, n   n !

The number of permutations of objects taken at a time, where , is


Definition 10.14: denoted by or and is given by .

Example 9:
a. Give all the permutations of three letters A, B and C.
b. Suppose we have 5 people to be seated in only 3 seats. In how many ways can they sit?
Solution:

a. The three letters A, B and C can be arranged in P  3, 3  3!  3  2 1  6 different permutations.


These are ABC, BAC, CBA, ACB, BCA and CBA.
b. The first chair can be filled by any one of the 5 people, the second by any one of the remaining 4
people and the third by any of the remaining 3 people. By the multiplication principle, this gives
5  4  3  60 possibilities.

Telegram channel https://t.me/johnson201485


18
6
5  4  3  2  1 5! 5!
60  5  4  3   
2 1 2!  5  3!
Permutation of duplicate items

Example 10: Find the number of permutations that can be made out of the letters of the word
"MATHEMATICS". In how many of these permutations
a. do the words start with C?
b. do all the vowels occur together?
c. do the words begin with H and end with S?

Solution:
a. In word "MATHEMATICS" there are 2 M‟ s, 2 A‟s and 2 T‟s
10!
So, there are ways that words start with C.
2!2!2!
4!7!
b. There are ways that all vowels occur together.
2!2!2!
9!
c. There are ways that words begin with H and end with S.
2!2!2!
Exercise 10.10

1. How many four – digit numbers can be formed from the digits 1, 3, 5, 7, 8 and 9 where a digit is used at
most once?
a. if the numbers must be even? b. if the numbers are less than 3000?
2. Two men and a woman are lined up to have their picture taken. If they are arranged at random, find the
number of ways that
a. the woman will be on the left in the picture.
b. the woman will be in the middle of the picture.
3. In a library there are 3 Mathematics, 4 Geography and 3 Economics books. If each of them will be put on
a shelf and each type of a book are identical, in how many ways can these books be arranged?
4. Verify that n Pn1  n Pn .

Telegram channel https://t.me/johnson201485


18
7
5. Four students are to be chosen from a group of 10 to fill the positions of president, vice-president,
treasurer and secretary. In how many ways can this be accomplished?
6. Find the number of permutations of the letters of these words:
a. COMMITTEE b. CINCINNATI
7. How many distinct permutations are there of the word “STATISTICS”?
Circular permutations: Is there a difference between arrangements of objects in a straight line and
around a circle? Consider three letters A, B, C and try to find the number of different permutations along a
circle. Since it is difficult to indicate the relative position of objects in a circle, we fix the position of one
object and arrange the remaining objects.

If n objects are to be arranged on a circle (along the circumference of a circle), then the number of circular

permutations is given by  n  1! .

Example 11:
a. 7 people are to sit around a circular table. In how many different ways can these people be seated?
b. In how many ways can 6 boys and 6 girls sit around a table of 12 seats, if no two girls are to sit
together?
c. In a playground, 3 sisters and 8 other girls are playing together. In a particular game, how many
ways can all the girls be seated in a circular order so that the three sisters are not seated together?
Solution:
a. Consider the following.

The number of ways these 7 people sit around a round table is  7  1!  6!  720 ways.

Telegram channel https://t.me/johnson201485


18
8
b. First allot seats to the boys, as shown in the diagram. Now the 6 boys can sit in  6  1!  5!  120
ways. Next the 6 girls can occupy seats marked (G). There are 6 such seats. This can be done in 6 P5  720
ways. By the Fundamental Principle of Counting, the required number of ways is 120  720  86, 400 ways.
c. There are 3 sisters and 8 other girls in total of 11 girls. The number of ways to arrange these 11 girls
in a circular manner is 11  1!  10! . These three sisters can now rearrange themselves in 3! ways. By the
multiplication principle, the number of ways so that 3 sisters always come together in the arrangement = 8! ×
3!. Hence, the required number of ways in which the arrangement can take place if none of the 3 sisters is
seated together is 10! 8! 3!  3,386,880 ways.
Combination: Before you define the concept of combinations, see the following example that helps to
illustrate how it is different from permutations. Three students A, B and C volunteer to serve on a committee.
How many different committees can be formed containing two students?
3!
Let us try to use permutations of two out of three: P  3, 2    6 . The possible arrangements are
 3  2 !
AB, AC, BC, BA, CA, CB. But AB and BA, AC and CA, BC and CB contain the same members. Hence AB
and BA cannot be considered as different committees, because the order of the members does not change the
committee. Thus, the required number of possible committee members is not six but three: AB, AC and BC.
This example leads us to the definition of combinations.
The number of ways objects can be chosen from a set of objects without
considering the order of selection is called the number of combinations of
Definition 13.15: objects taking of them at a time, denoted by

and .

To arrive at a formula for n Cr , observe that the r objects in n Pr can be arranged among themselves in r!
ways.
n!
Pr  n  r ! n!
Hence, C  n, r   n
  .
r! r!  n  r  !r !
Therefore, the number of possible combinations of n objects taken r at a time is given by the formula
n n!
   C  n, r   , 0r n.
r   n  r  !r !
From this, you can see that the number of ways that a committee of two members can be selected from three
3! 3!
individuals is given by C  3, 2     3 ways.
 3  2 !2! 1!2!
Example 12: Compute the following. a. C 8, 6 b. C 10, 4 
Solution:
8! 8! 8  7  6! 8  7
a. C 8, 6       28 .
8  6 !6! 2!6! 2!6! 2

Telegram channel https://t.me/johnson201485


18
9
10! 10  9  8  7  6! 10  9  8  7
b. C 10, 4      210 .
6!4! 6!4! 24

a.    
n 
b.  n    n    n  1
n
Example 13: Show each of the following. 
 
r nr
   r  r 1  r 
Solution:
 n  n!  n  n! n!
     
 n  r   n   n  r  ! n  r !
a.
 n  r   n  n  r ! n  r ! r ! n  r !
n! n
     C  n, r 
 n  r  !r !  r 
n  n  n! n!
b.    
 r   r  1  n  r !r !  n  r  1! r  1!
n  n  n! n!
   
 r   r  1  n  r !r !  n  1  r ! r  1!
n! rn !

n!

rn !  
 n  r  !r !  n  1  r  !r !  n  r !r !  n  1  r  n  r !r  r  1!
n! n!
 
 n  r !r !  n  1  r  n  r ! r  1!
n ! n  1  r   rn !

 n  1  r !r !
n !n  n ! rn ! rn !

 n  1  r  !r !
n !n  n ! n ! n  1  n  1!  n  1
   
 n  1  r !r !  n  1  r !r !  n  1  r !r !  r 

Example 14:
a. In an examination paper, there are 12 questions. In how many different ways can a student choose
eight questions in all, if two questions are compulsory?
b. In how many different ways can three men and three women be selected from six men and eight
women?
c. In how many ways can Bekele invite at least one of his friends out of 5 friends to an art exhibition?
d. A committee of 7 students has to be formed from 9 boys and 4 girls. In how many ways can this be
done when the committee contains
i. exactly three girls? ii. at least three girls? iii. 2 girls and 5 boys?
Solution:

Telegram channel https://t.me/johnson201485


19
0
a. Since 2 questions are compulsory, the student is left with a choice of selecting 6 questions from the
remaining 10 questions.

Hence, he/she can do it in C 10, 6  ways i.e. C 10, 6  


10!
 210 ways.
4!6!
10  8
b. Three men from six can be selected in   ways. Three women from 8 can be selected in  
 6  3
ways. Therefore, the total number of ways that a committee of three men and three women be selected out of

 6 8
6 men and 8 women is given by       20  56  1120 ways (by the Multiplication principle).
 3   3
c. At least one means that he can invite either one, two, three, four or five. Therefore, the total number
of ways in which he can invite at least one of his friends is given by (Addition principle)

C  5, 1  C 5, 2  C 5, 3  C 5, 4   C 5, 5  5  10  10  5  1  31


d. i. When exactly 3 girls are included in the committee, the remaining members will be 4 boys.

∴ The total number of ways of forming a committee is C  4, 3  C  9, 4  4 126  504 ways.

ii. At least 3 girls are included means the committee will consist of either 3 girls and 4 boys or 4 girls
and 3 boys.
∴ Total number of ways of forming a committee is given by
C  4, 3  C  9, 4    C  4, 4   C  9, 3     4  126   1  84   588 ways.

iii. Two girls and 5 boys can be selected in C  4, 2  C  9, 5  6  126  756 ways.

Exercise 10.11

1. Compute each of the following.

a. C 12, 9 b. C  20, 15

2. Show that C  n, 0   1 .

3. Show that C  5, 2  C  5, 3 .

4. If C  n, 6  C  n, 4  , then find n .

5. In how many ways can a committee of 5 be selected from 10 people willing to serve?
6. A committee of 5 students has to be formed from 9 boys and 9 girls. In how many ways can this be
done when the committee consists of

Telegram channel https://t.me/johnson201485


19
1
a. 2 girls and 3 boys? d. at least 3 boys?
b. all boys? e. at most 3 girls?
c. all girls?
7. In Ethiopia there are 20 Premier league Soccer teams.
a. In one round how many games are there?
b. If five of the teams represent one company, find the number of ways pairs of teams representing
different companies can play a game.
8. In a box there are 3 red, 4 white and 5 black balls. If we choose three balls at random, what is the
number of ways such that:
a. one ball is white? b. 3 of them are black? c. at most 2 are red?
9. A teacher has to choose three students from the class 11A and two students from the class 11B to the
recitation competition. There are 22 students in the 11A and 17 students in the 11B class. How many possible
choices does she have ?

10. Let A be the set of 4-digit numbers a1a2 a3 a4 where a1  a2  a3  a4 , then find n  A  .

11. Suppose that 20 pillars of the same height have been erected along the boundary of a circular
stadium. If the top of each pillar has been connected by beams with the top of all its non-adjacent pillars, then
find the total number of beams.
10.2.2.Binomial Theorem
Binomial theorem

For a non – negative integer n , the binomial expansion of  x  y  is given by


n

 x  y  C  n, 0  x n  C  n, 1 x n 1 y  C  n, 2  x n  2 y 2  C  n, 3  x n 3 y 3  ...  C  n, n  y n
n

n n  n  n  n
   x n    x n 1 y    x n  2 y 2    x n 3 y 3  ...    y n
0 1  2 3  n

Example 15: Expand each of the following:


4
1 2 
 x  y b.  ax  b  c.  2 x  3 d.   2x  e.  x  2 y 
4 3 5 4
a.
x 
Solution:

 x  y  C  4, 0  x 4  C  4, 1 x 41 y  C  4, 2  x 4 2 y 2  C  4, 3  x 4 3 y 3  C  4, 4  y 4
4
a.

Telegram channel https://t.me/johnson201485


19
2
 C  4, 0 x4  C  4, 1 x3 y  C  4, 2 x2 y 2  C  4, 3 x1 y3  C  4, 4 y 4

 x4  4x3 y  6x2 y 2  4xy3  y 4

 ax  b   C  3, 0  ax   C  3, 1 ax  b  C  3, 2  ax  b 2  C  3, 3  b 3
3 3 2
b.

  ax   3  ax  b  3  ax  b 2  b 3  a3 x3  3a2bx2  3ab2 x  b3
3 2

5 5 5 5


c.  2 x  3     2 x      2 x   3     2 x   3     2 x   3 
5 5 4 3 2 2 3

0 1   2  3

5 5
    2 x  3     3
4 5

 4 5

  2 x  3  32 x 5  240 x 4  720 x 3  1080 x 2  810 x  243


5

 4 1   4 1   4 1   4 1   4


4 4 3 2
1 2 
d. 
x
 2 x 

         2 x 2
       2x        2x      2x 
2 2 2 3 2 4

0 x   1  x   2 x  3 x   4


1 8
 4
  24 x 2  32 x5  16 x8
x x

 4 4  4 3  4 2  4  4
 x  2y     x      x   2 y      x   2 y      x  2 y      2 y 
4 2 3 4
e.
0  1  2 3  4

 x 4  4 x3  2 y   6 x 2  2 y   4 x  2 y   1 2 y 
2 3 4

 x4  8x3 y  24 x2 y 2  32xy3  16 y 4
Example 16: Find the coefficient of

x 2 y3 in expansion of  x  y  . c. a b in expansion of  2a  5b  .
5 6 5 11
a.

x 7 in expansion of  x  1 .
39
b.

Solution:
5 5 5 5 5 5
 x  y    x5    x 4 y    x3 y 2    x 2 y 3    xy 4    y 5
5
a.
0 1   2  3  4  5
5 5! 5 4
Thus, the coefficient of x 2 y 3 is      10 .
3 3!2! 2

 39  39!
  15,380,937 .
7
b. The coefficient of x is 
 7  32!7!

c. The coefficient of a b in expansion of  2a  5b  is C 11, 5 2a 


6 5 11 6
 5b 
5

Telegram channel https://t.me/johnson201485


19
3
 C 11, 5 2a   5b5    64a6  3125b5 
6 11!
6!5!
  462   64a 6  3125b5 

 92, 400,000a6 b5

Therefore the coefficient of a b is 92, 400,000 .


6 5

Exercise 10.12
1. Expand each of the following using the Binomial Theorem:
6
 1
 3x  4 y   
5
c.  a  
6
a. e. x 2  2
 a
7 9
a b  y
 2 x  3 d.    f.  3x  
4
b.
b a  3
2. Without writing all the expanded terms, answer the following

What is the coefficient of a b in the expansion of  a  b  ?


3 5 8
a.

What is the coefficient of the term containing a b in  a  b  ?


2 4 6
b.
9
 b
What is the coefficient of a b in the expansion of  3a   ?
5 4
c.
 3
8
 1
What is the coefficient of the term containing x in  x   ?
2
d.
 x
3. Find the constant terms in the following:
9 4
 1  10  3
a.  2x  2  b.  2x  1  c.  x  3 
 x   x  x 

In expanding  x  y  find the terms that have equal coefficients.


3
4.

10.2.3.Random Experiments and Their Outcomes


At the beginning of this section, you saw the basic definitions of experiment, event and sample space. In this
section, you will use these terms again and also see additional concepts.

A random experiment is an experiment (activity) which produces some well-defined


results. If the experiment is repeated under identical conditions it does not necessarily
Definition 10.16:
produce the same results.

Telegram channel https://t.me/johnson201485


19
4
Example 17: Give the outcomes for each of the following experiments.
a. Tossing a coin c. Rolling a die
b. Tossing a pair of coins d. Rolling a pair of dice
Solution:
a. {H, T} b. {HH, HT, TH, TT} c. {1, 2, 3, 4, 5, 6}
d.

NOTE: Outcomes of a random experiment are said to be equally likely when there is no reason to
expect any one of the outcomes in preference to another. That is, each element has equal chance of
being chosen.

Example 18: If a fair die is thrown, any one of the outcomes 1, 2, 3, 4, 5, 6 has an equal chance of appearing
at the top. Therefore, they are considered as equally likely.
NOTE: In a random experiment, the outcomes which insure the happening of a particular result are
said to be favourable outcomes to that particular result.

Example 19:
a. A fair die is thrown. How many favourable outcomes are there for getting an even number?
b. In picking a playing card from a pack of 52 cards, what is the number of favourable outcomes to
getting a picture card?
Solution:
a. There are 3 favourable outcomes. These are 2, 4 and 6.
b. There are 12 favourable outcomes - 4 Jacks, 4 Queens and 4 Kings.

10.2.4.Events
Recall that any subset of a sample space is called an event and is usually denoted by E . An event is a
collection of sample points.

Telegram channel https://t.me/johnson201485


19
5
Example 20: The four faces of a regular tetrahedron are numbered 1, 2, 3 and 4. If it is thrown and the
number on the bottom face (on which it stands) is registered, then list the events of this experiment.

Solution: The sample space = {1, 2, 3, 4}.


The possible events are {1}, {2}, {3} and {4}.

Types of events
a. Simple Event (Elementary Event) is an event containing exactly one sample point.

Example 21: In a toss of one coin, the occurrence of tail is a simple event.

b. Compound Event When two or more events occur simultaneously, their joint occurrence is known
as a compound event, an event that has more than one sample point.
Example 22: When a die is rolled, if you are interested in the event "getting even number", then the event
will be a compound event, i.e. { 2, 4, 6}.
We can determine the possible number of events that can be associated with an experiment whose sample
space is S . As events are subsets of a sample space, and any set with m elements has 2m subsets, the number
of events associated with a sample space with m elements is 2m . (Sometimes this is called the exhaustive
number of events).
Example 23: Suppose our experiment is tossing a fair coin. The sample space for this experiment is S = {H,
T}. Thus, this sample space has a total of four possible events that are subsets of S. The list of the possible
events is { }, {H}, {T}, and {H, T}.
Occurrence or Non-occurrence of an event
During a certain experiment, there are two possibilities associated with an event, namely, occurrence or non-
occurrence of the event.

Example 24: If a die is thrown, then S = {1, 2, 3, 4, 5, 6}. Let E be the event of getting odd number, then

E  1, 3, 5 . When we throw the die, if the outcome is 3, as 3  E , then we say that E has occurred. If in

another trial, the outcome is 4, then as 4  E , we say that E has not occurred (not E ).

c. Complement of an Event E , denoted by E ' (not E ) consists of all events in the sample space that
are not in E .
Example 25: Let a die be rolled once. Let E be the event of a prime number appearing at the top i.e.

E  2, 3, 5 . Give the complement of the event.

Solution: E '  1, 4, 6

Telegram channel https://t.me/johnson201485


19
6
NOTE:

Algebra of events

NOTE:
Since events are sets (subsets of the sample space) one can form union, intersection and complement of them.
The operations obey algebra of sets, like commutativity, distributivity, De Morgan‟s laws and so on.

d. Exhaustive Events are events where at least one of them must necessarily occur every time the
experiment is performed.
Example 26: If a die is thrown give instances of exhaustive events.
Solution: The sample space is S = {1, 2, 3, 4, 5, 6}. From this, the events {1}, {2}, {3}, {4}, {5}, {6} are
exhaustive events. The events {1, 2}, {3, 4}, {4, 5, 6} are also exhaustive events for this experiment.
More generally, events E1 , E2 ,..., En form a set of exhaustive events of a sample space S where

E1 , E2 ,..., En are subsets of S and E1 E2 .... En  S .

e. Mutually Exclusive Events are events that cannot happen at the same time.

Example 27: Say whether or not the following are mutually exclusive events.
i. When a coin is tossed once, the events {H} and {T}.
ii. When a die is rolled, E1  getting an even number E2  getting a prime number
Solution:
i. Either we get head or tail but we cannot get both at the same time. Thus, {H} and {T} are mutually
exclusive events.

ii. E1 and E2 are not mutually exclusive because 2 is even and prime at the same time.

Telegram channel https://t.me/johnson201485


19
7
f. Exhaustive and Mutually Exclusive Events: If S is a sample space associated with a random
experiment and if E1 , E2 ,..., En are subsets of S such that

i. Ei E j   for i  j and,

ii. E1 E2 .... En  S , then the collection of the events E1 , E2 ,..., En forms a mutually exclusive
and exhaustive set of events.
Example 28: If a die is thrown, the events {1}, {2}, {3}, {4}, {5}, {6} are mutually exclusive and exhaustive
events. But, the events {1, 2}, {3, 4}, {4, 5, 6} are not because {3, 4}∩{4, 5, 6} ≠ ∅.
g. Independent Events: Two events are said to be independent, if the occurrence or non- occurrence of
one event does not affect the occurrence or non-occurrence of the other.
Example 29: In a simultaneous throw of two coins, the event of getting a tail on the first coin and the event
of getting a tail on the second coin are independent.
Example 30: If a card is drawn from a well shuffled pack of cards and is replaced before drawing a second
card, then the result from drawing the second card is independent of the result of the first drawn card.
h. Dependent Events Two events are said to be dependent, if the occurrence or non-occurrence of one
event affects the occurrence or non-occurrence of the other.
Example 31: If a card is drawn from a well shuffled pack of cards and the card is not replaced, then the result
of drawing a second card is dependent on the first draw.
10.2.5.Probability of an Event

Probability can be measured by three different approaches.


a. The classical (mathematical) approach. c. The axiomatic approach.
b. The empirical (relative frequency) approach.
a. The classical approach
This is the kind of probability that you discussed in grade 9. In the classical approach to probability, if all the
outcomes of a random experiment are equally likely and mutually exclusive, then the probability of an event
E is
nE number of outcomes favoring E
PE   .
nS  number of all possible outcomes
Example 32: A fair die is tossed once. What is the probability that an even number appears?
nE 3 1
Solution: E  an even number shows up = {2, 4, 6}. Then, P  E     .
nS  6 2

Telegram channel https://t.me/johnson201485


19
8
b. The empirical approach
This approach is based on the relative frequency of an event (or outcome) when an experiment is repeated a
large number of times. Here, the probability of an event E is the proportion of outcomes favourable to E in
frequency of E f
the experiment. Thus, P  E    E .
total number of observations N
Example 33: If records show that 60 out of 100,000 bulbs produced are defective (D), then the probability of
a newly produced bulb being defective is given by
fE 60
P  D    0.0006 .
N 100, 000
c. The axiomatic approach
In this approach, the probability of an event is given as a function that satisfies the following definition:
Let S be the sample space of a random experiment. With each event E we associate a real number called the
probability of E, denoted by P(E), that satisfies the following properties called axioms (or postulates) of
probability.

1. 0  PE  1
2. P  S   1 , S is the sample space (the sure event)
3. If E1 and E2 are mutually exclusive events, then P  E1 E2   P  E1   P  E2 

NOTE:
P is a function with domain the set of subsets of S (Sample space) and its range is the set of real numbers
between 0 and 1 (both inclusive). Thus we note the following:
a. The probability of an event is always between 0 and 1.
b. If (the impossible event), then , and if E = S (the certain event), then P(S) = 1.

c. If then , and , where (not ).

Example 34: A box contains 6 red balls. One ball is drawn at random. Find the probability of getting a.
a red ball b. a white ball
Solution:
a. The box contains all red balls. Hence we are sure that red will occur. Then, the probability of getting
n  R 6
a red ball is one. Thus, P  R    1
nS  6

Telegram channel https://t.me/johnson201485


19
9
b. The box contains no white balls. The chance of getting white ball is impossible, and the probability is
n W  0
zero. That is, P W    0
nS  6
Example 35: A bag contains 3 red, 5 black, and 4 white marbles. One marble is drawn at random. What is
the probability that the marble is
a. black b. not black
Solution:
5
a. P  black  
12

P  not black   1  P  black   1 


5 7
b. 
12 12
5 7
Thus, P  black   P  not black    1
12 12

Example 36:Which of the following cannot be valid assignments of probabilities for outcomes of sample

space S  w1 , w2 , w3  where wi w j   , if i  j .

Solution:
a. is not valid assignment because the sum of the probabilities is not 1.
b. is valid; all the properties in the axiom above are satisfied.
c. is not valid because probability cannot be negative.

Odds in favour of and odds against an event

If m and n are probabilities of the occurrence and non-occurrence of an event respectively, then the ratio
m : n is called the odds in favour of the event. The ratio n : m is called the odds against the event.
Example 37: The odds against a certain event are 5 : 7 . Find the probability of its occurrence. Solution: Let

E be the event. Then, we are given that number  not E   5 and number  E   7 .

n  E   n  not E   n  E   5  7  12 .
7
 PE 
12

Telegram channel https://t.me/johnson201485


20
0
Rules of probability

In the last section, you have seen different types of events and approaches to probability. We will now discuss
some essential rules for probability and probabilities of the different types of events.
Example 38: Find the probability of obtaining a 6 or 4 in one roll of a die.
Solution: In one roll of a die, the sample space is S = {1, 2, 3, 4, 5, 6}. Obtaining 6 or 4 gives the event E = {4,
6}.
number of outcomes favouring E 2 1
Thus, P  4 or 6   P  E     .
number of possible outcomes 6 3
Trying to calculate probabilities by listing all outcomes and favourable outcomes may not always be
convenient. For more complex situations, there are rules we can use to help us calculate probabilities.

Addition rule of probability


From previous discussions, recall that, if E1 , E2 , E3 ,..., En form a set of exhaustive events of a sample space

S , then E1 E2 E3 ... En  S . Moreover, the probability of an event E , i.e. P  E  is given by

number of outcomes favouring E nE


PE  
total number of outcomes in the sample space n  S 
With these we can easily calculate probabilities of compound events by making use of the addition rule stated
below.
Addition Rule

If E1 and E2 are any two events, then, P  E1 E2   P  E1   P  E2   P  E1 E2  and

if the events are mutually exclusive,  i.e.E1 E2    then P  E1 E2   0

so that P  E1 E2   P  E1   P  E2  .

Example 39:
a. Find the probability of obtaining a 6 or 4 in one roll of a die.
b. Find the probability of getting Head or Tail in tossing a coin once.
c. A die is rolled once. Find the probability that it is even or it is divisible by 3.
Solution:
a. Let E1 be event of getting 6, E2 be event of getting 4. Then E1 and E2 are mutually exclusive

1 1 2 1
events  P  E1 E2   P  E1   P  E2      .
6 6 6 3

Telegram channel https://t.me/johnson201485


20
1
1 1
b. The events are mutually exclusive  P  H or T   P  H   P T    1.
2 2
c. S  1, 2, 3, 4, 5, 6

Let E1  getting even = { 2, 4, 6}.

E2  getting a number divisible by 3 = {3, 6}.

Then E1 and E2 are not mutually exclusive, because E1 E2  6

 P  even or divisible by 3  P  even   P  divisible by 3  P  even and divisible by 3


3 2 1 1 1 1 2
      
6 6 6 2 3 6 3

This shows the addition rule of probability with two events. What do you think the rule will be for three or
more events? The rule can be extended for a finite number of events, but becomes increasingly complicated.
For example, for three events it becomes:

NOTE:

Multiplication rule of probability


This rule is useful for determining the probability of the joint occurrence of events. It is based on the concepts
of independence or dependence of events, discussed earlier. Let us take a brief revision of independent and
dependent events. When the occurrence of the first event affects the occurrence of the second event in such a
way that the probability is changed, the events are said to be dependent.
Example 40: A bag contains 3 black and 2 white balls. We draw two balls one after the other with
replacement (the second is drawn after the first is replaced). Find the probability that the first ball is black and
the second ball is also black.
Solution:
Let event A be the first ball is black and Let event B be the second ball is black.
3 3
Then P  A  and P  B   (Since the ball is replaced, the sample space is not affected).
5 5
Recognizing dependence or independence is of paramount importance in using the multiplication rule of
probability. When occurrence of one event depends on the occurrence of another event, we say the second
event is conditioned by the first event. This leads into what is called conditional probability.

Telegram channel https://t.me/johnson201485


20
2
Conditional probability
If E1 and E2 are two events, the probability that E2 occurs given that E1 has already occurred is denoted

by P  E2 | E1  and is called the conditional probability of E2 given that E1 has already occurred. If the

occurrence or non-occurrence of E1 does not affect the probability of E2 , or if E1 and E2 are independent,

then P  E2 | E1   P  E2  . This is often called the multiplication rule of probability.

Multiplication rule of probability: If E1 and E2 are any two events, the probability that both events occur,

denoted by P  E1 and E2   P  E1 E2   P  E1 E2  is given by

P  E1 E2   P  E1   P  E2 \ E1  , whenever P  E1   0 .
 P  E2   P  E1 \ E2  , whenever P  E2   0 .
NOTE: If and are independent, then .
Hence, for independent events and .

Example 41:
a. A box contains 3 red and 2 black balls. One ball is drawn at random, is not replaced, and a second
ball is drawn. Find the probability that the first ball is red and the second is black.
b. A die is rolled and a coin is tossed. Find the probability of getting 3 on the die and a tail in the coin.
c. A bag contains 3 red, 4 blue and 3 white balls. Three balls are drawn one after the other. Find the
probability of getting a red ball on the first draw, a blue ball on the second draw and a white ball on the third
draw if
i. each ball is drawn, but then is replaced back before the next draw.
ii. the balls are drawn without replacement.
Solution:
a. Let E1  getting red in the first draw. And E2  getting black in the second draw.
3 2 6 3
P  E1 E2   P  E1   P  E2 | E1      .
5 4 20 10
b. Let E1  getting 3 on the die and E2  getting tail on the coin.
1 1 1
Since the two events are independent, P  E1 E2   P  E1   P  E2    
6 2 12
c. Let E1  getting red, in the first draw,
E2  getting blue in the second draw, and E3  getting white in the third draw.
i. The balls are replaced after each draw. The events are independent.
3 4 3 36 9
P  E1 E2 E3   P  E1   P  E2   P  E3       .
10 10 10 1000 250
ii. The balls are not replaced, so events are dependent .
3 4 3 1
P  E1 E2 E3   P  E1   P  E2 | E1   P  E3 | E1 and E2      .
10 9 8 20

Telegram channel https://t.me/johnson201485


20
3
Exercise 10.13
1. In throwing a die, consider the following events.
E1  the number that shows up is even
E2  the number that shows up is prime
E3  the number that shows up is more than 3
a. Determine the event E2 E3 d. Determine the P  E1 E2 
b. Determine the number of elements in E1 E2 e. Determine P  E1 E2 E3 
c. Determine the number of elements in
P  E1 E2 E3 
2. From a pack of 52 playing cards, one card is drawn. Find the probability that it is
a. either a King or a Jack; b. either a Queen or red.
3. A die is thrown twice. What is the probability of scoring a 3, followed by a 4?
4. A red ball and 4 white balls are in a box. If two balls are drawn without replacement, what is the
probability of
a. getting a red ball on the first draw and a white ball on the second?
b. getting two white balls?
5. Two cards are drawn from a pack of 52 cards. What is the probability that the first is an Ace and the
second is a King,
a. if the first card was replaced before the second was drawn?
b. if the cards were drawn without replacement?
6. A box contains 24 pens, 10 of which are red. A pen is picked at random. What is the probability that
the pen is not red?
7. The following table gives assignments of probabilities for outcomes from a sample space.

a. Which of the probabilities are invalid assignments? Why?


b. Why is (b) a valid assignment of probabilities.
8. In throwing a die what is the probability of getting even or prime number?
9. Two students are selected from a class of 28 girls and 22 boys one after the other. What is the
probability that the second student selected is a boy given that the first was a girl?
You have seen how to determine probability by using either of the product rules (for independent or
dependent events). It is also possible to show joint events using tree diagrams and tables, and calculate
probabilities from these.

Telegram channel https://t.me/johnson201485


20
4
Example 42: A fair coin is tossed twice. Find the probability that both outcomes will be Heads.
1 1 1
Solution: From the multiplication rule, P  HH   P  H   P  H     .
2 2 4
You can use a tree diagram and/or table to portray the possible outcomes.

1
Therefore, the probability that both outcomes are heads is .
4
Example 43: Suppose that a group of 10 students contain eight boys (B) and two girls (G). If two students
are chosen randomly without replacement, find the probability that the two students chosen are both boys.

   
Solution: P B1 and B2  P B1  P B2 | B1    108  79  56 
90 45
28
.

28
Hence, the probability that the two students chosen are both boys is .
45

Example 44: A bag contains 5 red balls, 4 blue balls, and 3 white balls. Two balls are drawn one after the
other, without replacement.
a. Find the probability that both are red.
b. Draw tree diagram representing the experiment.
5 4 20 5
Solution: P  R and R      .
12 11 132 33

Telegram channel https://t.me/johnson201485


20
5
Example 45: Two dice are thrown simultaneously. Find the probability that the sum of the numbers scored is
a. 7 b. greater than 9 c. less than 4
Solution:

From the table above n  S   36 .


6 1
 PE  
a. Let E  the sum of numbers at the top is 7. Then n  E   6 . 36 6
6 1
 PE  
b. Let E  sum of the numbers at the top is greater than 9 (i.e., 10 or 11 36 6
3 1
 PE  
c. Let E  = sum is less than 4 (i.e. 2 or 3). Then, n  E   3 . 36 12
Exercise 10.14
1. A box contains 5 red and 6 white balls. If one ball is drawn at random, find the probability that it will be
a. red or white? b. not red? c. yellow?
2. From a pack of 52 playing cards, three cards are drawn one after the other. What is the probability that all
are Kings if
a. drawing is made with replacement?
b. drawing is made without replacement?
3. There are 4 black, 2 red and 4 white balls in a box. If three balls are selected at random what is the
probability that
a. all the balls selected are black?
b. at least one ball is white?
c. all the balls are of different colour?
4. Two lamps are to be chosen from a pack of 12 lamps where four are defective and the rest are non-
defective. What is the probability that
a. both are defective? b. One is defective? c. at most one is defective?
5. If a plate of a car consists of two letters and four digits and one car is chosen at random, then find the
probability that the car has the letters at the beginning and at the end.

Telegram channel https://t.me/johnson201485


20
6
1. Quantitative data can be numerically described. Height, weight, age, etc. are quantitative.
2. Qualitative data cannot be expressed numerically. Honesty, beauty, sex, love, religion, etc. are qualitative.
3. A quantity which assumes different values is said to be a variable. A variable may be
iii. continuous, if it can take any numerical value within a certain range. Some examples are height,
weight, temperature.
ii.discrete, if it takes only discrete or exact values. It is obtained by counting.
4. Frequency means the number of times a certain value of a variable is repeated in the given data.
5. A grouped frequency distribution is constructed to summarize a large sample of data. The appropriate class
interval is given by
 Largest value   Smallest value 
 - 
Class interval   in ungrouped data   in ungrouped data 
.
Number of classes required
6. A measure of location is a single value that is used to represent a mass of data. The common measures of
location are mean, median, mode, quartiles, deciles and percentiles.
n

x i
Mean  x   i 1
for raw data
n
n

fx i i
 i 1
n
for discrete data
f
i 1
i

fm i i
 i 1
n
for grouped data ( m  class mark)
f
i 1
i

7. Given values x1 , x2 , x3 ,..., xn of a population function, if the mean is x , then

 x1  x    x2  x    x3  x   ....   xn  x   0 .
8. If a constant c is added to each value of a population function, then the new mean x ' is equal to the old
mean plus c .
x
9. Median of ungrouped data is given by
  n  1th 
Md    item, if n is odd
 2   After data is arranged in 
   
n
th
n 
th
 increasing or decreasing 
  item    1 item  order of magnitude. 
 
  2 
2
, if n is even
2

Telegram channel https://t.me/johnson201485


20
7
n 
 2  cfb 
10. Median for a grouped data is given by M d  BL   i .
 f c 
 
11. Mode is the value with the highest frequency.
12. If a distribution has a single mode it is "unimodal". If it has two modes, it is "bimodal". If it has more than
two modes, it is called "multimodal".
 d1 
13. For grouped frequency distributions, the mode is given by M o  BL   i .
 d1  d 2 
 tn 
 4  cfb 
14. Quartiles for grouped frequency distributions are given by QR  BL   i .
 f 
 
th
15. Similarly the t th Decile and i Percentile for grouped frequency distributions, are given by
 tn   in 
 10  cf b   100  cf b 
Dt  BL    i and Pi  BL    i respectively.
 f   f 
   
16. Variation is used to demonstrate the extent to which the individual item in the distribution varies from the
average.
17. The different measures of variation are Range, Variance and Standard Deviation.
 Range  xmax  xmin
n

 x  x
2
i
 Variance  i 1

n
n

 x  x
2
i
 Standard deviation (S) is the positive square root of variance, S  Variance  i 1
.
n
18. Probability of an event E is defined as follows If an experiment results in n equally likely outcomes and
m
m  n is the number of the ways favourable for event E , then P  E   .
n
19. Multiplication Principle
If an event can occur in m different ways and for every such choice another event can occur in n different ways,
then both events can occur in the given order in m  n different ways.
20. Addition Principle
If an operation can be performed in m different ways and another operation can occur in n different ways and
the two operations are mutually exclusive, (the performance of one excludes the other) then either of the two can
be performed in m  n ways.
21. f n is a natural number, then n factorial, denoted by n! , is defined by

Telegram channel https://t.me/johnson201485


20
8
n!  n   n  1   n  2  ...  2 1 .
22. The number of permutations of n objects taken r at a time, where 0  r  n , is denoted by P  n, r  or
n!
Pr and is given by P  n, r   .
n
 n  r !
23. The number of combination of n things taking r at a time is given by
 n  P  n, r  n!
C  n, r       , 0r n.
r  r!  n  r  !r !
24. The Binomial Theorem:
 x  y  C  n, 0  x n  C  n, 1 x n 1 y  C  n, 2  x n  2 y 2  C  n, 3  x n 3 y 3  ...  C  n, n  y n .
n

REVIEW EXERCISES ON UNIT 10

 n  1!
1. If  5 , then find n .
n!
2. How many three – digit numbers can be formed from the digits 2, 5, 7, 9
a. if each digit is used once only? b. if each may be used repeatedly?
3. A box contains 12 bulbs with 3 defective ones. If two bulbs are drawn from the box together, what is the
probability that
a. both bulbs are defective? c. one bulb is defective?
b. both are non-defective?

4. In the expansion of  a  b  , find


10

7 3 4 6
a. the coefficient of a b b. the coefficient of a b .
5. A committee of 5 members is to be selected from 7 men and 8 women. In how many ways can this be done so
as to include
a. 2 women? b. at least 2 men? c. at most 4 women?
6. A box contains 3 red and 8 white balls. If one ball is drawn from it, find the chance that the ball drawn is red.
7. From a pack of 52 playing cards, three cards are drawn one after the other without replacement. What is the
probability that Ace, King and Jack will be obtained respectively?
8. Suppose a pair of dice is thrown. What is the probability that the sum of the scores is 5?
9. The marks for passing in the examination in a subject is 33.4 out of 9 students who appeared at the examination
have failed and the marks of remaining students are 78, 40,97, 65, 50 . Find the median of the marks.

Telegram channel https://t.me/johnson201485


20
9
PRACTICE QUESTIONS ON UNIT 10
CHOOSE THE BEST ANSWER FROM THE GIVEN ALTERNATIVES

1. A school has three classrooms for grade 11, namely, 11A, 11B and 11C . The number of students in these

classrooms is 28, 20 and 22, respectively. All the students took examination and the average score of the

students of 11A, 11B and11C is 60, 70 and 70, respectively. What is the average score in this examination

for all grade 11 students? A. 66 B. 66.67 C. 65 D. 65.67


2. You are given a data on the age of students, in a Age 8 10 11 12 13
primary school. Which of the following is NOT true Number of students 2
5 15 8 10
about the data?
A. The median is 10.5 B. The mode is 10 C. The mean is 10.5 D. The range is 5
3. The following is the table of simple frequency distribution of a data with variable x .
x 1 3 4 5 7
Frequency 2 5 6 5 2
The standard deviation of the data is equal to:

A. 3 B. 3 C. 2.3 D. 3.6
4. Which of the following is true about the Arithmetic Mean of a given data?
a. It can be obtained even in the absence of some of the values in the data.
b. It can also be used for qualitative data.
c. There can be two means for a given data.
d. It is affected by extreme values.
5. A three-digited library identification card is to be printed from the numbers 0, ,1, 2, 3, 4, 5 in such a way that
the first is non-zero and no number is to be repeated. How many such card can be printed?
A. 100 B. 120 C. 150 D. 180
6. A student needs to be select 3 books from 3 different mathematics, 3 different physics and 1 history book. What
is the probability that one of them is mathematics and the other two is either physics or history books?
3 9 15 18
A. B. C. D.
35 35 35 35
7. Items produced by a certain company are subjected to two kinds of defects D1 and D2 . Out of the total

production, if 5% have defect D1 , 10% have defect D2 and 2% have both defects, then what is the

probability for an item to have defect D2 , given that it has defect D1 .

Telegram channel https://t.me/johnson201485


21
0
A. 0.2 B. 0.05 C. 0.1 D. 0.4
8. Different codes each consisting of five characters are to be generated. In each code the first two characters are
either A or B and the remaining three characters are any of the digits 0, 1, 2, ..., 9 . How many distinct codes
can be generated so?
A. 4000 B. 3600 C. 3000 D. 2400
9. As shown in the table below, a measurement is grouped into five class intervals with the frequency distribution.
Class interval 45  55 55  65 65  75 75  85 85  95
Frequency 26 38 33 16 7
th
What are the first quartile Q1 and the 75 percentile P75 of the measurement?

A. Q1  30.50, P75  90.50 C. Q1  56.05, P75  90.50

B. Q1  30.50, P75  72.88 D. Q1  56.05, P75  72.88

10. The following table shows the frequency distribution of a group data in which the frequency of the class
interval, f1 , is missed.
Class interval 37 8  12 13  17 18  22
Frequency f1 9 4 1

If the mean of the data is 10 , what is the frequency f1 of the first class interval?

A. 4 B. 6 C. 7 D. 8
11. Below are the marks of grade 12 students in Mathematics out of 100. What is the variance of the frequency
distribution given below?
Marks 20  40 40  60 60  80 80  100
Number of students 4 10 4 2
A. 107.2 B. 4 C. 304 D. 17.5
12. Let A: getting head in one toss of a coin and B: getting number 5 in one roll of a die. Which one of the
following is true?

A. P A  B  16 B. PA  B  101 C. P A  B  
7
12
D. P A  PB  
1
10
13. Three men, M 1 , M 2 and M 3 are firing at a target independently and have probability 0.20, 0.25 and

0.30 respectively, of hitting the target. What is the probability that at least one of them hits the target? A.
0.30 B. 0.42 C. 0.58 D. 0.75

Telegram channel https://t.me/johnson201485


21
1
14. A committee consisting of 3 students is to be selected from 10 candidates among which 4 are girls. What is
the probability that at least one girl is selected?
5 2 1 1
A. B. C. D.
6 3 3 6
15. Among students who took a quiz, 15 students scored 6, 20 students scored 7, 10 students scored 8 and 5
students scored 10. What is the average score of the students?
A. 7.8 B. 7.5 C. 7.2 D. 7.0
16. In how many more ways can 4 people be arranged in a row than if they were arranged in a circle?
A. 1 B. 6 C. 18 D. 12
17. If distinct codes (words) of eight letters are formed by rearranging the letters in the word „ABBEBAYE‟,
how many of the codes begin with B or Y?
A. 840 B. 630 C. 1680 D. 420
18. A group of six students take their seats at random in a round table for discussion. What is the probability
that two specific students do NOT sit together?
3 2 2 1
A. B. C. D.
5 3 5 3
19. The following is the frequency distribution of a grouped data.
Class Intervals Frequency (f)
37 2
8  12 2
13  17 10
18  22 6
What is the mean and standard deviation of the distribution, respectively?

A. 15, 2 5 B. 15, 7.5 C. 12.5, 5 2 D. 12.5, 15

If Qi , Di and Pi are respectively the i  quartile, deciles and percentile of a data arranged in an
th
20.

increasing order, then which one of the following is necessarily true?


Q1  Q3
A. Q2  B. D3  P25 C. P25  Q1 D. Q 2 = mean of the data.
2
21. A company produced 25,000 bulbs and randomly tested 2% of the product. Among the tested bulbs, if 40
have defect of type D1 , 60 have defect of type D2 and 25 have both types of defects, what is the probability
that a bulb produced by the company has none of the defects?
A. 0.95 B. 0.80 C. 0.85 D. 0.20
22. If S is a set with 10 elements and A  S , what is the probability that A has 3 or more elements?

Telegram channel https://t.me/johnson201485


21
2
7 8 121 7
A. B. C. D.
10 11 128 128
23. Different codes, each of which consisting of five characters, are to be generated in such a way that the first
two characters are any of the English capital letters ( A to Z) and the remaining three are any of the digits
0, 1, ...9 . How many distinct codes can be generated so?
A. 468,000 B. 260 C. 676,000 D. 26
! 10!
24. Suppose that the first 3 letters (A, B and C) and number digits are to be used to form car plates in a small town.
How many different plates can be formed in total that contain 1, 2 or 3 letters and then followed by 3 letters?
A. 3,000 B. 27,000 C. 39,000 D. 100,000
25. A measurement is grouped into five class intervals with the following frequency distribution.
Class Interval 5  15 15  25 25  35 35  45 45  55
Frequency 22 40 68 50 20
th
What are the first quartile Q1 and 75 percentile P75 of the measurement?

A. Q1  20, P75  40 C. Q1  20, P75  39

B. Q1  22, P75  40 D. Q1  22, P75  39

26. Three persons P1 , P2 and P3 are firing at a target independently and have a probability 0.7, 0.5 and 0.4 ,

respectively, of hitting the target. What is the probability that at least one of them hits the target?
A. 0.95 B. 0.85 C. 0.91 D. 0.99
27. The following is a simple frequency distribution table of a data with variable X.
X 3 5 6 7
Frequency 2 5 2 1

What are the mean  x  and variance  of the data?


2

A. x  5,  2  0.7 B. x  6,  2  1.4 C. x  6,  2  0.7 D. x  5,  2  1.4


28. A box contains 10 items of which 3 are defective. If 2 items are randomly taken out of the box, what is the
probability that both items are not defective?
7 4 7 49
A. B. C. D.
10 7 15 100
29. Items produced by a certain company are subjected to two kinds of defects D1 and D2 . Out of the total

product, 5% have defect D1 , 10% have defect D2 , and 2% have both defects. What is the probability that a

randomly selected item has neither defect D1 nor defect D2 ?

Telegram channel https://t.me/johnson201485


21
3
A. 0.13 B. 0.5 C. 0.98 D. 0.87
30. There are three children in a room, ages three, four, and five. If a four-year-old child enters the room then
which one of the following is true?
A. mean age will stay the same but the standard deviation will increase.
B. mean age will stay the same but the standard deviation will decrease.
C. mean age and standard deviation will increase.
D. mean age and standard deviation will stay the same.
31. Two machines A and B work independently. The probability that both machines A and B work is 0.4 . If
the conditional probability that machine B works given that machine A works is 0.5 , then the conditional
probability that machine A works given that machine B works is_________.
A. 0.8 B. 0.3 C. 0.5 D. 0.7
32. A librarian would like to arrange 15 books in a row. A better method he has to use to find the number of all
possible ways is:
A. Principle of Combination C. Product Rule
B. Principle of Permutation D. Sum Rule
33. A three digit number starting with 2 is formed using the digits 1, 2, 3, 4. If the repetition of digits is not
allowed, then which of the following formulas is the : formula to calculate the number of such numbers?
P4, 3 C 4, 3
A. B. C. P 4, 3 D. C 4, 3
4 6
34. The odds in favor of an event are 3 : 8 . What is the probability of its occurrence?
3 3 11 8
A. P( E )  B. P( E )  C. P( E )  D. P( E ) 
8 11 21 3
35. In throwing a die, consider the following events:
E1  the number that show up is even.

E 2  the number that show up is prime.

E3  the number that show up is more than 3 .


Which one of the following is true?

C. PE1  E 2  E3  
1
A. The number of elements in E1  E 2 is zero.
6

B. PE1  E 2   D. PE1  E 2  E3  
1 5
2 6

Telegram channel https://t.me/johnson201485


21
4
UNIT ELEVEN: MATRICES AND DETERMINANTS
Unit Outcomes:

After completing this unit, you should be able to:

 know basic concepts about matrices.


 know specific ideas, methods and principles concerning matrices.
 perform operations on matrices.
 apply principles of matrices to solve problems.
 know basic concepts about matrices
 apply matrix concepts to model, solve and analyze real-world situations.

Main Contents:

11.1. MATRICES
11.2. DETERMINANTS AND THEIR PROPERTIES
11.3. INVERSE OF A SQUARE MATRIX
11.4. SYSTEMS OF EQUATIONS WITH TWO OR THREE VARIABLES
11.5. CRAMER’S RULE
Key terms

Summary

Review Exercises

Telegram channel https://t.me/johnson201485


21
5
INTRODUCTION
This unit deals with matrices and determinants. Matrices and determinants deal with an array of numbers with their
own rules for addition and multiplication.
In this unit, you will also learn how to solve systems of linear equations by systematically using Gauss‟ method.
Expressing linear systems of equations using matrices makes them easier to solve, using programmable calculating
machines.
Matrices have many scientific applications. They are useful to organize a vast amount of data. One can represent a
system of linear equations by using matrices. Matrix representation is better suited for solving systems of linear
equations by using computing devices.
OPENING PROBLEM
Consider a nutritious drink which consists of whole egg, milk and orange juice. The food energy and protein of
each of the ingredients are given by the following table.
Food Energy Protein (Grams)
(Calories)
1 egg 80 6
1 cup of milk 160 9
1 cup of Juice 110 2
How much of each do you need to produce a drink of 540 calories and 25 grams of protein?
To solve the above opening problem, you can formulate the problem as a system of linear equations. You can write
the system of linear equations by letting:
x be the number of eggs. y be the number of cups of milk. z be the number of cups of juice.
80 x  160 y  110 z  540
Then the system of linear equations becomes: 
6 x  9 y  2 z  25
8 x  16 y  11z  54
The system is equivalent to: 
6 x  9 y  2 z  25
 8 16 11 54 
Using matrix notation, this system of linear equations can be written as:  
 6 9 2 25 
One of the main target of this chapter is to solve such types and large size of system of linear equations.

11.1. MATRICES
DEFINITION 11.1 Let R be the set of real numbers and m and n be positive integers.
A rectangular array of numbers in of the form,

is called an m by n (m× n) matrix in R.

Telegram channel https://t.me/johnson201485


21
6
Consider the matrix A in the definition 14.1 above:
 The number m is called the number of rows of A.
 The number n is called the number of columns of A.
 The number aij is called the ijth element or entry of A which is an element in the ith row and jth column of A.

 Matrix A can be abbreviated by : A  (aij ) mn


 The rectangular array of entries is enclosed in an ordinary bracket or in a square bracket.
 m × n (read as m by n) is called the SIZE OR ORDER of the matrix.
REMARK: Matrices are usually denoted by capital letters, like A, B, C etc and entries of matrices by small letters a,
b, c like etc.
Example 1. Find the size and determine its elements/or entries for each of the following matrices.
 2 0  7 
   1    
a. A   5 1  b. S  (25) c. B    d. T   0 4 4 e.  3 
 0 3   2 11  E  3 / 2
 
  0 
 
 2 
Solution
a. A has three rows and two columns. Therefore, A is a matrix of size 3  2 (3 by 2).
The entries of A are:
i. a11 = 2 (the entry found in the first row and in the first column of A);
ii. a12 = 5 (the entry found in the first row and in the second column of A);
iii. a21  1 (the entry found in the second row and in the first column of A);
iv. a22  1 (the entry found in the second row and in the second column of A);
v. a31  0 (the entry found in the third row and in the first column of A);
vi. a32  3 (the entry found in the third row and in the second column of A)
b. S has exactly one row and one column. Therefore, S is a 1  1 matrix. The only entry of S is s11  25
c. B has two rows and two columns. Therefore, B is a matrix of order 2  2. The entries of B are : b11  1 , b12   ,
b21  2 and b22  11 .
d. T has one row and three columns. Therefore, T is a 1 by 3 matrix.
The entries of B are : t11  0 , t12  4 and t13  4 .
e. E has 4 rows and one column. Therefore, E is a 4  1 matrix.
The entries of B are : e11  7, e21  3, e31  3 2, e41  0 and e51  2 .
Example 2. Three students Chaltu, Solomon and Kalid have a number of 10, 50 and 25 cent coins in their
pockets. The following table shows what they have.
a. Represent the table in matrix form.
b. What is represented by the columns?
c. What is represented by each row?
d. Suppose aij denotes the entry in the ith row and jth column. What does a31 tell you? What about a23 ?

Telegram channel https://t.me/johnson201485


21
7
Student‟s Name

Chaltu Solomon Kalid

Number of coins
10 cent coins 2 6 4

50 cent coins 3 2 0

25 cent coins 4 0 5

Solution
a. 2 6 4
 
A  3 2 0
4 5 
 0

b. The columns represent the number of the various kinds of coins each student has.
c. The rows represent the number of coins of a certain fixed value that the students have.
d. a31 = 4. It means Chaltu has four 25-cent coins in her pocket. a23 = 0. This means Solomon has no 50-cent
coins.
Let A  (aij )32 where aij  2 j  i . Determine A.
2
Example 3.
Solution. A  (aij )32 shows that i  1, 2 and j  1, 2 and 3 .

Since aij  2 j  i , we have


2

a11  2(1)  12  1 a12  2(2)  12  3 a21  2(1)  22  2


a22  2(2)  22  0 a31  2(1)  32  7 a32  2(2)  32  5
Therefore, matrix is given by  1 3 
 
A   2 0 
 7 5 

NOTATION
For a matrix A  (ai j )m n , the ith row of A is denoted by Ai and the j th column of A is denoted by A . That is if
j

 a11 a12 ... a1n   a1 j 


   
a
A   21
a22 ... a2 n  , then Ai   ai1 ai 2 ... ain  and A j   a2 j 
 ... ... ... ...   ... 
   
 am1 am 2 ... amn   amj 
where i  1, 2, 3, ..., m and j  1, 2, 3, ..., n
 3 0 
 
For example; for the matrix A   2 5  ,
0 11 

i) the rows of A are A1   3 0 , A2   2 5 and A3   0 11 , and
 3   0 
ii) the columns of A are A1   2  and A2   5  .
   
 0   11 
   

Telegram channel https://t.me/johnson201485


21
8
Some Important Types of Matrices
1. A matrix with only one column is called a COLUMN MATRIX. It is also called a COLUMN VECTOR.
For instance;
2 
 
5 
i) A   is a column matrix or a column vector of size 4  1.
0 
 
 2
ii) B   5 is a column matrix or a column vector of size 1  1.
2. A matrix with only one row is called a ROW MATRIX (also called a ROW VECTOR).
For instance;
i) C   3 5 4 is a row matrix or a row vector of size 1  3.

ii) D   2  is row matrix or a row vector of size 1  1.


3. A matrix with the same number of rows and columns is called a SQUARE MATRIX.
NOTE: A matrix of size n  n is said to be a square matrix of order n . Hence, a square matrix of order n is often
expressed in the form

The PRINCIPAL OR MAIN DIAGONAL of a square matrix A = (aij)nn of order n is the list
( ) ( )
of its n diagonal elements.
The other elements with i  j are called the OFF-DIAGONAL ELEMENTS of A.
For instance;
 2 10 
i) A    is a square matrix of order 2.
 5 8 
The list of the main diagonal elements are and
1 5 3  
 
0 3
ii) B   0 1 is a square matrix of order 4.
2 
 
 1 1  2 0 0 
7 0 1 
 1
The list of the main diagonal elements are ,
4. A matrix with all entries 0 is called a ZERO or NULL MATRIX which is denoted by 0.
For instance;
i) C   0 0 0 is a zero matrix of order 1 by 3.

Telegram channel https://t.me/johnson201485


21
9
0 0 0 0
ii) D    is a null matrix of size 2  4.
0 0 0 0
0 0 0
 
iii) E   0 0 0  . is a zero square matrix of order 3.
0 0 0
 
5. A DIAGONAL MATRIX is a square matrix that has zeros everywhere except possibly along the main
diagonal (top left to bottom right).

A diagonal matrix of dimension n can be written in the form


For instance;
 1 0 0 0  0 0 0
   
0 0 0 0
i) S    and ii) E   0 0 0  are diagonal matrices.
 0 0 5 0 0 0 0
   
 0 0 0 9
6. The IDENTITY (UNIT) MATRIX is a diagonal matrix where the elements of the principal diagonal are all
ones.
NOTE: An identity matrix of order n is often denoted by I n .
1 0 0 0
1 0 0  
1 0   0 1 0 0
For instance; I1  (1) , I2    , I 3   0 1 0  and I 4   are identity matrices of
0 1 0 0 1 0 0 1 0
   
0 0 0 1
order 1, 2, 3 and 4, respectively.
A diagonal matrix is said to be a SCALAR MATRIX if its diagonal elements are equal,
that is, a square matrix B  (bi j )n  n is said to be a scalar matrix if
0 for i  j
bi j   for some real number k.
k for i  j
Observe that a scalar matrix is an identity matrix when k = 1. But every identity matrix is clearly a scalar matrix.
For instance;
5 0 0
 1 0   
A  (2) , B    and C   0 5 0  are scalar matrices of order 1, 2 and 3, respectively.
 0 1  0 0 5
 
7. A LOWER TRIANGULAR MATRIX is a square matrix whose elements above the main diagonal are all
 
zero. That is a square matrix A  ai j
nn
is a lower triangular matrix if

Telegram channel https://t.me/johnson201485


22
0
ai j  0 for i  j.
0 0 0 0
 5 0 0  1 0 0  0 0 0  
 2 0        0 0 0 0
For instance; A   , B   9 0 0  , C   0 1 0  , D   3 0 0  and E   are
 3 1  2 0 1 0 0 1  2 1 0  0 0 0 0
       
0 0 0 0
lower triangular matrices.
NOTE: A square matrix A  ai j   nn
is said to be A STRICTLY LOWER TRIANGULAR MATRIX if

ai j  0 for all i  j.
For instance;
 0 0 0 0 0 0  0 0 0
0 0      
A , B   2 0 0  , C   0 0 0  and D   3 0 0 
5 0  3 0 0 0 0 0  2 1 0 
     
8. An UPPER TRIANGULAR MATRIX is a square matrix whose elements below the main diagonal are all zero.
 
That is a square matrix A  ai j
nn
is an upper triangular matrix if

ai j  0 for i  j.
For instance;
 1 
2 0 0 3 0 1
2 
 0 5     
M   , S  0 2 0 , and T   0 1 0  
0 0  0 0 2 0 0 9 4 
   
0 0 2 
 0
are upper triangular matrices.
NOTE: A square matrix A  ai j   nn
is said to be A STRICTLY UPPER TRIANGULAR MATRIX if

ai j  0 for all i  j.
 0 1  3
 0 1 4  0 0 0  
0 2     0 0 0 5
For instance; A    , B   0 0 8  , C   0 0 0  and
0 0 0 0 0 0 0 0 0 0 0 3 
     
0 0 0 0
Note that all diagonal matrices are triangular matrices.
Equality of matrices
DEFINITION 11.2: Two matrices A  (ai j ) m  n and B  (bi j ) m  n of the same order are said to be

EQUAL, written as A = B, if their corresponding elements are equal, i.e. ai j  bi j for all 1  i  m and 1  j  n
.

Telegram channel https://t.me/johnson201485


22
1
 x  3 z  4 2y  7  0 6 3y  2 
   
Example 4. If  6 a  1 0    6 3 2c  2  , then find a, b, c, x , y and z.
 b  3 21 0   2b  4 21 0 

Solution. As the given matrices are equal, therefore, their corresponding elements must be equal. Comparing the
corresponding elements, we get
x + 3 = 0, z + 4 = 6, 2y – 7 = 3y – 2,
a – 1 = – 3, 0 = 2c + 2 b – 3 = 2b + 4,
Simplifying the above equations, we get a = – 2, b = – 7, c = – 1, x = – 3, y = –5, z = 2
Addition and Subtraction of Matrices
Suppose Fatima has two factories at places A and B. Each factory produces sport shoes for boys and girls in three
different price categories labelled 1, 2 and 3. The quantities produced by each factory are represented as matrices
given below:

Suppose Fatima wants to know the total production of sport shoes in each price category. Then the total production
In category 1 : for boys (80 + 90) , for girls (60 + 50)
In category 2 : for boys (75 + 70) , for girls (65 + 55)
In category 3 : for boys (90 + 75) , for girls (85 + 75)
This can be represented in the matrix form as
This new matrix is the sum of the above two matrices. Thus the sum of two matrices is defined as follow:

DEFINITION 11.3
Let A  (ai j ) m  n and B  (bi j ) m  n be two matrices. Then the sum of A and B, denoted by A + B, is an m  n matrix
obtained by adding the corresponding elements, while the difference of A and B, denoted by A – B, is an m  n
matrix obtained by subtracting the corresponding elements i.e.,
i) A  B  (ai j )m  n  (bi j )m  n  (ai j  bi j )m  n , and

ii) A  B  (ai j )m  n  (bi j )m  n  (ai j  bi j )m  n


Example 5.
 5 3 1  1 11 4 
If M    , and N    are two matrices, then find M + N and M  N.
 0 7 2  3 0 2 
Solution.

Telegram channel https://t.me/johnson201485


22
2
 5 3 1  1 11 4   5  1 3  ( 11) 1  4   4 8 5
M N     
0 7 2   3 0 2   0  3 7  0 2  ( 2)   3 7 0 
 5 3 1  1 11 4   5  1 3  (11) 1  4   6 14 3 
M N     
0 7 2   3 0 2   0  3 7  0 2  (2)   3 7 4 
Properties of Matrix Addition
1. A + B = B + A ……………………………………(Commutative property)
2. (A + B) + C = A + (B + C) …………………..(Associative property)
3. A + 0 = A = 0 + A …………………………….(Existence of additive identity)
4. A + (−A) = A  A= 0 …………………………..(Existence of additive inverse)
Multiplication of Matrices by Scalars: Given a matrix, you can get another matrix by
multiplying each of its elements by a constant. This is known as Multiplication of Matrices by Scalars
DEFINITION 14.3: If k is a scalar (i.e. a real number) and A  (ai j )m  n is a given matrix, then kA is
the m  n matrix obtained from A by multiplying each element of A by k. i.e kA  k (ai j )m  n  (kai j )m  n .
That is

 4 5 2 1
Example 6. If A    , then find a) 5A b) A c) 0.A d) 1.A
 0 2  1  2

 4 5 2   5  (4) 5  5 5  2   20 25 5 2 
Solution a) 5 A  5    
 0 2 1   5  0  5  (2)  5  (1)   0 10 5 
1 1 1   5 2
 (4) (5) ( 2)   2 
1 1  4 5 2 2 2 2
b) A     2 2 
2  0 2 1   1  1 
(1)   0
2 1 1
 (0) (2) 1  
2 2 2   2 
 4 5 2   0(4) 0(5) 0( 2)   0 0 0 
c) 0. A  0    
 0 2 1   0(0) 0( 2) 0( 1)  0 0 0
 4 5 2  1(4) 1(5) 1( 2)   4 5 2
d) 1. A  1   
 0 2 1  1(0) 1(2) 1(1)   0 2 1 
Properties of Scalar Multiplication of Matrix
If A and B are matrices of the same order and r and s are any scalars (i.e., real numbers), then:
a. r(A + B ) = rA + rB c. (rs)A = r(sA) e. 0A = 0e.
b. (r + s) A = rA + sA d. 1A = A and 0A = 0 f. 1A = A

Telegram channel https://t.me/johnson201485


22
3
Exercise 11.1
 1.52 0 
 
1. For a matrix A  (ai j )   3 7 11  , determine a) a11 b) a31 c) a32 d) size of A
 0 3 

2. Determine the entries of each of the following matrices.
7 
 
 1 10   3 
   1    
a) A  (ai j )32   0 3.1 b) B  (bi k ) 22   c) E  (eij )51   3 / 2 
 9 3   2 11 
  0 
 
 2 
d) S  ( si j )11  (7) e) T  (ti j )13   0 4 4

3. a) Let A  (aij )22 where ai j  j  3i . Determine A.


i j  2i for i  j
a) Let A  (aij )22 where ai j   2 . Determine A.
 j  i  3 for i  j
4. a) Find the values of the unknowns in each of the following, if A = B.
 x 5   3 1  z   1 a  b 
i) A  and B     
0 2y  0 2y  ii) A   a  2b 11  and
 0 3 

 c 2a  3b 
 
B   3a  b 11 
 0 3 

b) Find the values of the unknowns in each of the following, if 3A = 2B.
 2 x 3   3 z 
i) A  and B   
 3 5y   9 x  2 y 
 1 2a  b   c 2a  3b 
   
ii) A   a  2b 4  and B   3a  b 6 
 0 2   0 3 
 
 h  2k 0 0 
 
5. If A   0 2 0  is a scalar matrix, then find the values of h and k .
 0 0 k  3h 

6. Write the following matrices in tabular form:
i. A =  ai j  , where i = 1, 2, 3 and j = 1, 2, 3, 4
ii. B =  bi j  , where i = 1 and j = 1, 2, 3, 4

Telegram channel https://t.me/johnson201485


22
4
iii. C =  ci k  , where j = 1, 2, 3 and k = 1

 1 1 3   3 2 1 
   
7. Let A   0 2 5 and B   1 5 2  , then find
 2 0 3   2 0 2 
   
a) B c) 3B  2 A e) A  3B
b) 2 A  B d) a matrix C such that B  2C  A
1 2 5   3 2 7   2 3 0 
8. Let A    , B  and C    . Then find
 0 3 3  1 2 1  1 1 4 
a) C  2 A b) B  (C  A) c) (3B  A)  2C
9. Graduating students from a certain high school sold cinema tickets on two different occasions, in two kebeles,
in order to raise money that they wanted to donate to their school. The following matrices show the number of
students who attended the occasions.
st nd
1 occasion 2 occasion
Kebele 1 Kebele 2 Kebele 1 Kebele 2
Boys  175 221  Boys  120 150 
   
Girls
 190 150  Girls  199 181 
a. Give the sum of the matrices.
b. If the tickets were sold for Birr 2.50 a piece on the 1st occasion and Birr 3.00 a piece on
the second occasion, how much money was raised from the boys? from the girls? In kebele 1. What is the total
amount raised for the school?
Multiplication of Matrices
To study the rule for multiplication of matrices, first let us define the rule for row and column matrices of orders 1
× p and p × 1, respectively.
Let A and B be a row and column matrices of orders 1 × p and p × 1, respectively, given by
 b11 
 
A   a11 a12 ... a1 p  and B   b21  . Then the product AB in the given order is the 1 × 1 matrix given by
 ... 
 
 a p1 
 b11 
 
b21 
AB   a11 a1 p 
 ...   11 11
a12 ...   a b a12b21 ... a1 p b p1 
 
 bp1 
 0
 
3
Example 7. If A   a14    4 0 2 1 and B  (b41 )    , then find AB.
 5
 
 7 

Telegram channel https://t.me/johnson201485


22
5
 0 
 
3
Solution. AB   a14  b41    4 0 2 1     4(0)  0(3)  2(5)  (1)(7)   17  which is a
 5 
 
 7 
1 by 1 matrix.
NOTE:
From the example given above, we can observe that in multiplication of a row matrix A   a1  p  and a column

matrix B   bp  1  ,
1. The number of columns of A = the number of rows of B = p
2. The operation is done row by column in such a way that each element of the row is multiplied by the
corresponding element of the column and then the products are added.
 2 3 

Now, let us consider two matrices A  1 0
 and B  1 1 0 4  . Then
   
 0 5   2 5 3 12  4
 3  2
1  1
A1 B1   2 3     2(1)  (3)(2)   (8) , A1 B 2   2 3     2(1)  (3)(5)   (17)
 2  5
0  4 
A1 B 3   2 3     2(0)  (3)(5)   (15) , A1 B 4   2 3     2(4)  (3)(1)   (11)
 3  1
1  1
A2 B1  1 0     1(1)  (0)(2)   (1) , A2 B 2  1 0     1(1)  (0)(5)   (1)
 2  5
0  4 
A2 B 3  1 0     1(0)  (0)(3)   (0) , A2 B 4  1 0     1(4)  (0)(1)   (4)
 3  1
Then the matrix determined by these products
 c11 c12 c13 c14   A1 B A1B A1B A1B   8 17 15 11 
1 2 3 4

C  (cij ) 24   
  
4 
 21 22 23 24   2  1  1 0  4 
1 2 3
c c c c A B A2 B A2 B A2 B
is called the PRODUCT of A and B.
Under this idea we define multiplication of matrix by matrix is defined as follow.
DEFINITION 11.4
Let A  (ai j ) m  p and B  (bk l ) p  n be matrices in ℝ , where , and are positive integers. The product of A and

B, denoted by AB , is the  matrix AB  (ci j )m  n where


ci j  ai1b1 j  ai 2b2 j  ai 3b3 j  ........  ai pbp j , for , …, and 𝑗, …, .
h
That is, the 𝑗 entry of the product is the sum of the product of the corresponding
h
entries of the row of A with the 𝑗 h column of B.
Thus,

Telegram channel https://t.me/johnson201485


22
6
c
AB   a b      ac  bd 
c
i) If A   a b and B    , then AB is a 1 by 1 matrix given by d 
d 
a
ii) If A    and B   c d  , then AB is a 2  2 matrix given by
b
a  ac ad 
AB     c d    
b  bc bd 
a b  e g
iii) If A    and B    , then AB is a 2  2 matrix given by
c d  f h
 a b  e f   ae  bg af  bh 
AB     
 c d  g h   ce  dg cf  dh 

REMARK
From the definition of product of matrices, you can observe that, if A is an m  s matrix and B is a t  n matrix,
then the product AB is defined only when s  t and the product will be a matrix of order m  n . That is, the
product AB is defined only when the number of columns of the first matrix A is equal to the number of rows of the
second matrix P.
In this case, we say that the two matrices are COMPATIBLE for matrix multiplication.
 2 0 
Example 8. If A   5 3 0 and B   3 5  , then find AB.
1 4
 
Solution. Since the number of columns of A = number of rows of B, A and B are compatible matrices and the
product AB is a 1  2 matrix given by
 2 0 
 
AB   5 3 0   3 5    5(2)  3(3)  0(1) 5(0)  3(5)  0(4)   16 15 
1 4
 
2 0
1 2   
Example 9. If A    and B   1 5  , then find AB.
7 2  4 3 
 
Solution. Since the number of columns of A  number of rows of B, A and B are not compatible matrices and
the product AB is not defined.
1 2
 1 0 2 1
Example 10. If M   2 3  and T  

 , then find MT.
0  2 3 1 5 
 4 
Solution. M is a 3  2 and T is 2  4 matrices. Since the number of columns of M = number of rows of T, then
M and T are compatible matrices and the product MT is a 3  4 matrix given by

Telegram channel https://t.me/johnson201485


22
7
1 2 1  4 0  6 2  2 1  10   3 6 4 9
   1 0 2 1    
MT   2 3      2  6 0  9 4  3 2  15    8 9 1 17 
 0 4   2 3 1 5   0  8 0  12 0  4 0  20   8 12 4 20 
     
1 2 2 3
Example 11. Let C    and D    , then find CD and DC. Is CD = DC?
0 4 0 7
1 2  2 3   2 17   2 3 1 2   2 16 
Solution. CD      and DC     
 0 4  0 7   0 21  0 7  0 4   0 14 
Here you can see that CD  DC. This shows that matrix multiplication is not commutative.
ACTIVITY 11.2
1) Let . / and . /. Find and . What do you observe?

2) Let ( ) and ( ). Find and . What do you conclude? Is ?

In Activity 14.2, you have observed that 1) , and 2)


That is
1) is the identity matrix for multiplication of matrices of order ; and
2) is the identity matrix for multiplication of matrices of order 3.
The IDENTITY (UNIT) MATRIX is a diagonal matrix where the elements of the principal diagonal are all ones.
NOTE: An identity matrix of order n is often denoted by I n .
For instance;
1 0 0 0
1 0 0  
1 0   0 1 0 0
I1  (1) , I2    , I   0 1 0  and I 
0 1
3
0 0 1
4
0 0 1 0
   
0 0 0 1
are identity matrices of order 1, 2, 3 and 4, respectively.
Example 12. Let A be a square matrix given by . / . Then
( ) ( ) ( ) ( )
. /. / ( ) . / and
( ) ( ) ( ) ( )
( ) ( ) ( ) ( )
. /. / ( ) . /
( ) ( ) ( ) ( )
Hence
Extending the above for any square matrix of order n, you will have the following result.
REMARK: For any square matrix A of order n, AI n  A  I n A , where I n is an identity matrix of order n

given by

( )

Telegram channel https://t.me/johnson201485


22
8
The identity matrix earns its name because it represents a MULTIPLICATIVE IDENTITY on the “algebra”
of all n  n matrices. That is, is the unique n  n-matrix with the property that AI n  A  I n A for every n  n-
matrix A.
Typical notation suppresses the subscript n in that indicates the dimension of the identity matrix.
Properties of Multiplication of Matrices
Let A and B be two matrices in R and ∈ ℝ. Assume that the indicated operations on matrices are compatible.
Then
a. A( B  C)  AB  AC (Multiplication of matrices is left distributive over addition).
b. ( A  B)C  AC  BC (Multiplication of matrices is right distributive over addition).
c. A( BC)  ( AB)C (Multiplication of matrices is associative).
d. k ( AB)  (kA) B
e. If 0 is the zero matrix of appropriate size, then A0 = 0 and 0A = 0 .
ACTIVITY 11.3
Let . /, . / and . /. Then find and . What do you observe? Is AB = AC? Is
B = C?
In activity 6,3 above, you can observe that
. / and . /. That is AB  AC . But B  C .
This shows that, cancellation of multiplication does not work on matrices
Recall that That is, cancellation of multiplication works on the set of real numbers. That is, for non-zero real
numbers a , b and c ; if ab  ac then b  c.
Transpose of a matrix and Its Properties
ACTIVITY 11.4

Consider matrices . / and ( ) . Can you mention how B is obtained from A?

Compare the 1st row of A with the 1st column of B, and the 2nd column of A with the 2nd column of B. What do you
observe?
From your responses in Activity 6.4, you have observed that, the rows of A are columns of B
and the columns of A are rows of B.
That is, B is obtained from A by changing rows of A to columns of B and columns of A to rows
of B and this process is called TRANSPOSING A MATRIX.
DEFINITION 11.5:The Transpose of a matrix A  aij   mn
, denoted by AT or At, is the n × m matrix

obtained by interchanging the rows and columns of A. That is AT  B   b ji  of order n × m such that
b ji  aij . Thus,
Example 13. Find the transpose of each of the following matrices.

Telegram channel https://t.me/johnson201485


22
9
 2 1  7
   1 5  
a) A   5 0 1 3 b). E   0 3  c) M    d) S   2 
 8 10   0 3 0
   
Solution.
 5 
 
0
a) A is a 1  4 row matrix. Therefore, A is a 4  1 column matrix given by A   
T T
 1 
 
 3
 2 0 8 
b) E is a 3  2 matrix. Therefore, ET is a 2  3 matrix given by AT   
 1 3 10 
1 0
c) M is a 2  2 square matrix. Thus, MT is a 2  2 square matrix given by M T   
5 3
d) S is a 3  1 column matrix. Hence, ST is a 1  3 row matrix given by S T   7 2 0 .
Properties of transposes of matrices
The following are properties of transposes of matrices:
a) (AT)T = A
b) (A + B)T = AT + BT, A and B being of the same order.
c) (rA)T = r(AT ), where r is any scalar.
d) (AB)T = BTAT; provided that AB is defined.
 1 5   2 4
Example 14. Let A    and B    . Then find
 7 0  3 1 
 
T
d)  AB 
T T
a) ( A  B)T b) 3A c) B
Solution. From the properties of transpose of matrix stated above;
 1 5   2 4 
T T

a) ( A  B)  ( A  (B))  A  (B)  A  B =    
T T T T T T

 7 0   3 1 
 1 5   3(1) 3(5)   3 15 
b) 3A  3   
 7 0   3(7) 3(0)   21 0 
T
  2 4 T   2 3 T  2 4
c)  B     3 1     4 1    3
T T
B
      1

 1 5  2 4   1(2)  5(3) 1(4)  5(1)   17 1 


d) AB      
 7 0  3 1   7(2)  0( 3) 7(4)  0(1)   14 28 

 17 1   17 14 
T

Hence,  AB    
T

 14 28   1 28 

Telegram channel https://t.me/johnson201485


23
0
Symmetric and Skew Symmetric Matrices
ACTIVITY 11.5

Consider square matrices . / and ( ).

1. Find a) AT b) BT c) A  AT d) B  BT
2. What do you observe? Is AT  A ? BT   B ?
From your responses in Activity 6.5, you can observe that
A  AT  0 . That is A  A . Such types of matrices are known as symmetric matrices.
T
i)
B  BT  0 . That is B   B . Such types of matrices are known as skew-symmetric matrices.
T
ii)
DEFINITION 11.5
a) Let ( ) be a square matrix of order . Then A is said to be a SYMMETRIC
MATRIX, if . That is, A is a symmetric matrix, if 𝑗 = 𝑗 for 1 ≤ , 𝑗 ≤
b) Let ( ) be a square matrix of order . Then A is said to be a SKEW-
SYMMETRIC MATRIX, if . That is, A is a skew-symmetric matrix, if 𝑗 =  𝑗
for 1 ≤ , 𝑗 ≤ .
Note that, if a matrix ( ) is skew-symmetric matrix, then for , for all 1 ≤ ≤ .
Example 15. Identify the following matrices as symmetric, skew-symmetric or neither.
 2 0 3   0 1 3  2 1 4
    1 0  
a. A   0 1 5  b. M   1 0 5  c. B    d. E   1 0 5 
 3 5 0  3 5 0 0 1  3 5 1
     
Solution.
 2 0 3   2 0
T
3

a. A  0
  
T
 1 5    0 1 5   A . Therefore, A is symmetric matrix.
 3 5 0   3 5 0 
   
 0 1 3   0 1 3 
T

  
b. M  1 0 5  1

T
   0 5    M . Hence, M is skew-symmetric matrix.
 3 5 0   3 5 0 
   
T
1 0 1 0
c. B       B . Therefore, B is symmetric matrix.
T

0 1 0 1
 2 1 4   2 1 3 
T

   
d. E  1 0 5
   1 0 5  . Here, you can see that E  E and E   E .
T T T

 3 5 1  4 5 1 
   
That is E is neither symmetric nor skew-symmetric.

Telegram channel https://t.me/johnson201485


23
1
 2 0 3 
Example 16. Consider a square matrix A   7 1 5  . Then find matrices
 
5 4 3
 
1 1
a. B  ( A  AT ) b. C  ( A  AT ) c. B + C
2 2
 2 7 5 
Solution. The transpose of A is AT   0 1 4  . Then
 
 3 5 3 
 
 2 0 3   2 7 5   4 7 2 
     
A  A   7 1 5    0 1 4    7 2 1 and
T

 5 4 3   3 5 3   2 1 6 
     
 2 0 3   2 7 5   0 7 8 
     
A  A   7 1 5    0 1 4    7 0 9  ,
T

 5 4 3   3 5 3   8 9 0 
     
Here, you can observe that A  AT is symmetric and A  AT is skew-symmetric matrices.
 7 
 2 2 1 
 4 7 2   
  7
B   A  A    7 2 1 
a. 1 1 1  which is symmetric matrix.
T
1 
2 2  2 2

 2 1 6   
 1  1 3 
 2 
 7 
 0  2 4 
 0 7 8   
b. C  1 ( A  AT )  1  7 0 9    7 0  9  which is skew-symmetric matrix.
2 2   2
  2
 8 9 0  
 4 9 0 
 2 
c. Adding B and C, we obtain
 7   7 
 2 2 1  0 
2
4 
     2 0 3 
 7 1 7 9  
BC  1   0    7 1 5   A
 2 2   2 2 
     5 4 3 
 1  1   9
3  4 0 
 2   2 
This shows that A  B  C   A  AT    A  AT  .
1 1
2 2
symmetric skew  symmetric

Telegram channel https://t.me/johnson201485


23
2
Thus, any square matrix A can be written as the sum of symmetric matrix
1
2
 A  AT  and skew-symmetric
matrices 1  A  AT  .
2
3 0 1
Example 17. Express a square matrix M   1 5 0  as the sum of symmetric and skew-symmetric
 4 2 8 
 
matrices.
1 1
Solution. M  M  0  (2M  0)  [( M  M )  ( M T  M T )] 
2 2
1
2
 M  M T   M  M T  .
1
2
symmetric skew  symmetric

 3 0 1   3 1 4 
T


Now, M  1 5 0
  
   0 5 2  .
T

 4 2 8   1 0 8 
   
 5 1
 3 
 3 0 1   3 1 4    2 2

1    
1
2
 M  M T    1 5 0    0 5 2     
2  
5 1  and
1
2 
  
 4 2 8   1 0 8    5 
 1 8 
 2 
 1 3
 0 2 2
 3 0 1   3 1 4    
1      1
1
2
 M  M    1 5 0    0 5 2    
T

2  
0 1  .

   1 0 8    2 
 4 2 8   
 3 1 0 
 2 
 1 5  1 3
 3 2 2   0 2 2
   
Then M   M  M    M  M   
1 T T  1 
5 1   1
0 1 
2  2   2 
   
 5 
1 8   3
1 0 
 2   2 
symmetric skew symmetric

Telegram channel https://t.me/johnson201485


23
3
Exercise 11.1
1. Construct a 2 × 3 matrix A  (aij ) whose elements are given by:
1 𝑗 𝑗
a) aij  (ij  4i) b) {
2 𝑗 𝑗
x y  x 6   4 x  y
2. Given 3     . Find the values of x, y, z and w.
 z w   1 2w   z  w 3 
3. Let A, B, C, D and E be matrices with order 2  3, 2  2, 4  3, 2  3 and 3  4, respectively. Find the orders
of the following, if defined.
a. (A  D)E b. DCTA d). (BA  D)E c. (E + CT)B e. AEC
 2 1
 0 1 2     3 2 
4. If A    , B   3 0  and C    , then find
 3 1 3   1 4   1 5 
 
a) CB b) AB + C c) BA  3I3 .
 1 1 
If A   c. A  3 A  2 I 2
2 2
5.  , find a. A b. 2 A  3I 2
2 0 
 cos  0  sin    cos  0 sin  
   
6. If A   0 1 0  , and B   0 1 0  , then show that AB  I3 .
 sin  0 cos     sin  0 cos  
   
 1 2   1 0
7. If A    and B    , then show that
 0 1  1 2 
a) ( A  B)( A  B)  A2  2 AB  B2 b) ( A  B)( A  B)  A2  B2
 0 3 1 
 
8. Write the square matrix M   2 1 4  as the sum of symmetric and skew-symmetric matrices.
 2 0 5 
 
 2 0   4 2 
   
9. Let A   1 1  and B   1 1  . If 3B  2 X T  5 A , then find the matrix X .
3 5  0 3 
   
 cos  sin    sin   cos  
10. Simplify cos     sin   .
  sin  cos    cos  sin  
 cos  sin  
11. If A    , then show that A A  I 2 .
T

  sin  cos  
 2 0 1
 
12. If A   1 2 0  and (2 A  B)T  AT A , then find the matrix B.
 0 0 1
 

Telegram channel https://t.me/johnson201485


23
4
5 2  3 6
13. Find matrices X and Y, if X    and Y    .
0 9  1 2 
x y  x 6   4 x  y
14. Given 3    .
 z w   1 2w   z  w 3 
 cos x  sin x 0 
 
15. If F ( x)   sin x cos x 0  , then show that F ( x) F ( y)  F ( x  y) .
 0 1 
 0
 3 2  1 0
16. If A    and I    , then find k so that A  kA  2I .
2

 4 2  0 1
 1 2 3
 
17. If A   3 2 1  , then find A3  23A  40I  0 .
 4 2 1
 
 cos  sin     sin  cos  
18. Simplify cos     sin   
  sin  cos    cos  sin  
19. A trust fund has Br. 30,000 that must be invested in two different types of bonds. The first bond pays 5%
interest per year, and the second bond pays 7% interest per year. Using matrix multiplication, determine how to
divide Br. 30,000 among the two types of bonds. If the trust fund must obtain an annual total interest of: (a) Br.
1800 (b) Br. 2000
 cos   sin  
Let A    . If A  A  I , then find the value of  .
T
20.
 sin  cos  
0 2y z 
 
Find the values of x, y and z if  x y  z  satisfy the equation A A  I 3
T
21.
 x y z 
 
 1 2 0  0 
  
22. For what value of x 1 2 1  2 0 1  2   0
 1 0 2  x 
  
 3 1
If A    , then show that A  5 A  7 I 2  0
2
23.
 1 2 
 1 0 2  x 
  
24. Find x , if  x 5 1  0 2 1  4   0
 2 0 3  1 
  
If A is a square matrix such that A  A , then show that  I  A   7 A  I , provided that I is an identity
2 3
25.
matrix having the same order as A.

Telegram channel https://t.me/johnson201485


23
5
11.2. DETERMINANTS AND THEIR PROPERTIES
Every square matrix can be associated with a real number called its determinant. It is helpful in solving
simultaneous equations. Only square matrices have determinants. The matrices which are not square do not have
determinants
The determinant of a square matrix is a real number that is assigned to the matrix according to certain rules. The
determinant gives you valuable information about the matrix.
NOTATION: The determinant of a square matrix A is denoted by det( A) or A or .
The determinant of a square matrix A is associated with A according to the following definition.
Determinants of 1 × 1 and 2 × 2 matrices
DEFINITION 11.6
1. The DETERMINANT of a 1 × 1 matrix A = (a) is the real number given by |A| = a.
a a12 
2. The DETERMINANT of a 2 × 2 matrix A   11  is defined to be the number a11a22  a12 a21 .
 a21 a22 
a11 a12
Thus, A  det( A)   a11a22  a12 a21
a21 a22
 3 5
Example 18. The determinant of a square matrix A    is
 1 2 
3 5
det( A)  A   3(2)  5(1)  6  5  11
1 2
1 0
Example 19. Find the determinant of an identity matrix I 2   .
0 1
Solution. The determinant of the identity matrix I2 for multiplications is given as
1 0
I2   1(1)  0(0)  1
0 1
 2 1  x 
Example 20. If D    and det(D) = 0, then find the value of x. .
 3 x 
2 1  x
Solution. det( D)  0
3 x
 2( x)  (1  x)  2 x 1  x  0
  x 1  0  x  1
Determinants of a 3 × 3 matrices
Determinant of a matrix of order three can be determined by expressing it in terms of second order determinants.
To define the determinant of a 3 × 3 matrix, it is useful to first define the concepts of minor and cofactor.

Telegram channel https://t.me/johnson201485


23
6
ACTIVITY 11.6

Let ( ) . Then find

a) matrix B obtained by crossing-out the 2nd row and the 1st column from A.
b) det(B) and the value of (1)21 |B|.
c) matrix C obtained by crossing-out the 1st row and the 2nd column from A.
d) det(C) and the value of (1)12 |C|.
From your responses for Activity 6.6, observe that
(i) both . / and . / are 2  2 matrices;

(ii) the determinant of B : | | | | ( ) ( ) , is called the MINOR of the


entry a21 , the entry in the second row and the first column of A.
The signed minor (1)21 |B| = (12) = 12 is called the COFACTOR of the entry a21 .

(iii) the determinant of C : | | | | ( ) ( ) is called the MINOR of the


entry a12, the entry in the first row and the second column of A.
The signed minor (1)12 |C| = (4) = 4 is called the COFACTOR of the entry a12.
DEFINITION 11.7
Let n be a positive integer with n > 1., and suppose ( ) be a square matrix of order n, for
and 𝑗 .
1) Cross-out the row and the 𝑗 column of A and obtain a square matrix of order ( )
( ) and denote this matrix by .
That is

Then det(Aij) is called the MINOR of the entry 𝑗 of A, denoted by M 𝑗.


Thus, t( )
2) The signed minor ( ) is called the COFACTOR of an entry , denoted by .
That is the cofactor of an entry of A is given by
𝑗
( ) {
𝑗

Example 21. Let ( ). Then determine the minors and cofactors of the entries , and

Telegram channel https://t.me/johnson201485


23
7
Solution. The minor of = t( ) | | . It is found by crossing out
the first row and the first column as in the figure given below.

The cofactor of = =( )
In a similar manner,
The minor of = | |,

while the cofactor of = =( ) | |

The minor of = | |,

while the cofactor of = =( ) | |


NOTE:
Note that the „sign‟ (−1)i + j accompanying the minors form a chess board pattern with „+‟s on the main
diagonal as shown :

. / for n = 2, and ( ) for n = 3.

Example 22. Determine the minors and the cofactors of all the entries of the 3  3 matrix

( ).

Solution. The minors of all the entries of A are:


t( ) | | | | | |

In a similar manner | | | | | |

| | | | | |
Then The cofactors of all the entries of A are:

ACTIVITY 11.7
Let ( ) . Then find

a) ------------ Expansion along the first row


b) ------------ Expansion along the second column
c) ------------ Expansion along the third row

Telegram channel https://t.me/johnson201485


23
8
Using the values in example 21 above,
a) ( ) ( )( ) ( )
b) ( ) ( ) ( )
c) ( )( ) ( ) ( )
From your responses in Activity 11.7, observe that the answer for all questions is the same
number and this number is called the determinant of A.
You can now define the determinant of order 3 as follows:

DEFINITION 11.8: Let ( ). Then the DETERMINANT of A along any row or

any column is given by one of the formulas :


ith row expansion : |A| = ai1ci1 + ai2ci2 + ai3ci3, for any row i (i = 1,2 or 3), or
jth column expansion: |A|= a1jc1j + a2jc2j + a3jc3j, for any column j (j = 1,2 or 3).
Definition 11.8 shows how the determinant of a square matrix obtained by cofactor expansion method.
NOTE:
The definition of determinant states that to find the determinant of a square matrix, follow the following procedure:
STEP 1. Choose a row or column;
STEP 2. Multiply each entry in it by its cofactor;
STEP 3. Add up these products.
This sum is the determinant of the matrix.

Example 23. Let ( ). Then, find the determinant of A using cofactor expansion along the

three rows of A.
Solution:
a. The determinant of A using the cofactor expansion along the first row is given by :
| | ( ) ( ) ( )
. | |/ ( ). | |/ . | |/
( ) ( )( ( ( )) ( )
b. The determinant of A using the cofactor expansion along the second row is given by :
| | ( ) ( ) ( )
. | |/ ( ). | |/ . | |/
( )( ) ( )
c. The determinant of A using the cofactor expansion along the third row is given by :
| | ( ) ( ) ( )
. | |/ . | |/ . | |/
( ) ( ) ( )
Thus, the determinant of A using the cofactor expansion along any of its three rows is the same,
that is, | | .
In the similar manner, if you calculate the determinant of A using the cofactor expansion along any of its three
columns, you obtain the same result | | .
Therefore, the determinant of a square matrix is independent of the choice of rows and columns.

Telegram channel https://t.me/johnson201485


23
9
NOTE:
In determining the determinant of a square matrix by cofactor expansion along a row (or a column), the best
strategy is to use a cofactor expansion along a row (or a column) with large number of zeros.

Example 24. Compute the determinant of the matrix ( ) ( ).

Solution. As the second column has two zero entries, more zero entries than the other columns or any rows, you
can use a cofactor expansion along the second column. Thus,
| |
| | | | | | | | ( )

REMARK: In the above discussion, the determinant of a 3 by 3 matrix is calculated by the use of minors and
cofactors which is a general rule for calculating determinants of any squared order. But, for only 3 by 3 matrix,
the following approach can also be used to calculate determinants.

Example 25. Calculate the determinant of the matrix ( ).

Solution. To calculate such a determinant, you follow the following steps:


Step 1. Write the first two columns beside the third column. ( )

Step 2. Multiply and add the triple numbers presented left to right.

( )( ) ( )( ) ( )( )( )
Step 3. Multiply and add triple numbers presented right to left.

( )( ) ( )( ) ( )( )( )
Step 4. Subtract the result of step 2 from result of step 3. The resulting number is the determinant. Therefore, the
determinant of A is |A| = 2  21 = 23
Properties of Determinants
ACTIVITY 11.8
For a matrix ( ) , do each of the following.

1. a) Calculate | | and | |. b) What do you conclude from these result? Is | | = | |?


2. Let B be the matrix found by interchanging row 2 and row 3 of matrix A.

Telegram channel https://t.me/johnson201485


24
0
a) Calculate | | and compare it with | |. b) What relationship do you see between |B| and |A|?
3. Let C be the matrix found by multiplying row 1 of A by 4.
a) Calculate | | and compare it with | |. b) What relationship do you see between |C| and |A|?
4. Let D be the matrix found by adding 5 times column 1 on column 2.
a) Calculate | | and compare it with | |. b) What relationship do you see between |D| and |A|?
From your responses in Activity 6.8, observe that

1. | | | | and | | | |

Here you can see that | | | |

2. ( ) and | | | | ( ) ( )

Here you can see that | | | |


 Note that you obtain the same result if you repeat this process on any two columns.

3. ( ) ( ) and | | | |

Here you can see that | | | | ( ) .


 Note that you obtain the same result if you repeat this process on any column.
( )
4. ( ( ) ) ( ) and | | | | .
( )
Here you can observe that the determinant of A is unchanged. That is | | | |
 Note that you obtain the same result if you repeat this process on row.
Thus, without proof we can state the following properties of determinants of square matrices. You can use more
additional square matrices to illustrate these matrices.
THEOREM 11-1 (Properties of Determinants)
The following properties hold. All the matrices considered are square matrices:
1. | | | |
2. If B is a matrix found by interchanging two rows (two columns) of A, then |B| = − |A|.
3. If B is a matrix found by multiplying one row (one column) of A by a scalar k, then |B| = k|A|.

That is, for instance, | | | | and | | | |

4. If B is a matrix obtained by adding a multiple of a row (a column) of A to another


row (column) of A, then |B| = |A|.
5. If A has a row (or a column) of zeros, then the determinant of A is zero.

For instance, consider a matrix ( ) . then using cofactor expansion along the

third column, we obtain

| | | | ( ) ( ) ( )

6. If A has two identical rows (or columns), then the determinant of A is zero.

Telegram channel https://t.me/johnson201485


24
1
For instance, consider a matrix ( ). Here rows 1 and 3 are identical. Then using

cofactor expansion along the second row, we obtain

| | | | ( ) ( ) ( )

7. If some or all elements of a row (or a column) of a determinant are expressed as sum of two (or more)
terms, then the determinant can be expressed as sum of two (or more) determinants. That is, for instance;

| | | | | |

8. Let A and B be two square matrices of order n. Then AB  A B


We shall illustrate Theorem 6.1 with the following examples.

Example 27. Calculate the determinant of ( ).

Solution. Using cofactor expansion along the third row, we obtain | | | | ( )

( ) ( )

Example 28. Let | | | | . Then compute each of the following determinants.

a) | | b) | | c) | |

d) | | e) | |

Solution. Let ( ). Hence | |

a) ( ) is obtained by interchanging the 1st and 3rd column of .

Hence, | | | | | |

b) ( ) has identical columns. Hence, | |

c) Matrix ( ) is obtained by multiplying the 2nd row of A by 3. Hence,

| | | | | |

Telegram channel https://t.me/johnson201485


24
2
d) Matrix ( ) is obtained by replacing the 3rd row of A by 3rd row of A plus (2)

times the 1st row of A. No other operation has been done on A. Hence,

| | | |

e) A sequence of operations has been done on matrix A to obtain matrix | |.

( )
| | | | | | ( )| |

( )| | | | ( )

6 1 3 2
Example 29. Without expansion, show that 2 0 1 4
0.
3 6 1 2
4 0 2 8

Solution. Using property (3) of Theorem 6.1, In the L.H.S, taking 2 common from fourth row,
6 1 3 2 6 1 3 2 6 1 3 2
2 0 1 4 2 0 1 4 2 0 1 4
 2  2(0)  0 ,
3 6 1 2 3 6 1 2 3 6 1 2
4 0 2 8 2( 2) 2(0) 2(1) 2(4) 2 0 1 4
since rows 2nd and 3rd are identical in the last determinant.
Example 30. Let A and B be two square matrices of order 3 with |A | = 4 and |B| = 5. Then compute a) AB

b) 2 AT B c) A3

Solution. Using properties of Theorem 6.1,


a) AB  A B  4(5)  20
b) 2 AT B  23 AT B  8 A B  8(4)(5)  160

A3  A A A  A  43  64
3
c)
Using Property (8) of Theorem 6.1 and the associative property of matrix multiplication, if A
Am  A
m
is a square matrix and m is a positive integer, then

Example 31. Let A be two square matrices of order 3 with |A | = 2 . Then compute A5 .

A5  A  (2)5  32
5
Solution.

Telegram channel https://t.me/johnson201485


24
3
NOTE: One consequence that follows from property (3) of Theorem 6.1 is, if you multiply a square matrix of
order n by a scalar , then the determinant of the new matrix is times the
determinant of the given matrix.
That is, if A is a square matrix of order and is a scalar, then | | | |
Example 32. Let | | . Then compute | |.

Solution. | | | |

Example 33. Let M be a square matrix of order 5. If | | , then find | |.


Solution. | | | | ( )

Example 34. Let | | . Then find the value(s) of .

Solution. | | ( )| | | | | |

( )( ( ) ( )) ( ( ) ( )) ( ( ) ( ))
( )( ) ( ) ( )
( ) ( ) ( )

or

Example 35. Show that | |

Solution. Applying property (7) above, we have

| | | | | |

| | | | ( )
⏟ ⏟
( ) ( )

Example 36. Show that | | ( )

Solution. By cofactor expansion along the second row,

| | | | | | | | | |

, ( ) ( ) ( )- , ( ) ( ) ( )-
, ( ) ( ) ( )- , ( ) ( ) ( )-

( ) ( ) ( )

Telegram channel https://t.me/johnson201485


24
4
Note that determinant of a triangle can be used in coordinate geometry to find the equation of a a line and the area
of a triangle.
AREA OF A TRIANGLE
The area of a triangle whose vertices are (x1, y1), (x2, y2) and (x3, y3), is given by the expression
x1 y1 1
1
A  x2 y2 1 --------------------------- (1)
2
x3 y3 1
Example 37. Find the area of the triangle whose vertices are (3, 8), (– 4, 2) and (5, 1).
Solution. Letting ( x1, y1 )  (3, 8), ( x2 , y2 )  (4, 2) and ( x3 , y3 )  (5,1) , we have
3 8 1
1 1
A  4 2 1  [3(2  1)  8(4  5)  (4  1)]  35 sq.u
2 2
5 1 1
REMARKS
i) Since area is a positive quantity, we always take the absolute value of the determinant in (1).
ii) The area of the triangle formed by three collinear points is zero.
Remark (ii) above helps us to find the equation of a line passing through two points and to check the collinearity of
three lines.
EQUATION OF LINE and COLLINEAR POINTS
1. The equation of a line passing through two points (x1, y1) and (x2, y2) given by
x y 1
x1 y1 1  0 --------------------------- (2)
x2 y2 1
2. Three distinct points (x1, y1), (x2, y2) and (x3, y3) are collinear (or lie on a line) if
x1 y1 1
x2 y2 1  0 --------------------------- (3)
x3 y3 1
Example 38. Find the equation of a line passing through points ( 4, 2) and (1, 3).
Solution. Using equation (3), the equation of a line passing through points ( 4, 2) and (1, 3) is
x y 1 x y 1
y 1 x 1 x y
x1 y1 1  0  4 2 1  0  4  (2) 1 0
3 1 1 1 1 3
x2 y2 1 1 3 1
 4( y  3)  2( x 1) 1(3x  y)  0
 4 y  12  2 x  2  3x  y  0
 5x  3 y 14  0
Example 39. Find the value of k so that points ( 4  k, 2) , (1, 3) and ( 2k + 1, 0) are collinear.
Solution. Using equation (3), if points (x1, y1) = ( 4  k, 2) , (x2, y2) = (1, 3) and
(x3, y3) = ( 2k + 1, 0) are collinear, then

Telegram channel https://t.me/johnson201485


24
5
x1 y1 1 4k 2 1
x2 y2 1  0  1 3 1 0
x3 y3 1 2k  1 0 1
3 1 1 1 1 3
 (4  k ) 2 1 0
0 1 2k  1 1 2k  1 0
 3(4  k )  2(1  2k 1)  1(6k  3)  0
 12  3k  4  4k  6k  3  0
19
 7k  19  0  k 
7
Exercise 11.2
1. Evaluate the following determinants:

sin s 3 1 4
a) | | b) | sin sin | c) 6 2 8
s sin 1 5 7
2. Show that
1 1 2 1
3 1 4 5
a) | | b) | | ( )( )( ) c)  60
7 6 1 2
1 1 3 4
1 1 1
3. Find if a) | | | | b) | | | | c) t 1 t 0
1 t 2 3
4. Find equation of the line joining (3, 1) and (9, 3) using determinants.
5. Find the area of a triangle having vertices P(1, 5), Q(2, 0) and R(1, 3).
 2 1 3 
 
6. The 3×3 matrix A is defined in terms of the scalar constant k by A   k 2 4 .
k  2 3 k  7
 
If A  8 , then find the possible values of k.
7. Verify each of the following statements is TRUE. Assume that all letters represent non-zero
real numbers.
1 a bc
x tw x t x w a  rb b a b
a)   b)  c) 1 b c  a  0
y su y s y u c  rd d c d
1 c ab

Telegram channel https://t.me/johnson201485


24
6
11.3. INVERSE OF A SQUARE MATRIX
In this subunit, you will learn how to find inverses of some square matrices. Finding inverses of square matrices,
whenever they exist, is important in solving systems of linear equations.
ACTIVITY 11.9
Consider a matrix a matrix . /. Find a square matrix B order 2 such that
AB  I 2  BA , where I 2 is an identity matrix of order 2. If this matrix exist what do we call it?
a b
To solve the problem in Activity 6.9, let B    . Then AB  I 2
c d
 2 3  a b   1 0   2a  3c 2b  3d   1 0
       
 1 2  c d   0 1   a  2c b  2d   0 1
 2a  3c  1  2b  3d  0
  and 
 a  2c  0  b  2d  1
2 3 1 2
Solving these two systems of equations, we obtain values a   , b  , c  and d  .
7 7 7 7
 2 2
 7 7
Thus, B    . Checking the other side product,
 1 2
 
 7 7
 2 3  4 3 2 2
 7 
7  2 3   7 7
   
7 7 1 0
BA      
 1 2  1 2  2 2 3 4  0 1
     
 7 7  7 7 7 7 
Therefore, AB  I 2  BA . This shows that B is the multiplicative inverse of A in “matrix algebra”, as in
multiplicative inverse in algebra of real numbers.
DEFINITION 11.9
A square matrix A is said to be INVERTIBLE (or has an INVERSE) or NON-SINGULAR, if and only if there
is a square matrix B such that AB = BA = I, where I is the identity matrix that has the same order as A. In this case
we say that matrix B is the INVERSE of matrix A.
The inverse of matrix A, whenever it exists, is denoted by A−1. Thus, B = A−1 .
3 1  2 1
Example 40. Show that B    is the inverse of A   .
5 2  5 3 
 2 1 3 1   2(3)  1(5) 2(1)  1(2)   1 0 
Solution. AB       and
 5 3  5 2   5(3)  3(5) 5(1)  3(2)   0 1 
 3 1  2 1  3(2)  1(5) 3(1)  1(3)   1 0 
BA      
 5 2  5 3   5(2)  2(5) 5( 1)  2(3)   0 1 

Telegram channel https://t.me/johnson201485


24
7
3 1
This implies that AB  I 2  BA . Thus, B is the inverse of A and we write as A1   .
5 2
In this case we say that A and B are inverse to each other.
NOTE:
1. Only a square matrix can have an inverse.
2. A 1 and A have the same order.
3. A matrix that does not have an inverse is called SINGULAR.
REMARK: The inverse of a square matrix, if it exists, is unique.
Proof: Let A be an invertible square matrix. Suppose B and C are inverses of A.
Then AB = BA = I. and AC = CA = I (by definition of inverse)
Now, B = BI = B (AC) = (BA) C = IC = C. Hence, the inverse of A is unique.
Now it remains to answer one big question.
BIG QUESTION: when does the inverse exist?
ADJOINT OF A MATRIX
DEFINITION 11.10
The ADJOINT of a square matrix A  (ai j ) is defined as the transpose of the matrix C  (ci j ) where ci j are the

cofactors of the elements ai j . Adjoint of A is denoted by adj A.

That is adj A = C  (ci j ) .


T T

 1 5 
Example 41. Find the adjoint of the square matrix E   .
 3 1
Solution. Let E  (ei j ) and ci j = the cofactor of the entry ei j . Then
c11  (1)11 1  1 , c12   | 5 | 5 , c21   | 3 | 3 and c22   |1| 1

 1 5 
Hence, the matrix of cofactors of E is C   .
 3 1 
 1 5   1 3 
T

Therefore, the adjoint of E is given by adj E  C  T


  
 3 1   5 1 
1 2 1
Example 42. Find the adjoint of the square matrix A  1 4 
3 .

1 2 0 

Solution. The cofactors of the entries of A are
4 3 1 3 1 4
c11    6 , c12   3 , c13    2
2 0 1 0 1 2
2 1 1 1 1 2
c21   2 c22    1 c23   0
2 0 1 0 1 2

Telegram channel https://t.me/johnson201485


24
8
2 1 1 1 1 2
c31   2 c32    2 c33   2
4 3 1 3 1 4
 1 5 
Hence, the matrix of cofactors of A is C   .
 3 1 
 1 5   1 3 
T

Therefore the adjoint of A is given by adj A  C  


T
  
 3 1   5 1 
c12   | 5 | 5 , c21   | 3 | 3 and c22   |1| 1

 6 3 2 
 
Hence, the matrix of cofactors of A is C   2 1 0  .
 2 2 2 
 
 6 2 2   6 3 2 
T

  
Therefore the adjoint of A is given by adj A  C  3 1 0  2 1 2
T 
   
 2 2 2   2 0 2 
   
ACTIVITY 11.10
Consider a matrix . /. Then find a) |A| b) adj A c) A(adj A) d) (adj A)A e) |A| I2
f) what do you observe from the above results?
From your responses in Activity 6.10, observe that (adj A) A | A | I 2  A(adj A)
This property of adjoint matrix is holds true for a square matrix of order n.
Statement (ii) above implies that if A  0 , then A is invertible or non-singular and the inverse of A is given by
1
A1  (adj A) .
| A|
Example 43. Let A be a square matrix of order 3 and |A| = 2. Then find adj A .
Solution. From the relation (adj A) A | A | I3 , taking determinant of both sides, we have

(adj A) A  | A | I3
adj A A | A |3 I3 …………… (since | AB || A || B | and | k I 3 | k | I 3 | )
3

 adj A | A |2 …………… (since | I n | 1 and | A |  0 )

 adj A  22  4
ACTIVITY 11.11
a b
Consider a square matrix of order 2 given by A    . Then find
c d
d)  adj A 
T T
a) A b) adj A c) adj ( AT )
e) what do you observe from the above results?
From your responses in Activity 6.10, observe that

Telegram channel https://t.me/johnson201485


24
9
a c   d b 
a) AT    b) adj A   
b d   c a 
That is to find Adjoint of a square matrix of order 2.
 Interchange the diagonal elements, and  Change the signs of non-diagonal elements.
 d c   d c 
c) adj ( AT )    d) (adj A)T   
 b a   b a 
Comparing the results of (iii) and (iv), you can observe that adj ( AT )  (adj A)T .
The above property of adjoint of square matrix can also extend for any square matrix order n > 2.
In general, from the above discussions, we have the following properties of adjoint of matrices.
PROPERTIES OF ADJOINT OF SQUARE MATRICES
Given a square matrix of order n,
1 1
i) (adj A) A | A | I n  A(adj A) ii) If A  0 , then  (adj A) A  I n   A(adj A)  .
| A| | A|
n 1
iii) If A  0 , then adj A  A .
1
iv) An identity matrix I is its own inverse. That is I  I v). adj ( AT )  (adj A)T

 Now, you are at an appropriate stage of computing the inverses of invertible square
matrices.
From the definition of inverse matrix and property (ii) of adjoint matrices, if A  0 , then
1 1
 (adj A) A  I n   A(adj A)  .
| A| | A|
1
This shows that A is invertible and its inverse is given by A1  (adj A) . This proofs the following theorem of
| A|
inverse.
THEOREM 11-2 : A square matrix A is invertible, if and only if A  0 . If A is invertible, then,
1
A1  (adj A)
| A|
 0 2 3 
Example 43. Find the inverse of the square matrix A   1 3 3  , if invertible.
 1 2 2 
 
Solution. First check whether A is invertible.
0 2 3
3 3 1 3 1 3
A 1 3 3 0  (2)  (3)  1
2 2 1 2 1 2
1 2 2
 | A | 0 . This implies that A is invertible.
The cofactors of A are

Telegram channel https://t.me/johnson201485


25
0
c11  0 , c12  1 c13  1
c21  2 , c22  3 c23  2
c31  3 , c32  3 c33  2

 0 1 1 
 
Then the matrix of cofactors of A is C   2 3 2  .
 3 3 2 
 
 0 1 1 
T
0 2 3
The adjoint of A is adj A  C  2 3 2   1 3 3 
T  
   
 3 3 2  1 2 2
   
Therefore, the inverse of A is
0 2 3   0 2 3   0 2 3 
1 1      
A1  (adj A)    3 3   (1)  1 3 3    1 3 3 
| A| 1    1 2 2   1 2 2 
1 2 2     
 1 2 1
Example 43. Show that matrix A   3 4 5  is singular.
 4 2 6 
 
1 2 1
4 5 3 5 3 4
Solution. A  3 4 5  1 2  (1)  1(24  10)  2(18  20)  1(6  16)  0
2 6 4 6 4 2
4 2 6
Hence, A is singular (has no inverse).
a b
REMARK: Given a 2  2 square matrix A   .
c d
 d b 
1. The adjoint of A is given by adj A   .
 c a 
1 1  d b 
A1  (adj A)   
A ad  bc  c a 
2. If A  0 , then A is non-singular and
Example 44. Check whether each of following matrices are non-singular.
 4 3   1 2 
a) A  b) B   
 1 3   3 6 
Solution.
4 3
a) A  4(3)  (3)(1)  9  0 .
1 3

Telegram channel https://t.me/johnson201485


25
1
1 1
1 1  d  b  1  3 3  3 3
Hence, A is non-singular and A      
ad  bc  c a  9  1 4   1 4
 
9 9
1 2
b) B   1(6)  (2)(3)  6  6  0 Hence, B is singular (has no inverse).
3 6
THEOREM 11-3 : If A and B are two invertible matrices of the same order, then ( AB)1  B1 A1
Proof: ( AB)( B1 A1 )  A(BB1 ) A1  AIA1  AA1  I , and in a similar manner
( B1 A1 )( AB)  B1 ( A1 A) B  B1IB  B1B  I
This shows that ( AB)( B1 A1 )  I  ( B1 A1 )( AB)
1 1
Thus B A is the inverse of AB. Therefore, ( AB)1  B1 A1
a b e f
For instance; consider two matrices of order 2 given by A    and B    , such that
c d g h
A  ab  bc  0 and B  eh  fg  0 . Then
AB  A B  (ab  bc)(eh  fg )  0 . This implies that AB is invertible.

a b  e f   ae  bg af  bh 
AB     
c d  g h   ce  dg cf  dh 

1  cf  dh (af  bh) 
 ( AB)1    …………………………> (*)
| AB |  (ce  dg ) ae  bg 
1  d b  and 1  h f 
A1    B 1   
| A |  c a  | B |  g e 
1  h  f  1  d b  1  h  f  d b 
 B 1 A1   .     
| B |  g e  | A |  c a  | A || B |   g e   c a 
1  dh  cf (bh  af )  …………………………> (**)
  
| A || B |  (dg  ce) bg  ae 
Comparing (*) and (**) , we obtain the relation ( AB)1  B1 A1
1
REMARK: If A is an invertible matrix of order n, then A1  A
1 1
Proof: A is an invertible matrix implies that A  0 . Now, from A  (adj A) , we have
A
…………………….. since kA  k | A |
n n
1  1 
A1  ( adj A)  
 A 
 adj A
A  
n 1
…………………….. …………… since adj A | A |
n
 1  n 1

 A 
 A
 
1 1
  A
A

Telegram channel https://t.me/johnson201485


25
2
Example 45. If A is a square matrix order 3 and A  5 , then find A 1 .

Solution. A1  1  1 .
| A| 5

Exercise 11.3
1 0 2  11 2 2 
   
1. Show that matrices  2 1 3  and  4 0 1  are inverse to each other.
4 1 8  6 1 1
   
2. Find the inverses of each of the following matrices, if exist.
 4 0 3   2 1 0 
 2 6   4 2     
a)   b)   c)  1 2 1  d)  2 2 3 
1 2   12 6   0 3 2   1 3 1 
   
3. If A is a square matrix of order 3 and | A | = 5, then find |5A|.
4. If A is a square matrix of order 3 and | A | = 10, then find A(adj A) .

 4 k 3
 
5. Find k if A   7 3 6  is singular matrix.
2 3 1
 
2 2 4 
 
6. Show that A   5 x  2 2  is non-singular for all values of real numbers x.
 1 x 
 3
 4 4 4 
 
7. Given the 3×3 matrix A   1 0 1  . Determine the values of the constant  , so that A   I is
 7 6 7 
 
singular.
8. The 3×3 matrices A and B are defined in terms of a scalar constant k by
 cos  sin  0
 
9. Given A    sin  cos  0  , show that A1  AT .
 0 1 
 0
 k 9 2  1 3 2 
   
10. A   1 k 0  and B   k 2 1 . Find the possible values of k so that the product AB is
 5 1 1  4 1 1 
   
singular.
3 1 1
11. It is given that A and B are 3×3 matrices that satisfy det( AB)  . If det( A )  , then find
4 20
det( B1 ).

Telegram channel https://t.me/johnson201485


25
3
 3 1 1   1 2 2 
1    
12. If A   15 6 5  and B   1 3 0  , then find ( AB)1 .
 5 2 2   0 2 1 
   
2 3   1 2 
13. If A    and B  
1 1 1
 . Then verify that ( AB)  B A .
 1 4   1 3 
 1 1 3 
 
14. The 3×3 matrix A is given in terms of the scalar constant k by A   2 1 k 
0 1 1
 
a) Find the inverse of A in terms of k.
b) State the condition that k must satisfy, so that the inverse matrix exists.
11.4. SYSTEMS OF EQUATIONS WITH TWO OR THREE
VARIABLES
Systems of linear equations have many applications. For instance, they are used to give mathematical models of
traffic flows and electrical networks. There are different ways of solving systems of two linear equations in two
variables, using substitution or elimination methods. Gauss‟ method is a systematic application of the elimination
method. The method of elimination is more systematic than the method of substitution. It can be expressed in
matrix form and matrix operations can be done by computers. The method of elimination is based on equivalent
systems of equations.
In this subunit, you will learn how to find solutions of systems of linear equations by using Elementary Row
Operations (Gauss Method).
Elementary Operations on Matrices
Elementary Row Operations
Let and be positive integers and A be an  matrix with entries in . An elementary row operations on A
is any one of the following operation.
Swapping : Interchanging two rows of a matrix:
Swapping of the i t h row and the j t h row of is denoted by Ri  R j
Rescaling : Multiplying a row of a matrix by a non-zero constant:
Multiplying the i t h row by a nonzero scalar k is denoted by Ri  kRi .
Pivoting : Adding a constant multiple of one row of the matrix onto another row :
Adding times the i t h row of A onto the j t h row is denoted by R j  R j  kRi
Elementary Column Operations
Let and be positive integers and A be an  matrix with entries in . An elementary column operations on
A is any one of the following operation.
Swapping : Interchanging two columns of a matrix.
Swapping of the i t h column and the j t h column of is denoted by C i  C j
Rescaling : Multiplying a column of a matrix by a non-zero constant.

Telegram channel https://t.me/johnson201485


25
4
Multiplying the i t h column by a nonzero scalar k is denoted by C i  kC i .
Pivoting : Adding a constant multiple of one column of the matrix onto another column. Adding times the i t h
column of A onto the j t h column is denoted by C j  C j  kC i
2 3 
Example 46. Let A    . Apply each of the following elementary row operations on A and obtain
 1 4 
the indicated new matrices.
a) AR2  R1 B R 3 R
b) A 
1
C
1 R  R  2R
c) A 
2 2
D1

Solution.
 2 3   1 4 
a) . A  R2  R1  B
 1 4   2 3 
 2 3  R1  3 R1  3(2) 3(3)   6 9 
b) A      C
 1 4   1 4   1 4 
 2 3   2 3   2 3 
c) A  R2  R2  2 R1   D
 1 4   1  2(2) 4  2(3)   3 2 
These three new matrices B, C and D are said to be equivalent matrices.
REMARK: In this book, we use only elementary row operations in solving system of linear equations.
DEFINITION 11.11
Two matrices are said to be ROW (OR COLUMN) EQUIVALENT, if and only if one is obtained from the other
by performing successive applications of a finite number of elementary row operations.

Thus, matrices A, B, C and D obtained in Example 46 are row equivalent matrices, denoted by
A  B  C  D.
2 4 6 
Example 47. Let A   4 5 6  . Apply appropriate elementary row operations to change A an upper
 2 7 12 
 
triangular matrix.
2 4 6  1 2 3 
Solution. A   4 5 6  R1  R1  
1 1
2  4 5 6  (Multiplying R1 by 2 .
 2 7 12   2 7 12 
   

R2  R2  4 R1
1 2 3 
 
  0 3 6  (Replacing R2 by R2  4R1 and R3 by R3  2R1
R3  R3  2 R1
0 3 6 
 
1 2 3 
 
R3  R3  R2  0 3 6  (Replacing R3 by R3 + R2 )
0 0 0 
 

Telegram channel https://t.me/johnson201485


25
5
1 2 3 
 
Then the matrix  0 3 6  so obtained is upper triangular matrix, which is equivalent to A.
0 0 0 
 
DEFINITION 11.12
A matrix is said to be in ROW ECHELON FORM (REF) if,
1. a zero row (if there is) comes at the bottom. (ENTIRE ZERO ROW)
2. the number of zeros preceding the first non-zero element (or LEADING ZEROS) in each non-zero
row except the first row is greater than the number of such zeros in the preceding row.
Example 48. Which of the following matrices are in row echelon form?
 1 2 4   0 0 1 2  0 0 1 2
     2 1 3 0   
A   0 1 2  B   1 7 4 9  C   D  0 0 0 0
0 0 0   3 2 5 1  0 10 1 2   0 0 0 1
     
Solution.
 A is in echelon form.
 B is not in row echelon form because the number of zeros preceding the first non-zero element in the first
row is greater than the number of such zeros in the second row.
 C is in row echelon form.
 D is not in row echelon form because the zero row is not at the bottom.
DEFINITION 11.13
A matrix is said to be in ROW REDUCED ECHELON FORM (RREF), if and only if,
1. it is in echelon form, and
2. the first non-zero element in each nonzero row is 1, and it is the only nonzero number in its column. The
first non-zero element of a row is called LEADING ENTRY OF THE ROW.
Example 49. Which of the following matrices are in row reduced echelon form?
1 2 0 1 0 0
  1 0 2 5  1 0 2 0    1 0 
A  0 1 1 
B 0 1 2 6
 C   1 0 0 0 D  0 0 1  
0 1
 0 0 0 1 E 
0 0 0 0 0 0 0

I4  
0

1 0 0 0 1 0 0 0
    0

0 1 0

   
0 0 0 1 0 0
Solution.
 The matrices B, C, I4 and E are in RREF, because all the four matrices satisfy the
requirements of the definition of a matrix in RREF.
 Matrix A is not in RREF, because the leading element of the second row of A, which is 1, is
not the only nonzero element in its column.
 Matrix D is not in RREF, because the leading zeros of the second row is two and the leading
zeros of the third row is one.
However, you can reduce both A and C in the above example to RREF by applying appropriate
elementary operations. Thus,
i) Matrix A is in REF. then applying elementary row operations : first R2  R2  2R1 and then
applying R1  R1  2R2 on the resulting matrix:

Telegram channel https://t.me/johnson201485


25
6
1 2 0 1  2(0) 2  2(1) 0  2(1)   1 0 2 
     
A   0 1 1  R1  R1  2 R2  0 1 1   0 1 1 
0 0 0  0 0   0 0 0 
   0
1 0 0
 
R1  R1  2 R2  0 1 1   B
0 0 0
 
Thus, B is a matrix in RREF obtained by applying elementary row operation twice on A.
Hence, Matrices A and B are row equivalent matrices.
Example 49 illustrates that you can use proper elementary operations to reduce a matrix into a matrix in Reduced
Row Echelon Form (RREF).
REMARK:
i) To reduce a given matrix into REF, apply a sequence of elementary operations starting from
the top and working downward.
ii) To reduce a given matrix into RREF, apply a sequence of elementary operations starting from
the bottom non-entire zero row and working upward.
Example 50. Reduce each of the following matrices into row reduced echelon form.
 1 3 2  1 2 1 2 
 2 0 1 3     
a) A  b) B   0 1 2 c) C   1 3 1 3 
 1 2 5 1   4 1 2  1 6 1 6 
   
Solution.
 2 0 1 3  1 0  12 12   1 0  12 1

a) A  R1  2 R1 
1
 R2  R2  R1 
2

 1 2 5 1   1 2 5 1   0 2 2
11 1
2 
 1 0  12 12 
R2   12 R2  11 
0 1  4  4 
11

 1 3 2   1 3 2   1 3 2 
     
b) B   0 1 2  R1   R1  0 1 2  R3  R3  4 R1 0 1 2 
 4 1 2   4 1 2   0 11 6 
     
 1 3 2   1 3 2 
   
R3  R3  11R2  0 1 2  R3   281 R3  0 1 2  (This is REF of B)
 0 0 28  0 0 1
   
 1 3 2  1 0 2 1 0 0
     
R2  R2  2 R3  0 1 0  R1  R1  3R2  0 1 0  R1  R1  2 R3  0 1 0   I3
0 0 1 0 0 1 0 0 1
     
Which is the RREF of B.

Telegram channel https://t.me/johnson201485


25
7
1 2 1 2  1 2 1 2  1 2 1 2 
  R2  R2  R1    
c) C  1 3 1 3   0 1 0 1  R3  R3  4 R2  0 1 0 1  (REF of C)
 1 6 1 6  R3  R3  R1 0 4 0 4 0 0 0 0
     
1 0 1 0 
 
R1  R1  2 R2  0 1 0 1  (RREF of C)
0 0 0 0
 

Exercise 11.4
1. Reduce each of the following matrices into row echelon form.
 5 0 1 1 1 3 6 
1 1 1 5     
a) A  b) B   1 1 0  c) C   5 3 2 4 
4 8 1 6 0 1 4  1 3 4 11 
   
2. Reduce each of the following matrices into row reduced echelon form.
 0 2   2 0 1 1 0 1 
     
a) A
2 1  b) B   2 3 0  c) C   1 2 2 
 1 1 4   3 1
 1 3 
     4 
 0 2  2 0 3 

System of Linear Equations


One of the main reasons of studying matrices is to solve systems of linear equations.
DEFINITION 11.14
An equation of the form a1 x1  a2 x2  a3 x3  .....  an xn  b ,
where a1 , a2 , a3 ,....., an  is called a linear equation in the variables x1 , x2 , x3 ,....., xn .
The variables x1 , x2 , x3 ,....., xn are also called unknowns.
If b = 0, then the linear equation is said to be HOMOGENEOUS.
Example 51.
i) The equation 3x  5 y  1 is a linear equation in two variables x and y .
ii) The equation x  5 y  117 z  0 is a homogeneous linear equation in three variables x , y and z .
iii) The equation 2 x1  5x2  x3   x4  7 is a linear equation in four variables x1 , x2 , x3 and x4 .
REMARK:
A linear equation does not involve product of variables, quotient of variables or roots of
variables and all the variables that occur in the equation are only to power one.
Example 52. The equations
i) 2x  3 yz  0 3x x  y  0,
ii) 1  2z  0 , iii)
y
iv) x2  2x  5 y  1
are not linear equations.

Telegram channel https://t.me/johnson201485


25
8
DEFINITION 11.15
A linear system with m equations in n unknowns (variables) x1 , x2 , x3 ,....., xn is a set of equations of the
a11 x1  a12 x2  a13 x3  .....  a1n xn  b1
a x  a x  a x  .....  a x  b
 21 1 22 2 23 3 2n n 2

 x 
 31 1 32 2
a a x  a x  .....  a x  b
form 
33 3 3n n 3
…………………………..> ( * )
.....................................................
.....................................................

am1 x1  am 2 x2  am3 x3  .....  amn xn  bm

where ai j ; i  1, 2,...., m and j  1, 2, ..., n are real numbers
If b1  b2  b3  ...  bm  0 , then the system is said to be HOMOGENEOUS
Example 53.
2 x  y  2
i)  is a system of two linear equations in two variables x and y.
5 x  3 y  0
3x1  x3  0
ii)  is a homogeneous system of two linear equations in three variables x1 , x2 and x3 .
3x1  x2  5 x3  0
3x  4 y  z  2w  0

iii)  x  y  3z  3 is a system of three linear equations in four variables x1 , x2 and x3 .
5 x  y  z  2w  1

REMARK:
The system of linear equations (*) can be written in matrix form as
AX  B …………> (**)
 a11 a12 a13 ... a1n 
   x1   b1 
 a21 a22 a23 ... a2 n     
 a a a ... a3n   x2   b2 
where A   ai j    31 32 33 , x  and b 
mn X  3 B 3 
 ...........................   ...   ... 
 ...........................   ...   ... 
     
 am1 am 2 am3 ... amn   xn   bm 
Matrix A is called THE COEFFICIENT MATRIX of the system.
 a11 a12 a13 ... a1n b1 
 
 a21 a22 a23 ... a2 n b2 
The matrix form  a a32 a33 ... a3n b3 
( A | B )   31 
 ........................... 
 ........................... 
 
 am1 am 2 am 3 ... amn bm 
obtained by adjoining B to A is called the AUGMENTED MATRIX of the system.

Telegram channel https://t.me/johnson201485


25
9
Example 54. Describe each of the following systems of equations in the form of (**) and in augmented matrix
form.
3x  4 y  z  2w  0
2 x  y  2 3x  x  0 
i)  ii)  1 3 iii)  x  y  3z  3
5 x  3 y  0 3x1  x2  5 x3  0 5 x  y  z  2w  1

Solution
2 x  y  2  2 1   x  2
i)  A  , X   , B 
5 x  3 y  0 5 3   y 0
 2 1 x   2 
Then the system can be rewritten in matrix form as AX  B that is      
 5 3  y   0 
 2 1 2 
The augmented matrix of the system is ( A | B)   
5 3 0 
 x2 
3x1  x3  0 3 0 1    0
ii)  A  , X   x2  and B   
3x1  x2  5 x3  0  3 1 5  x  0
 3
Then the system can be rewritten in matrix form as
 x1 
AX  B that is 3 0 1   0
   x2    
 3 1 5     0 
 x3 
The augmented matrix of the system is
3 0 1 0 
( A | B)   
 3 1 5 0 
x
3x  4 y  z  2w  0  3 4 1 2     0 
iii) The matrix form of the system of equation  x  y  3z  3 is   y   
  1 1 3 0   z    3 
5 x  y  z  2w  1  5 1 1 2     1 
    
 w
 3 4 1 2 0 
 
The augmented matrix of the system is ( A | B)   1 1 3 0 3 
 5 1 1 2 1 
 
The coefficient matrix and the augmented matrix of a given system of linear equations play important roles in
determining the solution set of the given system.
Consider a system AX  B of linear equations in variables. An order tuple of numbers
 c1 ... ... cn  is a SOLUTION to the system AX  B if AC  B and the set of all solutions of
T
c2 c3
AX  B is called the SOLUTION SET of the given system.

Telegram channel https://t.me/johnson201485


26
0
Solutions of Systems of Linear Equations Using Augmented Matrix
In this sub-section, you will learn how to solve systems of linear equations using the concept of elementary row
operations.
DEFINITION 11.16
Two systems of linear equations over R, the set of real numbers, with the same number of unknowns are said to be
EQUIVALENT if every solution of one system is a solution of the other system.
Example 54. Show that the systems of equations are equivalent
x  y  1  x  y  3
 and 
 y2  x  y 1
Solution
x  y  1
For  , substitute the second equation y  2 in the first gives
 y2
x  y  1  x  2  1  x  1
x  y  1
Thus, the solution set of the first system  is {(1, 2)}
 y2
 x  y  3
For the system  , adding the two equations downward, we obtain
 x  y 1
2y  4  y  2
Then substitute this in one of the two equations and solving for x :
 x  y  3  x  y  3  2  3  1
 x  y  3
Thus, the solution set of the second system  is {(1, 2)}
 x  y 1
from the above discussion, you observe that the two systems of equations have the same
x  y  1  x  y  3
solution set. Therefore, the two systems  and  are equivalent.
 y2  x  y 1
NOTE:
A system of linear equations that is equivalent to a given system AX  B can be obtained by
applying appropriate elementary row operations on the augmented matrix (A | B).
Consider a system of linear equation AX  B .
(a) If the matrices (A | B) and (C | D) are row equivalent augmented matrices of two systems,
then the systems AX  B and CX  D have the same solutions.
(b) The method of solving the system AX  B by reducing (A | B) into Row Echelon Form
(REF) is called GAUSSIAN ELIMINATION METHOD..
(c) The method of solving the system AX = B by reducing (A | B) into Reduced Row
Echelon Form (RREF) is called GAUSS-JORDAN REDUCTION METHOD.

Telegram channel https://t.me/johnson201485


26
1
Example 55. Solve the following system of linear equations using Gaussian Elimination Method.
x  2 y  z  2

2 y  3z  1 .
2 x  y  z  1

Solution
 1 2 1 2 
 
The augmented matrix of the system is  0 2 3 1
 2 1 1 1 
 
 1 2 1 2 
 
R3  R3  2 R1  0 2 3 1 (a zero is obtained at the position of a31 )
 0 3 1 5 
 
 1 2 1 2 
 
R2  12 R2  0 1  23  12  (A leading entry 1 is obtained in row 2.)
 0 3 1 5 

 1 2 1 2 
 
R3  R3  3R2  0 1  32  12  (a zero is obtained at the position of a32 )
0 0  72 72 

 1 2 1 2 
 
R3   R3  0 1  23  12  (A leading entry 1 is obtained in row 3 at a33 )
2
7
0 0 1 1 

 1 2 1 
 
The left side of the last matrix  0 1  32  is the REF of the coefficient matrix A.
0 0 1 

The system of equation corresponds to the last matrix is given by:
x  2 y  z  2

 y 2 z 2
3 1

 z  1

Now, starting from the bottom, we substitute back into the variables and find the solution.
From the last equation, you have z  1
Consider the second equation, y  32 z   12 , with z  1 from above and solving for y gives
 y  32 z  12  23 (1)  12   24  2
Use the first equation x  2 y  z  2 , and z  1 and y  2 from above to solve for x .
Then, solving for x gives you that  x  2 y  z  2  x  2  2 y  z  2  2(2)  (1)  1

Telegram channel https://t.me/johnson201485


26
2
Therefore, the solution set of the given equation is {(1,  2, 1)} . This method is Gauss method. (Gauss
elimination method)
 Gauss‟ method, as we discuss above, is just changing the augmented matrix of a linear system to an upper
triangular matrix and then solving the system using back substitution.
 Changing the system to upper triangular matrix is done by expressing the system into echelon form (This
is Gauss‟ method). On the other hand, if we change the system into reduced echelon form (a modification by
Jordan and hence Gauss – Jordan method), we do not need to use back substitution.
Example 56. Solve the following system of linear equations using Gauss-Jordan Method. Elimination
 x  2 y  1
Method.  .
 2 x  3 y  2
 1 2 1 
Solution The augmented matrix of the system is  
 2 3 2 
 1 2 1   1 2 1   1 2 1
  R1   R1   R2  R2  2 R1   (Left side is in REF)
 2 3 2   2 3 2  0 1 0 
 1 0 1
R1  R1  2 R2   (Left side is in RREF)
0 1 0 
The system of equation corresponds to the last matrix is given by:
 x  1

y  0
This is Jordan Method. It doesn‟t need back substitution to solve the system.
Therefore, the solution set of the system is
S.S = {5, 2)}
NOTE:
In solving system of linear equations using augmented matrix, there are three possibilities.
1. When the augmented matrix is changed into either echelon form or reduced-echelon form and if the number of
non-zero rows is less than the number of variables, then the system has an INFINITE SOLUTIONS.
2. When the augmented matrix is changed into either echelon form or reduced-echelon form and if the last
non-zero row has numerical coefficients which are all zero while having non-zero constant part, then the system
has NO SOLUTION.
Given a system of equations, there are four possibilities for solution sets of the systems.
CASE 1: The system may have EXACTLY ONE SOLUTION − such a system of linear
equations is called CONSISTENT.
CASE 2: The system may have NO SOLUTION−such a system of linear equations is called
INCONSISTENT.
CASE 3: The system may have an INFINITE NUMBER OF SOLUTIONS−such a system of
linear equations is called DEPENDENT.
 x  2 y  4z  0

Example 57. Solve the following system of linear equations   x  y  2 z  0 .
3x  3 y  6 z  0

Telegram channel https://t.me/johnson201485


26
3
Solution
 1 2 4 0 
 
The augmented matrix of the system is  1 1 2 0 
 3 3 6 0 
 
 1 2 4 0  1 2 4 0 
R2  R2  R1    
 0 1 2 0  R2   R2  0 1 2 0 
R3  R3  3R1    0 3 6 0 
 0 3 6 0   
1 2 4 0 
 
R3  R3  3R2  0 1 2 0  This is REF of A.
 0 0 0 0 
 
1 0 0 0 
 
R1  R1  2 R2  0 1 2 0  This is RREF of A.
 0 0 0 0
 
Here the number of non-zero rows is less than the number of variables, then the system has an infinite solutions.
The corresponding equation form of the last matrix gives the system
 x0

 y  2z  0
This has solution x = 0, y = −2z.
The solution set is given by S.S = {(0, −2z, z) | z is a real number}.
Notice that the solution set is infinite.
x  2 y  z  0
Example 58. Solve the system of linear equations 
2x  y  z  3
1 2 1 0 
Solution. The augmented matrix of the system is  
 2 1 1 3 
1 2 1 0   1 2 1 0 
R2  R2  2 R1   R2   3 R2  0 1  1 1 (this is REF of A)
1

 0 3 1 3   3 
the number of non-zero rows is less than the number of variables. Therefore, the given solution has infinite number
of solutions.
x  2 y  z  0
The given system is equivalent to the system of equations 
 y  3 z  1
1

The variable which corresponds to leading 1‟s are x and y. This variables are known as DEPENDENT
VARIABLES. The other variable Z known as FREE VARIABLES.
The solutions of such types of system are written as “the dependent variable interms of the free variables.
Now, from the second equation, solving for y in terms of z , you get y  13 z  1  y  13 z 1

From the first equation, x  2 y  z  2  13 z  1  z  13 z  2


Therefore, the solution set of the given system of equation is

Telegram channel https://t.me/johnson201485


26
4
S.S =  1
3 z  2 , 13 z  1, z  : z   which is an infinite set.
 x1  3x2  x3  1

Example 59. Solve the system of linear equations 2 x1  x2  4 x3  1 .
6 x  7 x  8 x  7
 1 2 3

 1 3 1 1 
Solution. The augmented matrix of the system is  2 1 4 1
 6 7 8 7 
 
 1 3 1 1   1 3 1 1   1 3 1 1 
  R2  R2  2 R1    1
 2 1 4 1   0 7 6 1  R2  7 R2
1
0 1  7 7 
6

 6 7 8 7  R3  R3  6 R1  0 11 2 13   0 11 2 13 
     
 1 3 1 1   1 3 1 1 
   1
R2  R2  0 1  76 71  R3  R3  11R2
1
7 0 1  7 7 
6

 0 11 2 13   0 0 80 80 
   7 7 

 1 3 1 1 
 
R3  R3  0 1  76 71  (the first 3 columns are REF of A)
7
80
0 0 1 1 

1 0 1 2  1 0 0 1
R2  R2  76 R3    
 0 1 0 1 R1  R1  R3  0 1 0 1
R1  R1  3R2   0 0 1 1
0 0 1 1  
This implies that the given system is equivalent to the system of equations
 x1  1

 x2  1
x  1
 3
Hence, the solution of the given system has a unique solution : x1  1, x2  1, x3  1 .
Therefore; the solution set is S.S = {(1, 1, 1)}
NOTE:
The system of n  n linear equations AX = B has unique solution if A  0

 x  2 y  3z  5

Example 60. For what values of k does the system  2 x  y  z  8
kx  y  2 z  14

a) has a unique solution? b) has no solution? c) Has an infinite solution?

Telegram channel https://t.me/johnson201485


26
5
 1 2 3 5 
Solution. The augmented matrix of the system is  2 1 1 8 
 k 1 2 14 
 
 1 2 3 5   1 2 3 5 
  R2  R2  2 R1  
 2 1 1 8   0 5 5 2 
 k 1 2 14  R3  R3  kR1  0 1  2k 2  3k 14  5k 
   
 1 2 3 5   1 2 3 5 
   
R2   R2 
1
5 0 1 1 5 2
 R3  (2k  1) R2  R3  0 1 1 5 
2

 0 1  2k 2  3k 14  5k   0 0 3  k 6821k 
   5 
 1 2 3 5 
 
R3  (2k  1) R2  R3  0 1 1 52 
 0 0 3  k 6821k 
 5 
a) From the last matrix, the system has unique solution if 3  k  0  k  3
An alternative way is to use the fact that the system has a unique solution is if A  0

1 2 3
1 1 2 1 2 1
2 1 1  1 2  (3)
1 2 k 2 k 1
k 1 2
 (2  1)  2(4  k )  3(2  k )  5k 15
| A |  5k 15  0  k  3
68  21k
b) The system has no solution if 3  k  0 and 5 0  k  3 and k  68
21

68  21k
c) The system has infinite solution if 3  k  0 and 5  0  k  3 and k  68
21

11.5. CRAMER‟S RULE


When the coefficient matrix A of a system of linear equations AX = B is such that A  0 , determinants can be
used to solve systems of linear equations with equal number of equations and unknowns. This method is known as
CRAMER’S RULE. The method is practicable, when the number of variables is either 2 or 3.
To solve the system of equation AX= B, where matrix A is of order more than 3, use of Cramer‟s rule is impractical
due to the large number of calculations that are required. In such cases Gauss‟ methods are preferable. Besides,
Cramer‟s Rule does not work when │A│ = 0.
ax  by  e a b
For instance, consider a 2 by 2 system of equation given by  with A  0.
cx  dy  f c d
a b  x e
Then the system is AX  B whereA  , X    and B    .
c d  y f
a b 1 1  d b 
Since A   0 , A 1 exists and A 1 adj A   .
c d | A| (ad  bc)  c a 
Now, AX  B  A 1( AX )  A 1B  ( A 1 A) X  A 1B
I 2 X  A 1B  X  A 1B
1  d b  e  1  de  bf 
X  A 1B       
(ad  bc)  c a  f  (ad  bc)  af  ce 
 de  bf  de  bf af  ce
  x   ad  bc   x , y
   ad  bc ad  bc
 y   af  ce 
 
 ad  bc 
e b a e
Here, we know that de  bf  and af  ce 
f d c f
e b a e
Then the solution of the system can be written as f d c f A
x , y  2
a b a b A
c d c d
where A1 is a 2  2 matrix obtained from A by replacing its 1 column by the constant column matrix B and A2 is
st

a 2  2 matrix obtained from A by replacing its 2nd column by the constant column matrix B.
This rule of solving system of equation is called CRAMER‟S RULE.
This rule is valid for any system of linear equations in so far as the coefficient matrix A is square matrix having
non-zero determinant. But, the repeated calculation of determinants may be tiresome.
CRAMER‟S RULE FOR 2  2 SYSTEM OF LINEAR EQUATIONS:
a x  b1 y  c a1 b1
Consider a system of linear equation  1 with A  0.
a2 x  b2 y  d a2 b2
a b1 c b1 a c
Let D 1 , Dx  and Dy  1 .
a2 b2 d b2 a2 d
Dx Dy
Then the system has unique solution given by x , y
D D
2 x  y  3
Example 60. Solve the system of equations  using Cramer‟s Rule.
3 x  5 y  1
 2 1 3
Solution. The coefficient matrix A is A    and the column matrix of constants is B   .
 3 5   1 
2 1
Since D | A |   2(5)  (1)(3)  7  0 , Cramer‟s Rule is applicable.
3 5
3 1 2 3
Hence, Dx   3(5)  (1)(3)  12 and Dy   2(1)  (3)(3)  7
1 5 3 1
Dx 12 Dy 7
This implies that x  , y  1
D 7 D 7
12
Therefore, the solution of the system is x 
, y 1
7
NOTE: Under similar conditions, the rule holds for 3  3 system of linear equations too.

Telegram channel https://t.me/johnson201485 26


7
CRAMER‟S RULE FOR 3  3 SYSTEM OF LINEAR EQUATIONS:
a11 x  a12 y  a13 z  b1

If the determinant of the coefficient matrix A of a 3  3 system a21 x  a22 y  a23 z  b2 is non-zero, then the
a x  a y  a z  b
 31 32 33 3

Dx Dy Dz
system has exactly one solution. The solution of the system is given by x  , y , z
D D D
a11 a12 a13 b1 a12 a13 a11 b1 a13 a11 a12 b1
where D  a21 a22 a23 , Dx  b2 a22 a23 , Dy  a21 b2 a23 and Dz  a21 a22 b2 .
a31 a32 a33 b3 a32 a33 a31 b3 a33 a31 a32 b3
 x  4 y  z  1

Example 61. Solve the system of equations 2 x  y  z  0 using Cramer‟s Rule.
x  y  z  1

 1 4 1 1
   
Solution. The coefficient matrix A is A   2 1 1  and the column matrix of constants is B   0  .
1 1 1 1 
   
1 4 1
D A  2 1 1  1(1  1)  4(2  1)  (1)(2  1)  5  0
1 1 1
This implies that the system has unique solution.
1 4 1 1 1 1 1 4 1
Dx  0 1 1  1, Dy  2 0 1  2 , Dz  2 1 0  4
1 1 1 1 1 1 1 1 1

Therefore, the solution of the system is x  Dx   1 , Dy 2 Dz 4


y  , z 
D 5 D 5 D 5
Thus, the solution set is S.S =  1 2 4 
  , , 
 5 5 5 

Exercise 11.5
1. Using the Gaussian Reduction Procedure, solve the system of linear equations
x y z x y z x y z
a) { x y z b) { x y z c) { x y z
x y z x y z x y z
2. Using the Gaussian Jordan Method, find the solution set of the equation
x y z x y z
 x1  x2  2
a)  b) { x y z c) {x y z
3x1  2 x2  0 x y z x z

3. For what value(s) of k the system { has unique solution?

4. For what value(s) of m will the system of equations { have


( )
a) a unique solution? b) no solution?c) an infinite number of solutions?

Telegram channel https://t.me/johnson201485 26


8
5. Solve the following systems by first finding and then using .

a) { b) ( )( ) ( )

6. Use Cramer‟s rule to solve each of the following.


x y z
x y
a) { b) { x y z c) {
x y
x y z
x y z
7. Determine the value of a and b for which the system { x y z has
x y z
a. A unique solution b. No solution c. Infinite many solution

8. If ( )4 5 ( ) and the determinant of the coefficient matrix is , then =

x ny
9. Determine the values of m and n for which the system { has
x y
a) A unique solution b) No solution c) Infinite many solution
REVIEW EXERCISES ON UNIT 11
INSTRUCTION: Workout each of the following questions correctly

1. If ( ) ( ) then determine the value of

a) b) ( )
2. Construct a 4 5 matrix ( ) where i

3. Given ( ) ( ) , then

a) find b) find matrix C satisfying .

4. For two matrices A and B, if . / . /, then find A and B.

5. If A = . /. /=. /, then find the value of a, b, c, and d.

6. Let . /. a) Find and b) If ( ) t n in ( )


x
7. If ( y z ) is symmetric matrix, then find the values of x, y and z.

8. For any square matrix A, show that is symmetric, while is skew-symmetric.

9.Describe a square matrix ( ) as a sum of symmetric and skew-symmetric matrices

10. Let ( ) and ( ) , then find the matrix B.

11. If ( ) and , where A is a 3  3 matrices, then find | A|.

Telegram channel https://t.me/johnson201485 26


9
12. If ( ), ( ), . / and . /, then find a)

b) c) d) ( )

13. Given a square matrix ( ). Then describe C as a sum of symmetric and skew-

symmetric matrices.
s sin
14. If . / and ( 𝑗 ) , then find the value of k.
sin s

15. Given a matrix ( ) ( ).

a) Find the minors of and b) Find the matrix of cofactors of E.


16. Reduced each of the following matrices given below into
i) row -echelon form. ii) row-reduced echelon form.

a. ( ) c. ( ) e. ( )

b. ( ) d.

( )

17. If A = ( ), then find a) adj A b) | | c) | |

d) the value of x such as 4x2  3| | + 2| |= 11 e) | |

18. If ( ) is square matrix with ( ), then what is the cofactor of .

19. Suppose that A and B are 3  3 matrices, I is identity matrix of order 3 such that AB = 2I. If
|B| = 6, then find |AT|.
20. If . / and . /, then find ( ) .

21. Find the inverse of the following matrices a) . / b) ( )

22. Let A = ( ) and f(x) = then find ( t( ))

23. If A is a square matrix of order 3 and det(A) = 5, then det(A.adj A) = __________


24. If A is a square matrix of order 3 and det(A) = 2, then | adj A | = __________

25. Show that | |

26. If M is a square matrix of order 3 and | M | = 5, then what is | 2M|?

Telegram channel https://t.me/johnson201485 27


0
27. If | | , then find the value of x.

28. If | | | |, then find the value of x.

29. Find a matrix D such that DA = B where ( ) and . /

30. Consider the system { . If the determinant of the coefficient matrix is 2, then

what is the solution set of the system?


x y
31. Write { in the form of and find the value of using
x y
a) has a unique solution b) has no solution c) has infinite many solution
32. Determine the values of k so that the following systems in unknowns x, y and z has:
i. A unique solution ii. More than one solution ii. No solution
x y z x y z
x y
){ b) {x y z c) { x y z
x y
x y z x y z
PRACTICE QUESTIONS ON 11
0 1 0 
 
1. If M   5 7 3 , then which one of the following is equal to the determinant of M ?
2 9 4 

A.  31 B.  8 C.  1 D. 14
2 3  1  1 
2. If A    and B    , then which of the following is equal to At  2 B ?
1 2   0  2 
4 5  5 3   4 1  3 4 
A.   B.   C.   D.  
1  2  1 2   3  2  1 0 
 x 3  2 3 
3. If   is the inverse of   ,then the values of x and y respectively are:
1 y  1 2 
1 3
A. 1, 3 B. 2, 2 C. 4, 1 D. ,
2 2
x  1  2 1  y y
4. What are the values of x and y so that      ?
1 0  4 1  2 1
A. x  3, y  2 B. x  0, y  1 C. x  4, y  5 D. x  2, y  3
 2x  y  4
5. What would be the value of k so that the system  has a unique solution?
 4 x  ky  1
A. k 0 B. k  4 C. k  2 D. k  1

Telegram channel https://t.me/johnson201485 27


1
1  y
 2 0 x  
6. Let A    and B   0 3  such that A  2BT  0 . Then which of the following is the
 2 y x  y  4 1  x 2 
 
value of y ?
13
A. 0 B.  C.  8 D. any real number
2
 x  y  2z  1

7. What is the solution set of the following system of equations  x  2 y  z  2 ?
 2 x  2 y  4 z  2

A. 0, 1, 0 C.  3k , k  1, k  | k   , 
B.  ,  D. 3k , k  1, k  | k   , 
 
2 0 0
  1
8. Let A and B be 3 3 matrices such that A  1 5 0  and B  . Which one of the following
  10
 0 1
1 
 2
T
is equal to 2 AB ? A. 1 B. 4 C. 100 D. 400

0 4 1
9. If  2 1 5  and B is a 3 3 matrix such that det2 B   40 , then det AB  is equal to:
 0 3 2
 
A. 200 B.  200 C. 50 D.  50
 x  y  z 1
10. What should be the value of k so that the system of equation  x  5 y  4 z  1 has one solution?
 2x  2 y  z  k

A. 0 B. 1 C.  4 D. 4
11. Suppose that A and B are 3 3 matrices, I is the identity matrix of order 3 such that AB  2 I . If
det B  B  6 . What is detAT  ?
1 4
A. B. C. 12 D. 48
3 3
1 2
12. Suppose A    . If X is a 2  2 matrix such that AX  AT  2 A , then what is the value of X ?
2 3
3 1  3 3 3 6 3 0
  B.   C.   D.  
3 
A.
1 3 3  6 9  0 3 
x  y  z  1
13. Consider the system  x  2 y  4 z  0 . If the determinant of the coefficient matrix is 2 , then what is the

5 x  y  z  0

solution of the system of equations?
 19  11   19  11   3  19 11   3 19  9 
A.  3 , ,  B.  3, ,  C.  , ,  D.  , , 
 2 2   2 2   2 2 2 2 2 

Telegram channel https://t.me/johnson201485 27


2
14. A box contains 5 white, 6 red and 4 black balls of identical size. If 3 balls are randomly taken out of the
box one after the other, what is the probability that the first ball is white and both the second and third balls are
red?
2 3 4 5
A. B. C. D.
15 15 75 91
 2 0  1
 
If A  1 2 0  and 2 A  B   AT A , then which one of the following is equal to B ?
T
15.
 0 0  1
 
1 0  2  0 0 0 8 0  4   1 2 0
       
A.  2 0 0 B.  0 0 0 C.  4 8 0 D.  0 0 0
0 0 4  0 0 0  0 0  4  2 0 4 
     
0 1 2
 
16. If M   3  1 0  and AT M  2 I , where A is a 3 3 matrix and I is the identity matrix of order 3,
5 2 4 

then what is det A ?
4 1
A. 0.2 B. C. 0.8 D.
17 17
Suppose AX  b , where A is a 3 3 matrix, b  b1 , b2 , b3  and X  x, y, z  . Which one of the
T T
17.
following is necessarily true about this system of linear equations?
A. The system has a solution only when det A  0.
B. The Cramer‟s rule is suitable to solve the system if two rows of A are identical.
C. If det A  0 and the second column of A is a multiple of b, then x  0 .
If b = 0, then X  0,0,0 is the only solution of the system.
T
D.
ax  by  2

18. Consider the following system of equations:  x  3 y  2 z  0 . If the determinant of the coefficient
2 x  y  z  0

matrix is 2 , then what is the solution set of the system?
 1 
A. 1, 3,  5 B.  ,
1
, 0  C.  2,  6, 10 D. 
 a b 
3  2 8 
 
19.  
If A   0 6 7  , then det AT A is equal to_____________.
0 4 5 
 
A. 12 B. 36 C. 30 D. 15
0 0
x 1 1 1
20. If A  1  1  
1  and A 1   3 0

0  , then what are the values of
x and y ?
0 y  1  2 0  1 
 
2 1 1 2
A. x  3, y  2 B. x  , y C. x  3, y  2 D. x  , y
3 3 3 3

Telegram channel https://t.me/johnson201485 27


3
x  y  z  1

21. What is the solution set of the system  x  2 y  3z  1 ?
2 x  3 y  4 z  2

A. 0, 2, 1 B. 1  k , 2k , k  | k   C. 2k  1,  k , k  | k   D. 
2 7 1 1 
 , then  AB  is equal to:
1
22. If A    and B 1  
1 3 0 2
4  3  2 5   3 11 4 0
  B.   C.   D.  
 5  2 
A.
4  2  4  1 3 0
0   6 x
     
23. Let A   2 2 1  , b   0  , and X   y  . If det( A)  3 , then what is the solution set of the system
3 2  0 z 
 1    

6,  2,  8  B.  0, 1 5  
 
T

AX  b ? A. C.  3, 1, 4 D. 
T T
,  
 a b  

 x  3y  2z  6

24. The solution of the system of linear equation of  2 x  4 y  3z  8 is:
 3x  6 y  8 z  5

A. x  1, y  3, z  2 C. x  1, y  3, z  2
B. x  1, y  3, z  2 D. x  1, y  3, z  2
25. For any n  n square matrix A , which one of the following is true?
A.  
det A   det AT , where AT is the transpose of A .
B. If k is a scalar, then detkA  k n det A .
C. If B is a matrix obtained from A by interchanging of two rows of A , then detB   det A .
D. If A is invertible, then det A  det A1 .  
 x  4 y  z  1

26. The solution of  2 x  y  z  0 using Cramer‟s rule is:
 x  y  z 1

1 2 4
A. x  1, y  2, z  5 C. x   , y  ,z
5 5 5
1 2 2
B. x , y ,z D. x  5, y  1, z  2
2 3 5
1 3 3 1
27. For what value of x is  x 1 A. 1 B. C. D.  1
x x2 2 2

Telegram channel https://t.me/johnson201485 27


4
3 2 1 
 
28. Let A   2 4  5  , then which of the following is equal to A  AT ?
3  5 6 
 
4 6 4 6 4 4  6 4 2   4 8  10 
       
A.  4  10 8  B.  4 8  10  C.  4 8  10  D.  6 4 2 
  10 4 12   4  10 12   6  10 12   6  10 12 
       
p rq 3 p 3r 2.4q
29. Suppose t y z  3 . Which of the following is equal to t y 0.8 z ?
a b c a b 0.8 c
A. 2.4 B. 9 C. 1.92 D. 7.2
30. Given two matrices A  aij   mn
 
and B  bij mn
, then which one of the following is NOT true about the
given matrices?
A. A  T T
A C. rB   rB , if r is any scalar.
T T

B.  ABT  AT B T , provided that AB is defined D.  A  B   AT  B T


T

 0  3  4
 
31. Let  k 0 8  be skew-symmetric matrix. Then what is the value of k ?
4  8 0 
 
3
A. 3 B. C. 3 D. 4
2
2 x  y  3z  4

32. The augmented matrix associated to the system of equations  x  z  1 is
4 x  y  z  0

 3 2 1 1  3 1 2 4  1 2 3 4  2 1 3 4
       
A.  1 1 1 3  B.  1 0 1 1  C.  1 1 1 1  D.  1 0 1 1 
 1 4 1 4   4 4 3 0   1 4 1 3   1 4 0 3 
   

33. If | | the solution set of ( )4 5 ( ) is

A. *( )+ B. 2. /3 C. 2. /3 D. 2. /3
x y z
34. If the system {x y z has infinitely many solutions, then the values of and are
x y z
A. ∈ C.
B. D. ∈

Telegram channel https://t.me/johnson201485 27


5
UNIT TWELVE:VECTORS AND TRANSFORMATION OF THE
PLANE
Unit Outcomes:
After completing this unit, you should be able to:
 know basic concepts and procedures about vectors and operation on vectors.
 know specific facts about vectors.
 apply principles and theorem about vectors in solving problems involving vectors.
 know basic concepts about transforming the plane.
 apply methods and procedures in transforming plane figures.

Main Contents:
12.1. REVISIONON VECTORS AND SCALARS
12.2. REPRESENTATION OF VECTORS
12.3. COMPONENTS OF VECTORS
12.4. PRODUCT OF VECTORS
12.5. APPLICATION OF VECTORS
12.6. TRANSFORMATION OF THE PLANE

Key terms

Summary

Review Exercises

Telegram channel https://t.me/johnson201485 27


6
INTRODUCTION
This unit deals with vectors and transformation of the plane. This chapter follows the concepts which you
discussed vectors in grade 9. Here, more discussions on vectors that include representation of vectors, scalar (dot)
product of vectors, applications of vectors and transformation of the plane will be treated.
12.1. REVISION ON VECTORS AND SCALARS
Most of the physical quantities can be divided into two categories as given below.
a. Physical quantities that possess magnitude only, known as SCALAR QUANTITIES, which are completely
determined once the magnitude of the quantity.
b. Physical quantities that possess both magnitude and direction known as VECTOR QUANTITIES, which are
not completely determined until both a magnitude and a direction are specified.
For example, wind movement is usually described by giving the speed and the direction, say 20 km/hr Northeast.
The wind speed and wind direction together form a vector quantity - the wind velocity.
VECTORS AND SCALARS
DEFINITION 12.1
Physical quantities expressed by a certain number associated with a suitable unit without any mention of direction
in space is known as SCALAR. The number describing the quantity of a particular scalar is known as its
magnitude.
Example 1. Time (15 sec), mass (20 kg), length (5km), area(7m2), volume(10m3), temperature (37°C) are
scalar quantities.
DEFINITION 12.2
A VECTOR is a physical quantity which can be expressed completely by stating both its magnitude with
particular unit and direction.
Example 2. A moving car is usually described by giving the speed and the direction, say 60 km/h northeast.
The car speed and car direction together form a vector quantity, velocity of the car.
Exercise 12.1
1. Classify the following physical quantities as scalar and vector quantities
a. Time e. Distance i. Force m. Acceleration q. Density
b. Mass f. Displacement j. Volume n. Area r. momentum.
c. Work g. Length k. Temperature o. Time
d. Torque h. Velocity l. Speed p. Weight
2. Based on your knowledge, classify the measures of the following situations as scalar or vector.
a. The width of your classroom.
b. A football is kicked on flat ground at a velocity of 15 m/s at an angle of 250.
c. The flow of a river.
d. The number of students in your class room.
e. A man walks 4 km north
f. The direction of your home from your school.
g. The wind is blowing towards east
h. When an open door is closed.
i. When you move nowhere in any direction

Telegram channel https://t.me/johnson201485 27


7
12.2. REPRESENTATION OF VECTORS
DEFINITION 12.3
1. If v is a vector in the plane whose initial point is the origin, O (0, 0) and whose terminal point is Q( , )
then the coordinate form of a vector v = ( ) or the column form of a vector v = . /. The numbers
and are called COMPONENTS or COORDINATES of a vector v.
Moreover i) the direction of the vector v can be given by the slope , and
ii) the magnitude (length) of v is given by | | √ .
2. If v is a vector in the plane whose initial point P( 1 , 1) and whose terminal point Q( 2 , 2), then the
coordinate form of a vector v = ( ) or the column form of a vector v = . /. The
numbers and are called COMPONENTS or COORDINATES of a vector v. In such case, vector
v is known as the POSITION VECTOR of vector .
Moreover i) the direction of the vector v can be given by the slope , and
ii) the magnitude (length) of v is given by | | √( ) ( ) .
Example 3. Find the coordinate and column forms, and the length of the
vector v that has initial point (0, 0) and terminal point (3, 6).
Solution: Let O = (0, 0) and Q = (3, 6) as shown in the coordinate plane.
Then
i) the coordinate form of vector v is v = (3, 6).
ii) the column form of vector v is v = . /
The length of v is v = √ √ √ units.
NOTE : If is a non-zero vector, then the NEGATIVE or OPPOSITE of , denoted by – , is a vector having
the same magnitude (length) as , opposite in direction with .

Example 3. Find the


coordinate and column forms of the vector v that has initial point (1, 2) and
terminal point (6, 7).
Solution: Let ( ) ( ) and ( ) ( ) as shown
in the coordinate plane given below.
The coordinate form of v is ( )
( ) ( )
The column forms of the vector v is v = . / . /

NOTE: If both the initial and terminal points of a vector v lie at the origin, then v is said to be the zero vector
and is given by v = (0, 0) or =. /

Telegram channel https://t.me/johnson201485


27
8
12.2.1 Addition and Subtraction of Vectors
A. Addition of Vectors
ACTIVITY 12.1
1. Consider vector as a displacement that Kebede walk 5m due east and then as a
displacement that Kebede walk 6m due north. Find the combined effect of
these two displacements as a single displacement.
2. Consider the following displacement vectors. Discuss how to determine
the combined effect of the vectors as a single vector.
From Activity 15.1, you can add two vectors geometrically by joining tail to
head rule called the resultant vector r = u + v which describe addition of
vectors which can be found by placing v such that its tail is at the same point as the head of u. The
resultant vector r or addition of vectors u + v then has its tail at the tail of u and its head at the head of v
as shown in the figure below which indicates triangular law of addition of vectors.
DEFINITION 12.4 (ADDITION OF VECTORS (TAIL − TO – HEAD RULE)
If u and v are two vectors, the sum u + v is the vector determined by translating vector v until its tail coincides
with the head of u. Then, the directed line segment from the tail of u to the head of v is the vector u + v.

NOTE : One can easily see that


u, v and u + v are represented by
the sides of a triangle, which is called the TRIANGLE LAW OF VECTOR ADDITION.
 The two useful properties of vector addition are given below.
THEOREM 12.1 (COMMUTATIVE PROPERTY OF VECTOR
ADDITION) If u and v are any two vectors, then u + v = v + u.

PROOF: Take any point O and draw the vectors = u and = v such
that the terminal point of the vector u is the initial point of the vector v, as
show in figure. Then, by definition of vector addition you have:
. . . . . . . . . . . . . . . . . . . . . . . . > (1)
Now, completing the parallelogram OABC whose adjacent sides are and , you
infer that , and .
Using the triangle law of vector addition, you obtain

v+u= . . . . . . . . . . . . . . . . . . . . . . . . > (2)


From (1) and (2), we have:
Hence, vector addition is commutative. This is also called the

Telegram channel https://t.me/johnson201485


27
9
PARALLELOGRAM LAW OF VECTORS ADDITION.
THEOREM 12.2 (ASSOCIATIVE PROPERTY OF VECTOR ADDITION)
If u, v and w are any three vectors, then (u + v) + w = u + (v + w).
PROOF: Exercise
B. Subtraction of VectorsSubtraction of vectors can be interpreted in terms
of vector addition. ( )
Geometrically, can be described as follow. This figure shows subtraction of
vector u – v as addition of and ( ) using the parallelogram law.
Moreover, you can also subtract these vectors using the triangular law of vector
addition as follows.
Draw the position u and v so that their initial points coincide. Here, the vector from
the terminal point of v to the terminal point of u is then the vector u – v as shown in
the figure below.
NOTE :
1. If v is any non-zero vector and –v is the negative of v, then v + (–v) = 0.
2. The subtraction of vectors is not commutative.
3. The subtraction of vectors is not associative.
12.2.2 Multiplication of Vectors by Scalars
Geometrically, the product of a vector u and a scalar k is the vector that is k times as long as u.
Suppose you have a vector u, then if this vector is multiplied by a scalar quantity k = −1, 2, 3,
−3, and so on, you will get the value as shown in the following figure.

Hence, from the above given set of vectors, you have observed that the direction of vector u remains the same
when the value of the scalar is positive and the direction becomes exactly opposite when the value of the scalar is
negative. In both cases the magnitude keeps changing depending upon the values of the scalar multiple.
DEFINITION 12.5: If v is a non-zero vector and k is a non-zero number (scalar), then the product kv is
defined to be the vector whose length is | | times the length of v and whose direction is
i) the same as that of v if k > 0 and
ii) opposite to that of v if k < 0.
If k = 0 or v = 0, then kv = 0.
A vector of the form kv is called a SCALAR MULTIPLE of v.
Example 4. Let v be any vector. Then 3v is a vector in the same direction as v and with length 3 times the
length of v.
Example 5. Let v be any vector. Then v is a vector in the opposite direction as v and with length half as
long as the length of v.

Telegram channel https://t.me/johnson201485


28
0
THEOREM 12.3 (PROPERTY OF SCALAR MULTIPLICATION OF VECTORS)
Let u and v be vectors and let k1 and k2 be scalars (real numbers). Then the following properties always holds true.
1. | | | || |
2. ( ) ( ) ……………………….> Associative Property
) ( ) ( )
3. ……………………….> Distributive Property
ii) ( )
4. ……………………….> Identity Property
5. ……………………….> Multiplicative Property of 0.
Example 6. Draw diagrams to illustrate the following vector equations.
a. b.
Solution.

Example 7. Sketch the resultant vector of the following force vectors using the tail-to-head method using a
Cartesian plane.
• in the positive y-direction
• F2 = 1,5 N in the positive x-direction
• in the negative y-direction
• in the negative x-direction
Solution.

Exercise 12.2
1. Observe the following vectors. Complete the chart for X-
component and Y- component for all the vectors.

Vector X-comp. Y-comp

2. A student walks a distance of 3km due east, then another 4km due south. Find the displacement relative to
his starting point.
3. What are the x and y components of the plane‟s velocity?

Telegram channel https://t.me/johnson201485


28
1
4. A car travels due east at 60km/hr for 15 minutes, then turns
and travels at 100km/hr along a freeway heading due north for 15
minutes. Find the displacement from its starting point.
5. In the triangle ABC given below, represents u and
represents v. If D is the midpoint of , then express , , and
in terms of u and v.
6. A car travels due east at 60km/hr for 15 minutes, then turns and travels at
100km/hr along a freeway heading due north for 15 minutes. Find the
displacement from its starting point.
7. If ABCDEF is a regular polygon in which represents a vector v and
represents a vector w, express each of the following vectors interms of v and
w. i) ii) iii) iv)
8. Find the resultant in the x-direction, , and y-direction, for the following forces:
• = 1.5 N in the positive x-direction • = 2 N in the negative x-direction
• = 1.5 N in the positive x-direction
9. Find the resultant in the x-direction, Rx, and y-direction, Ry for the following forces:
• = 3 N in the positive x-direction • = 2 N in the negative x-direction
• = 1 N in the positive x-direction • = 3 N in the positive y-direction

12.3. COMPONENTS OF VECTORS


In the discussion of vector addition, we saw that a number of vectors acting together can be combined to give a
single vector (the resultant). In much the same way a single vector can be broken down into a number of vectors
which when added give that original vector. These vectors which sum to the original are called components of the
original vector. The process of breaking a vector into its components is called resolving into components.
In practice, it is most useful to resolve a vector into components which are at right angles to one another, usually
horizontal and vertical.
Any vector can be resolved into a horizontal and a vertical component. If is a vector, then the horizontal
component of is and the vertical component is .
When resolving a vector into components that are parallel to the x- and y-axes we are
always dealing with a right-angled triangle. This means that we can use trigonometric
identities to determine the magnitudes of the components (we know the directions
because they are aligned with the axes).
Thus, from the triangle in the diagram, we know that
s and sin
This implies that and
Note that the angle is measured counter-clockwise from the positive x-axis.
Example 8. A force of 250 N acts at an angle of 300 to the positive x-axis. Resolve this force into components
parallel to the x- and y-axes.
Solution. The rough sketch and the resolution of the vector are as shown below.
As shown in the figure, using trigonometric identity, the magnitude of the components and , respectively, are
given by
| | s ( ) s √ (in the positive x-direction) and
| | sin ( ) sin (in the positive y-direction)

Telegram channel https://t.me/johnson201485


28
2
NOTE:
1. Evidently, a given vector has an infinite number of pairs of possible component vectors. However, if directions
of the component vectors are specified, the problem of resolving the vector into component vectors has a unique
solution.
2. Let u and v be two non-zero vectors. In the expression w = k1u + k2v,
a) the vectors k1u and k2v are said to be the COMPONENTS of w relative to u and v.
b) the scalars k1 and k2 are called the COORDINATES of the vector w relative to u and v.
Vector addition using components
Components can also be used to find the resultant of vectors. This technique can be applied to both graphical and
algebraic methods of finding the resultant. The method is straightforward:
1. Make a rough sketch of the problem.
2. Find the horizontal and vertical components of each vector.
3. Find the sum of all horizontal components, .
4. Find the sum of all the vertical components, .
5. Then use them to find the resultant, .
Consider the two vectors, and together with their resultant, , as shown below.
Each vector can be broken down into one component in the x-direction (horizontal) and one in the y-direction
(vertical). These components are two vectors which when added give you the original vector as the resultant.
Now, we can see that

, and
where ,
Note that the order that we add the vectors does not matter.
Example 9. Suppose two forces = 15 N acts at an angle of 300 to the horizontal and = 27 N acts at an
0
angle of 53 to the horizontal. Find the resultant force, .
Solution. Resolving each vectors into components gives:
| | s ( s ) ( )
| | sin (sin ) ( ) and
| | s ( s ) ( )
| | sin (sin ) ( )
Then the components of the resultant are

Telegram channel https://t.me/johnson201485


28
3
That is is 29.24 N to the right.
That is is 28.06 N upward.
Using the Theorem of Pythagoras, the magnitude R of the resultant is
( ) ( )
√ √
If is the angle between the resultant and the horizontal, then
t n Then t n ( )
Therefore, is 40.53 N at an angle of to the positive x-axis.
Example 10. Determine, by resolving into components, the resultant of the following four forces acting at a
point:
• to the positive x-axis. • to the positive x-axis.
• to the positive x-axis. • to the positive x-axis.
Solution. Resolving each vectors into components gives the
following:
| | s ( s )
| | sin (sin )
| | s ( s )
| | sin (sin )
| | s ( s )
| | sin (sin )
| | s ( s )
| | sin (sin )
Then the components of the resultant are
That is is 1.86 N to the right.
That is is 3.65 N upward.
Using the Theorem of Pythagoras, the magnitude R of the resultant is
( ) ( )
√ √
If is the angle between the resultant and the horizontal, then t n
Then t n ( )
Therefore, is 4.10 N at an angle of to the positive x-axis.
Example 11. A car weighting 8000N is on a straight road that
has a slope of 10o as shown in figure. Find the force that keeps the
car from rolling down.
Solution. The force vector acts in the downward direction.
 | |
Observe that o and (∠ )

Telegram channel https://t.me/johnson201485


28
4
 The force that keeps the car at D from rolling down is in the opposite direction of .
| | | | | |
sin(∠ )  sin
| | | |

 | | (sin )
REPRESENTATION OF VECTORS: As seen in definition 8.2, If v is a vector in the plane whose
initial point is the origin, O (0, 0) and whose terminal point is Q( , ) then the coordinate form of a
vector v = ( ) or the column form of a vector v = . /.
For instance, the coordinate form of a vector whose initial point is the origin and whose terminal point is
Q(3, 7) is v = (3, 7) or in column form v = . /.
If v is a vector in the plane whose initial point P( 1 , 1) and whose terminal point Q( 2 , 2), then the
coordinate form of a vector v = ( ) or the column form of a vector v=
. /.
For instance, the coordinate form of a vector whose initial point A( 2, 5) and whose terminal point is
B(3, 1) is ( ( ) ) ( ), or in column form v = . /.

the magnitude (length) of v is given by | | √ √ ( ) √


DEFINITION 12.6
Any vector whose magnitude is one is called a UNIT VECTOR. That is a vector is unit vector if and
only if | |

For instance, a vector . / is unit vector, since

| | √.√ / . / √
NOTE: The unit vectors in the direction of positive x-axis and positive y-axis, respectively, are (1, 0)
and (0, 1). These unit vectors are known as STANDARD UNIT VECTORS in the xy-plane.
Notation: the standard unit vectors in the coordinate plane are denoted by
̂ ( ) and ̂ ( ).
Thus, | ̂ | and | ̂ |
Now consider a vector ( ) in a coordinate plane.
Then by vector addition and scalar multiplication, can be expressed as
( ) ( ) ( ) ( )
( ) ( ) ̂ ̂
Thus, any vector ( ) in the plane can be expressed uniquely in the
form
( ) ̂ ̂
in this case we say that is expressed as a linear combination of ̂ and ̂.
Example 11. Express the following vectors in terms of the unit vectors ̂
and ̂ and findtheir norm. a. (7, –8) b. (–1, 5) c. (0, 3) d. ( , 2)

Telegram channel https://t.me/johnson201485


28
5
Solution
a. (7, –8) = ̂ ̂ and its nom is | ̂ ̂| √ ( ) √ √
b. (–1, 5) = ̂ ̂ and its nom is | ̂ ̂| √( ) ( ) √ √
c. (0, 3) = ̂ and its nom is | |̂ √ √

d. ( – )= ̂ ̂ and its nom is | ̂ ̂| √. / ( ) √ √


NOTE: If u is any non -zero vector, then
i. the unit vector in the direction of u is given as | |.
ii. the unit vector in the direction opposite to u is given as | |
.
This is to mean that if ̂ ̂ , then | | | ̂ ̂| √ .
̂ ̂
 ̂ ̂ is a unit vector in the direction of u, since
√ √ √

̂ ̂
| | | ̂ ̂| √( ) ( ) √
√ √ √ √ √

Example 12. For each of the following vectors, find the unit vector in the direction of the vectors.
a. u = (3, –2) b. u = ̂ ̂ c. u = ̂ d. u = ( , 0)
Solution
a. u = (3, –2) . Its nom is |( – )| √ ( ) √ √
( – )
Then the unit vector in the direction of u is | |
̂ ̂
√ √ √
b. u= ̂ ̂. Its nom is | ̂ ̂| √( ) √ √
̂ ̂
Then the unit vector in the direction of u is | |
̂ ̂
√ √ √

c. u= ̂. Its nom is | ̂| √ . / √

Then the unit vector in the direction of u is | |


. ̂/ . ̂/ ̂
. /

d. u = ( , 0). Its nom is |( )| √ √


( )
Then the unit vector in the direction of u is | |
̂ ̂ ̂
Example 13. For each of the following vectors, find the unit vector in the direction opposite to the given
vectors. a. u = (–1, 3) b. u = ̂ √ ̂
Solution
a. u = (–1, 3). Its nom is |( )| √( ) √ √
Then the unit vector in the direction opposite to u is given as

| |
(– ) . / ̂ ̂
√ √ √ √ √

b. u= ̂ √ ̂. Its nom is | ̂ √ ̂| √ √ √

Telegram channel https://t.me/johnson201485


28
6
̂ √ ̂ √
Then the unit vector in the direction opposite to u is given as | |
̂ ̂
Operation on Vectors in Component Forms : As you have discussed in Grade 9, operations of vector
addition and multiplication by a scalar are easy to work out in terms of coordinate forms of vectors. For
the moment, we shall restrict the discussion to vectors in the plane.
DEFINITION 12.7: If ( ) ( ), scalar k, then u + v = ( ) and
( )
Example 14. Given vectors u = (–1, 5) and v = (3, 2) find
a) 3v + u b) | |
Solution
a) 3v + u = (–1, 5) + (3, 2) = (–1 + 3, 5 + 2) = (2, 7)
b) ( ) ( ) . / . /

then | | |. /| √. / . /
Example 15. Given three vector u = (3, −2), v = (−2, −1) and w = (5, 1). Then find
a) u + 4v − w b) |3w − 2v  u|
Solution
a) u + 4v − w = (3, –2) + 4(–2, –1) – (5, 1) = (3 + 4(–2) – 5, –2 + 4(–1) – 1 = (–10, –9)
b) ( ) ( ) ( )
( ) ( )
Then | | |( )| √ √
DEFINITION 12.8: Two vectors u and v are parallel if u = kv for some non-zero k. furthermore, if k > 0
they are actually in the same direction, if k < 0, they are in opposite direction.
Example 16. If ̂ ̂ and ̂ ̂, which of the vectors below are parallel to u or v?
a) ̂ ̂ b) ̂ ̂ c) ̂ ̂ d) ̂ ̂
Solution. Determine whether or not they are parallel, you can check if their respective components can
be expressed as scalar multiples of each other or not.
Since u and v are not scalar multiple one to the other, they are not parallel.
a) ̂ ̂ ( ̂ ̂) ( ) . Therefore, and are parallel, but in opposite direction.
b) ̂ ̂ (̂ )̂ . Therefore, and are parallel, and in the same direction.
c) Suppose ̂ ̂ ̂ ̂
 and  and
This is contradiction. Hence, and are not parallel.
In a similar manner, Suppose  ̂ ̂ ̂ ̂
 and This is contradiction.
Hence, and are not parallel.
d) ̂ ̂ ( ̂ ̂) . Therefore, and are parallel, and in the same direction.

Telegram channel https://t.me/johnson201485


28
7
NOTE:
1. Two vectors ̂ ̂ and ̂ ̂ are EQUAL if and only if their components
are equal. i.e., and . or
2. Two or more vectors are said to be equal if they have the same magnitude and the same
direction.
Exercise 12.2
1. Determine whether the following two vectors are parallel or not. Then determine the
magnitude of the two vectors.
a. ̂ ̂ and ̂ ̂ b. ̂ ̂ and ̂ ̂
c. . / and . /
2. Sketch the resultant of the following force vectors using the tail-to-head method by first
determining the resultant in the x- and y-directions:
• = 2 N in the positive y-direction • = 1.3 N in the negative y-direction
• = 1.5 N in the negative y-direction • = 1 N in the negative x-direction
3. Resolve each of the following vectors into components:
a) = 105 N at to the positive x-axis. c) = 11.3 N at to the positive x-axis.
b) = 27 N at to the positive x-axis. d) = 149 N at to the positive x-axis.
4. Two vectors act on the same point. What should the angle between them be so that a
maximum resultant is obtained?

12.4. PRODUCTS OF VECTORS


There are two types of products of vectors : Dot (Scalar) Product and Cross (Vector) Product.
12.4.1 SCALAR (INNER OR DOT) PRODUCT OF VECTORS
In previous section, you have seen three vector operations : vector addition, subtraction and
multiplication by a scalar, each of which provides another vector. In this section, you will study
another vector operation like the inner or dot product. This product provides a scalar, rather than a
vector.
DEFINITION 12.9: If u and v are vectors and  is the angle between u and v, then the DOT (SCALAR
or INNER PRODUCT) of u and v, denoted by u.v, is defined by: | || | s , where

.
Example 17. For two vectors u and v, and  is the angle between u and v, find u.v,
a. if | | ,| | and c. if | | and | |
b. if | | ,| | and
Solution.

Telegram channel https://t.me/johnson201485


28
8

a. | || | s ( )( ) s √

b. | || | s ( )( ) s . / √
c. | || | s ( )( ) s
Example 18. For two vectors u and v, find u.v,
a. if ( ) and ( ) c. if ( ) and (√ )
b. if ( ) and ( )
Solution. To calculate u.v first determine the angle between u and v, and their norms. Let  is the angle
between u and v.
a. ( ) and ( ).
Clearily, u and v are parallel and lie in the same direction, since
( ) ( ) . Hence,  = 00
| | √ and | | √
Therefore, | || | s ( ) s ( )
b. ( ) and ( ).
( ) ( )
 u and v are parallel and in the opposite direction. Hence,  = 1800
| | √ ( ) and | | √
Therefore, | || | s ( ) s ( )
c. if ( ) and (√ )

| | √( ) ,| | √√ √ and  = 1200 (WHY?)


 | || | s ( √ ) s √ ( s ) √ . / √
REMARK:
Given two non-zero vectors and with an angle between them, there are five possibilities:

(a) (b)

(c)

(d) (e)
i) When . Then and are parallel and in the same direction. (See Figure (a))
 | || | s | || |
ii) When . (i.e. is acute) (See Figure (b)). This implies that s .

Telegram channel https://t.me/johnson201485


28
9
 | || | s
iii) When . (i.e. is right angle) (See Figure (c)). Thus, and are perpendicular.
This implies that s . Therefore, | || | s
Thus, we have the following statement to test whether two vectors are perpendicular.
Two non-zero vectors and are perpendicular if and only if
iv) When . (i.e. is obtuse) (See Figure (d)). This implies that s .
 | || | s | || |
v) When . (See Figure (e)). Then and are parallel and in different direction
This implies that s .
 | || | s | || |
Example 19. For two vectors u and v, if u.v = 6, | | and | | √ , then find the angle between u
and v.
Solution. From the formula for dot product : | || | s , we have

s | || | √ √
 is a second quadrant angle with reference angle rad. =
Therefore, or
NOTE:
1. = 0, since s. /
2. , since cos(0) = 1.
3. If either u or v is 0, then u v = 0.
4. u u > 0 if u ≠ 0, and u u = 0 if u = 0.
5. u v = v u. that is dot product of vector is commutative.
6. If the vectors u and v are parallel and pointing to the same direction, then u v = |u||v|.
7. If vectors u and v are parallel and pointing to the opposite direction, then u v = − |u||v|.
8. In particular, for any vector u, you have u u = |u|2 , you write u2 to mean |u|2.
9. If the vectors u and v are perpendicular or orthogonal, then u v = 0, since s. /
10. If u and v are nonzero vectors, then the cosine of the angle between u and v is given
by s | || |
DOT PRODUCT OF VECTORS IN COMPONENT FORM
Suppose u and v are two vectors in component forms ( ) and
( ) . Let an angle between u and v at their
common initial point.
Then u, v and u  v form a triangle, as shown in the figure below. By
the Law of Cosines, | | | | | | | || | s
( ) ( ) | || | s

Telegram channel https://t.me/johnson201485


29
0
= | || | s
 | || | s
 | || | s
Therefore, the dot product of two vectors and ( ) is given by
( )( )
Example 19. Find the dot product of the vectors
a) u = 3i + 2j and v = 5i  3j b. a = (1, 0) and b = (4, 5)
Solution: a). u.v = (3i + 2j).(5i  3j) = 3(5) + 2(3) = 15  6 = 9
b). a.b = (1, 0). (4, 5) = (1)(4) + 0(5) = 4 + 0 = 4
Example 20. If u = i  2j , v = 4i + 2j and w =  3j,
then find a). v.w  3u b). | (  )|
Solution.
a. v.w 3u.v = (4i +2j).(3j)3(i2j)(4i+2j)=(4(0)+2(3))3(1(4)+(2)(2))=63(0) = 6
b. (  ) ( ) ( ( ) ( )) ( ) ( ) ( )
 | (  )| |(  )| √ √
Example 21. Find the possible values k so that the angle between vectors u = 2ki  j and
v = 3i + (1k)j is a. acute b. obtuse c. right angle
Solution. Let  be the angle between u and v.
u.v = (2ki  j).( 3i + (1k)j) = 2k(3) 1(1k) = 6k 1 + k = 7k  1
a. If  is acute, then and s . u.v = |u||v| cos  > 0  7k  1 > 0 
b. If  is ottuse, then and s . u.v = |u||v| cos  < 0 7k  1 < 0 
c. If  is right angle then θ and s s .u.v = |u||v|cos  = 0 7k1 = 0 
The following theorem lists the most important properties of the dot product. They are useful in
calculations involving vectors.
THEOREM 8.4. If u, v and w be vectors and k be a scalar, then
1. Commutative property : u v = v u
2. k(u v) = (ku) v = u (kv)
3. Distributive property i) u (v + w) = u v + u w and ii) (v + w) u = v u + w u
Proof : Let u u and v v and w w
1. u v = (u u ) (v v ) u v u v
v u v u ……….. since addition is commutative over .
2. k(u v) = ((u u ) (v v )) (u v u v ) (u v ) (u v )
( u )v ( u )v ……….. since multiplication is associative over .
3. Exercise
Example 22. Find the measure of the angle between each pair of the vectors
a. u = 4i − j and v = 5i + 3j b. u = i − j and v =  i + j
Solution : Let be the angle between u and v.

Telegram channel https://t.me/johnson201485


29
1
a. u = 4i − j and v = 5i + 3j u.v = (4i − j).( 5i + 3j) = 4(5) + (1)(3) = 17
| | √ ( ) √ and | | √ √
Now, | || |
 17 = √ √
 ( )
√ √ √ √ √ √ √
This shows that is acute, and .
b. u = 5i − j and v =  i + j
u v = (5i − j).(  i + j) = 5( ) + ( )( ) =  . /

| | √ ( ) √ and | | √( ) ( ) √


Now, | || | . /. /
| || | √ √


 ( )
Example 23. Let u and v be vectors with |u| = 3 and |v| = 6 and angle between them is θ = .
a. Find |3u − 2v|. b. Find the cosine of the angle between vectors 3u − 2v and u.
Solution : In this problem, the property | | is important.
| || | ( ) . / . /
a. Using the properties of dot product, you have
| | ( )( )
| | | | ( ) ( ) ( )
Then | | √ √
b. Let be the angle between vectors 3u − 2v and u.
( ) ( ) ( ) | | ( ) ( )
Now, ( ) | || | s

 45 = ( )( √ ) s  s ( )( √ ) √
Example 24. For two non-zero vectors u and v, show that u.v = 0 |u + v|2 = |u|2 + |v|2.
Solution :
i) Let u.v = 0.
| | ( )( ) | | | |
| | | | ; since
| | | | ; since
2 2
ii) Let |u + v| = |u| + |v|2
 | | ( )( ) | | | |
 |u| + |v|
2 2
| | | |
 | | | | | | | |

Telegram channel https://t.me/johnson201485


29
2
Exercise 12.3
1. Find the dot product of the following vectors.
a. u = 4i + 2j and v = 3i −5j b. u = −2i + 2j and v = 7i + 4j
2. Find the cosine of the angle formed by the following two vectors
a. u = 4i + 3j and v = 4i – 3j b. u = −2i + 3j and v = 3i + 2j
3. Let u = (1, –1), v = (1, 1) and w = (–2, 3). Find the cosines of the angles between:
a. u and v b. v and w c. u and w
4. Say whether the vectors are orthogonal, parallel or neither
a. u = (−5, 3) and v = (6, −8) c. u = . / and v = (3, 6)
b. u = (4, 6) and v = (−3, 2) d. u = (2, 6) and v = (−3, − 9)
5. Find the angle between the two vectors 2i + 3i and 5i − 2j.
6. Show that (u − v).(u + v) = u2 − v2
7. Show that (u ∓ v)2 = u2 ∓ 2 u v + v2, where u2 = u u
8. Vectors u and v make an angle θ = between them. If |u| = 5 and |v| = 12, find
a. u v b. (u – v) (u + v) c. (u + v)2 d. |3u + v| d. the angle between |3u + v| and v
9. Given a triangle with vertices A (1, 5), B (2, 1), and C (4, 6), find angles of BCA.
10. Let |u| = 13, |v| = 19 and |u + v| = 24. Calculate a. u.v b. |u – v| c. |3u + 4v|
11. Let a = 3i + j, b = 2i – 2j and c = i + 3j be vectors. Find the unit vectors in the direction of
each of the following vectors. a) a + b b) –

12.5. APPLICATION OF VECTOR


12.5.1 Application of Vectors in Plane Geometry
From previous knowledge, you notice that vectors have many applications. This subunit is devoted to the
application of vectors that include verifying Pythagoras theorem and some of the properties related to
concurrence in triangles of the perpendicular bisectors of the sides, altitudes, and angle bisectors.
Example 25. Show that, in a right angled triangle, the square of the hypotenuse is equal to the sum of the
squares of the other two sides.
Solution. Let ABC be a given right-angled triangle with ( ) .
Consider the vectors, , and , as the side of ABC, as shown in figure.
To show :
Since , we have = 0.
By vector addition you have . Thus,
( )( )
( )
Therefore,
Example 26. Prove the diagonals of a parallelogram meet at
right angles if and only if it is a rhombus.
Solution. Consider a parallelogram OACB. Consider the
vectors, , , and , as the side of the parallelogram,
as shown in figure.
Let and
i) Let the diagonals and be perpendicular. That is
To show that

Telegram channel https://t.me/johnson201485


29
3
This implies that
Therefore, and
By vector addition you have and
Then ( )( ) | | | |
 | | | |  | | | |  | | | |
Hence, the adjacent sides of the parallelogram are congruent.
Therefore, the parallelogram OACB is rhombus.
ii) Let the parallelogram OACB be a rhombus. That is | | | |
To show the diagonals and be perpendicular. That is
Therefore, and
From the figure, and
( )( )
| | | | 
Therefore, the diagonals and are perpendicular.
Example 27. Let A(3, 4) and B(9, 2) be two points on a coordinate plane. Find the position vector of
a point T on the line through O and A which is closest to B.
Solution. The position vectors of A and B are and , respectively.
Since T is on the line OA, vectors and are parallel.
Let be a constant such that ( ).
Since T is closest to B, we have is perpendicular to .
 = 0 where ( ) ( ) ( )
 ( )( )=0
 ( ) ( )=0  =0
 
Therefore, the position vector of T is ( )
Example 28. Prove that the perpendicular bisectors of the sides of a triangle are concurrent.
Solution. Let ABC be a triangle and D, E, F the mid-points of the sides , , and , respectively.
Let and are perpendiculars to and , respectively. Join O to the mid-point F of .
To show
Let u, v, w be the vectors , and , respectively.
Then, and
From the figure, and

 ……………. Since, , we have

Then, since and are perpendicular, we have
| | | |
. / ( )  | | | | ………… ( i )
In a similar manner, and
Since and are perpendicular, you have

Telegram channel https://t.me/johnson201485


29
4
| | | |
. / ( )  | | | | ………… ( ii )
From (i) and (ii), we obtain | | | |
You can also obtain the relations and
Now, ( ). / | | | |
Therefore, and are perpendicular.
Exercise 12.4
1. If u, v, w, z are vectors from the origin to the points A, B, C, D, respectively, and v – u = w – z,
then show that ABCD is a parallelogram.
2. The figure shows the magnitudes and directions of six
coplanar forces (forces on the same plane).
3. Three forces F1 = 2i + 3j, F2 = i + 2j and F3 = 3i − j
measured in Newton act on a particle causing it to move from A =
i − 2j to B = 3i + 4j where AB is measured in meters. Find the
total work done by the combined forces.
12.5.2 Vectors and Lines
Geometrically, any two points in the plane determine a straight
line. A straight line in the plane is determined by its slope and a point through which it passes are known.
These lines have been determined to have a certain direction. Thus, related to vectors, you will see how
one can write equations of lines, circles and tangent line to a circle using vectors.
Let ( ) and ( ) be two points on a line ℓ in the plane. Then, as shown in the figure
below, the vector from Po to P1 is P1 – Po = ( – – )
Let and are position vectors corresponding to the points Po
and P1, respectively. Then we have
– =( – – )
Now, the line ℓ that passes through Po and P1 is parallel to the
vector ( – – )
Let ( ) be any point on ℓ. Then the position vector of P
is obtained from the relation.
The vector often known as a DIRECTION VECTOR of the line ℓ.
Now, since and are parallel, we have
From triangular form of vector addition, the position vector obtained by

 ,
 , where t is a scalar.
Then we have the following results.
1. If a direction vectors v of the line ℓ and a point ( ) on ℓ are given, then the VECTOR
EQUATION of the line determined by Po and v is given by
, where t is a scalar
Thus, the vector equation of a line ℓ containing and parallel to vector v is given by
, where t is a scalar
2. If ( ) , ( ) and ( ), then the above equation can be written as
( ) ( ) ( ) , where t is a scalar and ( ) ( ).

Telegram channel https://t.me/johnson201485


29
5
From the above equation we have ( ) ( )
Comparing the components of the vectors, we obtain
Thus, if a line ℓ passes through a point ( ) and in the direction of (parallel to) a vector
( ) ( ), then the system of equations
; where t is a scalar and ( ) ( ) is called the PARAMETRIC
EQUATION of the line ℓ.The scalar t is called a PARAMETER.
3. For n , solving the above equation for t, we obtain
and 
The above equation is called the standard (symmetric) equations of the line.
Thus, if a line ℓ passes through a point ( ) and in the direction of (parallel to) a vector ( ),
where n , then the equations .
is called the STANDARD (SYMMETRIC) EQUATION of the line ℓ.
Example 29. Consider the line passes through the point P0(3, 1) and in the direction of vector
( ) then find
a. the vector equation, b. the parametric equation and c. the standard equation of the line.
Solution: Let (  ) be the position vector of the point P0(3, 1) and ( ) be an arbitrary point
on the line. Let ( ) be the position vector of point ( ).
a. The vector equation of the line is given by , where ∈
( ) (  ) ( ) Or ( ) ( ), for scalar t.
b. Continuing from the above equation and comparing the components of the vectors, the
parametric equation of the line is given by , for scalar t
c. Solving the above equation for , we obtain and
Then the standard (or parametric) equation of the line is
The above equation can also written as ( ) ( ) 
Example 30. Consider the line passes through the point (1, 4) and (2, 2) then.
Find a. the vector equation b. the parametric equation
c. the standard equation of the line.
Solution: Let ( ) and ( ), as shown in Figure.
a. The vector in the direction of the line is given by
( ) ( ) ( )
Then the vector equation of the line is given by :
 ( ) ( ) ( ), ∈
b. From the above vector equation, the parametric equation of the line is
, ∈
c. From the above parametric equation, the standard equation of of the line is
or
NOTE : The standard equation of a line ℓ passes through a point ( ) and in the direction of
(parallel to) a vector ( ), where n , given by
can also written as follow:
. /
Letting , , and Then we have

Telegram channel https://t.me/johnson201485


29
6
Example 31. Find the parametric equation of the line passing through the point (1, 3) and perpendicular
to the line containing the vector .
Solution: ( ) is the point on the line to be determined.
Let be the direction vector of the line and perpendicular to .
  ( ) ( )
 
Letting . then Then ( )
Thus, the vector equation of is given by
( ) ( ) ( ); ∈
Therefore, the parametric equation of the line is ; ∈
REMARK: Two lines L1 and L2 with direction vectors u1 and u2 are parallel if u1 and u2 are parallel.
Example 32. Consider the line ( ) ( ) ∈ .. Find the vector equation of a line L2
, parallel to L1 that passes through the point M(−1, −13) .
Solution: The direction vector is u1 = ( ).
Since || L2, the direction vector u2 of L2 is equal to u1. Hence, u2 = ( ).
Thus, is a line that contains the point ( ) and having direction vector u2 = ( ).
Therefore, the vector equation of is given by ( ) ( ) ( ); ∈
REMARK: Two lines L1 and L2 with direction vectors u1 and u2 are perpendicular if u1 and u2 are
perpendicular. That is if .
Given a vector u1 = ( ) ( ). If u2 is a vector perpendicular to u1, then u2 = ( ) or u2 =
( ) . Indeed, u1. u2 = ( )( ) ( ) and
u1. u2 = ( )( ) ( )
Example 33. Consider two lines ( ) ( ) ∈ and
{ ∈ . Show that .
Solution: Clearly, the direction vectors of the given lines L1 and L2 are ( ) and
( ), respectively. u1. u2 = ( )( ) ( ) ( )( )
This shows that u1 and u2 are perpendicular. Therefore,
Example 34. Consider the line ( ) ( ) ∈ . Find the vector equation of a line L2
perpendicular to L1 that passes through the point N(−3, 0).
Solution: The direction vector u1 of is u1 = ( ). Let the direction vector of is u2.
Then u1. u2 = ( )  ( ). Therefore, the vector equation of is given by
( ) ( ) ( ); ∈
12.5.3 Vectors and Circles Consider a circle with centre ( ) and radius and any points
( ) on a circle such that| | . where and are position vectors of ( ) and ( ),
respectively, as shown in the figure below. | | ………….> ( i )
Squaring both sides gives | |
 ……………………..> ( ii)
Then equation (ii) represent an equation of the circle with center at (
), and radius r through a position vector of any point on the circle ( )
Substituting the corresponding components of R(x, y) and ( ) in
equation 1,
we obtain: | | |( )|
 ( ) ( ) ……………………..> ( iii)

Telegram channel https://t.me/johnson201485


29
7
Equation (iii) is STANDARD EQUATION OF A CIRCLE.
Example 35. Find an equation of the circle centered at C(2, 3) and having radius 4.
Solution : The Let ( ) be a point on the circle. Then the position vectors of R and C, respectively,
are ( ) and ( ). Then from equation (ii), we have

 ( )( ) ( )( ) ( )( )


Example 36. Find the equation of the circle whose end points of a
diameter are P(−1, −1) and Q(4, 11).
Solution : The centre of the circle is ( ) midpoint of
 ( ) . / . /
The radius of the circle is given by ( ) √( ) ( )
Let R(x, y) be a point on the circle and and be position vectors of R and C, respectively.
Substituting the corresponding components of and in equation (ii), you obtain:

 ( )( ) ( ). / . / . / . /

Thus, the equation of the circle is
12.5.4 Equation of Tangent Lines to a Circle
A tangent line to a circle is a straight line that touches the circumference of the circle at the point of
tangency. A line tangent to a circle is characterized by the fact that the radius at the point of tangency is
perpendicular (orthogonal) to the line.
Consider a circle with center ( ) and a radius of r units as
shown in the figure below.
Let ( ) be a point of tangency to the circle. Since ( )
is a point on the circle, it fulfilled the equation of the circle: (
) ( )
If ( ) is an arbitrary point on the tangent L, then
Thus, the equation of the tangent line is ( )(
)
( ) ) ( )( )
Adding ( ) ( ) to both sides of the equation above, you obtain
( )( ) (( )( ) ( ) ( )
Factorizing ( ) and ( ) from respective terms, you have
( )( ) ( )( )
( )( ) ( )( )
Thus, the equation of the tangent line L to a circle with center ( ), radius and point of tangency
to the circle at ( ) is given by ( )( ) ( )( )
This equation can be written as dot product :
Example 37. Find the equation of the tangent to the circle x2 + y2 – 4x + 6y + 4 = 0 at (2, 0).
Solution : By completing the square, the equation can be written as ( ) ( )

Telegram channel https://t.me/johnson201485


29
8
This implies that The circle has its centre at ( ) ( – ) and radius r = 3.
The point of tangency of the tangent is ( ) ( ).
Thus, the equation is: ( )( ) ( )( )  (the x-axis)
Note: If the circle is centred at the origin, then the above equation of tangent becomes:

Example 38. Find equation of tangent line through the point R1(3, 4) at a point on the circle x2 + y2 =
25.
Solution : Since the center of the circle is at the origin, the equation of the tangent is given by
 ( ) ( ) 
Exercise 12.5
1. Find the parametric and symmetric equations of the line passing through the point (-3, 5) and
parallel to the line x = 1 + 3t, y = –1 – 2t.
2. Find a) the vector equation b) the parametric equation and c. the standard equation of the
line though (2, –1) and (5, 3).
3. Determine whether each of the following pairs of lines are parallel, perpendicular or neither.
i) L1 : ; L2 :
ii) L1 : ; L2 : ( ) ( ) ( )
iii) L1 : ; L2 :
4. Find the vector equation of a line .
5. Find the line through P = (1, 1, 1) and perpendicular to the line ( ) ( ) ( ).
6. Find the vector equation of line passing through the point having vector 5i + 4j and having
direction vector ( −3, 4).
7. Find the standard form of equation of a line ( ) ( ) ( ) ∈ .
8. Given an equation of a line ℓ by P = (1, 0) + t (2, 2), ∈ ., find out whether the, points
A(1, 0), B(2, 2), C(–5, –6) and D (3, 0) lie on ℓ. For those of them lying on ℓ find the
respective values of the parameter t.
9. The standard equation of a line is . Rewrite the equation in parametric and vector
equation form.
10. Find a vector equation of a line L passing through the point ( ) and parallel to the line
containing points ( ) and ( ). Check whether the point P( ) lies on L.
11. Are the points A, B and C collinear?
a) A(1, –4), B (–2, –3), C(11, –11) b) A(–2, –3), B(4, 9), C (–11, –21)
12. Find the point of intersection of the pair of lines L1 : and
L2 : { ∈
13. Find the cosine of the angle between the lines L1 : , ∈ and
L2 :
14. Find an equation of the circle centred at C (1, −3) and radius 6.
15. Find the equation of the circle whose end points of a diameter are P(2, 2) and Q(5, 3).
16. Find an equation of the circle centred at origin and passes through (1, 3).
17. Find an equation of the circle cantered at C (−4, 2) and passes through (2, −5).
18. Find the equation of the tangent to the circle at the point (−4, 8).
19. Find equation of the tangent line through P(−4, −3) point on the circle .
20. Show that the given point lies on the circle and find the equation of the tangent at the point.
a) x2 + y2 – 2x – 4y – 9 = 0 at P(1, 4) b) (x+2)2 + y2 = 3 at P(–1, 2

Telegram channel https://t.me/johnson201485


29
9
12.6. TRANSFORMATION OF THE PLANE
Transformations are of practical importance, especially in solving problems and describing difficulties
in simpler forms. Transformations can be managed in different forms, those that maintain direction and
those that change direction. There are many versions of transformations, but, in this section, you are
going to consider three types of transformations namely translations, reflections and rotations.
ACTIVITY 12.2
1. Consider the following conditions, discuss what will happen on the shape or size or both of the
object:
a. when a spring is compressed or stretched
b. when the earth rotates about its axis
c. when a commercial jet flies from place to place at a specific time.
d. when you see your image in a plane mirror.
e. when you draw the map of your school compound.
2. Let T be a mapping of the plane onto itself given by T ((x, y)) = (x + 1, –y).
For example, T((4, 3)) = (4 + 1, – 3) = (5, –3).
If A = (0, 1), B = (–3, 2) and C = (2, 0), find the coordinates of the image of A, B and C.
Find the image of ABC under T. Is ABC congruent to its image?
3. Suppose T is a mapping of the plane onto itself which sends point P to point P'.

Let A(2, –3) and B (5, 4) be points on the plane.. Compare the lengths of and when
a. T((x, y)) = (x, 0) c . T ((x, y)) = (x, –y)

b. T ((x, y)) = (x + 1, y – 3) d. T ((x, y)) = . /.

From the above Activity, you have observed that some mappings, called Transformations of the plane
onto itself, of objects preserve shape, size or distance between any two points and some other mapping of
objects are not preserve shape, size or distance between any two points. Due to this reason,
transformations are classified as either rigid motion or non-rigid motion. But in this section, we will
focus only on rigid motions.

DEFINITION 12.10
A motion is said to be RIGID MOTION, if it preserves distance. That is for two distinct points P and Q,
PQ = P'Q' where P' and Q' are the images of P and Q, respectively. Otherwise it is said to be NON-
RIGID MOTION.

Telegram channel https://t.me/johnson201485


30
0
A transformation is said to be an IDENTITY TRANSFORMATION, if the image of every point is
itself. For example, if an object is rotated 3600 it is an identity transformation.
NOTE: Rigid motion carries any plane figures to a congruent plane figure. That means rigid
motion carries triangles to congruent triangles, parallelograms to congruent parallelograms, etc.
Within the rigid motion, there are three main types of transformations that we will learn in this unit.
These are translation, reflection and rotations.
Translation, Rotation, and Reflection all change the position of a shape, while the size remains the same.

12.6.1 Translation
Translation is a type of rigid motion that occurs when the object simply slides and maintains its direction.
In a translation, everything is moved by the same amount and in the same direction. Every translation has
a direction and a distance.
ACTIVITY 12.3
Consider a triangle ABC. Discuss what will happen to the shape, size and orientation of a triangle when
you slide it to the new position , as shown in the figure below.

From the above activity, you have observed that and have the same size, shape and
orientation. This type of transformation is said to be a TRANSLATION.
An easy way to remember what translation means is that a translation is a change in location.

DEFINITION 12.11:A TRANSLATION or PARALLEL MOVEMENT is a transformation that


occurs when every point of a figure is moved from one location to another location along the same
direction through the same distance.

Telegram channel https://t.me/johnson201485


30
1
If point P is translated to point ′, then the vector is called the TRANSLATION VECTOR. A
translation is usually denoted by T.
You can state a translation formula in terms of coordinates as follows:
If u = (h, k) is a translation vector of a translation T, then
( )
a. The origin is translated to (h, k). that means ( )→ ( )
b. The image of the point P(x, y) under the translation will be the point given by
T(x, y) = ′(x + h, y + k).
Example 39. Let T be a translation that takes the origin to (1, 2). Determine the translation vector and
find the images of the following points. a. P(2,–1) b. Q(–3, 5) c. S(x + 3, y  1)
Solution. The translation T takes the origin (0, 0) to the point (1, 2).
 T((0, 0)) = (1, 2) Then the translation vector is u(1  0, 2  0) = u(1, 2)
a. T(P) = T(2, –1) = (2 + 1, 1 + 2) = ′(3, 1)
b. T(Q) = T(–3, 5) = (–3 + 1, 5 + 2) = Q′(2, 7)
c. T(S) = T(x + 3, y  1) = ((x + 3) + 1, (y  1) + 2) = S′( x + 4, y + 1)
Similarly, if the translation vector is u = , where P = (a, b) and Q = (c, d), then
a. the origin is translated to (c  a, d  b). that means T((0, 0)) = (c  a, d  b).
b. the point P(x, y) is translated to (x  c  a, y  d  b)
Example 40. The image of point (3, 2) under translation T is (2, 5). What is the image of the point (0,
4) under the same translation?
Solution. The translation T takes the (3, 2) to the point (2, 5)  T((3, 2)) = (2, 5)
 The translation vector is u(2  (3), 5  2) = u(5, 3)
Then the image of the point (0, 4) under this translation is T((0, 4)) = (0 + 5, 4 + 3) = (5, 1)
THEOREM 12.5: A translation is a rigid motion.
Proof. Let the points ( ) and ( ) be translated by the vector ( ) to points
and , respectively.
Clearly, the image points are
( ) ( ) and ( ) ( )
Then, √( ) ( ) and

√(( ) ( )) (( ) ( )) √( ) ( )

Telegram channel https://t.me/johnson201485


30
2
This shows that .
Therefore, T is a rigid motion.
Example 41. If A(4, 4), B(2, 1) and C(1, 5) are the vertices of a triangle ABC. Find the
coordinates of the image of ΔABC under the translation T = (6, 5). Draw ΔABC and its image on the
same plane.
Solution. As A(4, 4), B(2, 1) and C(1, 5) are the vertices of ΔABC and the translation vector is T
= (6, 5), the coordinates of the vertices of image of ΔABC can be obtained as shown below: T(A) =
T(4, 4) = A′(4 + 6, 4 + 5) = A′(2, 1)
T(B) = T(2, 1) = B′(2 + 6, 1 + 5) = B′(4, 4)
T(C) = T(1, 5) = B′(1 + 6, 5 + 5) = C′(5, 0)
Thus, the image triangle ΔA′B′C′ of ΔABC has vertices A′(2, 1), B′(4, 4) and B′(4, 4).
The following figure shows ΔABC and ΔA′B′C′ on the same coordinate plane.
from the figure, you can observe that
( )

Translation of Line on a Plane


A translation maps lines onto parallel lines. Let ℓ' be the image of ℓ under T. Then,
Case 1. If ℓ is non-vertical line, then its image line ℓ' is also non-vertical and
Slope of ℓ = Slope of ℓ' = m
Thus, to find the image line ℓ' of a non-vertical line ℓ under translation T by translation vector
u = (h, k), we follow the following procedure.
Step 1: Determine the slope m of ℓ. Then Slope of ℓ' = Slope of ℓ = m
Step 2: Choose a point P on the given line ℓ and find the image P ' of P under T.
Then the image line ℓ' contains the image point P ' and having slope m.
Case 2. If ℓ is a vertical line given by , then its image line ℓ' under translation T by translation
vector u = (h, k) is also vertical and the equation of ℓ' is given by
( )
This is to mean that →

Telegram channel https://t.me/johnson201485


30
3
Example 42. If a translation T takes the origin to (0, 2), find the image of each of the following lines.
a. b. c. d.
Solution. T(0, 0) = (0, 2) implies that the translation vector is u = (h, k) = (0, 2)
a. ℓ: ; m = slope of ℓ = 3 = slope of ℓ'
P(0, 1) is a point on the given line ℓ. The image P' of P is given by
( )
P( ) → P( ) ( )
Then the image line ℓ' has slope 3 and contains the image point ( ).
Therefore, by slope – point form of equation of a line, the equation of the image line ℓ' is
ℓ' : ( )
ℓ' : ( )
ℓ' :
b. ℓ: ; m = slope of ℓ = 3 = slope of ℓ'
P(0, 1) is a point on the given line ℓ. The image P' of P is given by
( )
P( ) → P( ) ( )
Then the image line ℓ' has slope 3 and contains the image point ( ).
Therefore, by slope – point form of equation of a line, the equation of the image line ℓ' is
ℓ' : ( )
ℓ' : ( )
ℓ' :
c. 

 m = slope of ℓ = = slope of ℓ'

P(6, 2) is a point on the given line ℓ. The image P' of P is given by
( )
P( ) → P( ) ( )
Then the image line ℓ' has slope and contains the image point ( ).

Therefore, by slope – point form of equation of a line, the equation of the image line ℓ' is
ℓ' : ( )

ℓ' : ( )

ℓ' :
d. is vertical line. Its image ℓ' is also vertical given by

Telegram channel https://t.me/johnson201485


30
4
ℓ' :
ℓ' :
Translation of a Conic Sections on a Plane
As a translation is a rigid motion,
 to translate a circle it suffices to translate the center of the circle by the translation vector T (h, k)
and keep its radius for the image too.
 to translate an ellipse it suffices to translate the center of the ellipse by the translation vector T (h,
k) and keep its lengths of major and minor axes for the image too.
 to translate a parabola it suffices to translate the vertex of the parabola by the translation vector T
(h, k) and keep its focal length for the image too.
Example 43. Determine the equation of the circle ( ) ( ) when the origin is translated
to the point A(2, –1).
Solution. The translation vector is u = (2, –1).
The given circle C : ( ) ( ) has center O(1, –3) and radius r = 2.
( )
( ) → ( ( )) ( )
Thus, the image circle C' has center ( ) and radius r = 2.
Then the equation of the image circle is C' : ( ) ( ( ))
 C' : ( ) ( )
Example 44. Determine the equation of the curve 2x2 + 3y2 + 8x + 6y = 7 when the point A(1, 4) is
translated to the point B(1, 3).
Solution. The translation vector is u = (h, k) = (1 – (–1), 3 – 4) = (2, 1).
The curve to be translated is an ellipse E : 2x2 + 3y2 + 8x + 6y = 7

 ( ) ( )
 ( ) ( )
 ( ) ( )
( ) ( )

Thus, the given ellipse has center ( ), and lengths of semi-major and semi-minor axis √
and √
This implies that the image ellipse ha center

Telegram channel https://t.me/johnson201485


30
5
( ) ( ( )) ( ) is
( ) ( )
Therefore, the equation of the image ellipse is E' :

( ) ( )
 E' :  E' :
( )
 E' :  E' : ( )

Or E' :
Example 45. Determine the equation of the parabola x2 + 6y = 18 under translation by a translation
vector u = (3, 2).
Solution. The translation vector is u = (h, k) = (3, 2).
The curve to be translated is a vertical parabola P : x2 + 6y = 18 which opens downward.
Its standard form is P : ( ) ; 

Thus, the given parabola has vertex V( ) ( ) and focal length .

This implies that the image parabola ha vertex


( ) ( ) ( ) ( ) is
Therefore, the equation of the image parabola is P' : ( ) ( )

 P' : ( ) . /( )

 P' : ( ) ( )
P' :
Therefore, the standard form of the image parabola is P' :
Combined Translation
THEOREM 12.6: The composition of two (or more) rigid motions is a rigid motion.
Notation: Let S and T be two rigid motions. Then
i. The rigid motion S followed by T, denoted by , is given by: ( )( ) ( ( ))
ii. The rigid motion T followed by S, denoted by , is given by: ( )( ) ( ( ))
Example 46. A translation T takes the origin to A(3, –2). A second translation S takes the origin to B(–2,
–1). Find
i. where T followed by S takes the origin, and
ii. where S followed by T takes the origin.
Solution.

Telegram channel https://t.me/johnson201485


30
6
i. S(T(0, 0)) = S(3, –2) = (3+(–2), –2+(–1)) = (1, –3)
ii. T(S(0, 0)) = T(–2, –1) = (–2+3, –1+(–2)) = (1, –3)
REMARK :
Generally, if a translation vector ( , ) translates point ( , ) to point ( ), then we have the
relation and .
As a result and .
Then to obtain the equation of the image curve, we replace the variables and by and ,
respectively, in the given equation.
Thus, suppose a translation T maps the origin to the point ( ), then the translation vector is
( ). Then the image of an equation ( ) by translation vector is ( ) is given
( )
by ( ) → ( ) ( )
Therefore,
i) The image of a line ℓ : ( or ) after it has been translated by ( )
is given by
( ) ( or ( ) ( ) )
ii) The image of a circle C : ( ) ( ) (or ) after it has
been translated by vector ( ) is given by
(( ) ) (( ) )
(or ( ) ( ) ( ) ( ) )
Note that the equations of the images of ellipses, parabolas and hyperbolas can also obtained in the
similar manner.
NOTE : A translation carries a circle into a congruent circle, an ellipse into a congruent ellipse, a
parabola into a congruent parabola, and a hyperbola into a congruent hyperbola.
Example 47. If a translation T takes the point (–1, 3) to the point (4, 2), then find the images of the line ℓ
: y = 3x + 1 under the translation T.
Solution. The translation vector is (h, k) = (4 – (–1), 2 – 3) = (5, –1). Thus, the point P(x, y) is translated
to the point P'(x + 5, y – 1). Let ℓ' be the image of ℓ under T. Then,
( )
ℓ: → ℓ' : ( ) ( )
ℓ' :

Telegram channel https://t.me/johnson201485


30
7
Example 48. If a translation T takes the origin to ( 2, 2), then find the equation of the image for the
circle whose equation is ,
Solution. The translation vector is (h, k) = (2 – 0, 2 – 0) = (2, –2).
The given circle is C(x, y) : .
Therefore, the image circle is ( ) ( ) ( ( ))
 ( ) ( ) ( ) ( )
 ( )
Example 49. Determine the equation of the image of a curve when the origin
is translated to the point A(0, –3).
Solution. The translation vector is (h, k) = (0, –3).
( )
P( ) → P'( ( )) ( )
Therefore, the image curve is ( ) ( ) ( ) ( )
Or

Exercise 12.6
1. If a translation T takes the origin to the point A(–3, 2), find the image of the rectangle ABCD with
vertices A(3, 1), B(5, 1), C(5, 4) and D(3, 4).
2. Triangle ABC is transformed into triangle A'B'C' by the translation vector (4, 3).
If A = (2, 1), B = (3, 5) and C = (–1, –2), find the coordinates of A', B' and C'.
3. Find the equation of the image of the circle (x + 1)2 + (y – 3)2 = 5 when translated by the vector ,
where P = (1, –1) and Q = (–4, 3).
4. A translation T takes the origin to A(3, –2). A second translation S takes the origin to B(–2, –1).
Find where T followed by S takes the origin, and where S followed by T takes the origin.
5. If a translation T takes (2, –5) to (–2, 1), find the image of the line ℓ : 2x – 3y = 7.
6. If a translation T takes the origin to (4, –5), find the image of each of the following lines.
a. y = 3x + 7 b. 4y + 5x = 10
7. If the point A(3, –2) is translated to the point A'(7, 10), then find the equation of the image of
a. the ellipse 4x2 + 3y2 – 2x + 6y = 0 b. the parabola y2 = 4x
c. the hyperbola xy = 1 d. the function f (x) = x3 – 3x2 + 4
12.6.2 Reflections
As the name indicates, reflection transforms an object using a reflecting material.

Telegram channel https://t.me/johnson201485


30
8
A reflection fixes a mirror line in the plane and exchanges points from one side of the line with points on
the other side of the mirror at the same distance from the mirror.
Every reflection has a mirror line.
ACTIVITY 12.4
Consider a triangle ABC, discuss what will
happen the shape, size and orientation of a
triangle when you fold or flip over the fixed line
as shown in the Figure below.

From the above Activity, you have observed that ABC and its reflection A‟B‟C‟ have the same shape
and size, but the figures face in opposite directions. Having these concepts, let us define reflection as
follows.

DEFINITION 12.11
Let L be a fixed line in the plane. A reflection M about a line L is a transformation of the plane onto itself
which carries each point A of the plane into the point of the plane such that L

is the PERPENDICULAR BISECTOR of .


The line L is said to be the LINE OF REFLECTION
or the AXIS OF REFLECTION.
That is the point is the image of the point after it has been reflected along the line L if i) L bisects

, and ii) L is perpendicular to .


NOTATION: The reflection of a point A about the line L, is denoted by M(A) i.e. = M(A).
A REFLECTION LINE is a line over which the original object will be flipped to create a congruent,
mirror image of an object.
THEOREM 15.6: A reflection M is a rigid motion. That is, if P' = M(P) and Q' = M(Q), then PQ = P'Q'.
Basic Properties of Reflection
Let M be a reflection about a reflecting line L.
1. For two points A and B in the plane, if A = B, then M(A) = M(B). Hence, reflection is a function form
mapping the set of points in the plane into the set of points in the plane except reflection of points about
the axis of reflection.

Telegram channel https://t.me/johnson201485


30
9
2. L maps distinct points to distinct points, i.e., if A  B, then M(A)  M(B). Equivalently, it has the
property that, for points A and B in the plane, if M(A) = M(B), then A = B. Thus, reflection is a one-to-
one mapping.
3. For every point A' in the plane, there exists a point A such that M(A) = A'. If the point
A' is on L, then there exists A = A' such that M(A) = A'. Thus, reflection is an onto
mapping.
Next we shall consider how to find images under reflections along the line of the form and
, where m is the slope of the line.
A. Reflection in the line y = mx
Clearly the line of the form passes through the origin. Let L be a line passing through the origin
and making an angle  with the positive x-axis.
You will now find the image ( ) of a point P(x, y) under reflection M about the line L, as shown in
figure given below.

L L
A

Let  be the angle between the x-axis and the line . Let A be the point of intersection of L and ,
and .

Since L bisects and , we have (∠ ) (∠ )  .

The angle between and x-axis is ( )   


Using trigonometric identity, the coordinates of P can be written as and
In the similar manner, from the indicated right-triangle, the coordinates of can be written as
(  ) and (  )
Expanding (  ) and (  ), we obtain
(  )  s  sin  ysin , and
(  )  s  sin  , and
Thus, the coordinates of P'(x', y'), the image of the point P (x, y) when reflected about

Telegram channel https://t.me/johnson201485


31
0
the line y= mx, is given by x' = x cos 2 + y sin 2 and y' = x sin 2 – y cos 2
where  is the angle of inclination of the line L : y = mx. Recall that
Based on the value of , you will have the following four special reflections:
1. When  = 0, you will have reflection in the x-axis.
x' = x cos 2(0) + y sin 2(0) = x(1) + y(0) = x , and y' = x sin 2(0) – y cos 2(0) = x(0) – y(1) = – y

Thus, ( ) → ( )
2. In a similar manner, when  = , you will have reflection in the y-axis.

Thus, ( ) → ( )

3. When  = , you will have reflection in the line y = x. Thus, ( ) → ( )

4. when  = , you will have reflection in the line y = – x. Thus, ( ) → ( )

Example 49. Find the image of


a) The point P(2, 3) after it has been reflected along the x-axis.
b) The point P(2, 3) after it has been reflected along the y-axis.
c) The point P(3, 2) after it has been reflected along the y = x.
Solution

a) ( ) → ( ) c) ( ) → ( )

b) ( ) → ( )

Example 50. Find the image of the point A(5, 1) after it has been reflected along .
Solution. Since reflection along the line maps ( ) onto ( ), we have

( ) → ( )
Example 51. Find the image of the point A(2, 1) after it has been reflected along √ .
Solution. The reflecting line L : √ has the form with

Telegram channel https://t.me/johnson201485


31
1
t n √  ( √ ) .

Thus, the coordinates of the image point are


s ( ) sin ( ) s sin
√ √ √
. / . / , and

sin . / s . / sin s
√ √
. / . / √
√ √
Thus, the coordinates of the image point of A(2, 1) are . /

Example 52. Find the image of A(0, 3) after it has been reflected along the line .
Solution. The reflecting line L : has the form with
Thus, if is the angle of inclination of L, then t n . is a 1st quadrant angle

Let t n . Take and . Then √ √ √

 sin and s
√ √

From trigonometric identity, we have

sin sin s . /. / and s . / . /


√ √ √ √

Then the coordinates of the image of A(0, 3) are

. / . / , and

. / . / , and

Thus, the coordinates of the image point of A(0, 3) are . /

B. Reflection in the line


1. Reflection of a point in the line
Let L: , be the line of reflection.
Let A(x, y) be a point on a plane to be reflected, not on L.
Let A'(x', y') be the image of A (x, y) when reflected about the line L.

Let be a line that passes through the given point A and the image point A'. This is to say ⃡ . Since

we have slope of =

Then the equation of is determined by the point A the slope .

Let the lines L and t intersect at point B. then B is the midpoint of A and A'.
therefore, using points A and B, we can determine the coordinates of the image point .

Telegram channel https://t.me/johnson201485 31


2
Thus, to find the image A'(x', y') of a point A(x, y) when reflected about a line L : we
follow the following four steps.
STEP 1: Find the slope m of the line of reflection L.
STEP 2: Find the equation of the line , which passes through the point P(x, y) and has slope

STEP 3: Find the point of intersection B of L and t which serves as the midpoint of ̅̅̅̅̅.
STEP 4: Using A as the mid-point and P as one end point of ̅̅̅̅̅, find the coordinates of P'.
Example 53. Find the image of the point P(1, 3) after it has been reflected along the line .
Solution.
The reflecting line L : has slope and the point to be reflected is P(1, 3).
Let be the line passing through P(1, 3) and image P'(x', y').

Then the slope of is

 The equation of is ( )  ( )

The point of intersection of L and is obtained as follow: {


( )

  ,

This point of intersection . / is the mid-point of P(1, 3) and P'(x', y').

Then by the midpoint formula the coordinates of P' are and

 and
Thus, the image of the point P(1, 3) after it has been reflected along is ( ).
NOTE : The steps for determining the image of a point along the reflecting line also works
for the reflecting line .
Example 54. Find the image of the point A(2, 0) after it has been reflected along the line .

Solution: The reflecting line L : has slope and the point to be reflected is A(2, 0).

Let be the line passing through A(2, 0) and image A'(x', y').

Then the slope of is

 The equation of is ( ) 

The point of intersection of L and is obtained as follow: {


( )

Telegram channel https://t.me/johnson201485 31


3
  , ( )
This point of intersection ( ) is the mid-point of A(2, 0) and A'(x', y').
Then by the midpoint formula the coordinates of P' are
and  and

Thus, the image of the point A(2, 0) after it has been reflected along is ( ).
Example 55. If a reflection along a line L maps a point ( ) into the point ( ) , then find the
equation of line of reflection.
Solution: By definition of reflection, L is the perpendicular bisector of ̅̅̅̅̅.
( )
Slope of ̅̅̅̅̅ . Hence, slope of

L passes through the mid-point B of P and : . / . /

Thus, the line of reflection L passes through . / and having slope .

Therefore, the equation of reflecting line is ( )

i.e

2. Reflection of a line in the line


NOTE: If a line ℓ' is perpendicular to the axis of reflection L, then L' is its own image.
In reflection of a line along a line, there are two cases:
i) When the line to be reflected is parallel to the line of reflection, and
ii) When the line to be reflected is not parallel to the line of reflection.
CASE 1. If ℓ is a line parallel to the line of reflection L, to find the image ℓ' of ℓ when reflected about L,
we follow the following steps. ℓ

Step a: Choose any point P on ℓ.



Step b: Find the image P' of P along L. That is M(P) = P'
Step c: Find the equation of ℓ', which is the line passing
cccthrough P' with slope equal to the slope of ℓ.
Example 56. Find the image of the line after it has been reflected along the line

Solution. You can observe that the reflecting line L and the given line are perpendicular, since the
product of their slops is 1.
Therefore is its own image.
That is the image line is

Telegram channel https://t.me/johnson201485 31


4
Example 57. Find the image of the line after it has been reflected along the line
.
Solution. The reflecting line L : and the given line are parallel, since they have
equal slops.
The point ( ) is on Let ( ) be the image of A after it has been reflected along the line .

The line passing through and has slope , since it is perpendicular to L.

Then passes through A(0, 0) and having slope .

 The equation of is ( ) 

The point of intersection of L and is obtained as follow:

{   ,
( )

This implies that the point of intersection . / is the mid-point of A(0, 0) and A'(x', y').

Then by the midpoint formula the coordinates of A' are


and  and

Thus, the image of the point A(0, 0) after it has been reflected along is . /.

Therefore, by slope-point form of equation of a line, the equation of the image line of is

. / . / :

CASE 2. If s is a line not parallel to the line of reflection L, to


find the image s' of s when it has been reflected about L, we
follow the following steps. (Refer the figure below)
Step a: Choose any point A on s, not on the reflecting line L.
Step b: Find the image A' of A along L. That
is M(A) = A'
Step c: Find the point of intersection C of L and s.
The image line s' also passes through C.
Step d: Find the equation of s', which is the line passing through A' and C.
Example 58: Find the image of the line after reflection in the line .
Solution. The line to be reflected s : y = 3x + 2 and the reflecting line L : y = x + 4 are not parallel, since
they have different slope.

Telegram channel https://t.me/johnson201485 31


5
Let A (a, b) be any point on s, say A(0, 2), so that its image A'(a', b') lies on s', as shown in
figure above. Slope of L = m = 1
Now, since ̅̅̅̅̅ , we have

= Slope of ̅̅̅̅̅ =

   ----------- ( i )

The midpoint of (0, 2) and (a', ) which is . / . / lies on the reflecting axis L.

Then from , it follows  -------- ( ii )

Then solve equations (i) and (ii) simultaneously: 2  


( )
,
Thus, the image of the chosen point A(0, 2) is ( ).
The point C of intersection of the lines L and s is obtained as follow:

{   ,
( )
It follow that C has coordinates C(1, 5)
Then the image line passes through points ( ) and C(1, 5).
Therefore, by two-point form of equation of a line, the equation of the image line is


( )

Reflection of a point in a vertical line ∈


The image of a point ( ) after it has been reflecting along a vertical line

is ( ). That is ( ) → ( )
Example 59. Find the image of the point ( ) after reflection in the line .
Solution. Reflection with respect to the vertical line maps the point ( ) ( ) to the point
( ) ( ( ) ) ( ).
Reflection of a point in a horizontal line ∈
The image of a point ( ) after it has been reflecting along a horizontal line

is ( ). That is ( ) → ( )
Example 60: Find the image of the line after reflection in the line .
Solution: First choose a point on the given line to be reflected, say ( ).

( ) → ( ) ( ( ) ) ( )

Telegram channel https://t.me/johnson201485 31


6
The point C of intersection of the lines L and is obtained as follow:

{  ( )   &
( )
It follow that C has coordinates C(5, 3)
Then the image line passes through points ( ) and C(5, 3).
Therefore, by two-point form of equation of a line, the equation of the image line is
( ) ( )
( )
( )

Reflection of a Circle C in the Line


NOTE:
1. If the centre of a circle C is on the line of reflection L, then the image of C is itself.
2. If the centre O of a circle C has image O' when reflected about a line L, then the image circle has
centre O' and radius the same as C.
Example 61: Find the image of the circle after reflection in the line .
Solution: The center A and radius of the given circle C are A(0, 0) and √ , respectively. The
center A(0, 0) lies on the reflecting line . This shows that the image of the center after it has been
reflected along L is itself A′(0, 0).
Therefore, the image circle of the given circle C is itself
Example 62. Given an equation of a circle ( ) ( ) , find the equation of the
image circle after a reflection about the line y = x + 3.
Solution. Slope of L is
The center A and radius of the given circle C are A(1, 3) and , respectively.
Let ( ) be the image of A(1, 3) under this reflection.

 slope of ̅̅̅̅̅ =

This implies that the equation of the line ⃡ is given by


( )

Then the point of intersection B of and is obtained as follow:

{    &
( )

Thus, the point . / is the mid-point of A and . Then by mid-point formula

and  n

 the image point has coordinates ( ), which is the center of the image circle.
Therefore, the equation of the image circle is given by

Telegram channel https://t.me/johnson201485 31


7
( ) ( )  ( )

Exercise 12.7
1. The vertices of triangle ABC are A (2, 1), B (3, –2) and C (5, –3). Give the coordinates of the
vertices after: a. a reflection in the x-axis b. a reflection in the y-axis
c. a reflection in the line x + y = 0 d. a reflection in the line y = x.
2. Find the image of the point (–4, 3) after a reflection about the line ℓ : y = x −2.
3. If the image of the point (–1, 2) under reflection is (1, 0), find the line of reflection.
4. Find out some of the figures which are their own images in reflection about the line y = x.
5. Find the image of the line ℓ : y = x + 4 after it has been reflected about the line L : y = x – 3.
6. Find the image of the line ℓ : y = 2x + 1 after it has been reflected about the line L : y = 3x + 2.
7. The image of the circle x2 + y2 – x + 2y = 0 when it is reflected about the line L is
x2 + y2 – 2x + y = 0. Find the equation of L.
8. Find image of the circle after reflection in the line .
9. Given an equation of a circle ( ) ( ) , find the equation of the image
circle after a reflection about the line .
10. If T is a translation that sends (0, 0) to (3, −2) and M is a reflection that maps (0, 0) to (2, 4),
find a) T(M(1, 3)) b) M(T(1, 3))
11. In a reflection, the image of the line y − 2x = 3 is the line 2y – x = 9. Find the axis of
reflection.
12. Find the image of the circle after reflection in the line = .
13. Find the image of the circle ( ) ( ) after reflection in the line : = .
15.6.3 Rotation
Rotations are movements around a central point where distance from that point is maintained.
A rotation fixes one point and everything rotates by the same amount around that point. Every rotation
has a center of rotation and an angle of rotation.
ACTIVITY 15.5 Consider a triangle ABC, discuss what will be
happened the shape, size and orientation of a triangle when you
turn ABC counter clockwise it
around a fixed point as shown in the figure below
and try to define what does rotation of figures mean?
As you observed form the figure, when ABC is rotated about a point O or the centre of rotation to be
the origin, through 900 in counter clockwise direction, then and have the same shape and
size.

Telegram channel https://t.me/johnson201485 31


8
Thus, we define rotation formally as follow.

DEFINITION 12.12
A ROTATION R about a point O through an angle θ is a transformation of the plane onto itself which
carries every point P of the plane into the point of the plane such that
i) and ii) (∠ ) .
O is called the CENTRE OF ROTATION and θ is called the ANGLE OF ROTATION.
The rotation is in the counter clockwise (ccw) direction if θ > 0, and in the clockwise direction (cw) if θ
< 0, as shown in the following figure.

A. Rotation when the center of rotation is the origin (0, 0)


Example 63. Find the image of point A (2, 0) when it is rotated through 30° about the origin.
Solution. Let the image of the point A (2, 0) be A'(a, b) as shown in the Figure below.
From trigonometry, you have ( ) = (r cos θ, r sin θ) where r = 2 and θ = 30°

Therefore, ( ) ( s sin ) 4 . / . /5 (√ )

Therefore, the image the point A(2, 0) under rotation through an angle of
about the origin is . This is written as ( ) (√ )
NOTATION: If R is rotation through an angle θ, then the image of A (x, y) is denoted by ( ).
Now, let as derive the formula for the image ( ) of a point ( )
under rotation through an angle of θ about the origin ( ).
Let be the angle between ̅̅̅̅ and the x-axis.
Thus, the angle between ̅̅̅̅̅ and the x-axis is θ.
Let . Hence, , as shown in the figure.
From trigonometric identity, you have
( ) ( ) and
( ) ( ( ) ( ))
You know that ( )
( )
This proves the following theorem.

Telegram channel https://t.me/johnson201485 31


9
THEOREM 12.7: The image of a point ( ) under rotation through an angle of about the origin
O(0, 0) is given by ( ) (x y ) ( )
Example 64. Find the image of a point ( ) when it is rotated about the origin through

a) 60° b) 150° c)

Solution. Let ( ) ( ) ; and


Let the image of the point A(2, 1) when it is rotated about the origin ( ).
a)
( )
( )→ ( )
√ √
 s sin . / , and
√ √
sin s . / .

Therefore; the image of the point A(2, 0) when it is rotated about the origin through 60°
√ √
is ( ) . /

b) . The rotation in clockwise direction.



s( ) s and sin( )= sin and
√ √
 s( ) sin( ) . / . / , and
√ √
sin( ) s( ) . / . /

Therefore; the image of the point A(2, 1) when it is rotated about the origin through 60°
√ √
is ( ) . /

Example 64. Find the image of a point ( ) when it is rotated about the origin through
a) b) 900 ; cw f) 12600

Solution. Let ( ) ( ). That is and


√ √
a) .  s. / s. / = and sin . /= sin 
√ √
s. / sin . / . / , and
√ √
 sin . / s. / . /

Therefore; the image of the point A(0, 3) when it is rotated about the origin through
√ √
is ( ) . /

b)

Telegram channel https://t.me/johnson201485 32


0
 s( ) s and sin( )= sin .
 s( ) sin( ) ( ) , and
sin( ) s( ) ( )
Therefore; the image of the point A(0, 3) when it is rotated about the origin through is
( ) ( )
c) ( )  and are co-terminal angles.
 s( ) s and sin( ) = sin .
 s( ) sin( ) ( ) , and
sin( ) s( ) ( )
Therefore; the image of the point A(0, 3) when it is rotated about the origin through is
( ) ( )
In general, you can summarize special rotation R of ( ) centered at the origin as follow.
SPECIAL ROTATIONS ABOUT THE ORIGIN :
Let R be a counter-clockwise rotation through an angle θ about the origin. Then
1. If , then ( ) ( ).

2. If , then ( ) ( ).

3. If , then ( ) ( ).

4. If , for ∈ , then ( ) ( ).
B. Rotation when the center of rotation is the any point ( )
Let R be a rotation through an arbitrary point ( ). Then rotation R about the point ( , ) can be divided
the following three transformation.
i) Translation T of the center of rotation ( , ) by vector ( ,  ). This translation moves
( , ) to the origin (0, 0). Then the image of any point P(x, y) becomes
( ) ( ) ( )
ii) Then rotate ( ) through an angle about the origin
( ) ( ) ( s sin sin s )
= (( ) s ( ) sin ( ) sin ( ) s )
iii) Translation T of the center of rotation (0, 0) by vector ( , ). This translation moves the center of
rotation to the original position ( , ). Then the image of ( ) becomes
( ) ( ) ( )
=( ( ) s ( ) sin ( ) sin ( ) s )
This will prove the following statement.
COROLLARY 12.1: The image of a point ( ) under rotation through an angle of about the an
arbitrary point origin ( ) is given by ( ) (x y ) where
( ) s ( ) sin
( ) sin ( ) s If ( ) ( ),the rotation becomes about the origin.

Telegram channel https://t.me/johnson201485 32


1
Example 65. Find the image of the point A(2, 5) when it is rotated through about
the point ( )
Solution. Here the center of rotation is ( ) ( ) and the angle of rotation is .
If ( ) is the image of ( ) ( ) under this rotation, then
( ) s ( ) sin ( ) s ( ( )) sin
( ) sin ( ) s ( ) sin ( ( )) s
Therefore, the image of point A(2, 5) when it is rotated through about ( ) is ( ).
Example 66. Find the image of the point A(0, 0) when it is rotated through , clockwise, about the
point ( ).
Solution. , clockwise =
Here the center of rotation is ( ) ( ) and the angle of rotation is .
If ( ) is the image of ( ) ( ) under this rotation, then
( ) s ( ) sin ( ) s( ) ( ) sin
( ) sin ( ) s ( ) sin( ) ( ) s( )
Therefore, the image of point A(0, 0) when it is rotated through about ( ) is ( ).
C. Rotation of a line about a point (a, b)
To find the image of a line L under a given rotation R, follow the following steps:
1. Choose two points on the line L.
2. Find the images of the two points under the given rotation R.
Thus, the image line will be the line passing through the two image points.
Example 67. Determine the image of the line under a rotation R through an angle of
about the point ( ).
Solution:Here the center of rotation is ( ) ( ) and the angle of rotation is .

Choose two points on the line ; say ( ) and ( ).

sin and s

Then the images of A and B under this rotation are obtained as follow
Let ( ) is the image of ( ) ( ) under this rotation, then
( ) s ( ) sin ( ) s ( ) sin

( ) sin ( ) s ( ) sin ( ) s

Thus, the image of point A(0, 1) under this rotation ( ).


Let ( ) is the image of ( ) ( ) under this rotation, then
( ) s ( ) sin ( ) s ( ) sin

( ) sin ( ) s ( ) sin ( ) s

Thus, the image of point B(1, 3) under this rotation ( ).


Then, the image line passes through points ( ) and ( ).

Telegram channel https://t.me/johnson201485 32


2
Therefore, by two point form of equation of line, the equation of the image line is

 then
( ) ( )

Example 68:Find the image of the line under a rotation R through an angle of about

the point the origin.


Solution: Here the center of rotation is ( ) ( ) and the angle of rotation is .
√ √
sin and s

Points ( ) and ( ) are on the line. Then the images of A and B under R are
√ √
( ) (( )) . s sin sin s / . /

( ) (( )) . s sin sin s / ( √ )

√ √
Then, the image line passes through points . / and ( √ ).

Therefore, by two point form of equation of line, the equation of the image line is
√ √
√ √ √
√ √
 . /

√ √


C. Rotation of a circle about a point (a, b)
To find the image of a circle C under a given rotation R, follow the following steps:
1. Find the centre A and the radius r of the given circle.
2. Find the image of the centre of the circle under the given rotation.
Then the image circle has center and radius r.
Thus, the image circle will be the circle having center and radius r.
Example 69. Determine the image of the circle under a rotation R through an angle of

about the point ( ).

Solution.Here the center of rotation is ( ) ( ) and the angle of rotation is .

sin . / sin sin and s. / s s

The center of the circle is O(0, 0) and the radius of the circle is r = 2
Let ( ) be the image of ( ) ( ) under this rotation. Then
( ) s ( ) sin ( ) s( ) ( ) sin( )

Telegram channel https://t.me/johnson201485 32


3
( ) sin ( ) s ( ) sin( ) ( ) s( )

Thus, the image of center O(0, 0) under this rotation is ( ).


Thus, the image circle has center ( ) and radius r = 2.
Therefore, the equation of the image circle is ( ) ( )
( ) ( )

Example 70. Determine the image of the circle ( ) under a rotation R through an
angle of about the point ( ).

Solution: Here the center of rotation is ( ) ( ) and the angle of rotation is .



sin . / and s. /

The center of the circle is O(2, 0) and the radius of the circle is r = 1.
Let ( ) be the image of the center ( ) ( ) under this rotation. Then

( ) s ( ) sin ( ) s ( ) sin

( ) sin ( ) s ( ) sin ( ) s √ .

Thus, the image of the center O(0, 0) under this rotation is ( √ ).

Thus, the image circle has center ( √ ) and radius r = 1.

Therefore, the equation of the image circle is ( ) ( )



. / ( √ )

Example 71. Determine the image of the circle under a rotation R


through an angle of about the point ( ).
Solution:The standard form of the given circle is given by
( ) ( )
Here the center of rotation is ( ) ( ) and the angle of rotation is .
sin and s
The center of the circle is O(3, 1) and the radius of the circle is r = 3.
The image of the center O under this rotation is ( ) ( ) ( )
( ) ( )
Thus, the image of the center O(3, -1) under this rotation is ( ).
Thus, the image circle has center ( ) and radius r = 3.
Therefore, the equation of the image circle is ( ) ( )

Telegram channel https://t.me/johnson201485 32


4
( ) ( ) or

Exercise 12.8
1. Rectangle ABCD has vertices A (1, 2), B (4, 2), C (4, –1) and D (1, –1). Find the images of
the vertices of the rectangle when the axes are rotated about (0, 0) through an angle
2. Find the point into which the given point is transformed by a rotation of the axes through the
indicated angles, about the origin. a (– ) b (–2, 0); c. (–1, 2);
3. Find an equation of the line into which the line with the given equation is transformed under
a rotation through the indicated angle about the origin.
a. 3x – 4y = 7; acute angle such that t n b. 2x + y = 3;

4. Find an equation of the circle into which the circle with the given equation is transformed
under a rotation through the indicated angle, about the origin.
a. x y ;θ b. x y x y ;θ

Find the image of (1, 0) after it has been rotated −600 about the point (3, 2).
5. If M is a reflection in the line and R is a rotation about the origin through 900, find
a. M(R(3, 0)) b. R(M(0, 2)) c. R(R(1, 1))
6. In a rotation R, the image of A(6, 2) is ( ) and the image of B(7, 3) is (2, 6).
Find the image of (0, 0).
8. Find the image of (0, 1) after it has been rotated 450 about (5, 3).
9. Find an equation of the line into which the line with the given equation is transformed under a
rotation through the indicated angle about (0, 1).
a. – ; b. ;

10. Find an equation of the circle into which the circle with the given equation is transformed
under a rotation through the indicated angle, about the (1, 0).
a. ; b. ( ) ( ) ;

REVIEW EXERCISES ON UNIT 12


PART I: Workout each of the Following Questions Correctly
1. If 𝑗, 𝑗 𝑗 𝑗 and 𝑗 then find

a. ( )(v u) ( ) c. ( ) ( )

b. ( ) (v u)

Telegram channel https://t.me/johnson201485 32


5
2. If 𝑗 and 𝑗 , then
a. find ‖ ‖ and b. the unit vector in the direction of .

3. Vectors make an angle . If ‖ ‖ ‖ ‖ then find

a. ‖ ‖ b. ‖ ‖ c. ( )( )

4. Given that ‖ ‖ ‖ ‖ . Determine the value of x for which the vectors and

will be perpendicular to each other.


5. If 𝑗, then find the value of „k‟ if ‖ ‖ √ ?
6. If and ‖ ‖ ,‖ ‖ and ‖ ‖ √ , then find the angle between
.
7. From the given vectors : 𝑗, 𝑗, 𝑗, 𝑗, determine
a. the pairs of orthogonal vectors b. the pairs of parallel vectors.

8. Give two vectors , if ‖ ‖ ‖ ‖ and the angle between is , then

(i) find ‖ ‖ b. the angle between


a. Let | | = 6, | | = 10 and | | = 12, then find i) | | ii) | | iii) | |

9. ABCD is a rectangle with ‖ ‖ and ‖ ‖ , then calculate the magnitude and direction

of: a. b.
10. Find the component forms of √ ( ) and √ ( ) in , if and makes angle of

and , respectively, with the positive x- axes.

11. Find vector equation of the line that passes through the point ( ) and parallel to vector ̇
( )
12. Find the angle between a diagonal of a cube and one of its edges.
13. Given two vectors and such that 𝑗 and | | . If and are perpendicular
vectors, then the value of | | = ________
14. If ( ) and ( ), what is the unit vector in the direction of ?
15. A line given by the vector equation ( ) ( ) ∈ , is tangent to a circle at point
(1, 4). If its center is on the y-axis, then the radius of the circle is _____________
16. Using vector method, find the equation of tangent to the circle – at the point
( ).

17. If 𝑗 and is a unit vector such that | | , then the cosine of the angle

between and is equal to _____________

Telegram channel https://t.me/johnson201485 32


6
18. If is a unit vector in the direction of and | | = 4 , then is equal to __________
19. Two forces F1 and F2 with |F1| = 30N and | F2 | = 40N act on a point, if the angle between F1 and
F2 is 300, then find the magnitude of the resultant force.
20. A person pulls a body 50 m on a horizontal ground by a rope inclined at 300 to the ground. Find
the work done by the horizontal component of the tension in the rope, if the magnitude of the tension is
10 N.
21. Let be a vector with initial point (1, 5) and terminal point (4, 0). If 𝑗 is parallel to
, then what is the value of x?
22. Write the parametric vector equation of the line that satisfies the following conditions:
a. passing through the point ( ) and parallel to the vectors 𝑗

b. passing through the point ( ) and parallel to the line

c. passing through the point ( ) n ( )


23. A line given by vector equation ( ) ( ) ( ), ∈ , is tangent to a circle at a point
( ) If the radius of the circle is √ then find the center of the circle.
24. Find the parametric vector equation of the line passing through ( ) and parallel to line
.

25. Suppose that the magnitude of vectors is 24 and its direction is (from the positive x – axis).

What are the x and y components of , respectively.


26. Find a unit – vector in parallel to the line –
27. Find a unit – vector in perpendicular to the line
28. A translation T takes the point (2, 3) into point (6, 4). Then find the images of the following under
T.
a. ( )
b. The triangle with the vertices ( ) ( ) n ( )
c. The line – ( – ) ( –( – ))
d. The circle – –
e. The parabola
f. The ellipse ( ) ( )
29. Find the image of each of the following under reflection along the line .
a. ( ) b. – c. – –
30. If the image of the line under a translation is , then what is the
translation vector of the translation line?

Telegram channel https://t.me/johnson201485 32


7
31. A translation takes ( ) to ( ) . What is the image of point (1, 3)
under this transformation?
32. Find the image of each of the following under reflection along the line .
a. ( ) b. – c.
33. Find the image of the point ( ) in the mirror line – –
34. Given a circle with equation given by – . Find the equation of the
image of the circle after it has been reflected about the line – ?
35. Find the image of the circle – in the mirror line –
36. If the image of (2, 3) under reflection along the line L is (5, 1), then find the equation of L.
37. Find the image of the line – after it has been reflected along the line .
38. Let R be a rotation of a plane through an angle of 900 in ccw direction about origin. Find the
images of the following plane figures under R.
a. The point P(2, 1) c. The circle ( ) ( )
b. The line d. The hyperbola ( )
39. Find the image of an ellipse ( ) ( ) under rotation by in clockwise
direction about origin?
40. Find the image of the circle ( ) +( ) = 1 when it is rotated through about the
point ( )
41. Find the image of the line after it has been rotated through 225 about the point
( )
42. If a plane is rotated through 120 about the point (0, 2), then under this rotation find the image of
a. ( ) c. – – – e. –
b. d. –
43. If T is a translation that sends ( )t ( ) and R is a rotation about the origin through

in clockwise direction, then find


a. ( ( )) b. ( ( ))
44. If T is a translation that sends ( ) ( ) and M is a reflection that maps ( ) to
( ), then find a. ( ( )) b. ( ( ))
45. If M is a reflection in the line and R is a rotation about the origin through , then find
a. ( ( )) b. ( ( ))
46. What is the image of the line given by ( ) ( ) ( ) ∈ under a translation that
takes (1, 0) to (0, 1) followed by the reflection about the line ?

Telegram channel https://t.me/johnson201485 32


8
PRACTICE QUESTIONS ON UNIT 12

CHOOSE THE BEST ANSWER FROM THE GIVEN ALTERNATIVES


1.Suppose a triangle has vertices ( ) ( ) and ( ) on a plane. What is the interior angle of
at vertex B?
A. B. C. D.
2. Let | | , is unit vector, and makes an angle . Then | | = ______

A. √ B. √ C. √ D. √
3. What are the x and y-components of vector with magnitude 15 and direction from +ive x-
axis?

A. . / B. . √ / C. ( √ ) D. . √ /

4.Let and be two vectors such that | | and | | | |. then is ____


A. 8 B. 10 C. 12 D. 24
5. If is a vector in the direction as vector ( ) having length three times the length of ,
what is the length of , where is the unit vector in the direction of +ve y-axis?

A. √ B. √ C. D.

6. Let ABCD be a parallelogram, and and its diagonals. Which of the following is NOT
TRUE?
A. C.
B. D.
7. In the xy-plane, if ( 𝑗) ( 𝑗) ( 𝑗), then the norm of the vector is
A. 11 B. √ C. 3 D. √
8. Which of the following is the reflection of the parabola y  x 2  1 in the line y   x ?

A. y  x 2  1 B. y  x 2  1 C. x  y 2  1 D. x   y 2  1

9. Consider the circle given by x 2  y 2  2 and the line l given by parametric vector equation
x, y   2, 0  t  1, 1 . Which of the following is true?
A. l is tangent to the circle at 1 7 . C. l intersects with the circle at two distinct points.
 , 
2 2 
 

B. l and the circle have no common point. D. The distance from the center of the circle to l is 2

Telegram channel https://t.me/johnson201485 32


9
10. Let l be the line whose equation is 2 x  y  10 . Which one of the following is the equation of the

image of l after a reflection in the line y  2 x  5 followed by a rotation through the angle of 90 0 about
the origin? A. x  2 y  0 B. 2x  y  0 C. x  2 y  5 D. x  2 y  5
11. Which one of the following is a vector equation of the line tangent to the circle
x 2  y 2  2 x  7  0 at 1, 2  ?

A. x, y   0, 3    1, 2 C. x, y   0, 3   1,  1

B. x, y   1, 2   2,  1 D. x, y   1, 2    1, 2

What is the image of the ellipse 3x 2   y  1  1 under translation that takes 0, 1 to 2, 2 
2
12.
followed by the reflection through the y  axis?

A. x  22  3 y  22  1 C. x  2  3 y 2  1
2

3x  2   y  2  1 D. 3x  2  y 2  1
2 2 2
B.

13. The equation of the image of the line y  x  1 after rotation about the origin through 90 0 in
counterclockwise direction is:
A. y  x B. y   x  1 C. y  x  1 D. y   x  1

14. If the line 3x  4 y  10  0 is translated by T to 3x  4 y  27  0 and the image of point 0, 1 by

T is 3, t  , then which of the following is the value of t?


A. 3 B.  1 C.  2 D. 0
15. If a straight line passes through the point 1, 0  and the center of the circle whose equation is

x  22   y  12  4 , which of the following is the parametric vector equation of the line?
A. x, y   1, 0  t 2,  1 C. x, y   2,  1  t 1, 0

B. x, y   1, 0  t  2, 1 D. x, y   1, 0  t  1, 1

16. What is the image of the line given by x, y    1, 0  t 3, 6, t   , under the translation that

takes 1, 0  to 0, 1 followed by the reflection about the line y  2x ?


A. y  2x  3 B. y  2 x  3 C. y  2x  6 D. y  2 x  5

What is the image of the ellipse whose equation is 2x  2   y  1  2 under translation that
2 2
17.

takes 2, 1 to 4, 0 followed by rotation 90 0 ?

x2  2y2  2 B. 2 x 2  y 2  2 C. 2x  4  y  2 D. x  4  2 y  2
2 2 2 2
A.

18. The image of the point  1, 3 when reflected along the line y  x is

Telegram channel https://t.me/johnson201485 33


0
A.  3,  1 B. 1, 3 C.  3, 1 D.  1,  3

19. If two lines y  x and y  x  4 are tangent to a circle at 2, 2 and 4, 0  , respectively, then
what is the equation of the circle?

A. x  22  y 2  4 C. x  3   y  1  2
2 2

B. x  42   y  22  4 D. x  1   y  1  10
2 2

20. What is the equation of a line that passes through the point  1, 2 and parallel to the vector 1,  1
?
A. 2x  y  1 B. x  y  1  0 C. x  2 y  3 D. y  2x  1  0

21. Which of the following points is the reflection of 1,  1 on the line y  x  2 ?

A.  1, 1 B.  3, 3 C.  7,  1 D. 1, 7 
 
22. If l is the line passing through 0, 2  and parallel to V  i  3 j , which one of the following is

true about l and the circle x  2   y  1  5 ?


2 2

A. l is tangent o the circle at 0, 2  .

B. l is tangent o the circle at some point P, where P  0, 2 .


C. l intersects the circle at two distinct points.
D. The distance between l and the center of the circle is greater than 5 .
23. If the translation T takes the circle x 2  y 2  2x  6 y  3  0 into the circle whose equation is
x  22   y  72  7 , then what is the image of the origin under T?
A.  3, 7 B. 1, 2  C. 1,  3 D.  2, 4
24. If a point 2, 5 is reflected under a line to the point  3,1 , what is the line of reflection?
A. 2x  3 y  7 B. x  3 y  7 C. 8x  10 y  19 D. 2 x  3 y  5  0
What is the image of the ellipse x  1  4 y 2  1 under the translation that takes 1, 1 to 0, 2
2
25.
followed by the reflection through the x  axis?
A. x 2  4 y  1  1 B. 4 x 2   y  1  1 C. x 2  4 y  1  1 D. 4 x 2   y  1  1
2 2 2 2

26. The image of a figure with vertices A1, 2, B3, 6, C  1, 2, and D 2,  2 after reflection
across the x  axis is:
A. A' 1,  2, B'  3,  6, C ' 1,  2, and D' 2, 2
B. A'  1, 2, B'  3, 6, C ' 1,  2, and D' 2,  2
C. A' 1,  2, B' 3,  6, C '  1,  2, and D'  2, 2
D. A' 1,  2, B' 3, 6, C '  1, 2, and D'  2,  2
27. If T is a translation that takes 1, 2  to 3,  2 , then which of the following is NOT true?

Telegram channel https://t.me/johnson201485 33


1
A. T x, y   2  x, y  4 C. TT x, y   4  2 x, 2 y  8
 
B. T 1T x, y   x, y  D. T 1 x, y   x  2, y  4
28. Which of the following ordered pairs is the result of the reflection of 4, 1 with respect to the line
y  x 1 ? A.  1, 8 B.  3, 4 C. 0, 5 D.  4, 3
29. If T is a translation that takes 1, 2  to 3, 4  , then which of the following is NOT true?
A. T 5, 6  7, 8 B. T 0, 0  2, 2 C. T 1 0, 0   2,  2 D. T  3, 2   2, 4

What is the image of the circle with equation  x  2    y  2   1 , when it is rotated through
2 2
30.
6
about the origin?

     y         y   
2 2 2 2
A. x  3 1 3 1 1 C. x  3 1 3 1 1

B.  x   3  1    y   3  1  D.  x   3  1    y   3  1 
2 2 2 2
1 1
31. Which of the following is NOT true about transformation?
A. Rotation is a rigid motion. C. Rotations move triangles into congruent triangles.
B. Reflection is not a rigid motion. D. Translations move angles to congruent angles.
32. The image of the circle x 2  y 2  x  2 y  0 when it is reflected about the line L is
x 2  y 2  2x  y  0 . The equation of L is equal to:
1
A. y  x B. y  2x C. y   x D. y  x
2

Telegram channel https://t.me/johnson201485 33


2
UNIT THIRTEEN:SEQUENCE AND SERIES
Unit Outcomes:
After completing this unit, you should be able to:
 Understand sequences and series;
 Compute terms of a sequence from a given rule;
 Use the given terms to develop a formula that represent sequence;
 Identify different types of sequences and series;
 Compute partial and infinite sum of some sequences; and
 Apply the knowledge of sequence and series in real life problems.

Main Contents:
13.1. SEQUENCE
13.2. ARTHMETIC SEQUENCE AND GEOMETRIC SEQUENCE
13.3. THE SIGMA NOTATION AND PARTIAL SUMS
13.4. INFINITE SERIES
13.5. APPLICATION OF SEQUENCE AND SERIES IN DAILY LIFE

Telegram channel https://t.me/johnson201485 33


3
13.1. SEQUENCE
Definition : A sequence is a function whose domain is a set of all integers greater than or equal to a given
integer m where m is usually 0 or 1. Thus, a sequence can be denoted by * n + n . The functional
values: n are called the terms of a sequence, and n is called the general term,
or the n term of the sequence.
There are two types of sequence depending on its last term.
Finite Sequence: It is a sequence that has a last term. The domain of a finite sequence is 1, 2, 3 . . . n
Infinite sequence: a sequence that does not have a last term. The domain of an infinite sequence is
n
Example 1: List the first five terms of the sequences whose general term is n . / where n is a
positive integer:
Solution
Since n is positive integer it start from 1.
Thus . / . /

. / . /

. / . /

. / . /

. / . /
n
Therefore the first five terms of n . / are 1, - , - and
Example 2: The monthly rent of a machine, Birr 200, is to be paid at the end of each month. If it is not
paid at the end of the month, the amount due will increase Birr 3 per day. What will be the first five terms
of the amount to be paid after a delay of n days?
Solution The amount to be paid after n days delay forms the general term (in Birr) n 200 + 3n Thus
=200+3 = 203
=200+3(2) =200+6 = 206
=200+3(3) =200+9 = 209
=200+3(4) =200+12 = 212
=200+3(5) =200+15 = 215
∴ The first five terms of the amount to be paid after a delay of n days are 203, 206, 209, 2012 and 215
Fibonacci and Mulatu Sequences
 Fibonacci sequence
Fibonacci sequence is defined as follows:
i n
n { i n
n n rn
 Mulatu sequence
Professor Mulatu Lemma introduced a sequence of the form:
i n
n { i n
n n rn
Example: List the first eight terms of
a. Fibonacci sequence and b. Mulatu‟s sequences
Solution: the first eight terms are

Telegram channel https://t.me/johnson201485 33


4
a. 1, 1, 2, 3, 5, 8, 13, 21 b. 4, 1, 5, 6, 11, 17, 28, 45
13.2 ARITHMETIC AND GEOMETRIC SEQUENCES
13.2.1 Arithmetic Sequences
Definition Arithmetic sequences or arithmetic progression is a sequence in which each term except the
first is obtained by adding a fixed number (positive or negative) to the preceding term. The fixed number
is called common difference of the sequence
Example 1: For the arithmetic sequence 2, 5, 8, 11, 14… the 1st term is 2, the 3rd term is 8 and 3 is
common difference of the sequence.
The 6th term of the sequence is obtained by 5th term + 3 = 14 + 3 = 17.
Example 2: Given that the 1st term of an arithmetic progression is 10 and common difference is -4, find
the terms from 2nd to 5th term.
Solution: 2nd term = 1st term + (-4) = 10 + (-4) = 6.
3rd term = 2nd term + (-4) = 6 + (-4) = 2.
4th term = 3rd term + (-4) = 2 + (-4) = -2.
5th term = 4th term + (-4) = -2+ (-4) = -6.
Therefore, the terms from 2nd to 5th term are 6, 2, -2, and -6
General term of arithmetic sequence
Theorem: If n is an arithmetic sequence with the first term and a common difference , then the n
term of the sequence is given by n (n )
Proof: suppose * n + be arithmetic sequence.
From the definition of arithmetic sequence we know that

. . . .
. . . .
. . . .
n n (n ) (n )
Example 1: Find the general term of the sequence n , when the first term is 5 and common difference 4.
Solution: Given n
From theorem 1.1 we have n (n )
So n (n ) n n
Example 2: What is the 31st term of the sequence 1, 4, 7, 10...?
Solution: The given sequence is arithmetic sequence with n
The nth term is n (n )
Then ( ) ( )
Further on arithmetic sequence
Example 1: When the third term is 10, and the sixth term is 1,
a. Find the general term of sequence n
b. Find
Solution: a. Applying arithmetic sequence formula and substituting existing values yields:
{ Subtracting these two equations
We obtain
( )
Therefore, the general term becomes n (n )
n (n )( )
n n
n n
b. sin n n t n
( )

Example 2: Determine whether the sequences with the following general terms are arithmetic
a. n n b. n n
Solution: To solve such types of problem, we have to show the difference between successive terms is
constant
a. n n
n (n )
n
n
That is n n n ( n ) n n
Since, the difference between successive term is constant, it is arithmetic sequence.
b. n n
n (n )
(n n )
n n n n
n n n n ( n ) n n n n
Since, the difference between successive terms is not constant, it is not arithmetic sequence
Arithmetic Mean between two numbers
The term(s) of arithmetic sequence that lie between two given terms are called arithmetic mean.
Example 1: given that x is arithmetic sequence, find x
Solution: Since it is arithmetic sequence, the difference between two consecutive terms is constant.
x x
x x
x
x
Example 2: the first and sixth terms of an arithmetic sequence are 4 and 29. Find the values of four terms
between them.
Solution: Let the four terms be
So 4, , 29 form an arithmetic sequence.
Then n (n )

Thus n (n )
( )

( )
( )
( )

Telegram channel https://t.me/johnson201485


33
6
( )

13.2.2 Geometric Sequences


Definition: A geometric sequence is one in which the ratio between consecutive terms is a non-zero
constant. This constant is called the common ratio. * + is geometric sequence if and only if
∈ where is common ratio.
Example 1: For the following geometric sequence 3, 6, 12, 24, 48, … find the common ratio, , r and the
6th term.
Solution: from the definition we
Thus
And
Determining nth term of Geometric Sequence
Theorem 13.2
If * + is a geometric sequence with the first term and common ratio , then the term of the
sequence is given by .
Example 1: Find when the first term is 3, common ratio is 2.
Solution: Given and then .
( )= .
Example 1: Find the term, of the sequence1, -2, 4, -8, 16...
Solution: Given and then .
(( ) ) = ( ) .
Geometric Mean between two numbers
When are a geometric sequence, then is called the geometric mean between and .
( ) In a geometric sequence, the ratio between consecutive terms is constant.


Example: When is a geometric sequence, find ( )
Solution: As the ratio between the consecutive terms is the same,


Example: Find the term of the geometric sequence whose term is 5 and is
Solution:

( )

13.3 THE SIGMA NOTATION AND PARTIAL SUMS

Telegram channel https://t.me/johnson201485


33
7
Partial sums
Definition Let *An +∞n 1 be a sequence. The sum of the first n terms of the sequence, denoted by Sn s
called the partial sum of the sequence. Such summation is denoted as follows. Sn ∑nk 1 AK A1
A2 A3 A4 A5 An Where k is index of summation, 1 is is lower limit of summation and n
is upper limit of summation and is sigma notation or summation notation.
Example: Let an 3n 1 find S6
Solution: an 3n 1
a1 4 , a2 7 , a3 10 , a4 13 a5 16 a6 19
S6 a1 a2 a3 +a4 a5 a6 = 4+7+10+13+16+19 = 69
1
Example: Given the general term an n(n 1) find the sum of the first
a. 99 terms b. n terms
Solution: By using partial fraction decomposition
1 A B
n(n 1) n n 1
1 A(n 1) Bn
Solve for A &B n(n 1) n(n 1)
An Bn A
n(n 1)
(A B)n A 1
A 1
A B 0
1 B 0
B 1
1 1 1
Thus an
n(n 1) n n 1
1 1 1 1 1 1 1
a. S99 .1 /+. / .3 / .99 99 1/
1 1 2 2 1 3 1
1 1 1 1 1 1 1 1 99
.1 /+.2 / .3 / .99 / 1 0 99
2 3 4 100 100 100
1 1 1 1 1 1 1 1 n
b. Sn .1 2/+.2 3/ .3 4/ .n n 1 / 1 n 1 n 1
Note: Such a sequence is said to be telescoping sequence.
Sigma notation
Sigma notation is a method used to write out a long sum in concise way. We use sigma notation for
writing finite and infinite numbers of terms in a sequence.
Example: ∑8k 1 3k 3(1) 3(2) 3(3) 3(4) 3(5) 3(6) 3(7) 3(8)
3 6 9 12 15 18 21 24 108
13.3.1 Sum of Arithmetic Sequences
Definition: An arithmetic sequence (or arithmetic progression) is a sequence of numbers in which the
difference of any two successive members is a constant. This difference is called the "common
difference." The sum of the first nn terms of an arithmetic sequence can be found using the formula:
n
Sn (2a (n 1)d)
2
where:
 Sn is the sum of the first nn terms,
 a is the first term,
 d is the common difference,
 n is the number of terms.
Example: Let's find the sum of the first 5 terms of the arithmetic sequence 3, 7, 11, 15, 19.

Telegram channel https://t.me/johnson201485


33
8
Solution: Here, the first term a = 3, the common difference d=4, and n=5.
Using the formula

n
Sn (2a (n 1)d)
2
5
S5 (2x3 (5 1)4) 55
2
So, the sum of the first 5 terms of the sequence is 55
13.3.2. Sum of geometric Sequences
Definition: A geometric sequence is a series of numbers where each term after the first is found by
multiplying the previous term by a fixed, non-zero number called the common ratio. The sum of the first
nn terms of a geometric sequence is called the geometric series.
The formula to find the sum of the first nn terms of a geometric sequence is:
a(1 rn )
Sn
1 r
where:
 Sn is the sum of the first nn terms,
 a is the first term,
 r is the common difference,
 n is the number of terms.
Example:
Let's find the sum of the first 4 terms of the geometric sequence 2,6,18,54,…2, 6, 18, 54,….
1. Identify the first term (aa): a=2
2. Identify the common ratio (r): =6/2=3
3. Number of terms (n): n=4
a(1 rn)
Using the formula: Sn 1 r
2(1 34 ) 2(1 81)
S4 80
1 2 1 2
So, the sum of the first 4 terms of the geometric sequence 2,6,18,542, 6, 18, 54 is 80.
13.4. INFINITE SERIES
Definition: An infinite series is the sum of the terms of an infinite sequence. In other words, it's a sum in
which the terms continue indefinitely.
The general form of an infinite series is given by:
n
Where are the terms of the series.
Example

This is an infinite series where the terms are decreasing powers of 2.


Convergence and Divergence:An infinite series can either converge or diverge:
 If the sum of the series approaches a finite value as more and more terms are added, the series
converges.
 If the sum grows without bound or does not settle to a fixed number, the series diverges.

Sum of a Geometric Infinite Series:

Telegram channel https://t.me/johnson201485


33
9
A common example of an infinite series is a geometric series. A geometric series has the form:

r r r

where:

 a is the first term.


 r is the common ratio (each term is multiplied by r to get the next term).

Formula for the sum of an infinite geometric series:

Consider the geometric series: ∑n rn r r r


Where is the first term and is the common ratio between consecutive terms.

If | | (the common ratio is between -1 and 1), the series converges and the sum is given by:

If | | the series diverges.

SOLVED PROBLEMS ON SEQUENCE AND SERIES


1. Find the sum of the infinite geometric series:

Solution:
Here, the first term and the common ratio
Since | | , the series converges.
The sum of the infinite geometric series is: : .
Thus, the sum of the series is 6
2. Find the sum of the infinite geometric series:

Solution:
Here, the first term and Since | | the common ratio the series diverges.
Therefore, this infinite series does not have a sum, as it grows without bound.
3. Find the sum of the infinite geometric series:
Solution:
Here, the first term and the common ratio
Since | | , the series converges.
The sum of the infinite geometric series is:
Thus sum of the infinite geometric series is: :
Thus, the sum of the series is
4. Find the 20th term of the arithmetic sequence where the first term and the common difference
d=3
Solution:
In an arithmetic sequence, the general formula for the n-th term is: n (n )

Telegram channel https://t.me/johnson201485


34
0
Here and we want to find the 20th term, so
Substitute the values into the formula: ( )
So, the 20th term of the sequence is 62.
5. Find the 7th term of the geometric sequence where the first term and the common ratio .
Solution:
In a geometric sequence, the general formula for the nnn-th term is:

Here, ., and we want to find the 7th term, so n=7.


Substitute the values into the formula:
So, the 7th term of the sequence is 1458.
6. Find the 8th term of the Fibonacci sequence, where the first two terms are defined as and
, and each subsequent term is the sum of the two preceding ones.
Solution:
The Fibonacci sequence is defined as: for
With and ,, we can compute the subsequent terms:

Thus, the 8th term of the Fibonacci sequence is 13


7. Which one of the following represents a geometric sequence?
1 1 1 1 1 1 1 1
A. 3,1, , , ,... B. , , , , ,... C. 1,3,6,10,15,… D. -3,6,-9,12,-15,…
3 9 27 2 3 4 5 6

Solution: The given sequence is said to be geometric sequence if where is a


common ratio between each terms of the sequence

A. , so A is a geometric sequence.

B. , so, B is not geometric sequence.

C. , so, C is not a geometric sequence.


D. , so D is not geometric sequence.
Therefore, A is the only geometric sequence
8. The population of certain country is currently 80 million with growth rate 0f 2% per year.
Given: (0,02)9 = 5.12 x 10-16, (1.02) 9= 1.19
(0.02)10 = 1.024 x 10-17, (1.02)10 = 1.22
Which one of the following is the best approximation of the population after 10 years?
A. 81.9 million B. 86.8 million C. 95.2 million D. 97.6 million
Solution: To find the approximation of the population after 10 years we use the concept of
geometric sequence. Then
The number of populations after
i i n million =80+ million=80(1+ )million=81.6 million (1 year)

Telegram channel https://t.me/johnson201485


34
1
i i n million =80( ) million=80(1.0404)million=83.232 million
Generally, after 10 years ( ) (( ) ( ) million
Therefore, the approximation of the populations after 10 years is million the answer is D
35
9. If {an} is a sequence such that a1 = 2, and an+1 = an + 4 for all n> 1, then a
n 1
n is equal to:

A. 2460 B. 2458 C. 2450 D. 2442


Solution: To find the sum of such an arithmetic sequence we use the general formula
n
n ( (n ) ) , where n n
n ( ( ) ) ( ) ( )
Therefore, the correct answer is C

10. What is the sum of all multiples of 3 between 20 and 200?


A. 7,227 B. 6,570 C. 6,150 D. 5,166

Solution: The list of a numbers between 20 and 200 which are a multiple of 3 are 21, 24, 27, …198
Then the sequence is an arithmetic sequence with common difference is 3.
n
n ( (n ) ) , where n n , In this case to determine

The number of items, n (n ) (n ) n

n ( ( ) ) ( )

Therefore, the correct answer is B

11. If An n 1 is an arithmetic sequence such that its 1st term A1 = -5 and its 5th term A5 =15, then its 11th term A11 is
equal to:
A. 40 B. 50 C. 45 D. 55
Solution: n (n ) ( )
Then A11= A1+(11-1)d A11
Therefore the correct answer is C
12. What is the sum of all multiples of 4 that are between 30 and 301?
A. 12,882 B. 11,288 C. 6,288 D. 6,882
Solution: The list of a numbers between 30 and 301which are a multiple of 32 are 21, 24, 27, …300
Then the sequence is an arithmetic sequence with common difference is 4.
n
n ( (n ) ) , where n n , In this case to determine

The number of items, n (n ) (n ) n

n ( ( ) ) ( ) ( )
Therefore the correct answer is B
13. The nth term of the sequence: 1, -4, 9, -16, . . . is:
A. an = (-2)n B. an = (-1)n n2 C. an = (-1)2n n2 D. an = (-1)n-1 n2
Solution:

Generally: ( )
Therefore, the correct answer is D

Telegram channel https://t.me/johnson201485


34
2
n
2
14. The sum of  5 3  is
n 0
.

10
A. 0 B. 15 C. D. 5
3
n
Solution: In this sequence 2 some of the lists of terms are
 5 3 
n 0

Then this sequence is a geometric sequence with common ratio is | | it is convergent

n
Therefore 2 =15 the correct answer is B
 5 3 
n 0

15. Which one of the following is a convergent sequence?


( )
A. 2. / 3 B. 2 3 C. 2 3 D. 2 3

Telegram channel https://t.me/johnson201485


34
3
UNIT FOURTEEN: INTRODUCTION TO LINEAR PROGRAMMING
Unit Outcomes:
After completing this unit, you should be able to:
 identify regions of inequality graphs.
 create real life examples of linear programming problems using inequalities and solve them.

Main Contents:
14.1. REVISION ON LINEAR GRAPHS
14.2. GRAPHICAL SOLUTIONS OF SYSTEMS OF LINEAR INEQUALITIES
14.3. MAXIMUM AND MINIMUM VALUES
14.4. REAL LIFE LINEAR PROGRAMMING PROBLEMS
Key terms
Summary
Review Exercise

Telegram channel https://t.me/johnson201485 34


4
INRODUCTION

Many real-life problems involve finding the optimum (maximum or minimum) value of a function under certain
conditions. In particular, linear programming is a field of mathematics that deals with the problem of finding the
maximum or minimum value of a given linear function, known as the objective function, subject to certain
conditions expressed as linear inequalities known as constraints. The objective function may be profit, cost,
production capacity or any other measure of effectiveness, which is to be obtained in the best possible or optimal
manner. The constraints may be imposed by different resource limitations such as market demand, labour time,
production capacity, etc

14.1.REVISION ON LINEAR GRAPHS


Given a non-horizontal line on the xy  coordinate plane, it intersects with the x  axis at exactly one point.
The angle  measured from the x  axis to in the counter clockwise direction is called the inclination of the

line 0    180 . 
In order to determine the equation of , we pick two points P  x1 , y1  and Q  x2 , y2  on as shown in the
rise y2  y1 y1  y2
figure below. Then we define the slope m of by m   , for x1  x2 .
run x2  x1 x1  x2
Opposite side y2  y1
Since tan    , we have m  tan  .
Adjacent side x2  x1

Example 1: The slope of a line passing through the points P  3,  2  and Q  1, 3 is given by
y2  y1 3   2  5
m   .
x2  x1 1  3 4
NOTE:

1. Two non-vertical lines 1 and 2 with slopes m1 and m2 , respectively, are parallel if and only if they have the
same slope; i.e. m1  m2 .
2. Two lines 1 and 2 with slopes m1 and m2 , respectively, are perpendicular if and only if m1  m2  1 .
An equation of a line is an equation in two variables x and y such that a point P  x, y  is on if and only if
x and y satisfy the equation.
Recall that if a line has slope m and passes through a point P  x1 , y1  , then the point slope form of equation of
is given by y  y1  m  x  x1 
If the line passes through  0, 0  , its equation is y  mx .
If the y  intercept of a line with slope m is  0, b  , then its equation in the slope-intercept form is y  mx  b
2   3 23
Example 2: If a line passes through P 1,  3  and Q  2, 2  , then its slope m   5.
2 1 1
Its equation in slope-intercept form is
y  2  5  x  2  5x 10 or y  2  5x  8  slope 5, y  intercept  0,  8  .

Telegram channel https://t.me/johnson201485 34


5
Example 3: Suppose 1 and 2 are perpendicular lines intersecting at  2, 1 . If the angle of inclination of 2 is
45 , what is the equation of 1 ?
Solution:
Slope of 2 is m2  tan 45  1 . Since 1 and 2 are perpendicular lines, the slope of 1 is m1  1 .
Then, the equation of 1 is
y  y1  m1  x  x1   y  1    x  2 , because 1 passes through  2, 1
 y  x  2 1  x  3
Therefore equation of 1 : y  x  3
Example 4: Line passes through  0, 5 and  5, 0  . Find the inclination angle in radian measure.
05
Solution: The slope of , m 1
5  0

Inclination angle  tan   m  1    tan 1 1 
4

Therefore, the inclination angle is .
4
3
Example 5: Find the equation of the line passes through  0, 1 with angle of inclination of .
4
 3 
Solution: The slope of , m  tan   1
 4 
Then, the equation of the line 1 is y  y1  m1  x  x1   y  1  1 x  0  y  x 1

Exercise 14.1
1. Determine the equation of the line
a. that has slope 4 and passes through P  1, 3 . D that passes through the points P 1, 2  and Q  4, 1

b. whose slope is 2 with y  intercept  0, 5 . E. passes through 1,  1 ; parallel to y  


3
x  3.
4
c. passes through 1,  2  ; perpendicular to  x  2 y  2 .f. passes through  5, 2  ; perpendicular to
5x  y  3 .
2. Find the equation of each line given below.
a. c.

b. d.

Telegram channel https://t.me/johnson201485 34


6
3. Determine the value of k k so that the line with equation 4 x  ky  8 is parallel to the line with equation
x  2y  0 .
4. Draw the graphs of the following lines using the same coordinate axes.
a. y  2x 1 b. y  2x  3 c. 3x  2 y  4

14.2.GRAPHICAL SOLUTIONS OF SYSTEMS OF LINEAR


INEQUALITIES
In this section, you use graphs to determine the solution set of a system of linear inequalities in two variables.
Every line : ax  by  c in the plane divides the plane into two regions, one on each side of the line. Each of these
regions is called a half plane. A vertical line : x  a divides the plane into left and right half planes. A point
 x, y  is to the left of the line x  a , if and only if,
x  a . Hence the graph of the inequality x  a is the half
plane lying to the left of the line x  a . Similarly, the graph of the inequality x  a a is the half plane lying to the
right of the line x  a .
Example 1: Let be the vertical line x  2 .

Observe that in a the left half plane x  2 contains the points on the line x  2 and hence the line is a bold
(unbroken) line; whereas in b the right half plane x  2 does not include the points on the line x  2 (broken
line).
A non-vertical line divides the plane into two regions which can be called upper and lower half planes.

Example 2: Consider the graph of the linear equation 2x  y  3 and the related linear inequalities 2x  y  3 and
2x  y  3 . First graph the line 2x  y  3 by plotting two points on the line. To identify which half plane
belongs to which inequality, test a point that does not lie on the line (usually the origin).

Test  0, 0  ;2  0   0  0  3 . Observe the broken line for 2x  y  3 and solid line for 2x  y  3 .
A system of linear inequalities is a collection of two or more linear inequalities to be solved simultaneously. A
graphical solution of a system of linear inequalities is the graph of all ordered pairs  x, y  that satisfy all the
inequalities. Such a graph is called the solution region (or feasible region).

Telegram channel https://t.me/johnson201485 34


7
Example 3: Find a graphical solution to the system of linear inequalities.
x  y  3

2 x  y  0
Solution: First draw the lines x  y  3 and 2x  y  0 by plotting two points for each line. Then shade the
regions for the two inequalities. The solution region is the intersection of the two regions. To find the point of
x  y  3
intersection of the two lines, solve  simultaneously, to get the point 1, 2  .
2 x  y  0

Example 4: Draw the solution region of the system of linear inequalities.


y  2x  4

2 x  y  4
Solution: Draw the two lines 1 : y  2x  4 and 2 : 2 x  y  4 and identify their point of intersection P  0, 4 .
The solution region, which is the intersection of the two half planes, is shaded in the following figure.

Example 5: Draw the solution region of the system of the following linear inequalities.
 2
 y   3 x  1  2
 y   x 1
a.  b.  3
y  1 x  2  x  3
 3
Solution:
2 1
a. Draw the two lines 1 :y x  1 and 2 : y  x  2 and identify their point of intersection
3 3
P  3,  1 . The solution region, which is the intersection of the two half planes, is shaded in the following
figure.

Telegram channel https://t.me/johnson201485 34


8
x  1 and 2 : x  3 and identify their point of intersection P  3,  1 .
2
b. Draw the two lines 1 :y
3
The solution region, which is the intersection of the two half planes, is shaded in the following figure.

Definition 14.1: A point of intersection of two or more boundary lines of a solution region is called a vertex (or
a corner point) of the region.
Example 6: Solve the following system of linear inequalities.
2 x  y  22 
x  y  13 

2 x  5 y  50 
x0 

y  0 
Solution: The last two inequalities x  0 and y  0 are known as non-negative inequalities (or non-negative
requirements). They indicate that the solution region is in the first quadrant of the plane.
Draw the lines 1 : 2 x  y  22, 2 : x  y  13 and 3 : 2 x  5 y  50 .
To determine the solution region test the point O  0, 0  which is not in any of these 3 lines, and find the
intersection of all half planes to get the shaded region in the following figure.

This solution region has five corner points. The vertices O  0, 0 , P  0, 10  and Q 11, 0 can be easily
determined. To find the other two vertices R and S solve simultaneously the following two pairs of equations:
1 : 2 x  y  22  2 : x  y  13 
 and 
2 : x  y  13  3 : 2 x  5 y  50 

to get S  9, 4  to get R  5, 8
Observe that the point of intersection of 1 and 3 is not a corner point of the solution region.
Definition 14.2: A solution region of a system of linear inequalities is said to be bounded, if it can be enclosed
by a rectangle, otherwise it is unbounded. Thus the solution region of Example 6 is bounded, while that of
Example 4 is unbounded.

Telegram channel https://t.me/johnson201485 34


9
Example 7: Find the linear inequalities for which the shaded region in the given figure is the solution set.

Solution:
i. Consider 2 x  3 y  3 . We observe that the shaded region and the origin lie on opposite side of this line
and  0, 0  , satisfies 2 x  3 y  3 . Therefore, we must have 2 x  3 y  3 as linear inequality corresponding
to the line 2 x  3 y  3 .
ii. Consider 3x  4 y  18 We observe that the shaded region and the origin lie on the same side of this line
and  0, 0  satisfies 3x  4 y  18 . Therefore, 3x  4 y  18 is the linear inequality corresponding to the
line 3x  4 y  18 .
iii. Consider 7 x  4 y  14 . It is clear from the figure that the shaded region and the origin lie on the same
side of this line and  0, 0  satisfies the inequality 7 x  4 y  14 .
iv. Consider x  6 y  3 . It may be noted that the shaded portion and origin lie on the same side of this line
and  0, 0  satisfies x  6 y  3 .
v. Also the shaded region lies in the first quadrant only. Therefore, x  0, y  0 .
Hence, in view of (i), (ii), (iii), (iv) and (v) above, the linear inequalities corresponding to the given
2 x  3 y  3
3 x  4 y  18

solution set are :  7 x  4 y  14
x  6 y  3

 x, y  0

Exercise 14.2
1. Find a graphical solution for each of the following.
x  y  2 x  0  x, y  0
 x  3 y  6  x  3 y  6 x  y  2 y  0 2 x  3 y  60
  
a.  b.  c.  d.  e. 
4 x  3 y  3  4 x  y  1 x  2 y  8 x  y  8 2 x  y  28
 x  4 3 x  5 y  30 4 x  y  48
2. Find the linear inequalities for which the shaded region in the given figure is the solution set.
a. b.

Telegram channel https://t.me/johnson201485 35


0
2 x  y  8
3. Show that the solution set of the following system of linear inequalities is an unbounded region  x  2 y  10 .


x  0

y  0
4. Show that the following system of linear inequalities has no solution x  2 y  3 .
3 x  4 y  12


x  0

y 1

14.3.MAXIMUM AND MINIMUM VALUES


Many applications in business and economics involve a process called optimization, in which you are asked to find
the maximum or minimum value of a quantity. In this section you will study an optimization strategy called linear
programming.
Definition 14.3: Suppose f is a function with domain I  x | a  x  b

i. A number M  f  c  for some c in I is called the maximum value of f on I, if M  f  x  ,for all x in I.


ii. A number m  f  d  for some d in I is called the minimum value of f on I, if m  f  x  ,for all x in I.
iii. A value which is either a maximum or a minimum is called an optimum (or extremum) value of f on I.
Many optimization problems involve maximizing or minimizing a linear function (the objective function) subject
to one or more linear equations or inequalities (constraints).
In this section, problems with only two variables are going to be considered since such problems can easily be
solved by a graphical method.
Example 1: Find the values of x and y which will maximize the value of the objective function
x0
Z  f  x, y   2 x  5 y , subject to the linear constraint: y0
3x  2 y  6
2 x  4 y  8
Solution: First you sketch the graphical solution S of the given constraints using the methods of Section 10.2.
This bounded region S is also called the region of feasible solution or feasible region.
Any point in the interior or on the boundary of S satisfies all the above constraints. Next you find a point  x, y  of
the feasible region that gives the maximum value of the objective function Z . Let's first draw some lines which
represent the objective function for values of Z  0, 5, 10 and 15 ; i.e., the lines
2x  5 y  0 2 x  5 y  10
2x  5 y  5 2 x  5 y  15

From the above figure you can observe that as the value of Z increases, the lines are moving upwards and the line
for Z  15 is outside the feasible region. The maximum possible value of Z will be obtained if we draw a line
between Z  10 and Z  15 parallel to them that just "touches" the feasible region.

Telegram channel https://t.me/johnson201485 35


1
This occurs at the vertex (corner point) P which is the point of intersection of the lines
3x  2 y  6 1 9
  x  and y 
2 x  4 y  8 2 4
The value of Z at this point is: 1  9  49 1 .
Z  2x  5 y  2    5     12
2 4 4 4
Thus the maximum value of Z under the given conditions is Z  12
1 .
4
As a generalization of this example, we state the following:
Theorem 14.1: Fundamental theorem of linear programming
If the feasible region of a linear programming problem is non-empty and bounded, then the objective function
attains both a maximum and a minimum value and those occur at corner points of the feasible region. If the
feasible region is unbounded, then the objective function may or may not attain a maximum or minimum value;
however, if it attains a maximum or minimum value, it does so at corner points.

Steps to solve a linear programming problem by the graphical method


1. Draw the graph of the feasible region.
2. Compute the coordinates of the corner points.
3. Substitute the coordinates of the corner points into the objective function to see which gives the optimal value.
4. If the feasible region is unbounded, this method is misleading: optimal solutions always exist when the feasible
region is bounded, but may or may not exist when it is unbounded.
1 9
To apply this to Example 1, we find the vertex points  0, 0  ,  2, 0  ,  ,  and  0, 2  and test their values as
2 4
shown in the following table.

49 1 9
Comparing the values of Z , you get the maximum value of Z  obtained at  ,  .
4 2 4
We also have the minimum value Z  0 at  0, 0  .
Example 2: Use the graphical method to solve the following linear programming problem.
x  y  3
Maximize R  4 x  11y Subject to : 2 x  y  4 .

x  0
 y  0
Solution: We need to graph the system of inequalities to produce the feasible set. We will start by rewriting each
inequality as an equation, and then number the equation for each line.
The x  and y  intercepts for line x  y  3 are  3, 0  and  0, 3 , respectively. Since the inequality x  y  3
contains an equal sign, a solid line can be drawn through those two intercepts. We need to choose a test point to
substitute into the original inequality to determine which half-plane to shade. We will choose the point  0, 0  :

 x y 3
00  3
03
The point  0, 0  satisfies the inequality, so we will shade the half-plane containing  0, 0  .

Telegram channel https://t.me/johnson201485 35


2
The x  and y  intercepts for line 2 x  y  4 are  2, 0  and  0, 4  , respectively. Since the inequality 2 x  y  4
contains an equal sign, a solid line can be drawn through those two intercepts. We need to choose a test point to
substitute into the original inequality to determine which half-plane to shade. We will choose the point
 0, 0  0  4 . The point  0, 0 satisfies the inequality, so we will shade the half-plane containing  0, 0 .
The inequalities x  0 and y  0 together represent the first quadrant, so Quadrant I should be shaded.
The feasible set, shown below, is where all shaded regions intersect, along with the solid boundary of the shaded
region.

We can see from the diagram that the feasible set is bounded, so this problem will have an optimal solution.
Next, we need to find the vertices (corner points) of the feasible set. By observing the graph we see that the
vertices are  0, 0 ,  0, 3 , 1, 2  and  2, 0  .
Finally, we substitute each corner point into the objective function to determine the optimal solution.

The maximum value is 33 and it occurs at  0, 3 .


Example 3: Use the graphical method to solve the following linear programming problem.
5 x  y  5
Minimize S  2x  7 y Subject to : x  3y  9 .


x  0

y  0
Solution: We need to graph the system of inequalities to produce the feasible set. We will start by rewriting each
inequality as an equation, and then number the equation for each line.
The x  and y  intercepts for line 5x  y  5 are 1, 0  and  0, 5 , respectively. Since the inequality 5x  y  5
contains an equal sign, a solid line can be drawn through those two intercepts. We need to choose a test point to
substitute into the original inequality to determine which half-plane to shade. We will choose the point
 0, 0  0  5 . The point  0, 0 does not satisfies the inequality, so we will shade the half-plane does not
contains  0, 0  .
The point  0, 0  satisfies the inequality, so we will shade the half-plane containing  0, 0  .

The x  and y  intercepts for line x  3 y  9 are  9, 0  and  0, 3 , respectively. Since the inequality x  3 y  9
contains an equal sign, a solid line can be drawn through those two intercepts. We need to choose a test point to
substitute into the original inequality to determine which half-plane to shade. We will choose the point
 0, 0  0  9 . The point  0, 0  does not satisfies the inequality, so we will shade the half-plane does not
contains  0, 0  .

Telegram channel https://t.me/johnson201485 35


3
The inequalities x  0 and y  0 together represent the first quadrant, so Quadrant I should be shaded.
The feasible set, shown below, is where all shaded regions intersect, along with the solid boundary of the shaded
region.

 3 20 
The vertices of the feasible set are  0, 5  ,  ,  and  9, 0 
7 7 
.Next, we substitute each point into the objective function to determine the optimal solution.

The minimum value appears to be 18, which occurs at  9, 0  . Since the feasible set is unbounded, this may or may
not represent the minimum. Let us test some points in the feasible set near the suspected optimal solution of  9, 0 
For test point 8, 1 : S  2 x  7 y  2 8  7 1  23
For test point  9, 1 : S  2 x  7 y  2  9   7 1  25
For test point 10, 0 : S  2 x  7 y  2 10  7  0  20
The above is not a formal proof since we have only chosen a few points, but we could go on testing and would not
find any points in the feasible set which yield a function value less than 18. The minimum value is 18, which
occurs at  9, 0  .
Example 4: Solve the following linear programming problem. Find the maximum value of the objective function
x  2 y  4
x  y  1

Z  3x  2 y , subject to the following constraints:  .
x  0
 y  0
Solution: From the constraints you sketch the feasible region shown in the following figure. The vertices of this
region are  0, 0 , 1, 0  ,  2, 1 and  0, 2  .

Telegram channel https://t.me/johnson201485 35


4
Their functional values at Z are given in the following table:

Thus, the maximum value of Z is 8, and occurs when x  2 and y  1.


Example 5: Solve the following linear programming problem. Find the maximum value of the objective function
x0
y0
Z  4 x  6 y , subject to the following constraints:  x  y  11
x  y  27
2 x  5 y  90
Solution: From the constraints you sketch the feasible region shown in the following figure. The vertices of this
region are  0, 0 ,  27, 0 , 15, 12 , 5, 16  and  0, 11 .

Testing the objective function at the vertices gives

Thus, the maximum value of Z is 132 , and occurs when x  15 and y  12 .


Example 6: Solve the following linear programming problem. Find the minimum values of the objective function
x  2 y  8
T  5x  6 y , subject to the following constraints: 
6 x  4 y  36
0  x  7

Solution: From the constraints you sketch the feasible region shown in the following figure. The vertices of this
region are  0, 9  ,  5, 3  and  7, 1  .
   
 2  2

Telegram channel https://t.me/johnson201485 35


5
Testing the objective function at the vertices gives

3
Thus, the maximum value of Z is 34 , and occurs when x  5 and y  .
2
x  2 y  8
Example 7: Maximize Z  2 x  5 y subject to:  .
6 x  4 y  36
0  x  7

Solution: The feasible region is illustrated in the following figure. Since it is unbounded, we are not assured by
Theorem 14.1 that the objective function attains a maximum value. In fact, it is easily seen that since the feasible
region contains points for which both x and y are arbitrarily large and positive, the objective function can be
made arbitrarily large and positive. This problem has no optimal solution. Instead, we say the problem has an
unbounded solution.

xample 8: Find the values x and y that minimize Z  2 x  y subject to: 2 x  3 y  12 .



2 x  3 y  0
 x, y  0

Solution: In the following figure, we have drawn the feasible region of this problem. Because one of the
constraints is an equality constraint, the feasible region is a straight line segment with two extreme points. The
values of Z at the two extreme points are given in the following table.

Thus the minimum value of Z  2 x  y is 4 attained at  3, 2  .


Example 9: Find the values x and y which minimize the value of the objective function S  2x  4 y
x0
y0
subject to: .
x  2 y  10
3 x  y  10
Solution: From the given constraints the feasible region S is as shown in the figure below.

Telegram channel https://t.me/johnson201485 35


6
This region S is unbounded. The vertices are at  0, 10 ,  2, 4  and 10, 0  with values given below.

Here vertices  2, 4  and 10, 0 give the minimum value S  20 so that the solution is not unique. In fact every
point on the line segment through  2, 4 and 10, 0 gives the same minimum value of S  20 . From this
example we can observe that
i. an optimization problem can have infinite solutions.
ii. not all optimization problems have a solution, since the above problem does not have a maximum value for Z.

Exercise 14.3
Find the maximum and minimum values of
a. Z  2x  3 y , x0 x0
x0 y0 y0
y0 subject to: x  2y  4 subject to: 2 x  3 y  6
subject to:
2 y  x  16 3x  y  7 3x  2 y  9
x  y  10 x  2 y  7 x  5 y  20
b. Z  2x  3 y , d. Z  4 x  5 y , f. Z  3x  4 y ,
subject to: subject to: x0
x0 x0 y0
y0 y0
subject to: x  2 y  14
3 x  7 y  42 2 x  2 y  10 3x  y  0
x  5 y  22 x  2y  6 x y 2
c. Z  4x  2 y , e. Z  4x  3 y ,

14.4.REAL LIFE LINEAR PROGRAMMING PROBLEMS


In everyday life, we are often confronted with a need to allocate limited resources to best advantage. We may
want to maximize an objective function (such as profit) or minimize (say, cost) under some restrictions (which we
called constraints).
Despite the apparently quite restrictive nature of the Linear Programming problem format there are many practical
problems in industry, government and other organization which fall into this type. Below we give real life
examples of simple linear Programming problems, each of which represents a classic type of Linear Programming
problem.

Telegram channel https://t.me/johnson201485 35


7
Example 1: A manufacturer wants to maximize the profit for two products. Product I gives a profit of Birr 1.50
per kg, and product II gives a profit of Birr 2.00 per kg. Market tests and available resources have indicated the
following constraints.
a. The combined production level should not exceed 1200 kg per month.
b. The demand for product II is not more than half the demand for product I.
c. The production level of product I is less than or equal to 600 kg plus three times the production level of
product II.
Find the number of kg of each product that should be produced in a month to maximize profit.
Solution: The first step in solving such real life Linear Programming problems is to assign variables to the
numbers to be determined for a maximum (or a minimum) value of the objective function.
Let x  the number of kg of product I,
y  the number of kg of product II
These variables are usually called decision variables. The objective of the manufacturer is to decide how many
units of each must be produced to maximize the objective function (profit) given by:
P  1.5x  2 y
The above three constraints can be translated into the following linear inequalities
a. x  y  1200
1
b. y  x or  x  2 y  0
2
c. x  3 y  600 or x  3 y  600
Since neither x nor y can be negative, we have the additional non-negativity constraints of x  0 and y  0 . The
above information can now be transformed into the following linear programming problem.
x0
y0
Maximize P  1.5x  2 y subject to x  y  1200
 x  2y  0
x  3 y  600
The constraints above have region of feasible solutions shown in the figure below.

To solve the maximization problem geometrically, we first find the vertices by finding the points of intersection
of the border lines of S, to get O  0, 0 , A  600, 0  , B 1050, 150  and C 800, 400  .
Then a solution can be obtained from the table below:

35
Telegram channel https://t.me/johnson201485 8
Thus the maximum profit is Birr 2000 and it occurs when the monthly production consists of 800 units of product
I and 400 units of product II. (Observe that the minimum profit is Birr 0 which occurs at the vertex O  0, 0  .
Example 2: A manufacturer of tents makes a standard model and an expedition model for national distribution.
Each standard tent requires 1 labour-hour from the cutting department and 3 labour-hours from the assembly
department. Each expedition tent requires 2 labour-hours from cutting and 4 labour hours from assembly. The
maximum labour-hours available per day in the cutting department and the assembly department are 32 and 84,
respectively. If the company makes a profit of Birr 50.00 on each standard tent and Birr 80 on each expedition
tent, how many tents of each type should be manufactured each day to maximize the total daily profit? (Assume
that all tents produced can be sold.)
Solution: The information given in the problem can be summarized in the following table.

Then we assign decision variables as follows: Let x  number of standard tents produced per day
y  number of expedition tents produced per day
The objective of management is to decide how many of each tent should be produced each day in order to
maximize profit P  50 x  80 y . Both cutting and assembly departments have time constraints given by
1 x  2  y  32 .................. .cutting dept. constraint
3 x  4  y  84 .................. assembly dept. constraint
where x  0 and y  0................. non-negative constraints
x  2 y  32
The Linear Programming problem is then to maximize P  50 x  80 y , subject to : 3 x  4 y  84
x, y  0
To get a graphical solution, we have the feasible region S shown in the figure below. The vertices are at
 0, 0 ,  28, 0 ,  20, 6 and  0, 16 . The maximum value of profit can be obtained from the following table.

Thus the maximum profit of Birr 1,480 is attained at (20, 6); i.e. the manufacturer should produce 20 standard and
6 expedition tents each day to maximize profit.
Example 3: A patient in a hospital is required to have at least 84 units of drug A and 120 units of drug B each
day. Each gram of substance M contains 10 units of drug A and 8 units of drug B, and each gram of substance N
contains 2 units of drug A and 4 units of drug B. Suppose both substances M and N contain an undesirable drug
C, 3 units per gram in M and 1 unit per gram in N. How many grams of each substance M and N should be mixed
to meet the minimum daily requirements and at the same time minimize the intake of drug C? How many units of
drug C will be in this mixture?
Solution: Let us summarize the above information as:

35
Telegram channel https://t.me/johnson201485 9
Let x  number of grams of substance M used
y  number of grams of substance N used
Our objective is to minimize drug C from 3x  y . The constraints are the minimum requirements of
10x  2 y  84 ................ from drug A
and 10x  2 y  84 ................ from drug B
Since both M and N must be non- negative x  0, y  0 .
our optimization problem is to
10 x  2 y  84
Minimize C  3x  y subject to: 8 x  4 y  120
x, y  0
The sketch of the feasible region S is given in the figure below.

To obtain the minimum value graphically, we use the table

The minimum intake of drug C is 34 units and it is attained at an intake of 4 grams of substance M and 22 grams
of substance N.
We can summarize the steps in solving real life optimization problems geometrically as follows.
Step 1: Summarize the relevant information in the problem in table form.
Step 2: Form a mathematical model of the problem by introducing decision variables and expressing the objective
function and the constraints using these variables.
Step 3: Graph the feasible region and find the corner points.
Step 4: Construct a table of the values of the objective function at each vertex.
Step 5: Determine the optimal value(s) from the table.
Step 6: Interpret the optimal solution(s) in terms of the original real life problem.

Exercise 14.4
Solve each of the following real life problems:
a. A farmer has Birr 1,700 to buy sheep and goats. Suppose the unit price of sheep is Birr 300 and the unit
price of goats is Birr 200.
i. If he decided to buy only goats, what is the maximum number of goats he can buy?
ii. If he has bought 2 sheep what is the maximum number of goats he can buy with the remaining
money?
iii. Can the farmer buy 4 sheep and 3 goats? 2 sheep and 5 goats? 3 sheep and 4 goats?

36
Telegram channel https://t.me/johnson201485 0
b. A company produces two types of tables; Tables A and Table B. It takes 2 hours of cutting time and 4
hours of assembling to produce Table A. It takes 10 hours of cutting time and 3 hours of assembling to
produce Table B. The company has at most 112 hours of cutting labour and 54 hours of assembly labour
per week. The company's profit is Birr 60 for each Table A produced and Birr 170 for each Table B
produced. How many of each type of table should the company produce in order to maximize profit?
The officers of a high school senior class are planning to rent buses and vans for a class trip. Each bus can
transport 36 students, requires 4 supervisors and costs Birr 1000 to rent. Each van can transport 6 students,
requires 1 supervisor, and costs Birr150 to rent. The officers must plan to accommodate at least 420 students.
Since only 48 parents have volunteered to serve as supervisors, the officers must plan to use at most 48
supervisors. How many vehicles of each type should the officers rent in order to minimize the transportation
costs? What is the minimum transportation cost?
c. A calculator company produces a scientific calculator and a graphing calculator. Long-term projections
indicate an expected demand of at least 100 scientific and 80 graphing calculators each day. Because of
limitations on production capacity, no more than 200 scientific and 170 graphing calculators can be made
daily. To satisfy a shipping contract, a total of at least 200 calculators much be shipped each day. If each
scientific calculator sold results in a Birr 2 loss, but each graphing calculator produces a Birr 5 profit, how
many of each type should be made daily to maximize net profits?

1. The angle of inclination of a line L is the angle  measured from the x  axis to L in the counter clock wise
direction.
y2  y1
2. The slope of a line passing through P  x1 , y1  and Q  x2 , y2  is m  tan   , for x1  x2 .
x2  x1
3. If a line has slope m and passes through P  x1 , y1  , the slope-point form of its equation is given by
y  y1  m  x  x1  .
4. An equation of a line can be reduced to the form ax  by  c, a, b, c  with a  0 or b  0 .
5. A line divides the plane into two half-planes.
6. A system of linear inequalities is a collection of two or more linear inequalities to be solved simultaneously.
7. A graphical solution is the collection of all points that satisfy the system of linear inequalities.
8. A vertex (or corner point) of a solution region is a point of intersection of two or more boundary lines.
9. A solution region is said to bounded, if it can be enclosed in a rectangle.
10. A number M  f  c  for c in I is called the maximum value of f on I, if M  f  x  for all x in I.
11. A number m  f  d  for d in I is called the minimum value of f on I, if m  f  x  for all x in I.
12. A value which is either a maximum or a minimum value is called an optimal (or extreme) value.
13. An optimization problem involves maximizing or minimizing an objective function subject to constraints.
14. If an optimal value of an objective function exists, it will occur at one or more of the corner points of the
feasible region.
15. In solving real life linear programming problems, assign variables called decision variables.

36
Telegram channel https://t.me/johnson201485 1
REVIEW EXERCISES ON UNIT 14

1. Find the slope of the line


a. that passes through the points P  4,  1 and Q  0, 2  .
b. that has angle of inclination   45 . B. that is parallel to the line 2 x  4 y  6 .
2. Draw the graphs of the lines 1 : y  3x  4 and 2 : x  5 y  2 using the same coordinate axes.
2. Find graphical solutions for each of the following systems of linear inequalities.
x  5y  2 y  2x  4 x2 x0
a. b.
3x  y  4 y  2x  4 c. y0 d. y0
x y 5 3x  2 y  6
3. Find the maximum and minimum values of the objective function subject to the given constraints.
a. Maximize Z  3x  2 y, d. Minimize Z  3x  4 y,
subject to:
x0
x 1
y0
x  3 y  15 y0
4 x  y  16 subject to:
3 x  4 y  12
b. Maximize Z  2 x  3 y,
x0
x  2y  4
subject to: y  0 e. Maximize Z  13x  11y,
2 x  y  100 x0
x  2 y  80 subject to: y0
4 x  5 y  1500
c. Maximize Z  2x  4 y,
5 x  3 y  1575
x0 x  2 y  420
y0
subject to:
x  5 y  10
4x  y  8

4. Find the optimal solution of the following real life linear programming problems.
Ahadu company produces two models of radios. Model A requires 20min of work on assembly line I and 10
min of work on assembly line II. Model B requires 10 min of work on assembly line I and 15 min of work on
assembly line II. At most 22 hrs of assembly time on line I and 25 hrs of assembly time on line II are available per
week. It is anticipated that Ahadu company will realize a profit of Birr 10 on model A and Birr 14 on model B.
How many radios of each model should be produced per week in order to maximize Ahadu's profit?
a. A farming cooperative mixes two brands of cattle feed. Brand X costs Birr 25 per bag and contains 2
units of nutritional element A, 2 units of nutritional element B, and 2 units of element C. Brand Y costs
Birr 20 per bag and contains 1 unit of nutritional element A, 9 units of element B, and 3 units of element
C. The minimum requirements of nutrients A, B and C are 12, 36 and 24 units, respectively. Find the
number of bags of each brand that should be mixed to produce a mixture having a minimum cost.
5. In laying out a car park it is decided, in the hope of making the best use of the available parking space
 7200 sq. ft  , to have some spaces for small cars, the rest for large cars. For each small space 90sq. ft. is
allowed, for each large space 120sq. ft . Every car must occupy a space of the appropriate size. It is reliably
estimated that of the cars wishing to park at any given time, the ratio of small to large will neither less than
2:3 nor greater than 2 :1 . Find the number of spaces of each type in order to maximize the number of cars
that can be parked.

36
Telegram channel https://t.me/johnson201485 2
PRACTICE QUESTIONS ON UNIT 14
CHOOSE THE BEST ANSWER FROM THE GIVEN ALTERNATIVES
1. Which of the following point is not lie in the half-plane 2 x  3 y 12  0 ?
A.  2, 3 B.  2, 3 C.  2, 1 D. 1, 2 
x  2 y  120 
x  y  60 

2. The maximum value of the object function Z  5x  10 y , subject to the constraints x  2 y  0  is equal
x  0

y  0 
to:
A. 800 B. 300 C. 600 D. 400
x  4 y  24
3 x  y  21
3. Maximize Z  5x  10 y, , subject to: .
x y 9
x, y  0
a. 20 at 1, 0  B. 30 at  0, 6  C. 37 at  4, 5 D. 33 at  6, 3
2. The region represented by 2 x  3 y  5  0 and 4 x  3 y  2  0 is:
A. Not in first quadrant C. Bounded in first quadrant
B. Unbounded in first quadrant D. It is in second quadrant
3. A shopkeeper wants to purchase two articles A and B of cost price Birr 4 and 3 respectively. He thought that
he may earn 30 praise by selling article A and 10 praise by selling article B. He has not to purchase total
article worth more than Birr 24. If he purchases the number of articles of A and B, x and y respectively,
then linear constraints are:
A. x  0, y  0, 4x  3 y  24 C. x  0, y  0, 30 x  40 y  24
B. x  0, y  0, 30x  10 y  24 D. x  0, y  0, 4 x  3 y  24
4. A factory produces two products A and B. In the manufacturing of product A, the machine and the carpenter
requires 3 hour each and in manufacturing of product B, the machine and carpenter requires 5 hour and 3 hour
respectively. The machine and carpenter work at most 80 hour and 50 hour per week respectively. The profit
on A and B is Birr 6 and 8 respectively. If profit is maximum by manufacturing x and y units of A and B
type product respectively, then find the constraints for the function 6 x  8 y
A. x  0, y  0, 5x  3 y  80, 3x  2 y  50 C. x  0, y  0, 5x  3 y  80, 3x  2 y  50
B. x  0, y  0, 3x  5 y  80, 2 x  3 y  50 D. x  0, y  0, 3x  5 y  80, 3x  3 y  50
5. Which one of the following is the vertex of a feasible region by the linear constraints
3x  4 y  18, 2x  3 y  3 and x, y  0 ?
A.  0, 3 B.  0, 2  C.  4.8, 0 D.  0, 5
6. A toy manufacturer produces two types of dolls; a basic version doll A and a deluxe version doll B. Each doll
of type B takes twice as long to produce as one doll of type A. The company have time to make a maximum
of 2000 dolls of type A per day, the supply of plastic is sufficient to produce 1500 dolls per day and each type
requires equal amount of it. The deluxe version, i.e. type B requires a fancy dress of which there are only 600
per day available. If the company makes a profit of Birr 3 and Birr 5 per doll, respectively, on doll A and B,
then the number of each should be produced per day in order to maximize profit, is equal to:
A. 800, 500 B. 500, 600 C. 450, 450 D. 1000, 500

36
Telegram channel https://t.me/johnson201485 3
7. The corner point of the feasible region determined by the system of linear constraints are
 0, 0 ,  0, 40 ,  20, 40 ,  60, 20 ,  60, 0 . The objective function is Z  4x  3 y .
Compare the quantity in column A and column B

Column A Column B
Maximum of Z 325
A. The quantity in column A is greater
B. The two quantities are equal
C. The quantity in column B is greater
D. The relationship cannot be determined based on the given information.
8. A feasible solution to linear programming:
A. Must be a corner point of feasible region.
B. Must optimize the value of the objective function.
C. Need not satisfy all of the constraints, only some of them.
D. Must satisfy all of the problem‟s constraints simultaneously.
9. Which of the following represent the shaded region?

A. 2x  5 y  80, x  y  20, x  0, y  0 C. 2x  5 y  80, x  y  20, x  0, y  0


B. 2x  5 y  80, x  y  20, x  0, y  0 D. 2x  5 y  80, x  y  20, x  0, y  0
10. The sum of two positive numbers is at most 5 . The difference between two times a second number and the
first number is at most 4 . If the first number is x and second number is y , then which of the following is the
mathematical formulation for maximizing the product of these two numbers?
A. x  y  5, 2 y  x  4, x  0, y  0 C. x  y  5, 2 y  x  4, x  0, y  0
B. x  y  5,  2x  y  4, x  0, y  0 D. x  y  5, 2x  y  4, x  0, y  0
11. Find the solution of the set of constraints x  2 y  11, 3x  4 y  30, 2 x  5 y  30, x  0, y  0 .
A.  2, 3 B.  3, 2  C.  3, 4  D.  4, 3
12. The maximum and minimum values of the objective function Z  x  2 y subject to constraints
x  2 y  100
2x  y  0
occur respectively at:
2 x  y  200
x, y  0
A. one point and three points. C. one point and infinitely points.
B. two points and one point. D. one point and one point.
13. Which of the following is true about the constraint of a linear optimizing function Z  x1  x2 , given by
x1  x2  1, 3x1  x2  3, and x1 , x2  0 ?
A. There are two feasible regions. C. There is no feasible region.
B. There are infinite feasible regions. D. There are three feasible regions.

36
Telegram channel https://t.me/johnson201485 4
14. In Mathematics test there are multiple choice and workout part questions to be answered. The relevant data is
given in the following table:
Type of questions Time taken to solve Marks Number of questions
Multiple choice 5 minute 3 10
Workout 10 minute 5 14
The total mark is 100. Students can solve all questions. To secure maximum marks, a student solves x multiple
choice and y workout questions in three hours, then linear constraints except x, y  0 are:
A. 5x  10 y  180, x  10, y  14 C. x  10 y  180, x  10, y  14
B. 5x  10 y  180, x  10, y  14 D. 5x  10 y  180, x  10, y  14
15. What is the maximum value of the function Z  3x  y subject to: 2 x  3 y  1, x  y  3 , for x  0 and
y0?
9 3
A. 7 B. 9 C. D.
2 2
3x  y  6
x  y 1
16. Consider the following linear programming model: objective function Z  3x  5 y , subject to .
x  2y  6
x  0, y  0
Which of the following is the minimum value of Z ?
A. 5 C. It has no minimum value
B. 3 D. 0
17. Consider a line with equation y  mx  b . Which of the following is true?
A. The line intersects the x  axis, if m  0 and b  0 . C. The line passes through the origin.
B. The line intersects the y  axis, if m  0 and b  0 . D. The line has no slope.
18. A problem to be solved using linear programming method is given as follows:
“ A coffee packer blends coffee from Harar and from Sidama to prepare two kinds products, viz., “Super”
and “Deluxe” brands. Each kilogram of Super coffee contains 0.5kg of Harar coffee and 0.5kg of Sidama
coffee, whereas each kilogram of Deluxe coffee contains 0.25kg of Harar coffee and 0.75kg of Sidama
coffee. The packer has 120kg of Harar coffee and 160kg of Sidama coffee in store. Moreover, the plan is to
get 20 Birr profit on each kilogram of Super coffee and a profit of 30 Birr on each kilogram of Deluxe coffee.
In order to maximize profit the packer needs to know the amount (in kg) of each brand of coffee that should
be blended.”
If x  the amount of Super coffee in kilograms and
y  the amount of Deluxe coffee in kilograms, then which one of the following standard forms of linear
programming models should be used to solve the above problem?
A. Maximize P  30x  20 y C. Maximize P  20x  30 y
0.5 x  0.75 y  120 0.75 x  0.25 y  120
subject to: 0.5 x  0.25 y  160 subject to: 0.5 x  0.5 y  160
x  0, y  0 x  0, y  0
B. Maximize P  20x  30 y D. Maximize P  30x  20 y
0.5 x  0.25 y  120 0.5 x  0.5 y  120
subject to: 0.5 x  0.75 y  160 subject to: 0.25 x  0.75 y  160
x  0, y  0 x  0, y  0

Telegram channel https://t.me/johnson201485 36


5
UNIT FIFTEEN:MATHEMATICAL APPLICATIONS IN BUSINESS
Unit Outcomes:
After completing this unit, you should be able to:
 know common terms related to business.
 know basic concepts in business.
 apply mathematical principles and theories to practical situations.

Main Contents:
15.1. BASIC MATHEMATICAL CONCEPTS IN BUSINESS
15.2. COMPOUND INTEREST AND DEPRECIATION
15.3. SAVING, INVESTING, AND BORROWING MONEY
15.4. TAXATION
Key terms
Summary
Review Exercises

Telegram channel https://t.me/johnson201485


366
INTRODUCTION
The application of mathematics in business involves simple arithmetic operations to compute the profit or loss, or
to determine the corresponding percentage. Whenever people are engaged in business directly or indirectly, they
are involved with financial institutions such as bank, insurance, etc.
To this end, this unit deals with the concepts of basic mathematics in business, compound interest and
depreciation, saving, investing and borrowing money, and taxation.

15.1. BASIC MATHEMATICAL CONCEPTS IN BUSINESS


The concepts of ratio, rate, proportion and percentage are widely used whenever we deal with business in our
daily live activities. Hence, we will look at each of these concepts and their applications in this section.

A. Ratio
Consider the explanation given below :
Suppose the number of students and teachers in a given school are 3900 and 75, respectively. From this we can
make the statement that
i) “the ratio of teachers to students in the school is 1 to 52” or
ii) we can say that “the ratio of students to teachers in the school is 52 to 1”.
This tells us that for every 52 students in the school there corresponds one teacher.

DEFINITION 15.1
A RATIO is a comparison of two quantities that are measured in the same unit.

If we compare and , the ratio can be written as or or ; read as “the ratio of ”

The numbers appearing in a ratio are called TERMS of the ratio and they must be expressed in the same unit of
measurement.
For instance, if the ratio of boys to children is 2 to 5, then this is to mean that there are two boys for every 5
children. So, if there are 50 children in attendance, then there are 20 boys.
Thus, the ratio of boys to children is written as 2 : 5. This ratio is part-to-whole ratio.
The terms of this ratio are 2 and 5.
Ratios can be written in three ways:
 A ratio can be expressed in one of two ways:
i. part-to-whole ratio or ii. part-to-part ratio
Example 1. Suppose there are 20 students in a class, of which 12 are girls, as shown in the figure below.

Determine
a. Ratio of number of girls to number of boys and ratio of boys to girls.
b. Ratio of number of girls to number of whole students in the class

Telegram channel https://t.me/johnson201485


367
Solution.
a. In the class there are 12 girls and 20  12 = 8 boys. Therefore, ratio of number of girls
to number of boys is given by 3 : 2, which is the simplest form of .
In a similar manner, ratio of number of girls to number of boys is given by 2 : 3, which is
the simplest form of . This ratio is an example of part-to-whole ratio.
b. In the class there are 12 girls out of 20 students. Therefore, ratio of number of girls
to the whole number of students is given by 3 : 5, which is the simplest form of .
This ratio is an example of part-to-whole ratio of the students.

Example 2. The following table gives the number of teachers in a given school according to their education
level and sex.

a. What is the ratio of female diploma holders to the number of teachers in the school?
b. What is the ratio of male degree holder to all degree holder teachers in the school?
c. What is the ratio of diploma holders to degree holders in the school?
Solution:
a. The first question is asking the part-to-whole ratio, hence it is 16:100 or 4:25.
b. The second question is asking the part-to-whole ratio of male teachers, hence it is 46:72 or 23:36.
c. The third question is asking the part-to-part ratio, hence it is 42:58 or 21:29.
Example 3. What is the ratio of 1.6 meters to 180 centimetres?
Solution: To compare two measurements in different units you must change one of the units of measurement to
the other unit.
If you change 1.6 meters to centimeters, we have conversion rate 1 m = 100 cm. hence,
1.6 meters = 1.6  100 cm = 160 cm.

Therefore the ratio or 8 : 9.

REMARK. People commonly form a group and involve on a given business activity according to their
individual contribution for the business. In this case, their individual profit is allocated according to the ratio of their
investment.

Example 4. Allocate Birr 25,000 to four persons in the ratio 3 : 4 : 8 : 5.


Solution. Note that the terms in the ratio are positive integers. First, you need to
determine the total number of parts to be allocated.
That is 3 + 4 + 8 + 5 = 20

Telegram channel https://t.me/johnson201485


368
Then divide the total amount by the total parts to obtain the value of each single part : Birr 1,250 per part.

To allocate, multiply each term of the ratio by the value of the single part, i.e.
3 × 1,250 = Birr 3,750; 4 × 1,250 = Birr 5,000;
8 × 1,250 = Birr 10,000 and 5 × 1,250 = Birr 6,250.
Therefore, the allocation will be Birr 3,750, Birr 5,000, Birr 10,000 and Birr 6,250, respectively.

Example 5. A sum of money was divided between Aster, Ali, and Mesfin in the ratio ,

respectively. Aster has received Birr 3504. How much money was there to start with?

Solution: First write the terms of the ratio as the common denominator. Thus the common denominator for the
ratio is L.C.M(5, 3, 1) = 15.

Hence the ratio can be expressed as .

Therefore, the total part to be allocated is 6 + 20 + 30 = 56 parts.

From aster amount we can obtain the total amount as:

 TOTAL AMOUNT =

Exercise 15.1
1. A profit of Birr 19,560 is to be divided between four partners in the ratio of 3 : 2 : 1 : 6. How
much should each receive?
2. Allocate Birr 800 among three workers in the ratio of

B. Rate
Frequently we want to compare two different types of measurements, such as miles to gallons and and money per
hours. To make this comparison, we use a rate. For instance, to make a beam or a column of residential building,
cement, sand and gravel are mixed in the ratio , respectively. In this case cement is measured in quintals while
sand and gravel are measured using a cubic meter box. Hence the ratio involves different units of
measurement and this will lead us to the following definitions.

DEFINITION 11.2: A rate is a comparison of two or more quantities expressed in different units of
measurement.
Like ratios we usually write rates as fractions. We put the first given in the numerator and the second amount in the
denominator. When rates are simplified, the units remain in the numerator and denominator.

Example 6. The distance from Addis Ababa to Adama is 100 km. Ahmed travelled by minibus from Addis
Ababa to Adama early in the morning and it took him 1 hour and 20 minutes. What is the rate of speed of his
journey?

Solution. : The rate of speed of his journey is the ratio of the distance travelled and the time it took. Since the
distance is 100 km and the time taken is , the rate is:

Telegram channel https://t.me/johnson201485


369
REMARK. A special type of rate is called a UNIT RATE. A unit rate is a rate where the denominator is a 1.
Unit rates allow us to see relationships better.
For instance, per minute means per 1 minute

Example 7. A wild dog can run 30 miles in 1 hour. How many feet is this per second?
Solution. You need to convert to

Since = 5280 feet and 1 hour = 3600 sec., we have

In dealing with business, production, population, and so on, it is common to describe by what amount a quantity has
increased or decreased based on some starting or fixed level. This will lead us to the rate of change of a given
quantity given by the relation:

The rate of change will be a rate of increase if the amount of change is positive and a rate of decrease if the amount
of change is negative.

Example 8. The price of a quintal of cement in Addis Ababa in September 2008 was Birr 220, and ten months
later, on July 2009, its price was Birr 370. What is the rate of increase in the price of one quintal of cement from
September 2008 to July 2009?

Solution. We are given that: the original price = Birr 220 and the new price = Birr 370.
Hence change in price = Birr 370 – Birr 220 = Birr 150

0.682

Example 9. Aster has invested 20,000 Birr in a fruit wholesaler. A year later the audit report on the business
indicated that there was 16,200 Birr as a balance. Find the rate of decrease that resulted in one year.

Solution. Since the balance indicated that there is a decrease from the amount of capital invested, we have a
( )
decrease rate. 0.19

The negative sign indicates that there is a decrease in the investment which is a loss.

Exercise 15.2
1. A carpenter‟s daily production of school chairs increased from 20 units to 40 units. At the same time his income
(or revenue) increased from 1600 Birr to 2400 Birr. What is the rate of change of income per unit?
2. A steel company has imported 35 tons of raw material from South Africa in 1995. In 2008 the company
imported 54 tons of raw material from the same country. What is the rate of change of amount imported?
C. Proportion
DEFINITION 15.3

Telegram channel https://t.me/johnson201485


370
A PROPORTION is a statement of equality between two ratios.
For ∈ , with n , one way of denoting a proportion is a : b = c : d,

which is read as “a is to b” as “c is to d”. Of course, by definition, , which means that „a proportion is an


equation between two ratios”.
In the proportion a : b = c : d , with n , the four numbers n are referred as the TERMS of
the proportions. The first and the last terms a and d are called the
EXTREMES; the second and third terms b and c are called the MEANS.

In the proportion a : b = c : d , the product of the extremes is equal to the product of the means; that is, is
equivalently represented as a × d = b × c .

For three quantities a, b and c such that which is equivalent to b2 = a × c , b is called the MEAN
PROPORTIONAL between a and c .
Proportions and rates allow us to solve many applications.

Example 10. You are making cookies. A recipe calls for 29 grams of sugar and makes 2 dozen cookies. You
want to make 6 dozen cookies for a week. How much grams of sugar you will need?

Solution. We can use a proportion to figure out how much grams of sugar you will need.
Our first step is to write the rate 29 grams of sugar for 2 dozen cookies in fraction form.

You want to make 6 dozen cookies but do not know how much sugar we need. We will use this information to write
a proportion.

Notice the units are lined up. Both rates have grams in the numerator and dozens in the denominator. If we think of
these rates as equivalent fractions, we have

Hence, your total earnings for the week are 87 grams

Example 11. A secretarial pool (15 secretaries in all) on one floor of a corporate complex has access to 11
telephones. If on a different floor, there are 23 secretaries, approximately what number of telephones should be
available?

Solution Let x be the number of telephones available on the other floor. Then we
have the proportion 15 : 11 = 23 : x, that is,

Hence,

Therefore, 17 telephones are required.


From the above discussion you have seen how change in one variable quantity depends on a change in another
variable quantity (i.e., simple proportion). However, the value of a variable quantity most often depends on the
value of two or more other variable quantities. For example,

Telegram channel https://t.me/johnson201485


371
 The cost of sheet metal depends on the area of the sheet, thickness of the sheet, and the cost per unit area of the
metal.
 The amount of interest obtained on the money deposited in a bank depends on the amount of money deposited
in a bank, length of time it is deposited, and rate of interest per year.
DEFINITION 15.4
A COMPOUND PROPORTION is a situation in which one variable quantity depends on two or more other
variable quantities. Specifically, if a variable quantity y is proportional to the product of two or more variable
quantities, we say that y is JOINTLY PROPORTIONAL to these variable quantities, or y VARIES JOINTLY as
these variables.
If z is jointly proportional to x and y (or z is proportional to x and y), then in short we write it as z xy . Its
equivalent representation in terms of an equation is z = kxy, where k is a constant of proportionality.
Note that in a compound proportion, a proportion combination of direct and/or inverse variation may occur. If z is
directly proportional to x and inversely proportional to y, then we can write it as or equivalently

where k is a constant of proportionality.

Example 12. If z is proportional to y and to the square of x and z = 80 when x = 2 and y = 5, then find the
equation that relates the variables x, y and z.
Solution. We are given that which is equivalent to , where k is a constant of proportionality.
To determine the constant of proportionality, put the given values of the variables in .
z = 80 when x = 2 and y = 5 implies that ( )( ) this gives
Therefore the equation that relates the three variables is .
Example 13. The power (P) of an electric current varies jointly as the resistance (R) and the square of the
current (I). Given that the power is 12 watts when the current is 0.5 amperes and the resistance is 40 ohms, find the
power if the current is 2 amperes and the resistance is 20 ohms
Solution. , that is, , where k is a constant of proportionality. Putting the given values in the
equation and solving for k, we have ( )( )
 .
( )( )
Hence the relationship between the three variables is and the required power is
( )( ) watts.
Exercise 15.3
1. The ratio of the lengths of corresponding sides of two similar decagons is 1: 2. If the perimeter of the
smaller decagon is 76 cm, what is the perimeter of the larger decagon?
2. A cookie recipe requires 4 cups of flour to make 5 dozen cookies. If Amy needs to make 15 dozen cookies,
how many cups of flour will she need?
3. The president of the student body estimated that 2 out of every 3 students at school would attend the Spring
Festival. If there are 1,140 students at this school, according to the estimate, how many students will not
attend the Spring Festival?

C. Percentage
The concept of percentage is an extension of the material we have covered about fractions and ratio. To allow easy
comparisons of fractions we need to use the same denominator. As such, percentages use 100 as the denominator. In
other words, the „whole‟ is divided into 100 equal parts. The word “per cent” means per 100. Therefore 27% is .

Telegram channel https://t.me/johnson201485


372
To use percentage in a calculation, the simple mathematical procedure is to write the percentage as a fraction with a
denominator of 100.

For instance, 25% is . Thus to calculate 25% of 40, we could approach it as

Percentages are most commonly used to compare parts of an original.


For instance, the phrase „30% off sale!‟ indicates that whatever the original price, the new price will be 30% less.

DEFINITION 15.5 A PERCENTAGE is the numerator of a fraction whose denominator is 100. The term
percent is denoted by % which means “per one hundred”.

Example 14. Express each of the following fractions as percentages.


a. b. c.
Solution. First express the given fractions as decimal numbers and multiply by 100%.
a. Hence,
b. . Hence,
c. Since , we have
When percentages are involved in computations, the corresponding decimal representation is usually used.
Percentage is obtained by multiplying a number called the BASE by the percent, called the RATE.
Percentage = base × rate
Example 15. Find a. 3% of Birr 57? b. of Birr 900?
Solution
a. To find 3% of Birr 57, the base is 57 and the rate is 3% = 0.03, then
Percentage = base × rate = Birr
b. To find of Birr 900, the base is 900 and the rate is . Then
of Birr 900 = 900 = Birr 51.75.
Example 16.
a. What is the total amount whose 15% is 120?
b. Birr 62.50 is what percent of Birr 25,000?
Solution
a. Here we are looking for the total amount, whose percentage is 120 and the rate is 0.15.
Therefore, from percentage = base × rate
n
we have s units
b. Here the base is Birr 25, 000 and the percentage is Birr 62.50. it is to find the rate.
Since, percentage = base × rate
n
Hence, the rate is r t
Example 16. If the value added tax (VAT) on sales is 15%, find the VAT on a sale of
refrigerator that costs Birr 3,800. What is the total cost of the refrigerator?
Solution. The rate is 0.15 and the base is Birr 3,800, hence the percentage would be
Percentage = base × rate = 3,800 × 0.15 = Birr 570.
That is the VAT on the refrigerator is Birr 570.

Telegram channel https://t.me/johnson201485


373
The total cost of the refrigerator = cost + VAT
= Birr 3,800 + Birr 570
= Birr 4,370.00

Commercial Discount
In business activities, it is common to offer a sales discount due to clearance of available stock, changing the
business activity, approaching expiry date, and so on. In such cases the discount of an item is described in terms of
percentage. For example, you may have 20% discount, 30% discount, and so on.
If p is the original price of an item and r is the percentage of discount, then the amount of discount is given by:

Therefore, the sales price will be given by:


Discount sales price = Original price − Discount ( )
Example 17. A wool suit, discounted by 30% for a clearance sale, has a price tag of Birr 399. What was the
suit‟s original price? What is the amount of discount?
Solution. Let p be the original price of the suit. The amount of discount is 30% of p = 0.30 p.
Hence,
discount sales price = Original price − Discount ( )
 Birr 399
 Birr 570
Therefore, the original price is p = Birr 570 and
the amount of discount is 570 – 399 = Birr 171
Exercise 15.4
1. From 250 candidates who sat for a written examination for a job, 45 of them scored above 85%. The personnel
division suggested that those candidates who have scored above 85% in the written examination could sit for
interview. What percent of the candidates did not have a chance for interview?
2. A car dealer, at a year-end clearance, reduces the price of last year‟s models by a certain amount. If a certain
four-door model has been sold at a discounted price of Birr 51,000, with a discount of Birr 9,000, what is the
percentage of discount?

Markup: In order to make a profit, any institution or company must sell its products for more than the
product costs the company to make or buy. The difference between a product selling price and its cost is called
MARKUP.
Markup = Selling price – Cost price
Example 18. If the price of cement is Birr 250 per quintal and you sell it for Birr 330 per quintal, find the
markup per quintal.
Solution Markup = Selling price – Cost price
= Birr 330 per quintal – Birr 250 per quintal = Birr 80 per quintal
REMARK. Markup is usually expressed in terms of percentage with respect to selling price
and cost price.
i) Markup with respect to selling price is given by

ii) Markup with respect to cost price is given by

Telegram channel https://t.me/johnson201485


374
Example 19. : If you buy a gold ring for 498 Birr and sell it for 750 Birr, find the markup percent
a. with respect to selling price. b. with respect to cost.
Solution: Markup = Selling price – Cost price = Birr 750 – Birr 498 = Birr 252.
a. The markup percent with respect to the selling price is:
r u r nt
irr
b. The markup percent with respect to the cost price is:
r u r nt
irr
Example 20. A boutique buys a T-shirt for Birr 54.25 and wants a markup of 30% on retail. What is the selling
price?
Solution. Markup on retail means markup on selling price
Given cost = Birr 54.25.
Markup percent on selling price = 30%
We need to find selling price.
Cost = selling price – markup = 100% – 30% = 70 % of selling price.
This is called the complement of markup percent on selling price.
Hence, the selling price will be:
cost = 0.70 × selling price
54.25 Birr = 0.70 × selling price
 s in ri
 s in ri irr 77.50
REMARK.
In business, it is often necessary to make conversion between percent markups based on
cost and selling price.
i. To convert markup percent based on cost to markup percent based on selling price, use the
following relation:
n n
r u r nt n s in ri n ( n )
n n
n n
ii. To convert markup percent based on selling price to markup percent based on cost price, use
the following relation:
n n n
r u r nt n st ri ( n n )
n n
n n n
Example 21. What is the percent markup on selling price if the markup on cost is 25%?
Solution. Since we are given the markup on cost = 25%, we use the relation
r u r nt n st ri
r u r nt n s in ri
s in ri ( s r nt st)
n n
n n

Telegram channel https://t.me/johnson201485


375
Exercise 15.5
1. A pair of shoes costs Birr 110 and sells for Birr 155. Find the markup and the markup percent based on the
retail (selling price).
2. What is the percent markup on cost, if the markup on retail is 37%?
3. If W/ro Chaltu purchased a gallon of oil at Birr 258 and sold it at Birr 288, find
a. markup b. markup percent with respect to
4. Ato Dechassa wants to sell his ox at Birr 3,652 with a 10% markup on his cost. Find the cost of the ox.
5. Martha bought a shoe for Birr 280 and wants to sell it at 24% markup. Find
a. markup b. selling price of the shoe
6. Abebe sold a quintal of Teff at Birr 1,068 with 45% markup on selling price. Find the cost.
7. Find the percent markup on cost, if mark - up on selling price is 30%.
15.2. COMPOUND INTEREST AND DEPRECIATION
When money is deposited in a bank for some time, its total amount will increase in the future due to interest. On the
other hand, the value of an asset decreases or depreciates due to several reasons as time goes on. These two
concepts will be introduced in this sub-unit.
Simple Interest
When money is borrowed, or you deposit money in an account, a fee is paid for the use of the money. A fee paid for
the use of money is called INTEREST. From the investment point of view, interest is income from invested capital.
The capital originally invested is called the PRINCIPAL (OR PRESENT VALUE). The sum of the principal and
interest due (or paid) is called the AMOUNT (OR FUTURE VALUE OR ACCUMULATED VALUE).
For simple interest, the interest is computed on the original principal during the whole time, or term of the loan; at
the stated annual rate of interest. The computation of simple interest is based on the following formula:
Simple interest:
Where I is the simple interest, P is the principal, r is the interest rate per year or annual interest rate, and t is the
time in years.
Note:
The time period for r and t must be consistent with each other. That is, if r is expressed
as percentage per year, then t should be expressed in number of years.
To compute the future value of a simple interest, we use the formula:
The future value of a simple interest:
If a principal P is borrowed at a rate r of simple interest per year for t years, then the borrower will pay back the
lender an amount A given by
( )
Example 22. If Birr 2,500 is invested with a simple interest rate of 2% per month, find the amount of the
interest and future value at the end of the fourth month.
Solution. It is given that : the principal P = Birr 2,500, the interest rate per month r = 0.02 , and the time t =
4months. Hence,
where r is the interest rate per periods that is interest rate per month.
I = Prt = 2500 × 0.02 × 4 = Birr 200.
The value of the investment after four months (the future value) is
A = P + I = 2,500 + 200 = Birr 2,700.
Example 23. Zenebech wants to buy an electric stove priced at Birr 2,500 and agreed to pay Birr 700 initially
and the remaining amount to be equally paid monthly on simple interest rate of 13% per year in 9 months (i.e. the
remaining amount plus its interest). What is the monthly payment she has to do?

Telegram channel https://t.me/johnson201485


376
Solution. The amount of loan = Birr 2,500 – Birr 700 = Birr 1,800.
Hence, the Principal will be 1,800 Birr, interest rate r = 0.13 , time years, and
the number of times payment is made is times,
where m is the number of times payment is made per year.
( ) . /
Therefore, the periodic payment is Birr 219.50

15.2.1 Compound Interest


Simple interest is only based on the principal amount of a loan, while compound interest is based on the principal
and accumulated interest.
If at the end of a payment period the interest due is reinvested at the same rate, the interest as well as the original
principal will earn interest during the next payment period. Interest paid on interest reinvested is called
COMPOUND INTEREST.
If P is the principal earning interest compounded annually at a rate of r per year for n years, then the amount at the
end of one year can be calculated from the simple interest relation
A = P(1+ rt)
Let the amount accumulated at the end of nth year. Then
The amount at the end of the first year 1 A (i.e., when t = 1) is
( ) ( )
Since the amount at the end of the first year will serve as principal for the second year,
at the end of the second year the amount 2 A will be
( ) ( )( ) ( )
Since the amount at the end of second year will serve as principal for the third year, at
the end of the third year the amount 3 A will be
( ) ( ) ( ) ( )
th
Continuing this process, we see that the amount at the end of the year will be
( ) ( ) ( ) ( )
Therefore, if P Birr is invested at a rate of r per year compounded annually, then the total amount A accumulated
after n years will be given by ( ) …………………………….. ( i )
Example 24. Find the amount of interest on a deposit of Birr 1,000 in an account compounded annually with
annual interest rate of 6% for 5 years.
Solution. We are given P = Birr 1,000, r = 0.06 , t = 5 years and we need to find the future value A and then the
amount of interest. A = P(1 + r)n = 1,000(1 + 0.06)5 = 1,000(1.06)5 = Birr 1,338.23.
Hence the amount of the compound interest of the deposit is I = A − P = 1,338.23 – 1,000.00 = Birr 3,38.23.
REMARK. Interest is usually compounded more than once a year. The quoted rate of interest per year is called
ANNUAL or NOMINAL RATE and the interval of time between successive interest calculations is called
CONVERSION PERIOD or COMPOUND PERIOD.
If interest at an annual rate of r per year is compounded m times a year on a principal P,
then the simple interest rate per conversion period is
Since r is the annual interest rate and the interest is compounded m times per year, the year is divided into m
equal conversion periods and the interest rate during each conversion period is
that is, we get interest in every years.
Therefore, if P Birr is invested at a rate of r per year compounded m times in a year, then the total amount A
accumulated after t years will be given by . / …………………………….. ( ii)

Telegram channel https://t.me/johnson201485


377
where A is amount or future value, P is principal or present value, r is annual or nominal
rate, t is time in years, and m is the number of conversion periods per year.
In working with problems involving interest, we use the term of payment periods as follows:
 Annually means once a year, i. e. m = 1
 Semi-annually means twice a year, i. e. m = 2
 Quarterly means four times a year, i. e. m = 4and
 Monthly means 12 times a year, i. e. m = 12.
Example 25. If Birr 100 is deposited in the Commercial Bank of Ethiopia with interest rate of 10% per annum,
find the amount if it is compounded
a) Annualy b) semi-annually, c) quarterly and d) weekly
at the end of one year. (No withdrawal or deposit is made in the whole year).
Solution. You are given the principal P = Birr 100, the annual interest rate r = 0.1, for a period of time t = 1year,
and compound period of n = mt :
a. Annually means m = 1, so that the amount at the end of the year is
( )
. / . / ( ) Birr
b. Semi-annually means m = 2, so that the amount at the end of the year is
( )
. / . / Birr
c. Quarterly means m = 4, so that the amount at the end of the year is
( )
. / . / Birr
d. Weekly means m = 52, so that the amount at the end of the year is
( )
. / . / Birr
From the above example, you can observe that when the time, principal and interest rates are kept fixed and the
number of times the interest is compounded increases, the amount will increase.
Example 26. Suppose Birr 2,300 is invested at 8% interest rate compounded
a. annually b. monthly.
What is the amount after 5 years? Find the amount of interest in each case.
Solution Given P = Birr 2,300, r = 0.08, and t = 5 years.
a. When the interest is compounded annually, m = 1and hence the amount will be
( )
. / ( ) ( ) irr
The interest earned in five years without making withdrawal or deposit will be
I = A – P = 3,379.45 – 2,300 = Birr 1,079.45.
b. When the interest is compounded monthly, m = 12 and hence the amount A will be
( )
. / ( ) ( ) irr
The interest earned in five years without making withdrawal or deposit will be
I = A – P = 3,427.33 – 2,300 = Birr 1,127.33.
Example 27. Find the present value of an investment that will grow to Birr 600 after two years compounded
quarterly at the interest rate of 9% per year.
Solution. The given information is A = Birr 600, t = 2 years, m = 4, and r = 0.09 .
We want to find the present value P. The present value is given by
From the future amount formula . / , dividing both sides by . / gives us

Telegram channel https://t.me/johnson201485


378
. /
Therefore, the future value P of the investment is
( )
. / . / ( ) irr

Ordinary annuity
Many people are not in a position to deposit a large amount of money at a time in an account. Most people save
money by depositing relatively small amount at different times. If a depositor makes equal deposits at regular
intervals, he/she is contributing to an ANNUITY. The deposits may be made weekly, monthly, yearly, or any other
period of time.
If we deal with annuities in which the deposits (or payment) are made at the end of each of the deposit (or payment)
intervals, which coincides with the compounding period of interest, then this type of annuity is called ORDINARY
ANNUITY.
Suppose a person deposit an amount R at the end of each of an „m‟ deposit (or payment) interval periods, in an
account that pays interest per year compounded m-times a year over t years. Then he made a total of
deposits, one at the end of each interest payment period over t years.
The total future value will be calculated as follow:
The last deposit (the deposit) has no interest, so stays at Birr R. Thus,
The deposit has interest calculated for only one period, and it will accumulate to
( ) ( )
where R is the periodic payment and is the interest rate per period.
The payment has interest computed for two periods, and it will accumulate for the first period A = R (1 + i)
and for the second period as the amount for the first period serve as a principal to the second period
( ) ( ) .
The payment has interest computed for three periods, and it will accumulate for the first period A = R (1 + i)
and for the second period as the amount for the first period serve as a principal to the second period
( ) ( ) .
Continuing this process the first payment has interest computed for periods, and will accumulate to the
amount ( )
Thus, the amount of the ordinary annuity will be the sum S of the amount accumulated from each deposit made, that
is,
 ( ) ( ) ( ) ( ) ……………….. (*)
Multiplying both sides of equation (*) by ( ), we obtain
 ( ) ( ) ( ) ( ) ( ) ( ) …….. (**)
Subtract (*) from (**) and simplifying the difference give us ( ) ( )
 ,( ) -
,( ) -
This implies that
,( ) -
Thus, the future value S of an ordinary annuity is given by
where R is the periodic payment, is the interest rate per period and n is the number of periods.
The amount of interest of an ordinary annuity is .
In the above ordinary annuity formula, and , where which is the interest rate per year, m is the
number of times interest is compounded per year and t is time in years.

Telegram channel https://t.me/johnson201485


379
Example 28. Elizabeth deposits Birr 350 at the end of every month into a saving account that pays an interest
rate of 12% per year compounded monthly.
a. How much money is in her account at the end of 5 years? b. What is the amount of interest?
Solution. You are given R = Birr 350, r = 0.12, m = 12, and t = 5 years.
To use the above formula we need to find and n = mt =12(5) = 60.
a. The accumulated balance at the end of 5 years is given by
,( ) - ,( ) -
Birr 28,584.38
b. The amount of interest is I = S – nR = 28,584.38 – 60(350) = Birr 7,584.38.
Exercise 15.6
1. If Ato Abebe deposits a sum of money in a bank at 7.5% interest rate per year compounded monthly, then how
long will it take to double?
2. Ato Lemma works in XYZ-company earning a monthly salary of Birr 2,400. He is also a member of the credit
association of his company and deposits 20% of his monthly salary at the end of each month at 4% compounded
monthly.
a. What is Ato Lemma‟s accumulated balance by the end of three years?
b. How much interest has he earned?
3. If Dalelo deposited Birr 1,000 saving at 7% interest per year, how much will the amount be at the end of 10th
year?
4. Helen deposited Birr 2,000 at 8% interest compound annually. How many years will it take her to get Birr
3,000?
5. Suppose you deposit Birr 100 in an account at the end of every quarter with 8% interest compounded quarterly.
How much amount will you have at the end of 5 years?
6. An amount of Birr 500 is deposited in an account at the end of each six-month period with an interest computed
at 6% compounded semi-annually. How many years does it take for the amount to reach Birr 56,398.43?
15.2.2 Depreciation
Suppose a year ago your business purchased a new truck for Birr 30,000 to haul materials to various construction
sites. If your company were to sell that truck today, could the company sell it for Birr 30,000? The answer is no.
The truck is worth less today than it was a year ago due to wear and tear, the age of the equipment, and
obsolescence. This loss in equipment value over time is known as depreciation.
Any physical thing (tangible) or right (intangible such as, patents, copyrights and goodwill) that has money value is
an ASSET. There are two groups of assets known as CURRENT ASSETS (FINANCIAL ASSETS) and PLANT
ASSETS (or FIXED ASSETS).
Cash and other assets that may reasonably be expected to be recognized in cash or sold or consumed within one
year or less through the normal operation of the business are called CURRENT ASSETS.
Tangible assets used in business (not held for sales in the ordinary course of the business) that are of a permanent or
relatively fixed nature are called PLANT ASSETS or FIXED ASSETS.
Suppose a photographic equipment is used in the operation of a business. It is obvious that the equipment does wear
out with usage and that its usefulness decreases with the passage of time. The decrease in usefulness is a business
expense, called DEPRECIATION.
Plant assets include equipment, machinery, building, and land. With the exception of land, such assets gradually
wear out or otherwise lose their usefulness with passage of time, i.e. they are said to depreciate. Since we are
interested in this subsection how plant assets depreciate, from now on you consider plant assets to be simply assets.
The depreciation of an asset is caused mainly due to:
a. Physical Depreciation: wear out from use and deterioration from the action of the element.

Telegram channel https://t.me/johnson201485


380
b. Functional Depreciation: inadequacy and obsolescence. Inadequacy results if the capacity does not meet the
demand of increased production, while obsolescence results, if the commodity produced is no longer in demand
with respect to quality and cost of production.
Factors to be considered in computing the periodic depreciation of an asset are its original cost, its recoverable
cost at the time it is retired from service, and the length of life of the asset.
The estimated recoverable cost of depreciable asset as of the time of its removal from service is variously; termed as
residual value, scrap value, salvage value, or trade-in value.
There is no single method of computing depreciation for all classes of depreciable assets. Here we consider two
methods:
i. The fixed instalment method and
ii. Reducing-balance method
The Fixed Instalment Method
The fixed instalment method (or on cost method or the straight-line method) of determining depreciation allows for
equal periodic charges to expense (or cost) over the estimated life of the asset. That is, under this method, the
depreciation is charged evenly every year throughout the economic life of the asset. The periodic depreciation
charge of an asset is expressed as:

Example 29. A machine costing Birr 35,000 is estimated to have a useful lifetime of 8 years and a salvage value of
Birr 3,000. What is the accumulated depreciation at the end of 5 years? Find the book value of the asset at that time,
using the fixed instalment method (where book value = cost – accumulated depreciation)
Solution. We have the cost = Birr 35,000, salvage value = Birr 3,000 and useful life = 8 years.
The depreciation charge per year is r i ti n Birr 4,000
n
Hence the accumulated depreciation increases by Birr 4,000 every year.
The accumulated depreciation at the end of 5 years will be :
years × depreciation charge per year = 5 × 4,000 = Birr 20,000.
The book value of the asset at the end of 5 years will be :
Book value = cost – accumulated depreciation = 35,000 – 20,000 = Birr 15,000.
Example 30. A business buys a machine for Birr 250,000. It is expected to have a useful lifetime of 5 years, after
which time it will have a scrap value of Birr 50,000. Calculate the annual depreciation charge.
Solution. In this question, we have the cost = Birr 250,000, scrap value = Birr 50,000 and useful life = 5 years.
The depreciation charge per annum is r i ti n n
Birr 40,000
Hence the accumulated depreciation increases by Birr 40,000 every year.
The yearly depreciation and book value are shown in the following table.
Cost Yearly Accumulated Book
Year
depreciation depreciation value
Year 1 250,000 40,000 40,000 210,000
Year 2 250,000 40,000 80,000 170,000
Year 3 250,000 40,000 120,000 130,000
Year 4 250,000 40,000 160,000 90,000
Year 5 250,000 40,000 200,000 50,000
Therefore the machine will have a scrap value of Birr 50,000 after 5 years time.
Reducing balance method: The reducing balance method (or declining-balance method) yields a
declining periodic depreciation charge over the estimated life of the asset. Of the several variant techniques the most
common is to apply double straight-line depreciation rate, computed by:

Telegram channel https://t.me/johnson201485


381
The double reducing balance method uses the double rate applied to the cost of the asset for the first year of its use
and thereafter to the declining book value at the beginning of the year, i.e. cost minus the accumulated depreciation.

Example 31. Using the double reducing balance method of depreciation, determine the book value at the end of
the second year of an item that was bought on May 5 for Birr 30,000 and that has a salvage value of Birr 5,000 and
an estimated useful life of 40 years.
Solution. The depreciation rate per year = = 0.05
The depreciation for the first full year is 30,000 × 0.05 = Birr 1,500.
Hence the depreciation per month is Birr 1,500 per year  12 month per year = Birr 125 per month.
Since the item is bought on May 5, it is close to May 1. Hence at the end of the first year the depreciation is
Birr 125 per month × 8 months = Birr 1,000.
The book value at the end of the first year is 30,000 – 1000 = Birr 29,000.
Therefore, the depreciation for the second year is 29,000 × 0.05 = Birr 1450, and
the book value at the end of the second year is Birr 29,000 – Birr 1,450 = Birr 27,550.
Exercise 15.7
1. A dump truck is purchased for $110,000 and has an estimated salvage value of $10,000 at the end of the
recovery period. Prepare a depreciation schedule for the dump truck using the fixed instalment method with a
recovery period of five years.
2. New equipment was obtained at a cost of Birr 100,000 on January 5. The equipment has estimated lifetime of 5
years and an estimated residual value of Birr 8,000.
i. Determine the annual depreciation for each of the five years of the estimated useful life of the equipment.
ii.The accumulated depreciation at the end of each year.
iii. The book value of the equipment at the end of each year by using
a. the fixed instalment method. b. the double reducing balance method.
3. A dump truck is purchased for $110,000 and has an estimated salvage value of $10,000 at the end of the
recovery period. Prepare a depreciation schedule for the dump truck using the double reducing balance method with
a recovery period of five years.
4. A business buys a machine for Birr 200,000. It is expected to have a useful life of 10 years. After which time it
will have a scrap value of Birr 0. Calculate the annual depreciation charge.
15.3. SAVING, INVESTING AND BORROWING MONEY
What is Money?
It is very difficult to give a precise definition of money because various authors have defined money differently.
However we may define money in terms of functions it performs, i.e. “Money is that what money does" or
"Anything which is generally accepted as a medium of exchange in the settlement of all transactions
including debt and acts as a measure and store of value".
Functions of money: Money performs the following four important functions
I. Primary Functions of Money
a. Money as a medium of exchange: the most important function of money is to serve as a
medium of exchange.

Telegram channel https://t.me/johnson201485


382
b. Money as a measure of value: money serves as a common measure of value or unit of
account. It serves as a standard or yardstick in terms of which values of all goods and
services can be expressed.
II. Secondary Functions of Money
a. Money as a standard of deferred payment: money serves as a standard in terms of which future payments
can be expressed.
b. Money as a store of value: money being the most liquid of all assets is a convenient form
in which to store wealth. Furthermore, money helps in the transfer of value from one
person to another as well as from one place to another.
15.3.1 Saving Money
A. Reasons for Saving: You may be asking yourself why there is so much pressure to save money. If
you have enough to pay for everything you need, why should you worry about putting any aside each month? There
are a variety of reasons to begin saving money. Different people save for different reasons. Here are seven reasons
that you may consider for saving your money.
1. Save for emergency funds 5. Save for a new car
2. Save for retirement 6. Save for sinking funds
3. Save for a down payment on a house 7. Save for your education
4. Save for vacations and other luxury items
B. Planning a Saving Programme
If you think yourself as an employee or a business man, you need to plan on how to save, and this planning is
directly related to the reason on why you save money.
C. Savings as Investment
New issues of corporate stock: New corporations raising funds to begin operation, or existing corporations that
want to expand their current operations, can issue new shares of stock through the investment banking process.
People who buy these shares of stock hope to make money by having the price of the stock increase, and through
dividends that may be paid out of future profits.
New issues of bonds: New issues of bonds are issued by companies that want to borrow funds to expand by
investing in new factories, machinery, or other projects, and by government agencies that want to finance new
building, roads, schools, or other projects. The bonds are promises to repay the amount borrowed, plus interest, at
specified times. Individuals, banks, or companies that want to earn this interest purchase the bonds.
Borrowing from banks and other financial intermediaries: Companies (and individuals) can borrow funds from
banks, agreeing to pay interest, on a specified schedule. Banks and other financial intermediaries lend out money
that has been deposited by other people and firms. In effect, banks and other intermediaries are just a special kind of
“middleman,” making it easier for those with money to lend to find those who want to borrow funds. Of course,
banks also screen those who borrow money, to make sure they are likely to repay the loans.
D. Saving Institutions: Saving institutions are financial institutions that raise loanable funds by selling
deposits to the public. They accept deposits from individuals and firms and use these funds to participate in the debt
market, making loans or purchasing other debt instruments such as Treasury bills. The major types of saving
financial institutions are commercial banks, saving and loan associations, mutual saving banks, and credit unions.
Their main liabilities (sources of funds) are deposits, and their main assets are loans.
I. Commercial Banks: Commercial banks are business corporations that accept deposits, make loans, and
sell other financial services, especially to other business firms, but also to households and governments.

Telegram channel https://t.me/johnson201485


383
II. Savings And Loans Associations: Savings and loans associations (S & Ls) were originally designed
as mutual associations, (i.e., owned by depositors) to convert funds from savings accounts into mortgage
loans.
III. Mutual Savings Banks: Mutual savings banks are much like savings and loans, but are owned
cooperatively by members with a common interest, such as company employees, union members, or
congregation members.
IV. Credit Unions: Credit unions are non-profit associations accepting deposits from and making loans
to their members, all of whom have a common bond, such as working for the same employer. Credit
unions are organized as cooperative depository institutions, much like mutual savings banks. Depositors
are credited with purchasing shares in the cooperative, which they own and operate.
Exercise 15.8
What type of financial institutions would each of the following people be most likely to do business with
a. a person with Birr 10,000 in savings who would like to earn a decent return at low risk
and who does not know much about the stock and bond markets,
b. a person with Biir 350 who needs a checking account,
c. a person who needs a Birr 10,000 loan to open a pizza shop
d. a person who is recently married, is starting a family, and wants to make sure that his
children are well taken care of in the future,
e. the president of a small company who wants to list it on the stock exchange to obtain additional capital,
f. someone who has just received a large inheritance and wants to invest it in the stock market,
g. a person with no credit history who is buying her first car,
h. a family needing a mortage loan to buy a house,
i. a person who has declared bankruptcy in the past and is looking for a loan to pay off some past due bills.
15.3.2 Investment
INVESTMENT is the production and purchase of capital goods, such as machines, building, and equipment that
can be used to produce more goods and services in the future. Personal investment is purchasing financial securities
such as stocks and bonds, which are riskier than savings accounts because they may fall in value, but in most cases,
will pay a high rate of return in the long run than the interest paid on savings accounts.
A. Investment strategy
In finance, an investment strategy is a set of rules, behaviours or procedures, designed to guide an investor's
selection of an investment portfolio. Usually the strategy will be designed around the investor's risk-return tradeoff.
Some investors will prefer to maximize expected returns by investing in risky assets, others will prefer to minimize
risk, but most will select a strategy somewhere in between.
Passive strategies are often used to minimize transaction costs, and active strategies such as market timing are an
attempt to maximize returns. One of the better known investment strategies is buy and hold. Buy and hold is a long
term investment strategy, based on the concept that in the long run equity markets give a good rate of return despite
periods of volatility or decline.
B. Types of Securities
Stocks: Stocks can help you build long-term growth into your overall financial plan. History has repeatedly
demonstrated that stocks, as an asset class, have outperformed every other type of investment over long periods of
time. Stock represents an ownership or equity stake in a corporation. If you are a stockholder, you own a
proportionate share in the corporation's assets and you may be paid a share of the company's earnings in the form of
dividends.

Telegram channel https://t.me/johnson201485


384
Stocks are considered to be a riskier investment than bonds or cash. Stock prices tend to fluctuate more sharply-both
up and down than other types of asset classes.
Bonds: Corporations, governments and municipalities issue bonds to raise funds, and in return they typically
pay the bond owners a fixed interest rate. In this way, a bond is like a loan. Bonds may provide a regular income
stream or diversify a portfolio. Bonds are fixed income investments - most pay periodic interest and principal at
maturity.
Interest rates may be the most significant factors affecting a bond's value. When interest rates fall, the value of
existing bonds rise because their fixed-interest rates are more attractive in the market than the rates for new issues.
Similarly, when interest rates rise, the value of existing bonds with lower, fixed-interest rates tend to fall.
Inflation may erode the purchasing power of interest income. Generally, bonds with longer maturities are more
sensitive to inflation than bonds with shorter maturities. Economic conditions may cause bond values - particularly
corporate bonds-to fluctuate. An economic change that adversely affects a company‟s business may reduce the
ability of a company to make interest or principal payments.
C. How to Invest
As you may have noticed, there are several categories of investments, and many of those categories have thousands
of choices within them. So finding the right ones for you isn't a trivial matter. The single greatest factor, by far, in
growing your long-term wealth is the rate of return you get on your investment. There are times, though, when you
may need to park your money someplace for a short time, even though you won't get very good returns. Here is a
summary of the most common short-term savings vehicles:
Short-Term Savings Vehicles
Savings Account: Often the first banking product people use, saving accounts, earn a small amount in interest,
so they're a little better than that dusty piggy bank on the dresser.
Money Market Funds: These are a specialized type of mutual fund that invests in extremely short-term
bonds. Money market funds usually pay better interest rates than a conventional savings account does, but you'll
earn less than what you could get in certificates of deposit.
Certificate of Deposit (CD): This is a specialized deposit you make at a bank or other financial institution.
The interest rate on certificate of deposits is usually about the same as that of short- or intermediate-term bonds,
depending on the duration of the CD. Interest is paid at regular intervals until the certificate of deposit matures, at
which point you get the money you originally deposited plus the accumulated interest payments. Fools are partial to
investing in stocks, as opposed to other long-term investing vehicles, because stocks have historically offered the
highest return on our money. Here are the most common long-term investing vehicles:
Long-Term Investing Vehicles
Bonds: Bonds come in various forms. They are known as "fixed-income" securities because the amount of
income the bond generates each year is "fixed" or set, when the bond is sold. From an investor's point of view,
bonds are similar to CDs, except that the government or corporations issue them, instead of banks.
Stocks: Stocks are a way for individuals to own parts of businesses. A share of stock represents a proportional
share of ownership in a company. As the value of the company changes, the value of the share in that company rises
and falls.
Mutual Funds: Mutual funds are a means for investors to pool their money to buy stocks, bonds, or anything
else the fund manager decides is worthwhile. Instead of managing your money yourself, you turn over the
responsibility of managing that money to a professional. Unfortunately, the vast majority of such "professionals"
tend to under-perform the market indexes.
Exercise 15.9

Telegram channel https://t.me/johnson201485


385
DIRECTION: Mark an S if the situation involves saving, an I, if the situation involves
investing, a P if the situation involves personal investing, and an N if the situation
involves neither saving nor investing.
a. Kassech borrowed Birr 25,000 from a bank to purchase a computer and other equipment and supplies to open
her new internet home page business.
b. Bontu buys 100 shares of Alpha PLC, hoping that the price per share will increase.
c. Mike dies and leaves his estate of Birr100, 000 to his five children. They use it to take an around-the-world,
once-in-a-lifetime, one-year cruise.
d. Dawit, the head of Sunshine Computer Systems, issues new shares of stock in his company through an
investment banker, and uses those funds to build a new assembly line to produce the world‟s fastest
microprocessors.
e. A woman takes a new job and has Birr 20 a week deducted from each paycheck to be deposited directly into a
savings account at her bank.
f. Ford Motor Company issues a Birr 5,000 bond, which is purchased by Sara.
g. Medical Systems, Inc. builds a new plant to produce experimental pacemakers.
h. Mark quits his job to go back to school to study economics, hoping to earn more money with
a college degree.
15.3.3 Borrowing Money
Loans, overdrafts and buying on credit are all ways of borrowing. Different methods of borrowing suit different
types of people and situations. Whatever type of borrowing you choose, it is important to make sure you will be
able to afford the repayments.
Types of loan
Secured Loan: With a secured loan, the lender has the right to force the sale of the asset against which the
loan is secured if you fail to keep up the repayments. The most common form of secured loan is called a „further
advance‟ and is made against your home by borrowing extra on your mortgage. (Your mortgage is itself a secured
loan.) Because secured loans are less risky for the lender, they are usually cheaper than unsecured loans. Secured
loans are mostly suitable for borrowing large amounts of money over a longer term, for example, for home
improvements.
Unsecured Loan: An unsecured loan means the lender relies on your promise to pay it back. They're
taking a bigger risk than with a secured loan, so interest rates for unsecured loans tend to be higher. Unsecured
loans are often more expensive and less flexible than secured loans, but suitable if you want a short-term loan (one
to five years).
Credit Union Loan: Credit Unions are mutual financial organizations which are owned and run by their
members for their members. Once you've established a record as a reliable saver they will also lend you money but
only what they know you can afford to repay. Members have a common bond, such as living in the same area, a
common workplace, membership of a housing association or similar.
Money Lines: Money lines are community development finance institutions that lend and invest in deprived
areas and underserved markets that cannot access mainstream finance. They provide money for personal loans,
home improvements, back to work loans, working capital, bridging loans, property and equipment purchase, start up
capital and business purchase.
Overdraft: Overdrafts are like a 'safety net' on your current account; they allow you to borrow up to a certain
limit when there's no money in your account and can be useful to cover short term cash flow problems. Overdrafts
offer more flexible borrowing than taking out a loan because you can repay them when it suits you, but they're not

Telegram channel https://t.me/johnson201485


386
usually suitable for borrowing large amounts over a long period as the interest rate is generally higher than with a
personal loan. You need a bank account in order to have an overdraft.
Buying On Credit: Buying on credit is a form of borrowing. It can include paying for goods or services
using credit cards or under some other credit agreement.
a. Advantages and Disadvantages of Borrowing
The interest paid up on borrowed money is tax deductible. Therefore, cheaper Birr is paid back. Terms and
conditions of borrowing are fixed and are subject to change in relation to changes in market conditions like price
increments. As a result, costs will decrease and the value of the firm will increase.
The disadvantage of borrowing is that, if prices in the money market are going down, the borrower will be obliged
to pay much more money as interest on fund borrowed. This is because terms and conditions are fixed. Bond
indentures are burdensome due to inflexibility. In addition to this increase in debt may cause bankruptcy.
b. Source of Loan: The main sources of loan are saving institutions like commercial banks, saving and
loan associations and credit unions. Others include consumer finance companies, insurance companies and private
companies.
15.4. TAXATION
It is known that governments provide a number of public services. To provide public services, they need money
which has to be collected through tax. This sub-unit will discuss different objectives and principles of taxation.
As governments have played a growing role in all economies, they have used increasing amounts of resources for
their activities, and taxes have constituted increasing percentages of national income. Either directly or indirectly,
the various levels of government provide most education and pay a major proportion of medical bills. They provide
national defense, police and fire protection and provide or support a substantial amount of housing, recreation
facilities and parklands. They set health standards and ensure adequate water supplies, transportation and other
public facilities. They seek to attain a distribution of income regarded as equitable, to stabilize the economy from
periods of excessive inflation or unemployment, and to ensure an adequate rate of growth.
According to Richard Musgrave, governmental activities are divided into three parts.
1. Allocation: the activities involving the provision of various governmental services to society and thus
involving the allocation of resources to the production of these services. Some of the services are strict public
goods (e.g. national defence) some are ones involving externalities (e.g. education) some are provided by
government to avoid private monopoly and costs of collection of charges (e.g. highways).
2. Distribution: the activities involving in the redistribution of income welfare programs, progressive tax
structures and so forth.
3. Stabilization and Growth: the activities designed to increase economic stability by lessening
unemployment and inflation and influencing, if thought desirable, the rate of economic growth.
The main question that will follow is where does the government gets money in order to do all the above
mentioned activities?
A. Objectives of Taxation: Governments impose and collect taxes to raise revenue. Revenue
generation however is not the only objective of taxation, though it is clearly the prime objective. Taxes as a fiscal
policy instrument are used to address several other objectives such as:
1. Removal of inequalities in income and wealth: Government adopts progressive tax system and
stressed on canon of equality to remove inequalities in income and wealth of the people.
2. Ensuring economic stability: Taxation affects the general level of consumption and production; hence
it can be used as an effective tool for achieving economic stability. Governments use taxation to control inflation
and deflation.

Telegram channel https://t.me/johnson201485


387
3. Changing people‟s behaviors: Though taxes are imposed for collecting revenue to meet public
expenditure, certain taxes are imposed to achieve social
objectives for example, to discourage consumption of harmful products,
governments impose heavy taxes on production of tobacco and alcohol.
4. Beneficial diversion of resources: Governments impose heavy tax on nonessential and luxury goods
to discourage producers of such goods and give tax rate reduction or exemption on most essential goods. This
diverts producer‟s attention and enables the country to utilize limited resources for production of essential goods
only.
5. Promoting economic growth: Economic growth depends on the generation of income from industrial
agricultural and other areas. The rate of economic development goes up if more investment is available to all
sectors. Tax policy of a government is a key element in planning the economic growth of a country.
B. Principles of Taxation
The compulsory payment by individuals and companies to the state is called TAXATION. A government imposes
taxes to raise revenue to cover the cost of administration, the maintenance of law and order, defense, education,
housing, health, pensions, family allowances etc. Now, the government has started to subsidize farming, industries,
etc. In all these, taxes are imposed to provide revenue to cover government expenditure.
Adam Smith‟s Cannon of Taxation: Adam Smith has laid down principles or cannon of taxation in his
book “Wealth and Nations”. These cannons still constitute the foundation of all discussions on the principles of
taxation. To create an excellent system of taxation, it is necessary to first establish a set of high standard principles
for taxation. Little or no attention has been paid by governing bodies to establish such important principles.
C. Classification of taxes
In Ethiopia taxes are classified on the basis of impact (immediate burden) and incidence (ultimate burden) of tax.
Taxes are classified into two broad categories : DIRECT and INDIRECT TAXES.
1. Direct Taxes: Direct tax is one in which the payer himself is the ultimate sufferer of its consequence.
This means the incidence cannot be transferred to a third party. Direct taxes according to the Ethiopian tax law
include all income taxes such as employment income tax, business income tax and land use fee, mining income tax
and other income taxes. Generally direct taxes are income based taxes.
Schedules of Income: Recently, Ethiopia has launched a tax reform program to achieve the objectives of
democracy by considering taxation as one of the most important areas where attention is required. It resulted in the
outcome of many important proclamations. The present income tax proclamation (No 286/2002)
proclaimed after the tax reform program of the country, incorporated a number of tax bases as part of the
development activities of the government.
The government has identified many tax bases for direct taxes. These tax bases are categorized into different
schedules according to their nature in the proclamation. Thus the four schedules incorporated in direct taxes are
schedules „A‟, „B‟, „C‟ and „D‟. The bases for these schedules are.
Schedule A: Employment Income Tax: It is an income collected from employment.
The employer assesses employment income tax. The tax is deducted at source before paying the monthly salary.
For assessment of tax the employers make use of the following tax rates.

Telegram channel https://t.me/johnson201485


388
Example 32. Assume Ato Dagim earns a monthly salary of Birr 7850. His income tax will be calculated as
follows.
Total taxable income = Birr 7850
Less: the minimum amount not taxed = Birr 150
Remaining taxable income = 7850  150 = Birr 7700
Less: first Birr 500 taxed at 10% = 500 ×10% = Birr 50.00
Remaining taxable income = 7700  500 = Birr 7200
Less: second Birr 750 taxed at 15% = 750 ×15% = Birr 112.00
Remaining taxable income = 7200  750 = Birr 6450
Less: third Birr 950 taxed at 20% = 950 ×20% = Birr 190.00
Remaining taxable income = 6450  950 = Birr 5500
Less: forth Birr 1250 taxed at 25% = 1250 ×25% = Birr 312.50
Remaining taxable income = 5500  1250 = Birr 4200
Less: fifth Birr 1450 taxed at 30% = 1450 ×30% = Birr 435.00
Remaining taxable income = 4200  1450 = Birr 2750
Less: the last Birr 2750 taxed at 35% = 1450 ×35% = Birr 962.50
Total tax of the month = 50.00 + 112.00 + 190.00 + 312.50 + 435.00 + 962.50 = Birr 2061.00
Therefore, Ato Dagim‟s net income is then 7850 – 2061 = Birr 5789.00
Schedule B: Rental Income Tax: It is a type of tax collected from rental of building.
When leasing a building, certain items of expenses (deductible expenses) can be subtracted from the gross income
in order to arrive at the amount that is taxed. The expenses allowable against the rental income are those incurred
wholly or exclusively in connection with the leasing activity. Deductions include taxes paid with respect to the land
and building leased except income taxes and a total of an allowance of 20% of the gross rent received; for repairs,
maintenance and depreciation of such building, furniture and equipment. The tax rate for a body is 30% and others
are as in the following schedule.

Telegram channel https://t.me/johnson201485


389
Schedule C: Business Income Tax: It is a type of tax collected from business income.
The income tax proclamation (№ 286/2002) provides the tax rates that should be used for this purpose. The tax rate
is applied on the assessed taxable income of the business unit. Once the declaration is made by the business unit, its
accuracy is checked by the tax office through a process called tax assessment. Tax assessment is a tax review by a
tax official of a tax declaration and information provided by a taxpayer and a verification of the arithmetical and
financial accuracy of the declared tax liability. The procedure for the assessment of business income tax takes two
forms.
 Assessment by books of accounts and
 Assessment by estimation
Tax of those taxpayers who have different sources of income under schedule “C” will be assessed on the aggregate
of all income. The tax rates used for computation of income under schedule “C” are the same as that of schedule
“B”. Under schedule “C” there are three categories “a”, b” and “c” of which a and b categories are assessed by
books whereas category “c” is assessed by estimation.
Schedule D: Other income taxes: Other incomes which include royalties‟ income from technical services
rendered outside the country, income from games of chance, dividend income, causal rental of property, interest
income and gains from transfer of investment property.
People often get income from other sources in addition to (or other than) the income obtained from their
employment, their business activities or their renting activities. The income from other activities is taxed at a flat
rate as described below.
Source of income rate
Royalty 5%
Technical services 10%
Dividend 10%
Interest 5%
Game of chance 15%
Casual rental of property 15%
Gain on transfer of investment property: Gain on share capital 30%
Other capital gain 15%
Example 33. Ato Tekle leased his personal car for two months at Birr 6000 per month. Such income is referred
to as casual rental income by the tax expert.
i. How much is the tax to be paid? ii. Who is liable to pay the tax?
Solution
i. Tax on casual rental of property = 15% of gross rental income
= 15% × (6000 × 2) = 15% × 12,000.00 = Birr 1800.00

Telegram channel https://t.me/johnson201485


390
ii. The receiver of the income, Ato Tekle, is liable to withheld and pays the required tax to
tax authority.
Example 34. Ato Samuel acquired 1000 shares of Admas Co. for Birr 4,500 each and sold them at Birr 6,000
each. How much does he pay as capital gain tax?
Solution. Gain = (6000 – 4500) × 1000 = Birr 1,500,000.
Capital gain tax = 1,500,000.00 × 30% = Birr 450,000.
Example 35. Kurtu Trading Co. sold one of its building for Birr 980,000 which it acquired for 720,000.
Compute the capital gain tax.
Solution. Capital gain = 980,000 − 720,000 = Birr 260,000.
Capital gain tax = 260,000 × 15% = Birr 39,000.
2. Indirect Taxes: Indirect tax is a tax in which the burden may not necessarily be swallowed by business;
which means, indirect taxes can be shifted onto other persons. Generally the tax incidence of indirect tax is on the
ultimate consumer; however, sometimes a seller might absorb such indirect tax to be competitive in the market. This
action reduces its profit. Indirect taxes are consumption based taxes. In Ethiopia the indirect tax category includes,
value added tax (VAT), excise tax, turnover tax (TOT), custom duties and stamp duty.
Value Added Tax (VAT): VAT is a levy imposed on business at all levels of production and
distribution of goods and services. It is determined on the basis of the increase in price, or value, provided by each
stage in the chain of distribution. It is a general consumption tax assessed on the value added to goods and services.
Some goods are exempted from VAT. Supplies which are not exempted are called taxable supplies. Taxable
supplies and import goods are taxed at a flat rate of 15% in our country. Some taxable supplies are zero rated. Zero
rated supplies are those on which VAT on supply/sale is charged at zero rates.
In Ethiopia invoice credit method is used for collection of VAT. Under this method, VAT payable is the difference
between the tax charged on taxable transactions and tax paid on import of goods or on the purchase of supplies
where such supplies are used or to be used for the taxable transactions.
Example 36. Nokia Company purchased Mobiles for Birr 54,000. net and will be invoiced and will pay the
supplier Birr 62,100 of which 8,100 is VAT. Nokia sell these mobiles for 86,250 (Birr 75,000 + Birr 11,250 VAT.).
The VAT liability of Nokia company is Birr 3,150 (11,250 − 8,100).
The detail is illustrated below.

Turnover Tax (TOT): To enhance fairness in commercial dealings and to make a complete coverage of
the tax system, a turnover tax is imposed on those persons who are not required to register for VAT, but supply
goods and services in the country. As a result, persons who are engaged in the supply of goods and rendering of
service (which are taxable) and who are not required to register for VAT have to pay turnover tax on the value of
goods they supply or on the value of services they render. TOT is computed as per the proclamation No 308/2002.
The TOT rate is
 On goods sold locally : 2%
 On services rendered locally :
 Contractors, grain mills, tractors, and combine harvesters : 2%
 Others : 10%

Telegram channel https://t.me/johnson201485


391
Example 37. Elsa Stationery has daily sales of Birr 205. The stationery is open 280 days per year. How
much is the turnover tax payable by Elsa?
Solution. Annual sales = 280 × 205 = Birr 57,400.
TOT = Birr 57,400 × 2% = Birr 1148.
Exercise Tax: With a view to increase the revenue of the government to provide for public goods and
services and to reduce the consumption of specific goods, the governments of Ethiopia levies excise tax on selected
items of goods that are supplied in the country. As per the excise tax proclamation No 307/2002, the items of goods
that are subject to excise tax in Ethiopia are: goods imported to the country and goods produced locally. The tax is
imposed equally on both imported and locally produced goods at a rate defined in the proclamation. The major
items included are sugar, salt, tobacco, alcohol, textile, fuel, jewelers, vehicles and televisions.
Example 38. A company is importing sugar from China incurring costs of Birr 842,000 Birr 210,500 and Birr
165,500 for purchasing, insurance and freight respectively. Compute the excise tax payable. (Note:- Sugar is taxed
at a rate of 33%)
Solution. Total cost (purchase, insurance and freight)
(842,000 + 210,500 + 165,500) = Birr 1,218,000.
Excise tax payable = 1,218,000 × 33% = Birr 401,940.
Customs Duty: Customs duty refers to the tax tariff imposed by the tax authority directly on the activities of
import and export of goods and services. Custom duty is levied on cost of production for locally produced goods
and cost, insurance and freight (CIF) paid for imported items. Duties of customs are levied on goods imported to or
exported from Ethiopia at a rate ranging from 0 to 35% as follows.

Items like diplomatic and consular missions, personal effects, grants and gifts to Ethiopia, fire fighting
instruments and appliances, trade samples, defense and public security equipment, materials for handicapped
and similar items are exempted from customs duty.
Example 39. Kent tobacco importing company paid cost of purchase $ 120,000.00, insurance premium and
freight costs are, respectively, $12,000.00 and $8,000.00.The exchange rate is currently $1=12.50 Birr. Compute the
customs duty. (Note:- Tobacco is taxed at 35%.)
Solution. CIF = (120,000 + 12,000 + 8,000) × Birr 12.50 = 140,000 × Birr 12.50 = Birr 1,750,000.
Custom Duty = Birr 1,750,000 × 35% = Birr 612,500.
Exercise 15.10
1. Find the income tax of the following employees of Nyala Insurance sh. Company.
a. W/ro Mebrat with monthly salary of Birr 850.
b. Ato Tesfu with monthly salary of Birr 2,390.
c. Dr. Gebru with monthly salary of Birr 5,400.
2. Buna Bank declared to pay 20% dividend to its share holders. Find the dividend earned and
tax to be paid by the following share holders.
a. Mesfin with Birr 300,000 worth of shares.
b. Askale with Birr 100,000 worth of shares.
c. W/ro Almaz with Birr 450,000 worth of shares.

Telegram channel https://t.me/johnson201485


392
3. If Kassa won a lottery worth of Birr 150,000, find the amount of tax he is liable to and his net
income.
4. Zewdinesh rented her loader for 10 days to a contractor at a rate of Birr 5,000 per day.
Determine the amount she earns after tax.
5. A company purchased the following items from a stationary.

i. Complete the table


ii. What is the total VAT to be paid?
iii. What is the total price of the items including VAT?
iv. If the company want to pay for the stationary after subtracting a 2% withholding tax
before VAT,
a. what is the amount that will be subtracted by withholding?
b. what is the amount that the company has to pay for the stationary?
6. A company wants to buy five cars from MOENCO. If the price of each car including VAT is Birr 550,000, then
i. what is the total price of each car before VAT?
ii. if the company wants to subtract a 2% withholding tax before VAT, what is the amount to be subtracted?
iii. what is the amount that the company should pay to MOENCO after withholding 2% is subtracted?
7. A shoe dealer purchased net Birr 8,000 worth of shoes from a shoe company.
a. Find the amount it is to pay the company including VAT.
b. If the dealer sold the shoes for Birr 12,000, find the amount of VAT liable to the dealer.
8. An artist sold his new song to a producer company for Birr 350,000. What is the royalty that
should be paid by the artist.

KEY TERMS
annually jointly proportional rate
base liquidity ratio
book value markup reducing-balance method
commercial discount mean proportion restrictions
compound interest ordinary annuity safety
compound proportion percentage salvage value/residual value
depreciation present value semi-annually
earnings principal simple interest
fixed-installment method proportion simple proportion
future value proportionality constant taxes
interest quarterly terms

SUMMARY
1. A ratio is a comparison of two or more quantities expressed in the same unit of measurement.
2. A rate is a comparison of two or more quantities expressed in different unit of measurement.
3. A ratio can be a rate.
n n n n n n
4.
n n n n

Telegram channel https://t.me/johnson201485


393
5. A proportion is a statement of equality between two ratios.
6. A compound proportion is a situation in which one variable quantity depends on two or
more other variable quantities.
7. A percentage is the numerator of a fraction whose denominator is 100.
8. Percentage = base × rate
9. Markup = Selling price - Cost
10. The future value of a simple interest investment is obtained by ( )
11. The future value of a compound interest investment is obtained by . /
( )
12. The future value of an ordinary annuity is given by . / and the amount of
interest is .
13. Plant assets or fixed assets are tangible assets used in business that are of a permanent or
relatively fixed nature.
14. Depreciation of a plant asset is decrease in usefulness of the asset.
REVIEW EXERCISES ON UNIT 11
1. What is the ratio of 1.8 km to 800 meter?
2. In a family there are three daughters and a son. What is the ratio of the number of
a. females to the number of people in the family?
b. males to the number of females in the family?
3. Allocate a profit of Birr 21,400 of a company among three partners in the ratio of their share
of the company .
4. 15 workers can accomplish a job in 28 days. At the same rate by how many workers can the
work be accomplish in 8 days less time?
5. What percent of Birr 52 is Birr 3.12?
6. 8.35% of what amount is Birr 18.37?
7. A 6% tax on a pair of shoes amounts to Birr 10.20. What is the cost of the pair of shoes?
8. If the average daily wage of a labourer increased from Birr 16.00 to Birr 21.64 in the last three
years, what is the rate of increase?
9. A radio recorder sold for Birr 210 has a markup of 25% on the selling price. What is the cost?
10. Ato Alula deposited Birr 3,000 in a saving account that pays 6% interest rate per year,
compounded quarterly. What is the amount of interest obtained at the end of seven years?
(No deposit or withdrawal is made in these seven years)
11. Ato Alemu makes regular deposits of Birr 230 at the end of each month for 3 years. What is
the future value of his deposit, if interest rate per year is 9%. compounded monthly? What is
the amount of interest?
12. At the end of each month Ato Mohammed deposits 10% of his salary in a saving institution
that pays annual interest rate of 6% for one year and then 15% for the next 3 years. If the
salary of Ato Mohammed is Birr 1800, find the future value of his deposits at the end of the 4
years.
13. A piece of machinery costs Birr 50,000 with an estimated residual value of Birr 7,000 and a
useful life of 8 years. It was placed in service on April 1 of the current fiscal year. Determine
the accumulated depreciation and book value at the end of the following fiscal year using:
a. the fixed installment method. b. the double reducing balance method.

Telegram channel https://t.me/johnson201485


394
UNIT SIXTEEN: INTRODUCTION TO CALCULUS
Unit Outcomes:
After completing this unit, you should be able to:
 Deduce rate of change of different quantities.
 Calculate rate of change of different quantities.
 Understand gradient of function at a point.
 Analyze the geometric and mechanical meaning of derivatives.
 Find area of a region bound by a functions
 Understand definite integral.
 Value the real world contributions of having skills of derivatives and
integration
 Apply the knowledge of integral calculus to solve real life mathematical
problems
 Apply the concept of derivatives to solve basic problems in business,
economics, and taxation.

Main Contents:

Telegram channel https://t.me/johnson201485


395
16.1. INTRODUCTION TO DERIVATIVES.
16.2. APPLICATION OF DERIVATIVES.
16.3. INTRODUCTION TO INTEGRATION.
Key terms
Summary
Review Exercises

16.1. INTRODUCTION TO DERIVATIVES.


A derivative represents the rate at which a function changes as its input changes. In simpler terms, it describes how
the output of a function (dependent variable) changes with respect to a small change in the input (independent
variable). It is the fundamental concept in calculus and has applications in various fields, such as physics,
economics, and engineering.
Mathematically, the derivative of a function ( ) at a point is the limit or slope of a tangent line to the graph
of ( ) at a point ( ( ))of the average rate of change of the function as the interval approaches zero:
( ( ) ( ))
( ) i
The derivative is often denoted as ( ) or where ( )
Examples of derivatives:
Consider the function ( ) To find its derivative, we apply the definition of the derivative:
( ( ) ( )) ( ( ) )
( ) i i
Thus, the derivative of ( ) is ( ) , meaning the rate of change of at any point is
Understanding Rate of Change
The core idea behind derivatives is understanding how fast or slow something is changing. For example:

 If you are driving a car, your speed at any moment is the rate at which your position is changing with
respect to time. The speed is the derivative of the position function.

Telegram channel https://t.me/johnson201485


396
 If the temperature changes over time, the rate at which it is changing is given by the derivative of the
temperature function with respect to time.

Graphical Interpretation: Slope of Tangent Line

When you look at the graph of a function ( ) you can imagine a tangent line touching the graph at a particular
point. The derivative at that point is the slope of that tangent line. The slope tells you how steep the curve is at that
point.

 If the tangent line has a positive slope, the function is increasing at that point.
 If the tangent line has a negative slope, the function is decreasing at that point.
 If the tangent line is horizontal (slope = 0), the function has a local maximum, minimum, or a point of
inflection at that point.

How to calculate a derivatives

Rules to for finding derivatives

Telegram channel https://t.me/johnson201485


397
Higher-Order Derivatives: The derivative itself can also be differentiated. The second derivative, denoted
( ), tells us how the rate of change is changing, i.e., it gives us information about the curvature or concavity of
the function.

 The second derivative indicates whether the function is concave up (like a bowl) or concave down
(like a dome).
 The third derivative and higher-order derivatives are used in advanced analysis, but typically, the
first and second derivatives are the most important.
16.2. APPLICATIONS OF DERIVATIVES
16.2.1. Finding Tangent Lines:
Derivatives are used to find the equation of a tangent line to a curve at a given point. The slope of the
tangent line at is the value of the derivative ( )
Example: For ( ) , the slope at is ( ) The equation of the tangent line is:
( ) ( )( )
Substituting the values: ( )
16.2.2. Optimization(Finding Extrema (Maximum and Minimum): Derivatives are used in
optimization problems to find maximum or minimum values of a function. By setting the derivative equal
to zero ( ( ) ) we can find the critical points, which are potential maxima, minima, or points of
inflection.

Telegram channel https://t.me/johnson201485


398
Example: For ( ) , the derivative is ( ) . Setting ( ) gives
which corresponds to a maximum point.
Note: Optimization Problems: In real-world problems like maximizing profit or minimizing cost,
derivatives play a crucial role. For instance, companies use derivatives to determine the optimal level of
production where profit is maximized.
16.2.3. Motion in Physics: In kinematics, derivatives describe the motion of objects. The first derivative
of position with respect to time is velocity, and the second derivative is acceleration. This helps in
understanding the speed, direction, and rate of change of motion.

16.2.4. Engineering and Science: Derivatives are also used to describe physical phenomena, such as heat
transfer, fluid dynamics, and electrical circuits. They allow engineers and scientists to model and analyze
rates of change in systems.
Conclusion: The derivative is an essential concept in calculus that gives us a way to understand how
functions behave, how things change, and how we can optimize and control processes in various fields.
Whether you're studying the motion of a car, the growth of a population, or the design of a bridge,
derivatives provide the tools to analyze change effectively.
16.3. INTRODUCTION TO INTEGRATION
Integration is a fundamental concept in calculus that involves finding the total accumulation of a quantity.
It is often interpreted as the area under a curve or the accumulation of quantities over an interval.
Mathematically, the integral of a function ( ) over an interval , -is denoted as:

∫ ( )

This expression represents the total accumulated value of ( ) from to


Definite Integration and Its Rules: Definite integration refers to the process of integrating a function
over a specific interval , -. The result of a definite integral is a real number that represents the net area
under the curve of the function between the limits and

Fundamental Theorem of Calculus


The Fundamental Theorem of Calculus connects differentiation and integration, stating that if ( ) ( )
is an antiderivative of ( ) on , -, then:

∫ ( ) ( ) ( )
This theorem allows us to evaluate definite integrals by finding the antiderivative of the integrand and
computing its values at the boundaries of the interval.
Application Example: Area Under a Curve
Consider the function ( ) over the interval [1, 3]. To find the area under the curve from to
, we compute the definite integral:
∫ [ ] ( ) ( )

Therefore, the area under the curve ( ) over the interval from to is square units

Telegram channel https://t.me/johnson201485


399
Applications of Definite Integration
Definite integrals have numerous applications across various fields:
 Physics: Calculating displacement when given a velocity function.
 Economics: Determining consumer and producer surplus by integrating demand and supply
functions.
 Engineering: Finding the center of mass of an object by integrating its density function.
In summary, integration is a powerful mathematical tool used to compute total accumulated quantities,
such as areas and volumes, and has widespread applications in science, engineering, and economics.
SOLVED PROBLEMS ON INTRODUCTION TO CALCULUS
1. If (x) ( )
( ) and g‟(x)=10, then which one of the following is equal to f‟(2)?
8 4 8
A. -8 B. C. D.
9 3 9

Therefore, the correct option is A. -8.


2. Find the derivative of the function ( )
Solution:
To find the derivative of the function, we apply the power rule for differentiation.

Telegram channel https://t.me/johnson201485


400
( )
Now, differentiate term by term:
( ) ( ) ( ) ( ) ( ) ( )
( )
Thus, the derivative of the function is:
( )
3. Find the local maximum and minimum of the function ( )
Solution:
 First, find the first derivative ( )
( ) ( ) ( ) ( ) ( )
( )
 Set ( ) to find the critical point
divide by three(3)

Thus critical points are and


 Now, use the second derivative test to classify the critical points.
First, find the second derivative:
( ) ( )
Now, evaluate f′′(x) at the critical points:
At ( ) ( )
Since ( ) is local minimum
At ( ) ( )
Since ( ) is local maximum
Conclusion:
 Local maximum at
 Local minimum at
4. Find the tangent line to the curve at the point ( )
Solution:
 Find the derivative of the function:

 Evaluate the derivative at


⌉ ( )
So, the slope of the tangent line at is 6.
 Now, use the point-slope form of the equation of a line:
( )
Where is the slope and ( )is the point of tangency. Here, and .
Substitute into the equation:
( )

Thus the equation of tangent line is

5. Find the rate of change of the function ( ) n( ) at


Solution:
First, find the derivative of the function:

Telegram channel https://t.me/johnson201485


401
( ) n( )
Using chain rule:
n( )
Let so
( ) ( )
Now evaluate the derivative at :
( )
( )
( )
Thus, the rate of change of the function at is
6. Find the area under the curve from to
Solution:The area under the curve is given by the definite integral:
∫ ( ) , - ( ) ( )
So, the area under the curve is 12 square units.
7. Find the volume of the solid generated by rotating the region bounded by and the x-axis from to
about the x-axis.
Solution: The formula for the volume of a solid of revolution generated by rotating a curve around the x-axis is:
∫ , ( )-
Here ( ) and the limits are to
So the volume is ∫ , - ∫ [ ] ( )
So, the volume is cubic units
8. A force ( ) Newtons is applied to move an object along a straight line from to .
Find the work done by this force.
Solution: The work done by a variable force is given by the integral:
∫ ( )
Here ( ) and the limits are to
( ) ( )
So, the work is: ∫ 0 1 . / . /
So, the work done is 40 joules.
9. Find the average value of the function ( ) on the interval , -.
Solution: The formula for the average value of a function ( ) on the interval , - is:
v r v u ∫ ( )

Here ( ) and the limits are to


So, the average value is :

v r v u ∫ [ ] [ ] (. / . /)

So, the average value of the function is 21.


10. An object moves along a straight path with velocity given by ( ) meters per second, where
is the time in seconds. Find the distance traveled by the object from t .
Solution: The distance traveled is the integral of the velocity function over the given time interval:
ist n ∫ ( ) ∫ ( ) 0 1 . ( ) ( )/ . ( ) ( )/

Telegram channel https://t.me/johnson201485


402
So, the distance traveled is 24 meters

Supplementary question
DIRECTION: ALL OF THESE QUESTIONS ARE FROM THE TWO GRADE LEVELS AND
MULTIPLE CHOICES; SO, SOLVE THE QUESTIONS USING DEFINITIONS AND RULES ON
YOUR TEXT BOOK, CHOOSE THE CORRECT ANSWER FROM THE GIVEN
ALTERNATIVES AND PREPARE YOUSELF FOR SSLE
UNIT ONE: FURTHER ON RELATIONS AND FUNCTIONS
1. Which of the following function is power function?
A. f(x) = xx B. f(x) = 2x C. f(x) = 4x-3/4 D. f(x) = (6x +1)6
1
2. Let f(x) = , the domain of f(x) = ______. A. * + B. C. D.
| x | x
3. Which of the following is a one-to-one correspondence function?
A. f:[0, ) [0, )defined by f(x) = |x - 1| C. f: defined by f(x) = ex
B. f:[1, ) [0, )defined by f(x) = (x – 1)2 D. f:[1, ) [0, )defined by f(x) = (x – 1)2+1
4. Which of the following is rational expression?

A. B. C. D. 6x 2  5 x
x
5. Let f(x) = , g(x) = √ and assume that b = a + 1, then the value of f(a-1) + g(a+1)
x2
√ √ √ √
A. B. C. D.
x | x |
6. If x<0, then the simplest form of f(x) = is equal to:
x
A. 2x B. 2 C. -2 D. 0
x2 1
7. If f(x) = and g(x) =  2 , then f(g(x)) is equal to:
x2 x
x
A. x 2 B. x 2 C. x D.
x
 x 
8. If f(x) = ln   2  , for x>1, then which one of the following is the inverse off?
 x 1 
e 2
x
ex  2 ex x
A. G(x) = x B. G(x) = x C. G(x) = x 2 D. G(x) = x 1
e 3 e 1 e 1 e 2
9. What is the value of |x| + 2x if x<0?
A. -3x B. 3x C. –x D. x
1
10. If f(x) = x , then which of the following is equal to f-1(x)?
e 1
 1  1
A. n(1  x)  n( x) B. e-x+1 C. n  D.  x
 x 1 e 1
11. Given f(x) = In(x-1) and g(x) = 1 2 x , which one of the following is the domain of f  g ?
1 1
A. {x  R : x  1} B. {x  R : x  } C. {x  R : x  0} D. {x  R : x  }
2 2

Telegram channel https://t.me/johnson201485


403
12. If f ( x)  3 1  e  x , which of the following is equal to f 1 ( x) ?
 1  1
A. n 3  B. C. n(1  x 3 ) D. (1  e  x ) 3
 x 1  n( x  1)
3

13. Which of the following functions is a one-to-one correspondence?


A. F:    ,f(x)=tan x, where R‟ is the domain of f C. h:[0, )  [0, ), h(x)=x2
B. t:    , g(x) = 2x D. r:[0, )  [0, ), r(x)=x+5
14. Which one of the following is a one-to one correspondence function from A = 0,1 to B= 1,2 ?
A. f ( x)  x C. f ( x)  2 x  1
1
B. f ( x)  x 3  1 D. f ( x)  x 2  1
3
15. If the point (3, -2) is on the graph of y= f(x), which point is on the graph of y = f-1(x)?
1   1
A.  ,2  B. (3, -1) C. (-2,3) D.  3, 
3   2
16. In the decomposition of ( )( )
the partial fraction is:
A. ( ) ( )
B. ( ) ( )
C. ( ) ( )
D. ( ) ( )
17. The solution set of the equation + + = 0 is___________
A. * + B. {-2} C. {-1, -2} D. {-1}
18. Which one is not true about a rational function f(x) = ?
A. Has oblique asymptote at y =x+4 C. has no horizontal asymptote
B. Has vertical asymptote at x = 2 D. y-intercept at y =3/2
19. What is the solution set of 2?
A. (1, ) B. [1, ) C. {1, -5/2} D. (- 0)
20. Which one is not true about a rational function f(x) = ?
A. has oblique asymptote at y =x+4 C. has vertical asymptote at x = 2
B. has no horizontal asymptote D. y-intercept at y =3/2
1
1 2
4  (c  2) 2
a 1 ?
21. Which of the following is a simplified form of 2 
a  2a 2  1 a
4a 1 a 4a
A. B.  2 2 C. 2 D. 2
a a (a  1) a 1 a 1
22. Which one of the following functions has NO vertical asymptote?
x2 1 x2  9 x 1
A. f ( x)  n( x  1) B. f ( x)  3 C. f ( x)  D. f ( x)  2
x 8 x3 x x
2 x2 2x  2
23. What is the solution set of  2  1 ?
x x  2x 3x  2
A. {1,-2} B. {1,2} C. {-1} D. {1}
24. Which of the following functions could most likely be drawn as in the figure below?
x x 1
A. f(x)= C. f(x) = y=1
x2 x2

Telegram channel https://t.me/johnson201485


404
x 2  2x x3  2x
B. f(x) = D. f(x)= X
x2  4 x2  4
x=2
x 1
25. If f ( x)  and f(a)=5 then f(2a) is equal to:
x 1
A. 2 B. 4 C. 6 D. 8
26. Which one of the following is true?
A. A polynomial can have infinitely many vertical asymptotes.
B. The graph of a rational function can never cross its horizontal asymptote.
3x  1
C. The graph of f ( x) has no horizontal asymptote.
x 1
x3  x
D. The graph of f ( x) 2 has not vertical asymptote.
x x
x2 1
27. Which one of the following is true about the graph of f ( x)  ?
x  x2
A. x = 0 and x = 1 are its vertical asymptotes.
B. Y = 1 is its horizontal asymptote.
C. Y = x – 1 is its oblique asymptote.
D. It is almost the same as the horizontal line y = -1 as x   .
x2 1
28. The value(x) of x where the graph of the function y  crosses its horizontal asymptote is (are):
x3
A. x = -2 B. x = -1 and x = 1 C. x = 0 D. x = - 2 and x = 1 + 2
p( x) 1 1
29. If p(x) = 3x2 and q(x) = x2 + x, then what is the solution set of   ?
3q( x) x q( x)
A. {-1,2} B. {2} C. {-3,2} D. {-3}
30. Which of the following functions could most likely be drawn as in the Figure 1 below?
x3
A.
x2
x 2  2x
B. 2
x 4
 x2  x  6
C.
x2  4
 x2  x  6
D.
x2  4
1
1
31. What is the solution set of x  3x 2  x ?
1 1
1 2 1
x x
 1 1    1   1
A.  1,  B.   C. 3,  D.  
 3 3  3 3

Telegram channel https://t.me/johnson201485


405
32. Suppose f  x  
Q( x)
where Q(x) is a quadratic function. Which one of the following is
x( x 2  1)
necessarily true about the graph of f ?
A. x= 0, x = 1 and x = -1 are the vertical asymptotes of the graph of f .
B. The graph of f does not intersect with its horizontal asymptote.
C. The vertical asymptote of the graph of f is only x = -1 if Q(x) = x2-x.
D. The vertical asymptote of the graph of f is only x = 1 if Q(x) = 2x2.
x 2
33. Which one of the following is true about the horizontal asymptote(s) of the graph of y  ?
x
A. y = 2 is the only horizontal asymptote of the graph.
B. y = 1 and y = -1 are the horizontal asymptotes of the graph.
C. y = 2 and y =-2 are the horizontal asymptotes of the graph.
D. y = 1 is the only horizontal asymptote of the graph.
34. If line 5x + By -6 = 0 is perpendicular to the line 7y – 4x -1 = 0, then B is:
B. C. D.
35. Which of the following is equation of a circle with center at (2,4) that passes through the point(5,8)?
A. (x+2)2 +(y-4)2 =25 C. (x-2)2 +(y+4)2 =25
2 2
B. (x-2) +(y-4) =25 D. (x+2)2 +(y+4)2 = 254
f ( x)
36. If g(x) =  ( f ( x)) 2 , f ( z )  8andf ' (1)  2, then g‟ (1) is equal to
x 1
A. 36 B. 31 C. 25 D. 16
3
37. If a line with angle of inclination of  passes through (0,1), which one of the following is the
4
equation of the line?
A. y = -x + 1 B. y = x+1 C. y = -x -1 D. y = x-1
38. If x2-6x+y2+k = 0 is equation of a circle with radius w, then what is the value of k?
A. 13 B. 5 C. 4 D. –4
39. A parabola with focus at (3,-1) has directrix y = 3. Which one of the following is the equation of the
parabola?
A. (x-3)2 = -4(y+1) B. (x-3)2=-8(y-1) C. (x-3)2=4(y+1) D. (x-3)2=8(y-1)
Answer questions number 1 and 2 depending on the following data that shows students age
distribution from different grade level.
Age 10 – 14 15 – 19 20 – 24 25 – 29 30 – 34
Frequency 7 6 9 2 1
40. One of the following is the median class?
A. 2nd B. 3rd C. 4th D. 5th
41. What is the value of Q2 in the distribution?
A. 20.08 B. 22.375 C. 19.08 D. 14.2
42. If A and B are independent events such that P (A) = ½ and P (B) = , then what is P (AUB)?
A. ½ B. C. D.
43. If ( ) = 36, then one of the following is the value of „x‟.
A. 8 B. 9 C. 10 D. 11
2 3 4
44. If the coefficient of x in the expansion of (x + ) is 144, the value of „a‟ is

Telegram channel https://t.me/johnson201485


406
A. 24 B. 2 C. 2√ D. 2√
45. A school has three classrooms for grade 11, namely 11A, 11B and 11C the number of students in
these classrooms is 28,20 and 22 respectively. All the students took and examination and the average
score of the students of 11A, 11B and 11C is 60,70 and 70, respectively. What is the average score
in this examination for all grade 11 students?
A. 66 B. 66.67 C. 65 D. 65.67
46. You are given a data on the age of students, in a primary school.
Age 8 10 11 12 13
Number of students 5 15 8 10 2
Which of the following is NOT true about the data
A.The median is 10.5 C. The mean is 10.5
B. The mode is 10 D. The range is 5
47. The following is a table of simple frequency distribution of a data with variable x.
x 1 3 4 5 7
Frequency 2 5 6 5 2
The standard deviation of the data is equal to:
A. 3 B. 3 C. 2.3 D. 3.6
48. A three – digited library identification card is to be printed from the numbers 0,1,2,3,4,5 in such a
way that the first is non – zero and no number is to repeated. How many such cards can be printed?
A. 100 B. 120 C. 150 D. 180
49. A student needs to select 3 books from 3 different mathematics, 3 different physics and 1 history
book. What is the propballity that one of them is mathematics and the other two are either physics or
history books?
3 9 15 18
A. B. C. D.
35 35 35 35
50. Items produced by certain company are subjected to two kinds of defects D1 and D2. Out of the total
production, if 5% have defect D1 10% have defect D2 and 2% have both defects, then what is the
propbality for an item to have defect D2, given that is has defect D1.
A. 0.2 B. 0.05 C. 0.1 D. 0.4
51. Among students who took a quiz, 15 students scored 6,20 students scored y,10 students scored 8 and
5 students scored 10. What is the average score of the students?
A. 7.8 B. 7.5 C. 7.2 D. 7.0
52. How many four digit even numbers can be formed from 1,2,3,4 and 5 if the numbers start with 3?
A. 40 B. 50 C. 100 D. 120
53. The mark that students scored in an examination is grouped in class intervals as shown in the
following table.
Class Interval (Mark) Number of students
55-64 8
65-74 12
75-84 20
85-94 6
95-100 4

What is the median of the mark?


A. 25.0 B. 75.5 C. 77.0 D. 79.5

Telegram channel https://t.me/johnson201485


407
54. A committee consisting of 3 students is to be selected from 10 candidates among which 4 are girls.
What is the probability that at least one girl is selected?
5 2 1 1
A. B. C. D.
6 3 3 6
55. A box contains 5 white, 6 red and 4 black balls of identical size. If 3 balls are randomly taken out of
the box one after the other, what is the propbability that the first ball is white and both the second
and third balls are red?
2 3 4 5
A. B. C. D.
15 15 75 91
56. If the list of a measurement is 10,  , 5,  , 5,10,20,15,20,5 with mean x , then what is the value of 
in terms of x ?
A. 10 x - 90 B. 9 x – 90 C. 5 x – 90 D. 5 x – 45
57. The following is the frequency distribution of a grouped data.
Class Intervals Frequency (f)
3–7 2
8 – 12 2
13 – 17 10
18 – 22 6
What is the mean and standard deviation of the distribution, respectively?
A. 15, 2 5 B. 15, 7.5 C. 12.5, 5 2 D. 12.5, 15
58. If distinct codes (words) of eight letters are formed by rearranging the letters in the word
“ABBEBAYE”, how many of the codes begin with B or Y?
A. 840 B. 630 C. 1680 D. 420
59. A company produced 25,000 bulbs and randomly tested 2% of the product. Among the tested bulbs,
if 40 have defect of type D1, 60 have defect of type D2 and 25 have both types of defects, what is the
probability that a bulb produced by the company has none of the defects?
A. 0.95 B. 0.80 C. 0.85 D. 0.20
60. If S is a set with 10 elements and A  S , what is the probability that A has 3 or more elements?
7
8 121 7
A. 10 B. 11 C. 128 D. 128
61. The following is set of data representing the average mark of 13 students: 91, 89, 93, 91, 87, 94, 92,
85, 91, 90, 96, 93, 89. Then which one of the following statements is true about the data?
A. The median is 90.5. C. The range of the marks is 11.
B. The upper quartile is 92. D. The mean is 91.5
62. Different codes, each of which consisting of five characters, are to be generated in such a way that
the first two characters are any of the English capital letters (A to Z) and the remaining three are any
of the digits (0,1,…,9). How many distinct codes can be generated so?
A. 468,000 B. 260 C. 676,000 D. 26! x 10!
63. A ladder 6m long rests against a vertical wall. If the bottom of the ladder slides away from the wall
1
at a rate (speed) of m / sec , how fast is the angle between the top of the ladder and the wall
2

changing when the angle is rad?
4

Telegram channel https://t.me/johnson201485


408
2 2 2 2
A. rad / sec B. rad / sec C. rad / sec D. rad / sec
12 2 6 3
64. A city has two daily newspapers, X and Y. The following information was obtained from a survey of
100 residents of the city: 35 people subscribe to X,60 people subscribe to Y and 20 subscribe to both
newspapers. Then how many of the people in the survey do not subscribe to either of the
newspapers?
A. 5 B. 25 C. 40 D. 55
65. A measurement is groped into five class intervals with the following frequency distribution.
Class Interval 5 - 15 15 - 25 25 – 35 35 - 45 45 – 55
55Frequency 22 40 68 50 20
What are the first quartile Q1 and the 75th percentile P75 of the measurement?
A. Q1 = 20, P75 = 40, C. Q1 = 20, P75 = 39
B. Q1 = 22, P75 = 40 D. Q1 = 22, P75 = 39
66. Three persons P1, P2 and P3 are firing at a target independently and have a probability 0.7, 0.5 and
0.4, respectively, of hitting the target. What is the probability that at least one of them hits the target?
A. 0.95 B. 0.85 C. 0.91 D. 0.99
67. The following is a simple frequency distribution of a data with variable X.
X 3 5 6 7
Frequency 2 5 2 1
 2
What are the mean x and variance Q of the data?
2
A. x = 5, Q = 0.7 C. x = 6, Q 2 = 0.7
B. x = 6, Q 2 = 1.4 D. x = 5, Q 2 = 1.4
68. A box contains 10 items of which 3 are defective. If 2 items are randomly taken out of the box, what is
the probability that both items are not defective?
7 4 7 49
A. B. C. D.
10 7 15 100
69. Items produced by a certain company are subjected to two kinds of defects D1 and D2. Out of the total
product, 5% have the defect D1, 10% have the defect D2, and 2% have both defects. What is the
probability that a randomly selected item has neither defect D1 nor defect D2?
A. 0.13 B. 0.5 C. 0.98 D. 0.87
70. There are three children in a room, ages three, four, and five. If a four-year-old child enters the room
then which one of the following is true?
A. Mean age will stay the same but the standard deviation will increase.
B. Mean age will stay the same but the standard deviation will decrease.
C. Mean age and standard deviation will increase.
D. Mean age and standard deviation will stay the same.
71. In how many more ways can 4 people be arranged in a row than if they were arranged in a circle?
A. 1 B. 6 C. 18 D. 12
72. Two machines A and B work independently. The probability that both machines A and B work is 0.4.
if the conditional probability that machine B works given that machine A works is 0.5, then the
conditional probability that machine A works given that machine B works is .
A. 0.8 B. 0.3 C. 0.5 D. 0.7
73. If ( ) is inverse of. /, then the values of x and y respectively?
A. 1 and 3 B. 2 and 2 C. 4 and 1 D. ½ and -3/2

Telegram channel https://t.me/johnson201485


409
74. Given that A = . / and B-1 = . / then which of the following is (3AB)-1?

A. ( ) B. ( ) C. ( ) D. ( )

75. What is the solution of the following system of linear equation{ ?

A. *( )+ B. *( )+ C. *( )+ D. *( )+
76. Let ( ) and ( ) , then the values of and respectively such that
is a diagonal matrix.
A. B. C. D.

77. For what value of m the matrix A = ( ) is skew symmetric matrix.

A. 3 B. -3 C. 0 D. 5
1 y
2 0 x
78. Let A = and B = 0 3 Such that A + 2Bt = 0
2y x  y 4
1 x 2
Then which of the following is the value of y?
13
A. 0 B.  C. -8 D. any real number
2
 
2 0 0
A = 1 5 0  and |B| =
1
79. Let A and B be 3 x 3 matrices such that Which one of the
 1 10
0 1 
 2
following is equal to |2ABT|?
A. 1 B. 4 C. 100 D. 400
x  y  xz  1
80. What is the solution set of the following system of equations x  2y  x  2
 2 x  2 y  4 z  2
A. {(0,1,0)} C. {(-3k,k+1,k|k  (  , )}
B.  , ) D. {(3k,k-1,k|k  (  , )}
 1 2
81. Suppose A =   . If X is a 2x2 matrix such that AX – AT = 2A , then what is the value ofX?
 2 3
 3 1  3 3  3 6  3 0
A.   B.   C.   D.  
 1 3   3 3   6 9   0 3 
82. Suppose that A and B are 3x3 matrices, I is the identity matrix of order 3 such that AB = 2I. If det
B = |B| = 6. What is det (AT)?
1 4
A. B. C. 12 D. 48
3 3

Telegram channel https://t.me/johnson201485


410
ax  y  z  1
83. Consider the system x  2 y  4z  0 If the determinant of the coefficient matrix is 2, then
5x  y  z  0
what is the solution of the system of equations?
 19  11   3 19 11 
A.  3 ,  C.  , , 
 2 2   2 2 
 19  11   3 19  9 
B.  3, ,  D.  , , 
 2 2  2 2 2 
 2 0  1
 
84. If A   1 2 0  and (2A + B)T= ATA, then which one of the following is equal to B?
 0 0  1
 
1 0  2 0 0 0 8 0  4  1 2 0
       
A.  2 0 0  B.  0 0 0 C.  4 8 0  D 0 0 0
0 4  0 0  0  4   2 4 
 0  0  0  0
 0 1 2
 
85. If M   3  1 0  and ATM=2I, where A is a 3x3 matrix and I is the identity matrix of order 3,
 5 2 4
 
then what is det (A)?
4 1
A. 0.2 B. C. 0.8 D.
17 17
x  y  z  1

86. What should be the value of K so that the system of equation  x  5 y  4 z  1 has a solution?
2 x  2 y  z  k

A. 0 B. 1 C. -4 D. 4
87. Suppose AX = b, where A is a 3 x 3 matrix, b = (b1,b2,b3)T and X = (x,y,z)T. which one of the
following is necessarily true about this system of linear equations?
A. The system has a solution only when det (A)  0.
B. The Cramer‟s rule is suitable to solve the system if two rows of A are identical.
C. If det (A)  0 and the second column of A is a multiple of b, then x = 0.
D. If b = 0, then X = (0,0,0)T is the only solution of the system.
3  2 8
 
88. If A   0 6 7  , then det (ATA) is equal to __________
0 4 5
 
A. 12 B. 36 C. 30 D. 15
0 x 0  1 1 1 
  1
 
89. If  1  1 1  and A   3 0 0  , then what are the values of x and y?
 0 y  1  2 0  1
   
A. x = 3, y = -2 C. x = -3, y = 2

Telegram channel https://t.me/johnson201485


411
2 1 1 2
B. x  ,y  D. x  , y 
3 3 3 3
ax  by  2

90. Consider the following system of equations:  x  3 y  2 z  0 If the determinant of the coefficient
2 x  y  z  0

matrix is 2, then what is the solution set of the system?
A. {(1,3,-5)} C. {(-2,-6,10)}
 1 1 
B.  , ,0  D. 
 a b 
x  y  z  1
91. What is the solution set of the system  x  2 y  3z  1 ?
2 x  3 y  4 z  2

A. {(0,2,1)} C. {(2k+1,-k,k)|k   }
B. {(1-k,2k,k)|k   } D. 
2 7 1 1 
92. If A =   and B-1 =   , then (AB)-1 is equal to:
  0 2
1 3   
 4  3
A.   B.   2 5  C.   3 11 D.  4 0
 
 4  5 2  4  1  3   0 2
     
0    6 x
 
93. Let A =  2 2 1 , b   0  , and x=  y  . If det(A) = 3, then what is the solution set of the system
 
  0 z 
3 1 2     
 
AX = b?

 1 5  
6,2,8   
T

C.  3,1,4 D. 
T T
A. B.  0, ,  

 a b   
94. For any n  n square matrix A, which one of the following is true?
A. Det(A) = -det (AT), where AT is the transpose of A
B. If k is a scalar, then det (kA) = Kndet (A)
C. If B is a matrix obtained from A by interchanging of two rows of A, then det (B) = det (A)
D. If A is invertible, then det (A) = det (A-1)

 x  3x  2 z  6
95. The solution of the system of linear equation of 2 x  4 y  3z  8 is:
 3x  6 y  8 z  5

A. X = -1, y = -3, z = -2 C. x = 1, y = -3, z = 2
B. X = -1, y = -3, z = 2 D. x = 1, y = 3, z = -2
96. If is perpendicular to the n, what is the cosine of the angle between and ?
| | | | | | | |
A. B. C. D.
| | | | | | | |
97. Suppose = (3, x) and = (x, y-2) are vectors, then what is the value of „x‟ so that
3

Telegram channel https://t.me/johnson201485


412
A. 3 B. 5 C. 4 D.-4
98. Vector equation of a line passing through points (3, 2) and (1, -1) is:
A. (x, y) = (1, -1) + t(2, 3) C. (x, y) = (1, -1) + t(3, 2)
B. (x, y) = (3, 2) – t(2, 3) D. (x, y) = (3, 2 ) – t(2, 3
99. The image of the line l: y = x + 4 after it has been reflected along the line l: y = x – 3
A. y = 2x – 10 B. y = 2x +10 C. y = x- 10 D. y = x + 10
100. The image of the point (√ ) under the rotation of anticlockwise about the origin is:
A. (-2, 0) B. (4, -2) C. (-1, 2) D. (2, -3)
101. The exact value of .sin . / s . // is
A. -1 B.1 C. D.
th
102. What is 100 term of the sequence 3, 10, 17,24,31,…?
A. 463 B. 510 C. 696 D. 531
35
103. If {an} is a sequence such that a1 = 2,and an+1 = an + 4for all n 1, then a
n 1
n = __

A. 2460 B. 2458 C. 2450 D. 2442


104. Which one of the following relation holds for the sequence: -10, -4, 2, 8, 14,..?
A. an = an-1 – 6 B. an = an-1 + 6 C. an = an-1 – 7 D. an = an-1 +7
105. What is the fourth term of a geometric sequence whose third and eighth terms are 1 and
respectively.
A. B. C. D.
106. If x, 4x + 3 and 7x + 6 are consecutive terms geometric sequence, then value of „x‟ is:
A. -1 B. 1 C. 0 D. -2
107. For what value of k the line y = -8x + k is tangent to the curve y = 3x2 + 4x + 1?
A. -11 B. -2 C. 2 D. 11
108. Let f(x) = e-2x, what is the nth derivative of the function f at x = o, for positive integer n?
A. (-2)n B. 2n C. -2n D. 2-n
109. If f(x) = ln x 2  1 , then which of the following is equal to f ' ( x)
x x 2x 2x
A. B. 2 C. D. 2
x 1
2 x 1 x 1
2 x 1
th n
110. For which smallest value of n the n derivative f (x) = f(x) in the function f(x) = sinx.
A. 4 B. 6 C. 8 D. 10
111. Air is pumped in to spherical balloon at a rate of 50cm3/se, what is the rate of change of radius if the
diameter is 5cm?
A. B. C. D.
112. If f(x) = x x  4 on [-4, 0] for which value(s) of c  (-4, 0) that satisfy the conclusion of Roll‟s
theorem.
A. B. C. D.
3 2
113. Let f(x) = x – 9x + 24x + 3, then the interval in on which f is decreasing.
A. [0, 2] B. [2, 4] C. [4, ) D. ( , 2]  [4, )
114. If x2 + xy = 10, then what is the value of when x = 2?
A. - B. C. D.

Telegram channel https://t.me/johnson201485


413
115. An object is moving along the parabola Y=2√ in xy-plane at what point on its path does the object
be closest to the point (5, 0)?
A. (4,4) B.(1,2) C.(3, √ ) D.(3, √ )
116. What is the value of ∫ ( ) given that ∫ ( ) ∫ ( ) ?
A. 8 B. -3 C. 6 D. 5
117. Which one of the following is equal to ∫ ?
A. B. C. D.
x
118. Area of the region enclosed by the graph of f(x) = e and g(x) = x between the lines x = -1 and x = 1?
A. B. C. D.
119. Which of the following is equal to the volume generated when the region bounded by the graph of
y = 2√ and the x-axis, for 0 x 2, rotates about the x-axis?
A. 8 B. 8 C. 16 D. 16
120. Consider the following frequency distribution table of a grouped data, where a class interval includes
its lower class boundary and excludes its upper boundary. Then which of the following is true?
Value 5-15 15-25 25-35 35-40
Frequency 10 20 15 5
A. It is positively skewed C. The mode of the data is =23
B. It is negatively skewed D. The median of the data is=22.5
121. A group of 100 items have a mean of 62, if the mean of 40 of the items is 55. What is the mean of
the remaining items?
A. 69 B. 96 C. 66.67 D. 64.55
122. In a college a total of 1340 students are enrolled in the following departments ICT, AUTO,
Drafting and Electricity. If the number of students enrolled in ICT is 335, 40% in Drafting &15% in
Electricity, Then what percent of the total will be enrolled in AUTO department?
A. 10 B. 20 C. 15 D. 5
123. Suppose four students out of 25 score the highest mark in a test ,then it can be defined in degree
as
A. 500 B. 57.60 C. 75.60 D. 570
124. Mean deviation about the mean of the following data is:
X 0–5 6 – 11 12 – 17 18 - 23 24 - 29
F 5 8 7 10 3
A. 14. 14 B. 14.2 C. 14.4 D. 14.5
125. A salvage value of a machine that costs 18,000 is Birr 300 at the end of 8 years. What is the
depreciation of the machine for the fourth year?
A. 8850 B. 15000 C. 21000 D. 2212.5
126. Which of the following is the type of investment issued by government or corporation usually
known as fixed income security?
A. Saving account C. Certificate of deposit
B. Stock D. Bond
127. A car dealer fixed selling price of a car to be Birr 200000.In the meantime 10% tax was allotted. If
the car is sold for Birr240000, then the percentage profit gained is?
A.20 B.15 C.10 D.12
128. A washing machine that costs Birr 3100 has a service life of 20 years .If its monthly straight line
deprecation is Birr 18, what is its salvage value?
A.292 B.310 C.940 D.980

Telegram channel https://t.me/johnson201485


414
129. The price of an item after 20% reduction was 90 Birr, and then what is the original price?
A.312.5 B.250 C.225 D.112.5
The following table gives the number of teachers in a given school according to their educational
level and sex, then answer question 178 and 179 depending on this table.
Diploma Degree Total
Male 28 24 52
Female 26 22 48
Total 54 46 100
130.Using this table answer questions number 26 and 27 The ratio of male degree holders to the total in
the school is:
A. 28:100 B. 24:100 C. 52:100 D. 24:28
131.The ratio of diploma holders to degree holders in the school is:
A. 46:54 B. 54:46 C. 24:28 D. 22:26
132. Allocate birr 6000 in the ratio of 2:3:7to three students A, B, and C, then A get birr
A. 500 B. 1500 C. 1000 D. 3500
133. A farmer has birr 1,700to by sheep and goats suppose the unit price of a sheep is birr 400 and unit
price of goat is birr 200. If he has bought two sheep then what is the maximum number of goats he
can buy with the remaining money?
A. 5 B. 4 C. 10 D. 8
134. Marta bought a shoe for birr 280 and want to sell it at 24% mark up, then the mark up is:
A. 64 B. 56.4 C. 67.2 D. 70
135. The equation of a line passing through (-1, 2) and slope 4 is:
A. y = 4x – 7 B. y = -4x +7 C. y = 4x + 7 D. y = -4x – 7
136. The line with equation 4x + ky = 8 parallel to the x + 2y = 0, then one of the following is the value
of „k‟
A. 8 B. 4 C. ¾ D. 3/16
137. The maximum value of the objective function z = 2x + 3y subject to the given constraints

A. 30 B. 24 C. 20 D. 25
138.The value of x and y that gives the minimum value of the objective function z = 4x + 2y subject to

given constraints

A. x = 1, y = 4 B. x = 2, y = 1 C. x = 4, y = 0 D. x = 0, y = 4
139. The possible values of the point which minimizes the value of the objective function
Z= -2x+y Subject to

are:
A. (0, 0) B. (7, 4) C. (7,0) D. (5,2)

Telegram channel https://t.me/johnson201485


415

You might also like